You are on page 1of 134

SALES Case Digest (Atty.

Sarona)
Compiled by: Wigmore #wigmoreforever
PART I: CONCEPT OF SALE

1. BASIC CONCEPTS

1. Contract of Sale (Article 1458)


TEODORO ACAP vs. CA
G.R. No. 118114. December 7, 1995.
Padilla, J.
Doctrine: Ownership and real rights are acquired only pursuant
to a legal mode or process. While title is the juridical justification,
mode is the actual process of acquisition or transfer of ownership
over a thing in question.
FACTS: Teodoro Acap has been a tenant of a portion of land of
Lot No. 1130 Hinigaran Cadastre since 1960. Said lot was
formerly owned by Spouses Vasquez and Lorenza Oruma, which
upon their death was inherited by Felixberto. In 1975, Felixberto
sold the lot to Cosme Pido. Acap remained to be a registered
tenant of the said land and religiously paid his leasehold rentals
to Pido and thereafter, upon his death, to his widow Laurenciana.
On 1981, Pidos wife and children executed a notarized
document denominated Declaration of Heirship and Waiver of
Rights of the land in favor Edy delos Reyes. Delos Reyes
alleged that he and Acap entered into an oral lease agreement
whereby Acap undertook to pay him 10 cavans of rice per year
as lease rental. From 1983 onwards Acap refused to pay further
lease rentals. In defense, Acap denied having entered in an oral
lease agreement with delos Reyes and that he did not recognize
his ownership over the land. As a matter of fact he alleged that
he continued to pay Laurenciana, Pidos wife. Delos Reyes filed
a suit of recovery of possession against Acap and for the
payment of rentals accruing to him as owner of the said lot. Trial
court rendered decision in favor of delos Reyes ruling that there
was a perfected sale between heirs of Pido and delos Reyes
over the said lot and ordered Acap to deliver possession of the
same to delos Reyes. Upon appeal, CA affirmed the lower courts
decision. Hence, this petition.
ISSUE: Whether delos Reyes acquired ownership over the lot in
question.
HELD: NO. The Court noted that an asserted right or claim to
ownership or a real right over a thing arising from a juridical act,
however justified, is not per se sufficient to give rise to ownership
over the res. That right or title must be completed by fulfilling
certain conditions imposed by law. Hence, ownership and real
rights are acquired only pursuant to a legal mode or process.
While title is the juridical justification, mode is the actual process
of acquisition or transfer of ownership over a thing in question.
Under Article 712 of the Civil Code, modes of acquisition may
either be original or derivative. Original modes of acquisition
include occupation, acquisitive prescription, law or intellectual
creation. Derivative modes of acquisition on the other hand
include succession mortis causa and tradition as a result of
certain contracts such as sale, barter, donation, assignment or
mutuum. In the instant case, the Court determined whether delos
Reyes acquired ownership over the lot in question through any of
the modes mentioned. It was ruled that he had not acquired

ownership by virtue of sale, as opposed to the ruling of both RTC


and CA. The execution of the heirs of Pido the Declaration of
Heirship and Waiver of Rights was held to be not tantamount to
sale. Such declaration is only one whereby heirs adjudicate and
divide the estate left by the decedent among themselves as they
see fit. The Court further noted that waiver of hereditary rights is
different from sale of hereditary rights. Sale of hereditary rights
presupposes an existence of a contract of sale whereas waiver
of hereditary rights is an abdication or intentional relinquishment
of a known right with a knowledge of its existence and intention
to relinquish it in favor of other persons who are co-heirs in the
succession. As delos Reyes is a stranger to the succession of
Cosme Pido, he cannot claim ownership over the lot on the sole
basis of the document executed. Hence, private respondent
delos Reyes had not acquired ownership over Lot 1130 and
consequently had no right to exact lease rentals from petitioner
Acap.

Toyota Shaw Inc. vs. Court of Appeals, and Sosa


244 SCRA 320
May 1995
FACTS:
Luna L. Sosa and his son, Gilbert, went to purchase a yellow
Toyota Lite Ace from the Toyota office at Shaw Boulevard, Pasig
(petitioner Toyota) on June 14, 1989 where they met Popong
Bernardo who was a sales representative of said branch. Sosa
emphasized that he needed the car not later than June 17, 1989
because he, his family, and a balikbayan guest would be using it
on June 18 to go home to Marinduque where he will celebrate
his birthday on June 19. Bernardo assured Sosa that a unit
would be ready for pick up on June 17 at 10:00 in the morning,
and signed the "Agreements Between Mr. Sosa & Popong
Bernardo of Toyota Shaw, Inc., a document which did not
mention anything about the full purchase price and the manner
the installments were to be paid. Sosa and Gilbert delivered the
down payment of P100,000.00 on June 15, 1989 and Bernardo
accomplished a printed Vehicle Sales Proposal (VSP) No. 928
which showed Sosas full name and home address, that payment
is by "installment," to be financed by "B.A.," and that the
"BALANCE TO BE FINANCED" is "P274,137.00", but the spaces
provided for "Delivery Terms" were not filled-up.
When June 17 came, however, petitioner Toyota did not deliver
the Lite Ace. Hence, Sosa asked that his down payment be
refunded and petitioner Toyota issued also on June 17 a Far
East Bank check for the full amount of P100,000.00, the receipt
of which was shown by a check voucher of Toyota, which Sosa
signed with the reservation, "without prejudice to our future
claims for damages." Petitioner Toyota contended that the B.A.
Finance disapproved Sosas the credit financing application and
further alleged that a particular unit had already been reserved
and earmarked for Sosa but could not be released due to the
uncertainty of payment of the balance of the purchase price.
Toyota then gave Sosa the option to purchase the unit by paying
the full purchase price in cash but Sosa refused.
The trial court found that there was a valid perfected contract of
sale between Sosa and Toyota which bound the latter to deliver
the vehicle and that Toyota acted in bad faith in selling to another
the unit already reserved for Sosa, and the Court of Appeals
COMPILED BY: WIGMORE #WIGMOREFOREVER 1

SALES Case Digest (Atty. Sarona)


Compiled by: Wigmore #wigmoreforever
affirmed the said decision.

NDC to sell the leased property in its favor.

ISSUE:

Issue:
1. Whether or not there is a valid sale between NDC and PUP.

Was there a perfected contract of sale between respondent Sosa


and petitioner Toyota?
COURT RULING:
The Supreme Court granted Toyotas petition and dismissed
Sosas complaint for damages because the document entitled
Agreements Between Mr. Sosa & Popong Bernardo of Toyota
Shaw, Inc., was not a perfected contract of sale, but merely an
agreement between Mr. Sosa and Bernardo as private
individuals and not between Mr. Sosa and Toyota as parties to a
contract.
There was no indication in the said document of any obligation
on the part of Toyota to transfer ownership of a determinate thing
to Sosa and neither was there a correlative obligation on the part
of the latter to pay therefor a price certain. The provision on the
downpayment of P100,000.00 made no specific reference to a
sale of a vehicle. If it was intended for a contract of sale, it could
only refer to a sale on installment basis, as VSP No.928
executed on June 15, 1989 confirmed. The VSP also created no
demandable right in favor of Sosa for the delivery of the vehicle
to him, and its non-delivery did not cause any legally
indemnifiable injury.

Polytechnic University of the Philippines vs Court of


Appeals and Firestone Ceramics
National Development Corporation vs Firestone Ceramics
Inc.
[GR No. 143513 and 143590. November 14, 2001]
Bellosilo, J.:
Facts:
Petitioner National Development Corp., a government owned
and controlled corporation, had in its disposal a 10 hectares
property. Sometime in May 1965, private respondent Firestone
Corporation manifested its desire to lease a portion of it for
ceramic manufacturing business. On August 24, 1965, both
parties entered into a contract of lease for a term of 10 years
renewable for another 10 years. Prior to the expiration of the
aforementioned contract, Firestone wrote NDC requesting for an
extension of their lease agreement. It was renewed with an
express grant to Firestone of the first option to purchase the
leased premise in the event that it was decided "to dispose and
sell the properties including the lot..."
Cognizant of the impending expiration of the leased agreement,
Firestone informed NDC through letters and calls that it was
renewing its lease. No answer was given. Firestone's
predicament worsened when it learned of NDC's supposed plans
to dispose the subject property in favor of petitioner Polytechnic
University of the Philippines. PUP referred to Memorandum
Order No. 214 issued by then President Aquino ordering the
transfer of the whole NDC compound to the National
Government. The order of conveyance would automatically result
in the cancellation of NDC's total obligation in favor of the
National Government.
Firestone instituted an action for specific performance to compel

Ruling
A contract of sale, as defined in the Civil Code, is a contract
where one of the parties obligates himself to transfer the
ownership of and to deliver a determinate thing to the other or
others who shall pay therefore a sum certain in money or its
equivalent. It is therefore a general requisite for the existence of
a valid and enforceable contract of sale that it be mutually
obligatory, i.e., there should be a concurrence of the promise of
the vendor to sell a determinate thing and the promise of the
vendee to receive and pay for the property so delivered and
transferred. The Civil Code provision is, in effect, a "catch-all"
provision which effectively brings within its grasp a whole gamut
of transfers whereby ownership of a thing is ceded for a
consideration.
All three (3) essential elements of a valid sale, without which
there can be no sale, were attendant in the "disposition" and
"transfer" of the property from NDC to PUP - consent of the
parties, determinate subject matter, and consideration therefor.
Consent to the sale is obvious from the prefatory clauses of
Memorandum Order No. 214 which explicitly states the
acquiescence of the parties to the sale of the property.
Furthermore, the cancellation of NDC's liabilities in favor of the
National Government constituted the "consideration" for the sale.

Manila Metal Container Corporation vs Philippine


National Bank
[GR No. 166862, December 20, 2006]
Callejo, Sr., J.:
Facts:
Petitioner was the owner of 8,015 square meters of parcel
of land located in Mandaluyong City, Metro Manila. To secure
a P900,000.00 loan it had obtained from respondent Philippine
National Bank, petitioner executed a real estate mortgage over
the lot. Respondent PNB later granted petitioner a new credit
accommodation. On August 5, 1982, respondent PNB filed a
petition for extrajudicial foreclosure of the real estate mortgage
and sought to have the property sold at public auction. After due
notice and publication, the property was sold at public action
where respondent PNB was declared the winning bidder.
Petitioner sent a letter to PNB, requesting it to be granted an
extension of time to redeem/repurchase the property. Some PNB
personnel informed that as a matter of policy, the bank does not
accept partial redemption. Since petitioner failed to redeem the
property, the Register of Deeds cancelled TCT No. 32098 and
issued a new title in favor of PNB.
Meanwhile, the Special Asset Management Department
(SAMD) had prepared a statement of account of petitioners
obligation. It also recommended the management of PNB to
allow petitioner to repurchase the property for P1,574,560.oo.
PNB rejected the offer and recommendation of SAMD. It instead
suggested
to
petitioner
to
purchase
the
property
for P2,660,000.00, in its minimum market value. Petitioner
declared that it had already agreed to SAMDs offer to purchase
for P1,574,560.47 and deposited a P725,000.00.
COMPILED BY: WIGMORE #WIGMOREFOREVER 2

SALES Case Digest (Atty. Sarona)


Compiled by: Wigmore #wigmoreforever
Issue:
Whether or not petitioner and respondent PNB had entered
into a perfected contract for petitioner to repurchase the property
for respondent.

1.
2.
3.

Ruling:
The SC affirmed the ruling of the appellate court that there
was no perfected contact of sale between the parties.
A contract is meeting of minds between two persons
whereby one binds himself, with respect to the other, to give
something or to render some service. Under 1818 of the Civil
Code, there is no contract unless the following requisites concur:
Consent of the contracting parties;
Objection certain which is the subject matter of the contract;
Cause of the obligation which is established.
Contract is perfected by mere consent which is manifested
by the meeting of the offer and the acceptance upon the thing
and causes which are to constitute the contract. Once perfected,
the bind between other contracting parties and the obligations
arising therefrom have the form of law between the parties and
should be complied in good faith. The absence of any essential
element will negate the existence of a perfected contract of sale.
The court ruled in Boston Bank of the Philippines vs
Manalo:
A definite agreement as to the price is an essential
element of a binding agreement to sell personal or real property
because it seriously affects the rights and obligations of the
parties. Price is an essential element in the formation of a
binding and enforceable contract of sale. The fixing of the price
can never be left to the decision of one of the contracting parties.
But a price fixed by one of the contracting parties, if accepted by
the other, gives rise to a perfected sale.
In the case at bar, the parties to the contract is between
Manila Metal Container Corporation and Philippine National Bank
and not to Special Asset Management Department. Since the
price offered by PNB was not accepted, there is no contract.
Hence it cannot serve as a binding juridical relation between the
parties.

Spouses Cruz vs Fernando


477 SCRA 173 Civil Law Law on Sale Manner of Payment
Essential in a Contract of Sale
In 1983, Cruz executed a Kasunduan with the Gloriosos for the
consideration of the rear portion of a 223 sq m lot.
The Kasunduan provides that the lot will be sold at a P40 per sq
m. That the portion of the lot to be sold is the rear portion of it.
That upon selling, the Cruz will transfer their house from the front
portion to the rear portion of the land once it is bought. That they
will have a right of way from the front portion going to the back
end of the lot. The Cruz never gave anything to the Gloriosos for
there was an alleged failure to have the land surveyed. Due to
non payment, the Gloriosos instead sold the whole lot (back and
rear portion) to the Fernandos.
In 1994, after repeated demands, the Fernandos filed a case in
court for accion publiciana demanding the Cruz to vacate the lot
and to pay a rental of P500.00. The RTC ruled in favor of the
Fernandos. The CA affirmed the RTC ruling.
ISSUE: Whether or not what transpired between the Cruzes and
the Gloriosos was a contract of sale.

HELD: No. The absence of a specific manner of payment in the


terms and conditions of the contract makes it a contract to sell.
Ownership was never transferred to the Cruzes. This is because
the manner of payment of the purchase price is an essential
element before a valid and binding contract of sale can exist.
Although the Civil Code does not expressly state that the minds
of the parties must also meet on the terms or manner of payment
of the price, the same is needed, otherwise there is no sale.
Also, the Cruzes never transferred their house from the front
portion to the rear portion of the lot. It was evident in the contract
that they will transfer the house to the rear portion once they
were able to buy it.
The SC also ruled that the Fernandos were not buyers in bad
faith. There was no consummated sale between the Cruzes and
the Gloriosos. In a contract to sell, there being no previous sale
of the property, a third person buying such property despite the
fulfillment of the suspensive condition such as the full payment of
the purchase price, for instance, cannot be deemed a buyer in
bad faith and the prospective buyer cannot seek the relief of
reconveyance of the property. There is no double sale in such
case. Title to the property will transfer to the buyer
after registration because there is no defect in the owner-sellers
title per se, but the latter, of course, may be sued for damages by
the intending buyer.

Velarde vs CA
Facts: David Raymundo (private respondent) is the absolute and
registered owner of a parcel of land, located at 1918 Kamias St.,
Dasmarias Village Makati, together with the house and other
improvements, which was under lease. It was negotiated by
Davids father with plaintiffs Avelina and Mariano Velarde
(petitioners). A Deed of Sale with Assumption of Mortgage was
executed in favor of the plaintiffs. Part of the consideration of the
sale was the vendees assumption to pay the mortgage
obligations of the property sold in the amount of P 1,800,000.00
in favor of the Bank of the Philippine Islands. And while
their application for the assumption of the mortgage obligations is
not yet approved by the mortgagee bank, they have agreed to
pay the mortgage obligations on the property with the bank in the
name of Mr. David Raymundo. It was further stated that in the
event Velardes violate any of the terms and conditions of the said
Deed of Real Estate Mortgage, they agree that the down
payment P800,000.00, plus all the payments made with the BPI
on the mortgage loan, shall be forfeited in Favor of Mr.
Raymundo, as and by way of liquidated damages, w/out
necessity of notice or any judicial declaration to that effect, and
Mr. Raymundo shall resume total and complete ownership and
possession of the property, and the same shall be deemed
automatically cancelled, signed by the Velardes.
Pursuant to said agreements, plaintiffs paid BPI the monthly
interest loan for three months but stopped in paying the
mortgage when informed that their application for the assumption
of mortgage was not approved. The defendants through a
counsel, wrote plaintiffs informing the latter that their nonpayment to the mortgagee bank constituted non-performance of
their obligation and the cancellation and rescission of the
intended sale. And after two days, the plaintiffs responded and
advised the vendor that he is willing to pay provided that Mr.
COMPILED BY: WIGMORE #WIGMOREFOREVER 3

SALES Case Digest (Atty. Sarona)


Compiled by: Wigmore #wigmoreforever
Raymundo: (1) delivers actual possession of the property to
them not later than January 15, 1987 for their occupancy (2)
causes the release of title and mortgage from the BPI and make
the title available and free from any liens and encumbrances (3)
executes an absolute deed of sale in their favor free from any
liens and encumbrances not later than Jan. 21, 1987.
The RTC of Makati dismissed the complaint of the petitioners
against Mr. Raymundo for specific performance, nullity of
cancellation, writ of possession and damages. However, their
Motion for Reconsideration was granted and the Court instructed
petitioners to pay the balance of P 1.8 million to private
respondent who, in turn were ordered to execute a deed of
absolute sale and to surrender possession of the disputed
property to petitioners.
Upon the appeal of the private respondent to the CA, the court
upheld the earlier decision of the RTC regarding the validity of
the rescission made by private respondents.
Issue: Whether the rescission of contract made by the private
respondent is valid.
Held: There is a breach of contract because the petitioners did
not merely stopped paying the mortgage obligations but they
also failed to pay the balance purchase price. Their conditional
offer to Mr. Raymundo cannot take the place of actual
payment as would discharge the obligation of the buyer under
contract of sale.
Mr. Raymundos source of right to rescind the contract is Art.
1191 of the Civil Code predicated on a breach of faith by the
other party who violates the reciprocity between them. Moreover,
the new obligations as preconditions to the performance of the
petitioners own obligation were repudiation of an existing
obligation, which was legally due and demandable under the
contract of sale.
The breach committed by the petitioners was the nonperformance of a reciprocal obligation. The mutual restitution is
required to bring back the parties to their original situation prior to
the inception of the contract. The initial payment and the
mortgage payments advanced by petitioners should be returned
by private respondents, lest the latter unjustly enriched at the
expense of the other. Rescission creates the obligation
to return the obligation of contract. To rescind, is to declare a
contract void at its inception and to put an end to it as though it
never was.
The decision of the CA is affirmed with modification that private
respondents are ordered to return to petitioners, the amount they
have received in advanced payment.
2. Stages in the life of Contract of Sale

SAN MIGUEL PROPERTIES v SPS HUANG


[G.R. No. 137290. July 31, 2000]
FACTS: San Miguel Properties Philippines, Inc. is a domestic
corporation engaged in the purchase and sale of real properties.
Part of its inventory are two parcels of land totalling 1, 738

square meters at the corner of Meralco Avenue and General


Capinpin Street, Barrio Oranbo, Pasig City. On February 21,
1994, the properties were offered for sale for P52,140,000.00 in
cash. The offer was made to Atty. Helena M. Dauz who was
acting for respondent spouses as undisclosed principals. Atty.
Dauz signified her clients interest in purchasing the properties
for the amount for which they were offered by petitioner, under
the following terms: the sum of P500,000.00 would be given as
earnest money and the balance would be paid in eight equal
monthly installments from May to December, 1994.
However, petitioner refused the counter-offer. On March 29,
1994, Atty. Dauz wrote another letter proposing the following
terms for the purchase of the properties, viz: Enclosing herewith
the sum of P1,000,000.00 representing earnest-deposit money,
subject to the following conditions.
1. We will be given the exclusive option to purchase the property
within the 30 days from date of your acceptance of this offer.
2. During said period, we will negotiate on the terms and
conditions of the purchase; SMPPI will secure the necessary
Management and Board approvals; and we initiate the
documentation if there is mutual agreementbetween us.
3. In the event that we do not come to an agreement on this
transaction, the said amount of P1,000,000.00 shall be
refundable to us in full upon demand.
Isidro A. Sobrecarey, San Miguels vice-president and operations
manager for corporate real estate, indicated his conformity to the
offer by affixing his signature to the letter and accepted the
"earnest-deposit" of P1 million. Upon request of respondent
spouses, Sobrecarey ordered the removal of the "FOR SALE"
sign from the properties. Atty. Dauz and Sobrecarey then
commenced negotiations.
Sobrecarey informed Atty. Dauz that San Miguel was willing to
sell the subject properties on a 90-day term. Atty. Dauz
countered with an offer of 6 months within which to pay.The
parties again met during which Sobrecarey informed Atty.
Dauz that San Miguel had not yet acted on her counter-offer.
Atty. Dauz proposed a 4-month period of amortization.
On April 25, 1994, Atty. Dauz asked for an extension of 45 days
within which to exercise her option to purchase the property,
adding that within that period to finalize the agreement." Her
request was granted. On July 7, 1994, petitioner, through its
president and chief executive officer, Federico Gonzales, wrote
Atty. Dauz informing her that because the parties failed to agree
on the terms and conditions of the sale despite the extension
granted by petitioner, the latter was returning the amount of P1
million given as "earnest-deposit."
Sps Huang demanded the execution within 5 days of a deed of
sale covering the properties. Sps Huang attempted to return the
"earnest-deposit" but San Miguel refused on the ground that
respondents option to purchase had already expired. On August
16, 1994, spouses Huang filed a complaint for specific
performance against San Miguel. SAN MIGUEL
(1) the alleged "exclusive option" of respondent spouses lacked
a consideration separate and distinct from the purchase price
and was thus unenforceable and
(2) the complaint did not allege a cause of action because there
was no "meeting of the minds" between the parties and,
COMPILED BY: WIGMORE #WIGMOREFOREVER 4

SALES Case Digest (Atty. Sarona)


Compiled by: Wigmore #wigmoreforever
therefore, no perfected contract of sale.
TC Dismissed. CA REVERSED: held that all the requisites of a
perfected contract of sale had been complied with as the offer
made on March 29, 1994, in connection with which the earnest
money in the amount of P1 million was tendered by respondents,
had already been accepted by petitioner. The fact the parties had
not agreed on the mode of payment did not affect the contract as
such is not an essential element for its validity. CA also ruled that
Sobrecarey had no authority to sell the subject real properties
ISSUE
WON there was a perfected contract of sale between the parties
RULING: No. On alleged payment and acceptance Earnest
money, Sps Huang did not give the P1 million as "earnest
money" as provided by Art. 1482 of the Civil Code. They
presented the amount merely as a deposit of what would
eventually become the earnest money or downpayment should a
contract of sale be made by them. The amount was thus given
not as a part of the purchase price and as proof of the perfection
of the contract of sale but only as a guarantee that respondents
would not back out of the sale.
In the present case, the P1 million "earnest-deposit" could not
have been given as earnest money as contemplated in Art. 1482
because, at the time when petitioner accepted the terms of
respondents offer of March 29, 1994, their contract had not yet
been perfected.
The first condition: option period of 30 days sufficiently shows
that a sale was never perfected. Acceptance of this condition did
not give rise to a perfected sale but merely to an option or an
accepted unilateral promise. Art. 1479(2) states that an accepted
unilateral promise to buy or sell a determinate thing for a price
certain is binding upon the promisor only if the promise is
supported by a distinct consideration. There is no showing here
of any consideration for the option. Lacking any proof of such
consideration, the option isunenforceable.
Second condition: that, during the option period, the parties
would negotiate the terms and conditions of the purchase. In the
present case, the parties never got past the negotiation stage.
Nothing more than offers and counter-offers which did not
amount to any final arrangement containing the essential
elements of a contract of sale. While the parties already agreed
on the real properties which were the objects of the sale and on
the purchase price, the fact remains that they failed to arrive at
mutually acceptable terms of payment, despite the 45-day
extension given by petitioner.
Also, the manner of payment of the purchase price is an
essential element before a valid and binding contract of sale can
exist. An agreement on the manner of payment goes into the
price such that a disagreement on the manner of payment is
tantamount to a failure to agree on the price. The fact, therefore,
that the petitioners delivered to the respondent the sum as part
of the DP that they had to pay cannot be considered as sufficient
proof of the perfection of any purchase and sale agreement
between the parties herein under Art. 1482 of the new Civil
Code, as some essential matter - the terms of the payment - still
had to be mutually covenanted Thus, it is not the giving of
earnest money, but the proof of the concurrence of all the
essential elements of the contract of sale which establishes the

existence of a perfected sale. In the absence of a perfected


contract of sale, it is immaterial whether Isidro A. Sobrecarey had
the authority to enter into a contract of sale in behalf of petitioner.

2. ESSENTIAL CHARACTERISTICS OF A CONTRACT OF


SALE
1. Nominate and Principal
2. Consensual

QUIJADA vs. COURT OF APPEALS


G.R. No. 126444 December 4, 1998
FACTS: Petitioners, as heirs of the late Trinidad Quijada, filed a
complaint against private respondents for quieting of title,
recovery of possession and ownership of parcels of land with
claim for attorney's fees and damages. Petitioners are the
children of the late Trinidad Corvera Vda, de Quijada. Trinidad
was one of the heirs of the late Pedro Corvera and inherited from
Pedro the 2-hectare parcel of land subject of the case, situated in
the barrio of San Agustin, Talacogon, Agusan del Sur. On April 5,
1956, Trinidad together with her sisters Leonila Corvera and Paz
and brother Epapiadito executed a conditional deed of donation
of subject land in favor of the Municipality of Talacogon, the
condition being that the parcel of land shall be used solely and
exclusively as part of the campus of the proposed provincial high
school in Talacogon. Apparently, Trinidad remained in possession
of the parcel of land despite the donation.
On July 29, 1962, Trinidad sold (1) hectare of the subject parcel
of land to defendant-appellant Regalado Mondejar and verbally
sold the remaining (1) hectare also to defendant-appellant
without the benefit of a written deed of sale and evidenced solely
by receipts of payment. In 1980, the heirs of Trinidad (deceased)
filed a complaint for forcible entry against Mondejar, which
complaint was, however, dismissed for failure to prosecute.
In 1987, the proposed provincial high school having failed to
materialize, the Sangguniang Bayan of the municipality of
Talacogon enacted a resolution reverting the (2) hectares of land
donated back to the donors. In the meantime, Regalado
Mondejar sold portions of the land to (respondents) Fernando,
Rodolfo Goloran , Efren Guden and Ernesto Goloran. On July 5,
1988, Heirs of Trinidad filed this action alleging that their
deceased mother never sold, conveyed, transferred or disposed
of the property in question to any person or entity much less to
Regalado Mondejar save the donation made to the Municipality
of Talacogon in 1956 and that at the time of the alleged sale to
Regalado Mondejar by Trinidad Quijada, the land still belongs to
the Municipality of Talacogon, hence, the supposed sale is null
and void.
Respondents alleged that the land in dispute was sold to
Regalado Mondejar, the one (1) hectare on July 29, 1962, and
the remaining one (1) hectare on installment basis until fully paid.
ISSUE: WON the sale to Mondejar was valid
RULING: YES. The donation made by Trinidad Quijada and her
COMPILED BY: WIGMORE #WIGMOREFOREVER 5

SALES Case Digest (Atty. Sarona)


Compiled by: Wigmore #wigmoreforever
brother and sisters was subject to the condition that the donated
property shall be "used solely and exclusively as a part of the
campus of the proposed Provincial High School in Talacogon. It
further provides that should "the proposed PHS be discontinued
or if the same shall be opened but for some reason or another,
the same may in the future be closed" the donated property shall
automatically revert to the donor.
When the Municipality's acceptance of the donation was made
known to the donor, the former became the new owner of the
donated property donation being a mode of acquiring and
transmitting ownership notwithstanding the condition imposed
by the donee. Accordingly, ownership is immediately transferred
to the donee and that ownership will only revert to the donor if
the resolutory condition is not fulfilled (construction of the
school).
Thus, at the time of the sales made in 1962 towards 1968, the
alleged seller (Trinidad) could not have sold the lots since she
had earlier transferred ownership thereof by virtue of the deed of
donation. So long as the resolutory condition subsists and is
capable of fulfillment, the donation remains effective and the
donee continues to be the owner subject only to the rights of the
donor or his successors-in-interest under the deed of donation.
Since no period was imposed by the donor on when must the
donee comply with the condition, the latter remains the owner so
long as he has tried to comply with the condition within a
reasonable period. Such period, however, became irrelevant
herein when the donee-Municipality manifested through a
resolution that it cannot comply with the condition of building a
school and the same was made known to the donor.
What the donor sold was the land itself which she no longer
owns. It would have been different if the donor-seller sold her
interests over the property under the deed of donation which is
subject to the possibility of reversion of ownership arising from
the non-fulfillment of the resolutory condition. There is one thing
which militates against the claim of Quijadas. Sale, being a
consensual contract, is perfected by mere consent, which is
manifested the moment there is a meeting of the minds as to the
offer and acceptance thereof on three (3) elements: subject
matter, price and terms of payment of the price.
Ownership by the seller on the thing sold at the time of the
perfection of the contract of sale is not an element for its
perfection. What the law requires is that the seller has the right to
transfer ownership at the time the thing sold is delivered.
A perfected contract of sale cannot be challenged on the ground
of non-ownership on the part of the seller at the time of its
perfection; hence, the sale is still valid. Trinidad Quijada's heirs
and successors-in-interest became the owners of the subject
property upon the reversion of the ownership of the land to them.
Consequently, ownership is transferred to respondent Mondejar
and those who claim their right from him.
G.R. No. 137552 June 16, 2000
ROBERTO Z. LAFORTEZA, GONZALO Z. LAFORTEZA,
MICHAEL Z. LAFORTEZA, DENNIS Z. LAFORTEZA, and LEA
Z. LAFORTEZA vs. ALONZO MACHUCA
PARTIES: HEIRS OF FRANCISCO LAFORTEZA SELLER
ALONZO MACHUCA BUYER

SUBJECT: A house and lot located at No. 7757 Sherwood Street,


Marcelo Green Village, Paraaque, Metro Manila worth P630
000.00.
FACTS: In the exercise of the authority of Special Power Of
Attorney, on January 20, 1989, the heirs of the late Francisco Q.
Laforteza represented by Roberto Z. Laforteza and Gonzalo Z.
Laforteza, Jr. entered into a Memorandum of Agreement
(Contract to Sell) with the plaintiff over the subject property for
the sum of SIX HUNDRED THIRTY THOUSAND PESOS
(P630,000.00) payable as follows:
(a) P30,000.00 as earnest money, to be forfeited in favor of the
defendants if the sale is not effected due to the fault of the
plaintiff;
(b) P600,000.00 upon issuance of the new certificate of title in
the name of the late Francisco Q. Laforteza and upon execution
of an extra-judicial settlement of the decedent's estate with sale
in favor of the plaintiff (Par. 2, Exh. "E", record, pp. 335-336).
Significantly, the fourth paragraph of the Memorandum of
Agreement (Contract to Sell) dated January 20, 1989 (Exh. "E",
supra.) contained a provision as follows:
. . . . Upon issuance by the proper Court of the new title, the
BUYER-LESSEE shall be notified in writing and said BUYERLESSEE shall have thirty (30) days to produce the balance of
P600,000.00 which shall be paid to the SELLER-LESSORS upon
the execution of the Extrajudicial Settlement with sale.
On January 20, 1989, plaintiff paid the earnest money of THIRTY
THOUSAND PESOS (P30,000.00), plus rentals for the subject
property .
On September 18, 1998 3, defendant heirs, through their counsel
wrote a letter to the plaintiff furnishing the latter a copy of the
reconstituted title to the subject property, advising him that he
had thirty (3) days to produce the balance of P600,000.00 under
the Memorandum of Agreement which plaintiff received on the
same date.
On October 18, 1989, plaintiff sent the defendant heirs a letter
requesting for an extension of the THIRTY (30) DAYS deadline
up to November 15, 1989 within which to produce the balance of
P600,000.00. Defendant Roberto Z. Laforteza, assisted by his
counsel Atty. Romeo L. Gutierrez, signed his conformity to the
plaintiff's letter request. The extension, however, does not appear
to have been approved by Gonzalo Z. Laforteza, the second
attorney-in-fact as his conformity does not appear to have been
secured.
On November 15, 1989, plaintiff informed the defendant heirs,
through defendant Roberto Z. Laforteza, that he already had the
balance of P600,000.00 covered by United Coconut Planters
Bank Manager's Check dated November 15, 1989 . However, the
defendants, refused to accept the balance .Defendant Roberto Z.
Laforteza had told him that the subject property was no longer for
sale .
On November 20, defendants informed plaintiff that they were
canceling the Memorandum of Agreement (Contract to Sell) in
view of the plaintiff's failure to comply with his contractual
obligations .
COMPILED BY: WIGMORE #WIGMOREFOREVER 6

SALES Case Digest (Atty. Sarona)


Compiled by: Wigmore #wigmoreforever

Thereafter, plaintiff reiterated his request to tender payment of


the balance of P600,000.00. Defendants, however, insisted on
the rescission of the Memorandum of Agreement. Thereafter,
plaintiff filed the instant action for specific performance.
LOWER COURT: The lower court rendered judgment in favor of
the Alonzo Machuca and against the defendant heirs of the late
Francisco Q. Laforteza,.

determinate thing for a price certain is binding upon the


promissor if the promise is supported by a consideration distinct
from the price.
In the present case, the six-month period merely delayed the
demandability of the contract of sale and did not determine its
perfection for after the expiration of the six-month period, there
was an absolute obligation on the part of the petitioners and the
respondent to comply with the terms of the sale.

Petitioners appealed to the Court of Appeals, CA: This affirmed


with the decision of the lower court.
VDA. DE APE VS CA
Hence this petition wherein the petitioners raise the issues:
ISSUES:
(1) Whether or not the MOA is an OPTION CONTRACT,
CONTRACT TO SELL or a CONTRACT OF SALE.
2) WON the six-month period during which the respondent
would be in possession of the property as lessee was a
period within which to exercise an option.
HELD: In the case at bench, there was a perfected agreement
between the petitioners and the respondent whereby the
petitioners obligated themselves to transfer the ownership of and
deliver the house and lot located at 7757 Sherwood St., Marcelo
Green Village, Paraaque and the respondent to pay the price
amounting to six hundred thousand pesos (P600,000.00). All the
elements of a contract of sale were thus present.The
elements of a valid contract of sale under Article 1458 of the Civil
Code are (1) consent or meeting of the minds; (2) determinate
subject matter and (3) price certain money or its equivalent.
Even assuming for the sake of argument that the petitioners
were ready to comply with their obligation (and Machuca cannot),
we find that rescission of the contract will still not prosper. The
rescission of a sale of an immovable property is specifically
governed by Article 1592 of the New Civil Code, which reads:
In the sale of immovable property, even though it may have been
stipulated that upon failure to pay the price at the time agreed
upon the rescission of the contract shall of right take place, the
vendee may pay, even after the expiration of the period, as long
as no demand for rescission of the contract has been made upon
him either judicially or by a notarial act. After the demand, the
court may not grant him a new term.
It is not disputed that the petitioners did not make a judicial or
notarial demand for rescission.
2) WON the six-month period during which the respondent
would be in possession of the property as lessee was a
period within which to exercise an option.
The six-month period, during which the respondent would be in
possession of the property as lessee, was clearly not a period
within which to exercise an option. An option is a contract
granting a privilege to buy or sell within an agreed time and at a
determined price. An option contract is a separate and distinct
contract from that which the parties may enter into upon the
consummation of the option. An option must be supported by
consideration. An option contract is governed by the second
paragraph of Article 1479 of the Civil Code, which reads:
Art. 1479. An accepted unilateral promise to buy or to sell a

FACTS: Cleopas Ape died in 1950 and left a parcel of land (Lot
2319) to his 11 children. The children never formally divided the
property amongst themselves except through hantal-hantal
whereby each just occupied a certain portion and developed
each.
On the other hand, the spouses Lumayno were interested in the
land so they started buying the portion of land that each of the
heirs occupied. On 11 Apr 1973, one of the children, Fortunato,
entered into a contract of sale with Lumayno. In exchange of his
lot, Lumayno agreed to pay P5,000.00. She paid in advance
P30.00. Fortunato was given a receipt prepared by Lumaynos
son in law (Andres Flores). Flores also acted as witness.
Lumayno also executed sales transactions with Fortunatos
siblings separately.
In 1973, Lumayno compelled Fortunato to make the the delivery
to her of the registrable deed of sale over Fortunatos portion of
the Lot No. 2319. Fortunato assailed the validity of the contract
of sale. He also invoked his right to redeem (as a co-owner) the
portions of land sold by his siblings to Lumayno. Fortunato died
during the pendency of the case.
ISSUE: Whether or not there was a valid contract of sale?
HELD: No. Fortunato was a no read no write person. It was
incumbent for the the other party to prove that details of the
contract was fully explained to Fortunato before Fortunato signed
the receipt.
A contract of sale is a consensual contract, thus, it is perfected
by mere consent of the parties. It is born from the moment there
is a meeting of minds upon the thing which is the object of the
sale and upon the price. Upon its perfection, the parties may
reciprocally demand performance, that is, the vendee may
compel the transfer of the ownership and to deliver the object of
the sale while the vendor may demand the vendee to pay the
thing sold. For there to be a perfected contract of sale, however,
the following elements must be present: consent, object, and
price in money or its equivalent.
For consent to be valid, it must meet the following requisites:
(a) it should be intelligent, or with an exact notion of the matter to
which it refers;
(b) it should be free and
(c) it should be spontaneous. Intelligence in consent is vitiated
by error; freedom by violence, intimidation or undue influence;
spontaneity by fraud.
Lumayno claimed that she explained fully the receipt to
Fortunato, but Flores testimony belies it. Flores said there was
COMPILED BY: WIGMORE #WIGMOREFOREVER 7

SALES Case Digest (Atty. Sarona)


Compiled by: Wigmore #wigmoreforever
another witness but the other was a maid who also lacked
education. Further, Flores himself was not aware that the receipt
was to transfer the ownership of Fortunatos land to her mom-inlaw. It merely occurred to him to explain the details of the receipt
but he never did.

VILLANUEVA VS. PNB


G.R. NO. 154493, DECEMBER 6, 2006
FACTS:
The Special Assets Management Department (SAMD) of the
Philippine National Bank (PNB) issued an advertisement for the
sale of certain PNB properties in Calumpang, General Santos
City, including Lots 17 and 19 with advertised floor prices of
P1,409,000.00 and P2,268,000.00 respectively.
Villanueva offered to purchase the lots for P3,677,000.00. He
also manifested that he was depositing P400,000.00 to show
his good faith but with the understanding that said amount may
be treated as part of the payment of the purchase price only
when his offer is accepted by PNB.
At the bottom of said letter there appears an unsigned marginal
note stating that P400,000.00 was deposited into Villanueva's
account with PNB-General Santos Branch.
PNB forwarded the letter of Villanueva to Ramon Guevara, Vice
President, SAMD. Guevara informed Villanueva that only Lot
No. 19 is available and that the asking price therefor is
P2,883,300.00. Guevara further wrote:
If our quoted price is acceptable to you,
please submit a revised offer to purchase. Sale shall
be subject to our Board of Director's approval and to
other terms and conditions imposed by the Bank on
sale of acquired assets.
Instead of submitting a revised offer, Villanueva merely inserted
at the bottom of Guevara's letter a marginal note, which reads:
CONFORME:
PRICE
OF P2,883,300.00
(downpayment
of
P600,000.00 and the balance payable in two (2) years
at quarterly amortizations.)
Villanueva paid P200,000.00 to PNB which the latter issued a
receipt to acknowledge receipt of the "partial payment deposit
on offer to purchase." On the dorsal portion of Official Receipt
No. 16997, Villanueva signed a typewritten note, stating:
This is a deposit made to show the sincerity of my
purchase offer with the understanding that it shall be
returned without interest if my offer is not favorably
considered or be forfeited if my offer is approved but I
fail/refuse to push through the purchase.
Thereafter, however, Guevara wrote Villanueva that SAMD is
deferring negotiations with him over said property and returning
his deposit of P580,000.00.
Undaunted, Villanueva attempted to deliver postdated checks
covering the balance of the purchase price but PNB refused the
same.

Thus Villanueva filed a Complaint for specific performance


which the RTC granted anchoring its judgment on the finding
that there existed a perfected contract of sae between PNB and
Villanueva.
PNB appealed to the CA which reversed and set aside the
decision, stating that in the case at bench, consent, in respect
to the price and manner of its payment, is lacking. The record
shows that appellant, thru Guevara's July 6, 1990 letter, made a
qualified acceptance of appellee's letter-offer dated June 28,
1990 by imposing an asking price of P2,883,300.00 in cash for
Lot 19. The letter dated July 6, 1990 constituted a counter-offer
(Art. 1319, Civil Code), to which appellee made a new proposal,
i.e., to pay the amount of P2,883,300.00 in staggered amounts,
that is, P600,000.00 as downpayment and the balance within
two years in quarterly amortizations.
CA held that a qualified acceptance, or one that involves a new
proposal, constitutes a counter-offer and a rejection of the
original offer (Art. 1319). Consequently, when something is
desired which is not exactly what is proposed in the offer, such
acceptance is not sufficient to generate consent because any
modification or variation from the terms of the offer annuls the
offer.
ISSUE: W/N there was a perfected Contract of Sale between
respondents PNB and herein petitioner Villanueva. NO
HELD:
Contracts of sale are perfected by mutual consent whereby the
seller obligates himself, for a price certain, to deliver and
transfer ownership of a specified thing or right to the buyer over
which the latter agrees. Mutual consent being a state of mind,
its existence may only be inferred from the confluence of two
acts of the parties: an offer certain as to the object of the
contract and its consideration, and an acceptance of the offer
which is absolute in that it refers to the exact object and
consideration embodied in said offer. While it is impossible to
expect the acceptance to echo every nuance of the offer, it is
imperative that it assents to those points in the offer which,
under the operative facts of each contract, are not only material
but motivating as well. Anything short of that level of mutuality
produces not a contract but a mere counter-offer awaiting
acceptance. More particularly on the matter of the consideration
of the contract, the offer and its acceptance must be unanimous
both on the rate of the payment and on its term. An acceptance
of an offer which agrees to the rate but varies the term is
ineffective.
Tracing the transactions and letters between Villanueva and
PNB, it can be said that there was no perfected contract of sale
between the parties. The first letter of PNB stating that only Lot
19 was available was certainly not an acceptance but a mere
counter-offer. Further, such counter-offer imposed two more
conditions that Villanueva submit a revised offer to purchase
based on the new price and that such sae of property be
approved by the Board of Directors. However, Villanuevas reply
to said counter-offer was not an acceptance but a further
counter-offer since he qualified his acceptance proposing a twoyear payment
Moreover, Villanuevas contention that the repudiation was
belated since PNB already agreed to his counter-offer when it
COMPILED BY: WIGMORE #WIGMOREFOREVER 8

SALES Case Digest (Atty. Sarona)


Compiled by: Wigmore #wigmoreforever
accepted his downpayment, the Court ruled that acceptance of
Villanuevas payments did not amount to an implied acceptance
of his last counter-offer. PNB-GenSan Branch had no authority
to bind PNB to a contract of Sale with Villanueva. Neither did
SAMD have authority to bind PNB. Both clearly stated that
whatever is offered will be subject to approval of PNBs higher
authorities.
In sum, the amounts paid by petitioner were not in the nature of
downpayment or earnest money but were mere deposits or
proof of his interest in the purchase of Lot No. 19. Acceptance
of said amounts by respondent does not presuppose perfection
of any contract.

3. Bilateral and Reciprocal


CORTES VS CA
G.R. NO. 126083, JULY 12, 2006
FACTS:
For the purchase price of 3.7M, Villa Esperanza Development
Corporation and Antonio Cortes entered into a contract of sale
over the lots located at Baclaran, Paraaque, Metro Manila. The
Corporation advanced to Cortes the total sum of P1,213,000.00.
Later, in September 1983, the parties executed a deed of
absolute sale on the following terms:
The Corporation shall advance 2.2 M as
downpayment, and Cortes shall likewise deliver the
TCT
for
the
3
lots.
The balance of 1.5M shall be payable within a year
from the date of the execution.
The Corporation filed the instant case for specific performance
seeking to compel Cortes to deliver the TCTs and the original
copy of the Deed of Absolute Sale. According to the
Corporation, despite its readiness and ability to pay the
purchase price, Cortes refused delivery of the sought
documents. Cortes claimed that the owners duplicate copy of
the three TCTs were surrendered to the Corporation and it is the
latter which refused to pay in full the agreed down payment.
RTC rendered a decision rescinding the sale and directed
Cortes to return to the Corporation the amount of
P1,213,000.00, plus interest. CA reversed the decision and
directed Cortes to execute a Deed of Absolute Sale conveying
the properties and to deliver the same to the Corporation
together with the TCTs, simultaneous with the Corporations
payment of the balance of the purchase price of P2,487,000.00.
ISSUE: W/N there is delay in the performance of the parties
obligations that would justify the rescission of the contract
of sale.
HELD:
There is no doubt that the contract of sale in question gave rise
to a reciprocal obligation of the parties. Reciprocal obligations
are those which arise from the same cause, and which each
party is a debtor and a creditor of the other, such that the
obligation of one is dependent upon the obligation of the other.

They are to be performed simultaneously, so that the


performance of one is conditioned upon the simultaneous
fulfillment of the other.
In the present case, the Deed of Sale contained a stipulation
that the Corporation shall pay in full the downpayment upon
execution of the contract. However, based on Cortes
admission, he agreed that the Corporations full payment of the
downpayment would depend upon the delivery of the TCTs of
the three subject lots. As such, the corresponding reciprocal
obligation of the Corporations payment was the transfer of titles
by Cortes. His obligation is not only to affix the signature in the
Deed, but to set into motion the process that would facilitate
transfer of title of the lots.
As correctly found by the CA, Cortes never surrendered said
documents to the Corporation. Cortes avers that he delivered
the TCTs through the brokers son. He further avers that the
brokers son delivered it to the broker, who in turn delivered
them to the Corporation. However, Marcosa Sanchezs
unrebutted testimony is that, she did not receive the TCTs. She
also denied knowledge of delivery thereof to her son, Manny.
What further strengthened the findings of the Court of Appeals
that Cortes did not surrender the subject documents was the
offer of Cortes counsel at the pre-trial to deliver the TCTs and
the Deed of Absolute Sale if the Corporation will pay the
balance of the down payment. Indeed, if the said documents
were already in the hands of the Corporation, there was no
need for Cortes counsel to make such offer.
Considering that their obligation was reciprocal, performance
thereof must be simultaneous. The mutual inaction of Cortes
and the Corporation therefore gave rise to a compensation
morae or default on the part of both parties because neither has
completed their part in their reciprocal obligation. Cortes is yet
to deliver the original copy of the notarized Deed and the TCTs,
while the Corporation is yet to pay in full the agreed down
payment of P2,200,000.00. This mutual delay of the parties
cancels out the effects of default, such that it is as if no one is
guilty of delay.
Additionally, under Article 1169 of the Civil Code, from the
moment one of the parties fulfills his obligation, delay by the
other begins. Since Cortes did not perform his part, the
provision of the contract requiring the Corporation to pay in full
the down payment never acquired obligatory force.
ALMOCERA VS. ONG
546 SCRA 164
G.R. NO. 170479
FEBRUARY 18, 2008
FACTS:
Johnny Ong tried to acquire from Andre T. Almocera and First
Builder Multi-Purpose Cooperative (FBMC) a "townhome" in
Cebu City. As reflected in a Contract to Sell, the selling price of
the unit was P3,400,000.00 pesos.
Out of the purchase price, he was able to pay the amount of
P1,060,000.00.

COMPILED BY: WIGMORE #WIGMOREFOREVER

SALES Case Digest (Atty. Sarona)


Compiled by: Wigmore #wigmoreforever
Prior to the full payment of this amount, Ong claims that
defendants Andre Almocera and First Builders fraudulently
concealed the fact that before and at the time of the perfection of
the aforesaid contract to sell, the property was already
mortgaged to and encumbered with the Land Bank of the
Philippines (LBP). In addition, the construction of the house has
long been delayed and remains unfinished. On March 13, 1999,
Lot 4-a covering the unit was advertised in a local tabloid for
public auction for foreclosure of mortgage. It is the assertion of
Ong that had it not for the fraudulent concealment of the
mortgage and encumbrance by defendants, he would have not
entered into the contract to sell.
On the other hand, defendants assert that on March 20, 1995,
First Builders Multi-purpose Coop. Inc., borrowed money in the
amount of P500,000.00 from Tommy Ong, plaintiffs brother. This
amount was used to finance the documentation requirements of
the LBP for the funding of the Atrium Town Homes. This loan will
be applied in payment of one (1) town house unit which Tommy
Ong may eventually purchase from the project. When the project
was under way, Tommy Ong wanted to buy another townhouse
for his brother, Johnny Ong, plaintiff herein, which then, the
amount of P150,000.00 was given as additional partial payment.
However, the particular unit was not yet identified. It was only on
January 10, 1997 that Tommy Ong identified Unit No. 4 plaintiffs
chosen unit and again tendered P350,000.00 as his third partial
payment. When the contract to sell for Unit 4 was being drafted,
Tommy Ong requested that another contract to sell covering Unit
5 be made so as to give Johnny Ong another option to choose
whichever unit he might decide to have. When the construction
was already in full blast, defendants were informed by Tommy
Ong that their final choice was Unit 5. It was only upon knowing
that the defendants will be selling Unit 4 to some other persons
for P4million that plaintiff changed his choice from Unit 5 to Unit
4.
In trying to recover the amount he paid as down payment for the
townhouse unit, Johnny Ong filed a complaint for Damages
against Andre T. Almocera and FBMC alleging that they were
guilty of fraudulent concealment and breach of contract when
they sold to him a townhouse unit without divulging that the
same, at the time of the perfection of their contract, was already
mortgaged with the Land Bank of the Philippines (LBP), with the
latter causing the foreclosure of the mortgage and the eventual
sale of the townhouse unit to a third person.
In their Answer, Almocera and FBMC denied liability claiming that
the foreclosure of the mortgage on the townhouse unit was
caused by the failure of Johnny Ong to pay the balance of the
price of said townhouse unit.
ISSUES:
I.
WON it was a contract to sell or a contract of sale.
II.
WON the respondents refusal to pay the balance of the
purchase price is justified. -YES
HELD:
I.

It cannot be disputed that the contract entered into


by the parties was a contract to sell. In a contract
to sell, ownership is retained by the seller and is
not to pass to the buyer until full payment of the
price.

The contract was denominated as such and it contained the


provision that the unit shall be conveyed by way of an
Absolute Deed of Sale, together with the attendant
documents of Ownership the Transfer Certificate of Title
and Certificate of Occupancy and that the balance of the
contract price shall be paid upon the completion and delivery
of the unit, as well as the acceptance thereof by respondent.
All these clearly indicate that ownership of the townhouse
has not passed to respondent.
The unit shall be completed and conveyed by way of an
Absolute Deed of Sale together with the attendant
documents of Ownership in the name of the BUYER the
Transfer Certificate of Title and Certificate of Occupancy
within a period of six (6) months from the signing of Contract
to Sell.
II.

The respondent is justified in refusing to pay the


balance of the contract price.

From the terms of the contract, it is clear that petitioner and


FBMC had the obligation to complete the townhouse unit
within six months from the signing of the contract. Upon
compliance therewith, the obligation of respondent to pay
the balance of P2,400,000.00 arises. Upon payment thereof,
the townhouse shall be delivered and conveyed to
respondent upon the execution of the Absolute Deed of Sale
and other relevant documents.
The evidence adduced shows that petitioner and FBMC
failed to fulfill their obligation -- to complete and deliver the
townhouse within the six-month period. With petitioner and
FBMCs non-fulfillment of their obligation, respondent
refused to pay the balance of the contract price. Respondent
does not ask that ownership of the townhouse be transferred
to him, but merely asks that the amount or down payment he
had made be returned to him.
The contract subject of this case contains reciprocal
obligations which were to be fulfilled by the parties, i.e., to
complete and deliver the townhouse within six months from
the execution of the contract to sell on the part of petitioner
and FBMC, and to pay the balance of the contract price
upon completion and delivery of the townhouse on the part
of the respondent.
In the case at bar, the obligation of petitioner and FBMC
which is to complete and deliver the townhouse unit within
the prescribed period, is determinative of the respondents
obligation to pay the balance of the contract price. With their
failure to fulfill their obligation as stipulated in the contract,
they incurred delay and are liable for damages. They cannot
insist that respondent comply with his obligation. Where one
of the parties to a contract did not perform the undertaking to
which he was bound by the terms of the agreement to
perform, he is not entitled to insist upon the performance of
the other party.
Petitioner insists there was no delay when the townhouse
unit was not completed within six months from the signing of
the contract inasmuch as the mere lapse of the stipulated six
(6) month period is not by itself enough to constitute delay
on his part and that of FBMC, since the law requires that
there must either be judicial or extrajudicial demand to fulfill
COMPILED BY: WIGMORE #WIGMOREFOREVER 10

SALES Case Digest (Atty. Sarona)


Compiled by: Wigmore #wigmoreforever
an obligation so that the obligor may be declared in default.
He argues there was no evidence introduced showing that a
prior demand was made by respondent before the original
action was instituted in the trial court.

sale did not occur within a period of one year did not extinguish
the obligation of Fonacier to pay Gaite the balance of P65,000
because it does not seem to be the intention of the parties to the
contract.

We do not agree.

The Court looked into several circumstances which lead them to


conclude that the sale of the iron ore is but a suspensive term.
First, the words of the contract express no contingency in the
buyer's obligation to pay.

Demand is not necessary in the instant case. Demand by


the respondent would be useless because the impossibility
of complying with their (petitioner and FBMC) obligation was
due to their fault. If only they paid their loans with the LBP,
the mortgage on the subject townhouse would not have
been foreclosed and thereafter sold to a third person.

4. Onerous
GAITE VS. FONACIER
2 SCRA 831
G.R. NO. L-11827
JULY 31, 1961
FACTS:
Isabelo Fonacier executed a Deed of Assignment in favor of
Fernando Gaite as his true and lawful attorney-in-fact so that the
latter may enter into a contract for the exploration and
development of the mining claims owned by Fonacier. Gaite
executed a general assignment conveying the development and
exploitation of said mining claims to Larap Iron Mines owned by
him.
Fonacier decided to revoke the Deed of Assignment to which
Gaite assented on the condition that Fonacier is to pay him
P75,000 for the 24,000 metric tons of iron lodes already
extracted and to retain the company name Larap Iron MInes.
Fonacier already paid P10,000 leaving a balance of P65,000
which, as agreed by them, is to be derived from the local sale of
Iron ore made by Larap Iron Mines. On December 8, 1954,
Fonacier issued a security bond to secure payment of balance
with Far Eastern Surety and Insurance Co. but the surety
provided that liability to the company will only attach when there
had been actual sale of iron ore by Larap Iron Mines for an
amount of not less than P65,000 and that the bond will
automatically expire on December 8, 1955.
No sale of the iron ore was made thereafter. Gaite failed to pay
Fonacier the balance and the surety company refused to pay
contending that the bond expired automatically.

Second, in the usual course of business, an onerous contract is


most likely preferred by the parties in a sale.
Nothing is found in the record to evidence that Gaite desired or
assumed to run the risk of losing his right over the ore without
getting paid for it, or that Fonacier understood that Gaite
assumed any such risk. This is proved by the fact that Gaite
insisted on a bond a to guarantee payment of the P65,000.00, an
not only upon a bond by Fonacier, the Larap Mines & Smelting
Co., and the company's stockholders, but also on one by a
surety company; and the fact that appellants did put up such
bonds indicates that they admitted the definite existence of their
obligation to pay the balance of P65,000.00.
Assuming that there could be doubt whether by the wording of
the contract the parties indented a suspensive condition or a
suspensive period (dies ad quem) for the payment of the
P65,000.00, the rules of interpretation would incline the scales in
favor of "the greater reciprocity of interests", since sale is
essentially onerous. The Civil Code of the Philippines, Article
1378, paragraph 1, in fine, provides:
If the contract is onerous, the doubt shall be settled in
favor of the greatest reciprocity of interests.
There can be no question that greater reciprocity obtains if the
buyer' obligation is deemed to be actually existing, with only its
maturity (due date) postponed or deferred, that if such obligation
were viewed as nonexistent or not binding until the ore was sold.
The only rational view that can be taken is that the sale of the ore
to Fonacier was a sale on credit, and not an aleatory contract
where the transferor, Gaite, would assume the risk of not being
paid at all; and that the previous sale or shipment of the ore was
not a suspensive condition for the payment of the balance of the
agreed price, but was intended merely to fix the future date of the
payment.
5. Commutative

Gaite instituted the present case. Fonacier argued that the


payment of the P65,000 balance was subject to the condition
that it would be paid out of the first sale of the iron ore by Larap
Mines which did not happen.
ISSUE:
WON the obligation of Fonacier to pay Gaite the balance of
P65,000 was extinguished because the iron ore was not sold
within a year.
HELD:
The shipment or local sale of the iron ore is not a condition
precedent (or suspensive) to the payment of the balance of
P65,000.00, but was only a suspensive period or term. That the

BUENAVENTURA VS CA (Nov. 20, 2003)


FACTS:
Defendant spouses Leonardo Joaquin and Feliciana Landrito are
the parents of plaintiffs Consolacion, Nora, Emma and Natividad
as well as of defendants Fidel, Tomas, Artemio, Clarita, Felicitas,
Fe, and Gavino, all surnamed JOAQUIN. The married Joaquin
children are joined in this action by their respective spouses.
Sought to be declared null and void ab initio are certain deeds of
sale of real property executed by defendant parents Leonardo
Joaquin and Feliciana Landrito in favor of their co-defendant
children.
COMPILED BY: WIGMORE #WIGMOREFOREVER

11

SALES Case Digest (Atty. Sarona)


Compiled by: Wigmore #wigmoreforever
The petitioners argue that the deeds of sale are simulated as
they are null and void ab initio because:
1.
2.

3.
4.

There was no actual valid consideration for the deeds of


sale over the properties in litis;
Assuming that there was consideration in the sums
reflected in the questioned deeds, the properties are
more than three-fold times more valuable than the
measly sums appearing therein;
The deeds of sale do not reflect and express the true
intent of the parties (vendors and vendees); and
The purported sale of the properties in litis was the
result of a deliberate conspiracy designed to unjustly
deprive the rest of the compulsory heirs of their legitime.

The trial court ruled in favor of the defendants. The Court of


Appeals affirmed the trial courts decision.
ISSUE:
(I) WON petitioners have a legal interest over the properties
subject of the Deeds of Sale
(II) WON the Deeds of Sale are void for lack of consideration
(III) WON the Deeds of Sale are void for gross inadequacy of
price
RULING:
I. No, petitioners do not have a legal interest over the properties
subject of the Deeds of Sale. Petitioners failed to show any legal
right to the properties. In actions for the annulment of contracts,
such as this action, the real parties are those who are parties to
the agreement or are bound either principally or subsidiarily
or are prejudiced in their rights with respect to one of the
contracting parties and can show the detriment which would
positively result to them from the contract even though they did
not intervene in it.
Petitioners do not have any legal interest over the properties
subject of the Deeds of Sale. As the appellate court stated,
petitioners right to their parents properties is merely
inchoate and vests only upon their parents death. While still
living, the parents of petitioners are free to dispose of their
properties.
In their overzealousness to safeguard their future legitime,
petitioners forget that theoretically, the sale of the lots to their
siblings does not affect the value of their parents estate. While
the sale of the lots reduced the estate, cash of equivalent value
replaced the lots taken from the estate.
II. It is not the act of payment of price that determines the validity
of a contract of sale. Payment of the price has nothing to do with
the perfection of the contract. Payment of the price goes into the
performance of the contract. Failure to pay the consideration is
different from lack of consideration. The former results in a right
to demand the fulfillment or cancellation of the obligation under
an existing valid contract while the latter prevents the existence
of a valid contract.
A contract of sale is not a real contract, but a consensual
contract. As a consensual contract, a contract of sale becomes a
binding and valid contract upon the meeting of the minds as to
price. If there is a meeting of the minds of the parties as to the
price, the contract of sale is valid, despite the manner of

payment, or even the breach of that manner of payment. If the


real price is not stated in the contract, then the contract of sale is
valid but subject to reform.
Petitioners failure to prove absolute simulation of price is
magnified by their lack of knowledge of their respondent siblings
financial capacity to buy the questioned lots. On the other hand,
the Deeds of Sale which petitioners presented as evidence
plainly showed the cost of each lot sold. Not only did
respondents minds meet as to the purchase price, but the real
price was also stated in the Deeds of Sale. As of the filing of the
complaint, respondent siblings have also fully paid the price to
their respondent father.
III. Article 1355: Except in cases specified by law, lesion or
inadequacy of cause shall not invalidate a contract, unless
there has been fraud, mistake or undue influence.
Art. 1470. Gross inadequacy of price does not affect a
contract of sale, except as may indicate a defect in the
consent, or that the parties really intended a donation or
some other act or contract.
Petitioners failed to prove any of the instances mentioned in
Articles 1355 and 1470 of the Civil Code which would invalidate,
or even affect, the Deeds of Sale. Indeed, there is no
requirement that the price be equal to the exact value of the
subject matter of sale. All the respondents believed that they
received the commutative value of what they gave.
Courts cannot follow one every step of his life and extricate him
from bad bargains, protect him from unwise investments, relieve
him from one-sided contracts, or annul the effects of foolish acts.
There must be, in addition, a violation of the law, the commission
of what the law knows as an actionable wrong, before the courts
are authorized to lay hold of the situation and remedy it.

6. Sale is Title and Not Mode


SAN LORENZO DEVELOPMENT CORP. VS CA (Jan 21, 2005)
FACTS:
Respondents Miguel Lu and Pacita Zavalla, (hereinafter, the
Spouses Lu) owned two (2) parcels of land situated in Sta. Rosa,
Laguna covered by TCT No. T-39022 and TCT No. T-39023 both
measuring 15,808 square meters or a total of 3.1616 hectares.
According to Pablo Babasanta, Spouses Lu sold the two parcels
of land to him, for the price of P15.00 per square meter.
Babasanta made a downpayment of P50,000.00 as evidenced
by a memorandum receipt issued by Pacita Lu of the same date.
Several other payments totaling P200,000.00 were made by
Babasanta. Babasanta wrote a letter to Pacita Lu to demand the
execution of a final deed of sale in his favor so that he could
effect full payment of the purchase price. Babasanta notified the
spouses about having received information that the spouses sold
the same property to another without his knowledge and
consent.
The Spouses Lu alleged that Pacita Lu obtained loans from
Babasanta and when the total advances of Pacita reached
P50,000.00, the latter and Babasanta, without the knowledge
and consent of Miguel Lu, had verbally agreed to transform the
COMPILED BY: WIGMORE #WIGMOREFOREVER 12

SALES Case Digest (Atty. Sarona)


Compiled by: Wigmore #wigmoreforever
transaction into a contract to sell the two parcels of land to
Babasanta with the P50,000.00 to be considered as the
downpayment for the property and the balance to be paid on or
before 31 December 1987. Respondents Lu added that as of
November 1987, total payments made by Babasanta amounted
to only P 200,000.00 and the latter allegedly failed to pay the
balance of P260,000.00 despite repeated demands. Babasanta
had purportedly asked Pacita for a reduction of the price from
P15.00 to P12.00 per square meter and when the Spouses Lu
refused to grant Babasantas request, the latter rescinded the
contract to sell and declared that the original loan transaction just
be carried out in that the spouses would be indebted to him in
the amount of P200,000.00
SLDC (San Lorenzo Development Corporation) filed a Motion to
Intervene and alleged that it had legal interest in the subject
matter under litigation because the two parcels of land involved
had been sold to it in a Deed of Absolute Sale with Mortgage. It
alleged that it was a buyer in good faith and for value and
therefore it had a better right over the property in litigation.
RTC rendered a decision upholding the sale made to SLDC and
ordered Spouses Lu to pay Babasanta the sum of P200,000. CA
reversed the decision declaring the sale between Babasanta and
Spouses Lu valid and the sale to SLDC null and void on the
ground that it was a purchaser in bad faith.
ISSUE:
1. WON the agreement between Babasanta and Spouses
Lu was a contract to sell and not a contract of sale
2. WON the registration of the sale after the annotation of
the notice of lis pendens obliterate the effects of
delivery and possession in good faith which admittedly
had occurred prior to SLDCs knowledge of the
transaction in favor of Babasanta
RULING:
1.
The agreement between Babasanta and Spouses Lu was a
contract to sell and not a contract of sale.
A document was presented showing that Pacita Lu
acknowledged receipt of P50,000 and that she agreed to sell the
3.6 hectares at P15 per square meter. The receipt signed by
Pacita Lu merely states that she accepted the sum of
P50,000.00 from Babasanta as partial payment of 3.6 hectares
of farm lot situated in Sta. Rosa, Laguna. While there is no
stipulation that the seller reserves the ownership of the property
until full payment of the price which is a distinguishing feature of
a contract to sell, the subsequent acts of the parties convince us
that the Spouses Lu never intended to transfer ownership to
Babasanta except upon full payment of the purchase price.
Babasantas letter dated 22 May 1989 was quite telling. He stated
therein that despite his repeated requests for the execution of the
final deed of sale in his favor so that he could effect full payment
of the price, Pacita Lu allegedly refused to do so. In effect,
Babasanta himself recognized that ownership of the property
would not be transferred to him until such time as he shall have
effected full payment of the price. Moreover, had the sellers
intended to transfer title, they could have easily executed the
document of sale in its required form simultaneously with their
acceptance of the partial payment, but they did not. Doubtlessly,
the receipt signed by Pacita Lu should legally be considered as a

perfected contract to sell.


The distinction between a contract to sell and a contract of sale
is quite germane. In a contract of sale, title passes to the vendee
upon the delivery of the thing sold; whereas in a contract to sell,
by agreement the ownership is reserved in the vendor and is not
to pass until the full payment of the price. In a contract of sale,
the vendor has lost and cannot recover ownership until and
unless the contract is resolved or rescinded; whereas in a
contract to sell, title is retained by the vendor until the full
payment of the price, such payment being a positive suspensive
condition and failure of which is not a breach but an event that
prevents the obligation of the vendor to convey title from
becoming effective.
The perfection of a contract of sale should not be confused with
its consummation. In relation to the acquisition and transfer of
ownership, it should be noted that sale is not a mode, but merely
a title. A mode is the legal means by which dominion or
ownership is created, transferred or destroyed, but title is only
the legal basis by which to affect dominion or ownership. Under
Article 712 of the Civil Code, ownership and other real rights over
property are acquired and transmitted by law, by donation, by
testate and intestate succession, and in consequence of certain
contracts, by tradition. Contracts only constitute titles or rights to
the transfer or acquisition of ownership, while delivery or tradition
is the mode of accomplishing the same. Therefore, sale by itself
does not transfer or affect ownership; the most that sale does is
to create the obligation to transfer ownership. It is tradition or
delivery, as a consequence of sale, that actually transfers
ownership.
Explicitly, the law provides that the ownership of the thing sold is
acquired by the vendee from the moment it is delivered to him in
any of the ways specified in Article 1497 to 1501. The word
delivered should not be taken restrictively to mean transfer of
actual physical possession of the property. The law recognizes
two principal modes of delivery, to wit: (1) actual delivery; and (2)
legal or constructive delivery.
Actual delivery consists in placing the thing sold in the control
and possession of the vendee. Legal or constructive delivery, on
the other hand, may be had through any of the following ways:
the execution of a public instrument evidencing the sale;
symbolical tradition such as the delivery of the keys of the place
where the movable sold is being kept; traditio longa manu or by
mere consent or agreement if the movable sold cannot yet be
transferred to the possession of the buyer at the time of the sale;
traditio brevi manu if the buyer already had possession of the
object even before the sale; and traditio constitutum
possessorium, where the seller remains in possession of the
property in a different capacity.
Respondent Babasanta did not acquire ownership by the mere
execution of the receipt by Pacita Lu acknowledging receipt of
partial payment for the property. For one, the agreement
between Babasanta and the Spouses Lu, though valid, was not
embodied in a public instrument. Hence, no constructive delivery
of the lands could have been effected. For another, Babasanta
had not taken possession of the property at any time after the
perfection of the sale in his favor or exercised acts of dominion
over it despite his assertions that he was the rightful owner of the
COMPILED BY: WIGMORE #WIGMOREFOREVER 13

SALES Case Digest (Atty. Sarona)


Compiled by: Wigmore #wigmoreforever
lands. Simply stated, there was no delivery to Babasanta,
whether actual or constructive, which is essential to transfer
ownership of the property. Thus, even on the assumption that the
perfected contract between the parties was a sale, ownership
could not have passed to Babasanta in the absence of delivery,
since in a contract of sale ownership is transferred to the vendee
only upon the delivery of the thing sold.
2.

No, it did not obliterate the delivery and possession in good


faith.

It must be stressed that as early as 11 February 1989, the


Spouses Lu executed the Option to Buy in favor of SLDC upon
receiving P316,160.00 as option money from SLDC. After SLDC
had paid more than one half of the agreed purchase price of
P1,264,640.00, the Spouses Lu subsequently executed on 3
May 1989 a Deed of Absolute Sale in favor or SLDC. At the time
both deeds were executed, SLDC had no knowledge of the prior
transaction of the Spouses Lu with Babasanta. Simply stated,
from the time of execution of the first deed up to the moment of
transfer and delivery of possession of the lands to SLDC, it had
acted in good faith and the subsequent annotation of lis pendens
has no effect at all on the consummated sale between SLDC and
the Spouses Lu.
Section 52 of the Property Registration Decree (P.D. No. 1529)
which reads, thus: Sec. 52. Constructive notice upon registration.
Every conveyance, mortgage, lease, lien, attachment, order,
judgment, instrument or entry affecting registered land shall, if
registered, filed, or entered in the office of the Register of Deeds
for the province or city where the land to which it relates lies, be
constructive notice to all persons from the time of such
registering, filing, or entering.
However, the constructive notice operates as such by the
express wording of Section 52from the time of the registration of
the notice of lis pendens which in this case was effected only on
2 June 1989, at which time the sale in favor of SLDC had long
been consummated insofar as the obligation of the Spouses Lu
to transfer ownership over the property to SLDC is concerned.
More fundamentally, a notice of lis pendens only serves as a
warning to a prospective purchaser or incumbrancer that the
particular property is in litigation; and that he should keep his
hands off the same, unless he intends to gamble on the results
of the litigation. Precisely, in this case SLDC has intervened in
the pending litigation to protect its rights. Obviously, SLDCs faith
in the merit of its cause has been vindicated with the Courts
present decision which is the ultimate denouement on the
controversy.

NORKIS DISTRIBUTOR VS. CA


G.R. NO. 91029, FEBRUARY 7,1991; 193 SCRA 694
FACTS:
Petitioner Norkis Distributors, Inc. is the distributor of Yamaha
motorcycles in Negros Occidental. On September 20,
1979, private respondent Alberto Nepales bought from the Norkis
Bacolod branch a brand new Yamaha Wonderbike
motorcycle Model YL2DX. The price of P7,500.00 was payable

by means of a Letter of Guaranty from the DBP, which Norkis


agreed to accept. Credit was extended to Nepales for the price of
the motorcycle payable by DBP upon release of his
motorcycle loan. As security for the loan, Nepales would execute
a chattel mortgage on the motorcycle in favor of DBP.
Petitioner issued a sales invoice which Nepales signed in
conformity with the terms of the sale. In the meantime, however,
the motorcycle remained in Norkis possession. On January 22,
1980, the motorcycle was delivered to a certain Julian Nepales,
allegedly the agent of Alberto Nepales. The motorcycle met an
accident on February 3, 1980 at Binalbagan, Negros Occidental.
An investigation conducted by the DBP revealed that the unit
was being driven by a certain Zacarias Payba at the time of the
accident. The unit was a total wreck was returned.
On March 20, 1980, DBP released the proceeds of private
respondents motorcycle loan to Norkis in the total sum
of P7,500. As the price of the motorcycle later increased to
P7,828 in March, 1980, Nepales paid the difference of P328
and demanded the delivery of the motorcycle. When Norkis
could not deliver, he filed an action for specific performance
with damages against Norkis in the RTC of Negros Occidental.
He alleged that Norkis failed to deliver the motorcycle which
he purchased, thereby causing him damages. Norkis answered
that the motorcycle had already been delivered to
private respondent before the accident, hence, the risk of loss or
damage had to be borne by him as owner of the unit.
ISSUE:
Whether or not there has been a transfer of ownership of the
motorcycle to Alberto Nepales.
HELD:
No.The issuance of a sales invoice does not prove transfer of
ownership of the thing sold to the buyer. An invoice is nothing
more than a detailed statement of the nature, quantity and cost
of the thing sold and has been considered not a bill of sale. In all
forms of delivery, it is necessary that the act of delivery whether
constructive or actual, be coupled with the intention of delivering
the thing. The act, without the intention, is insufficient. When the
motorcycle was registered by Norkis in the name of private
respondent, Norkis did not intend yet to transfer the title or
ownership to Nepales, but only to facilitate the execution of a
chattel mortgage in favor of the DBP for the release of the
buyers motorcycle loan. The Letter of Guarantee (Exh. 5) issued
by the DBP, reveals that the execution in its favor of a
chattel mortgage over the purchased vehicle is a pre-requisite for
the approval of the buyer's loan. If Norkis would not accede to
that arrangement, DBP would not approve private respondent's
loan application and, consequently, there would be no sale
Article 1496 of the Civil Code which provides that in the absence
of an express assumption of risk by the buyer, the things sold
remain at sellers risk until the ownership thereof is transferred to
the buyer, is applicable to this case, for there was neither an
actual nor constructive delivery of the thing sold, hence, the risk
of loss should be borne by the seller, Norkis, which was still the
owner and possessor of the motorcycle when it was wrecked.
This is in accordance with the well- known doctrine of res perit
domino.

COMPILED BY: WIGMORE #WIGMOREFOREVER

14

SALES Case Digest (Atty. Sarona)


Compiled by: Wigmore #wigmoreforever
AZNAR V.YAPDIANGCO
13 SCRA 486

thereof by stealing the same while it was in the custody of the


latter's son.

FACTS:
Theodoro Santos advertised in the newspapers the sale of his
Ford Fairlane 500. After the advertisement, a certain de Dios,
claiming to be the nephew of Marella, went to the residence of
Santos and expressing his uncles intent to purchase the car.
Since Santos wasn't around, it was Irineo who talked with de
Dios. On being informed, Santos advised his son to see Marella,
which the son did. Marella expressed his intention to purchase
the car. A deed of sale was prepared and Irineo was instructed
by his father not to part with the deed and the car without
receiving the purchase price from Marella. When irineo and de
Dios arrived at the residence of Marella, the latter averred that
his money was short and had to borrow from his sister. He then
instructed de Dios and Irineo to go the supposed house of the
sister to obtain the money with an unidentified person. He also
asked Irineo to leave the deed to have his lawyer see it. Relying
on the good faith of Marella, Irineo did as requested. Upon
arriving at thehouse of Marellas supposed to be sister, de Dios
and the unidentified person then disappeared together with the
car. This prompted Santos to report the incident to the
authorities.

There is no adequate evidence on record as to whether Irineo


Santos voluntarily delivered the key to the car to the unidentified
person who went with him and L. De Dios to the place on
Azcarraga where a sister of Marella allegedly lived. But even
if Irineo Santos did, it was not the delivery contemplated by
Article 712 of the Civil Code. For then, it would be indisputable
that he turned it over to the unidentified companion only so that
he may drive Irineo Santos and De Dios to the said place on
Azcarraga and not to vest the title to the said vehicle to him as
agent of Vicente Marella. Article 712 above contemplates that the
act be coupled with the intent of delivering the thing.

Thereafter, Marella was able to sell the car to Aznar. And while in
possession of the car, police authorities confiscated the same.
This prompted Aznar to file an action for replevin.
ISSUE: W Aznar has the better title to the car. No
HELD: Vicente Marella did not have any title to the property
under litigation because the same was never delivered to him.
He sought ownership or acquisition of it by virtue of the
contract. Vicente Marella could have acquired ownership or title
to the subject matter thereof only by the delivery or tradition of
the car to him.
Under Article 712 of the Civil Code, "ownership and other real
rights over property are acquired and transmitted by law, by
donation, by testate and intestate succession, and
in consequence of certain contracts, by tradition." As interpreted
by this Court in a host of cases, by this provision, ownership is
not transferred by contract merely but by tradition or delivery.
Contracts only constitute titles or rights to the transfer
or acquisition of ownership, while delivery or tradition is the mode
of accomplishing the same.
For the legal acquisition and transfer of ownership and other
property rights, the thing transferred must be delivered,
inasmuch as, according to settled jurisprudence, the tradition of
the thing is a necessary and indispensable requisite in the
acquisition of said ownership by virtue of contract. So long as
property is not delivered, the ownership over it is not
transferred by contract merely but by delivery. Contracts only
constitute titles or rights to the transfer or acquisition of
ownership, while delivery or tradition is the method of
accomplishing the same, the title and the method of acquiring
it being different in our law. In the case on hand, the car in
question was never delivered to the vendee by the vendor as to
complete or consummate the transfer of ownership by virtue of
the contract. It should be recalled that while there was indeed a
contract of sale between Vicente Marella and Teodoro Santos,
the former, as vendee, took possession of the subject matter

The lower court was correct in applying Article 559 of the Civil
Code to the case at bar, for under it, the rule is to the effect that if
the owner has lost a thing, or if he has been unlawfully deprived
of it, he has a right to recover it, not only from the finder, thief
or robber, but also from third persons who may have acquired it
in good faith from such finder, thief or robber.
The said article establishes 2 exceptions to the general rule of
irrevindicabiltyto wit, the owner has lost the thing or has been
unlawfully deprived thereof. In these cases, the possessor
cannot retain the thing as against the owner who may recover it
without paying any indemnity, except when the possessor
acquired it in a public sale. Furthermore, the common law
principle that where one of two innocent persons must suffer a
fraud perpetrated by another, the law imposes the loss upon the
party who, by his misplaced confidence, has enable the fraud to
be committed, cannot be applied in this case, which is covered
by an express provision of law.

EQUATORIAL REALTY DEVELOPMENT, INC., vs. MAYFAIR


THEATER, INC., G.R. No. 133879 November 21, 2001

FACTS: This case stemmed from a Civil Case entitled "Mayfair"


Theater, Inc. v. Carmelo and Bauermann, Inc., et al.," where
Carmelo & Bauermann, Inc entered into a Contract of Lease with
Mayfair Theater Inc. ("Mayfair") for a period of 20 years. The
lease covered a portion of the second floor and mezzanine of a
two-storey building it owned which respondent used as a movie
house known as Maxim Theater.
Two years later,Mayfair entered into a second Contract of Lease
with Carmelo for the lease of another portion of the latter's
property .In that space, Mayfair put up another movie house
known as Miramar Theater. The Contract of Lease was likewise
for a period of 20 years.
Both leases contained a provision granting Mayfair a right of first
refusal to purchase the subject properties. However, on July 30,
1978 within the 20-year-lease term the subject properties
were sold by Carmelo to Equatorial Realty Development, Inc.
("Equatorial"), without their first being offered to Mayfair.
As a result of the sale of the subject properties to Equatorial,
Mayfair filed a Complaint for (a) the annulment of the Deed of
Absolute Sale between Carmelo and Equatorial, (b) specific
performance, and (c) damages. After trial on the merits, the
COMPILED BY: WIGMORE #WIGMOREFOREVER

15

SALES Case Digest (Atty. Sarona)


Compiled by: Wigmore #wigmoreforever
lower court rendered a Decision in favor of Carmelo and
Equatorial.

the part of the vendor, and the assumption of the same by the
vendee."

The Supreme Court made the following pronouncement for this


case: The Deed of Absolute Sale between petitioners Equatorial
Realty Development, Inc. and Carmelo & Bauermann, Inc.is
deemed rescinded; Carmelo & Bauermann is ordered to return to
petitioner Equatorial Realty Development the purchase price.
The latter is directed to execute the deeds and documents
necessary to return ownership to Carmelo & Bauermann of the
disputed lots. Carmelo & Bauermann is ordered to allow Mayfair
Theater, Inc. to buy the aforesaid lots.

It
is
clear
that
petitioner
never
took actual
control and possession of the property sold, in view of
respondent's timely objection to the sale and the continued
actual possession of the property. The objection took the form of
a court action impugning the sale which, as we know, was
rescinded by a judgment rendered by this Court in the mother
case. It has been held that the execution of a contract of sale as
a form of constructive delivery is a legal fiction. It holds true only
when there is no impediment that may prevent the passing of the
property from the hands of the vendor into those of the
vendee. When there is such impediment, "fiction yields to reality
the delivery has not been effected."

The foregoing Decision of this Court became final and executory.


Subsequently, Mayfair filed a Motion for Execution, which the
trial court granted.
However, Carmelo could no longer be located. Thus, following
the order of execution of the trial court, Mayfair deposited with
the clerk of court. The lower court issued a Deed of
Reconveyance in favor of Carmelo and a Deed of Sale in favor of
Mayfair. On the basis of these documents, the Registry of Deeds
of Manila canceled Equatorial's titles and issued new Certificates
of Title in the name of Mayfair.
Meanwhile, barely five months after Mayfair had submitted its
Motion for Execution before the RTC of Manila, Equatorial filed
action for the collection of a sum of money against Mayfair,
claiming payment of rentals or reasonable compensation for the
defendant's use of the subject premises after its lease contracts
had expired.
ISSUE: Whether or not Equatorial is entitled to back rentals
RULING: No. Equatorial is not entitled to back rentals.
No right of ownership was transferred from Carmelo to Equatorial
in view of a patent failure to deliver the property to the buyer.
Rent is a civil fruit that belongs to the owner of the property
producing it by right of accession. Consequently and ordinarily,
the rentals that fell due from the time of the perfection of the sale
to petitioner until its rescission by final judgment should belong to
the owner of the property during that period.
By a contract of sale, "one of the contracting parties obligates
himself to transfer ownership of and to deliver a determinate
thing and the other to pay therefor a price certain in money or its
equivalent."
Ownership of the thing sold is a real right, which the buyer
acquires only upon delivery of the thing to him "in any of the
ways specified in articles 1497 to 1501, or in any other manner
signifying an agreement that the possession is transferred from
the vendor to the vendee." This right is transferred, not merely by
contract, but also by tradition or delivery. And there is said to be
delivery if and when the thing sold "is placed in the control and
possession of the vendee." Thus, it has been held that while the
execution of a public instrument of sale is recognized by law as
equivalent to the delivery of the thing sold, such constructive or
symbolic delivery, being merely presumptive, is deemed negated
by the failure of the vendee to take actual possession of the land
sold.
In the Law on Sales, delivery may be either actual or
constructive, but both forms of delivery contemplate "the
absolute giving up of the control and custody of the property on

Hence, respondent's opposition to the transfer of the property by


way of sale to Equatorial was a legally sufficient impediment that
effectively prevented the passing of the property into the latter's
hands.
The execution of a public instrument gives rise, therefore, only to
a prima facie presumption of delivery. Such presumption is
destroyed when the instrument itself expresses or implies that
delivery was not intended; or when by other means it is shown
that such delivery was not effected, because a third person was
actually in possession of the thing. In the latter case, the sale
cannot be considered consummated.
Ownership is acquired, not by mere agreement, but by tradition
or delivery. Under the factual environment of this controversy as
found by this Court in the mother case, Equatorial was never put
in actual and effective control or possession of the property
because of Mayfair's timely objection.
In short, the sale to Equatorial may have been valid from
inception, but it was judicially rescinded before it could be
consummated. Petitioner never acquired ownership, not because
the sale was void, as erroneously claimed by the trial court, but
because the sale was not consummated by a legally
effective delivery of the property sold.

III. DISTINGUISHED FROM OTHER


TRANSACTIONS/CONTRACT

1.

Distinguished from Barter

2.

Distinguished from Donation

3.

Distinguished from Contract for Piece of Work


INCHAUSTI AND CO. vs. ELLIS CROMWELL, Collector of
Internal Revenue
G.R. No. L-6584

October 16, 1911

FACTS: Inchausti is engaged in the business of buying and


selling at wholesale hemp. It is customary to sell hemp in bales.
The operation of bailing hemp is designated among merchants
by the word "prensaje."
In all sales of hemp by the plaintiff firm, the price is quoted to the
buyer at so much per picul, no mention being made of bailing;
but with the tacit understanding, unless otherwise expressly
COMPILED BY: WIGMORE #WIGMOREFOREVER

16

SALES Case Digest (Atty. Sarona)


Compiled by: Wigmore #wigmoreforever
agreed, that the hemp will be delivered in bales and that,
according to the custom prevailing among hemp merchants and
dealers in the Philippine Islands, a charge, is to be made against
the buyer under the denomination of "prensaje." This charge is
made in the same manner in all cases, even when the operation
of bailing was performed by the plaintiff or by its principal long
before the contract of sale was made.
Plaintiff Inchausti has always paid to the defendant Collector of
Internal Revenue or to his predecessor in the office of the
Collector of Internal Revenue the tax collectible upon the selling
price expressly agreed upon for all hemp sold by the plaintiff firm,
but has not, until compelled to do so paid the said tax upon sums
received from the purchaser of such hemp under the
denomination of "prensaje."
Subsequently, the defendant, acting in his official capacity as
Collector of Internal Revenue of the Philippine Islands, made
demand in writing upon the plaintiff firm for the payment as tax
on the sums of money collected from purchasers of hemp under
the denomination of "prensaje."
The plaintiff firm paid to the defendant under protest that the tax
for the collected money under the denomination of "prensaje" is
illegal upon the ground that the said charge does not constitute a
part of the selling price of the hemp, but is a charge made for the
service of baling the hemp.
It is the contention of the defendant that the said charge made
under the denomination of "prensaje" is in truth and in fact a part
of the gross value of the hemp sold and of its actual selling price.
ISSUE: Whether or not the amount collected under the
denomination pensaje is part of the selling price of the hemp.
RULING: Yes. It is considered part of the selling price and the
tax was properly imposed.
The distinction between a contract of sale and one for work,
labor, and materials is tested by the inquiry whether the thing
transferred is one no in existence and which never would have
existed but for the order of the party desiring to acquire it, or a
thing which would have existed and been the subject of sale to
some other person, even if the order had not been given. It is
clear that in the case at bar the hemp was in existence in baled
form before the agreements of sale were made, or, at least,
would have been in existence even if none of the individual sales
here in question had been consummated. It would have been
baled, nevertheless, for sale to someone else, since, according
to the agreed statement of facts, it is customary to sell hemp in
bales. When a person stipulates for the future sale of articles
which he is habitually making, and which at the time are not
made or finished, it is essentially a contract of sale and not a
contract for labor. It is otherwise when the article is made
pursuant to agreement. Where labor is employed on the
materials of the seller he cannot maintain an action for work and
labor. If the article ordered by the purchaser is exactly such as
the plaintiff makes and keeps on hand for sale to anyone, and no
change or modification of it is made at the defendant's request, it
is a contract of sale, even though it may be entirely made after,
and in consequence of, the defendant's order for it.
A contract to make is a contract of sale if the article ordered is
already substantially in existence at the time of the order and
merely requires some alteration, modification, or adoption to the
buyer's wishes or purposes. It is also held in that state that a
contract for the sale of an article which the vendor in the ordinary

course of his business manufactures or procures for the general


market, whether the same is on hand at the time or not, is a
contract for the sale of goods to which the statute of frauds
applies. But if the goods are to be manufactured especially for
the purchaser and upon his special order, and not for the general
market, the case is not within the statute.
It is clear to our minds that in the case at bar the baling was
performed for the general market and was not something done
by plaintiff which was a result of any peculiar wording of the
particular contract between him and his vendee. It is undoubted
that the plaintiff prepared his hemp for the general market.

CELESTINO CO VS CIR (G.R. NO. L-8506)


FACTS: Celestino Co & Company is a duly registered general
co-partnership doing business under the trade name of Oriental
Sash Factory. From 1946 to 1951 it paid percentage taxes of
7% on the gross receipts of its sash, door and window factory, in
accordance with sec. 186 of the National Internal Revenue Code
which is a tax on the original sales of articles by manufacturer,
producer or importer.
However, in 1952 it began to claim only 3% tax under Sec. 191,
which is a tax on sales of services. Petitioner claims that it does
not manufacture ready-made doors, sash and windows for the
public, but only upon special orders from the customers, hence, it
is not engaged in manufacturing under sec 186, but only in sales
of services covered by sec 191.
Having failed to convince BIR, petitioner went to the Court of Tax
Appeal where it also failed. CTA, in its decision, holds that the
petitioner has chosen for its tradename and has offered itself to
the public as a Factory, which means it is out to do business, in
its chosen lines on a big scale.
As a general rule, sash factories receive orders for doors and
windows of special design only in particular cases but the bulk of
their sales is derived from a ready-made doors and windows of
standard sizes for the average home.. Even if we were to believe
petitioners claim that it does not manufacture ready-made sash,
doors and windows for the public and that it makes these articles
only special order of its customers that does not make it a
contractor within the purview of section 191 of the national
Internal Revenue Code. There are no less than fifty occupations
enumerated in the aforesaid section and after reading carefully
each and every one of them, we cannot find under which the
business of manufacturing sash, doors and windows upon
special order of customers fall under the category mentioned
under Sec 191.
ISSUE: Whether the petitioner company provides
services or is engaged in manufacturing.

special

HELD: The important thing to remember is that Celestino Co &


Company habitually makes sash, windows and doors, as it has
represented in its stationery and advertisements to the public.
That it manufactures the same is practically admitted by
appellant itself. The fact that windows and doors are made by it
only when customers place their orders, does not alter the nature
of the establishment, for it is obvious that it only accepted such
orders as called for the employment of such material-moulding,
COMPILED BY: WIGMORE #WIGMOREFOREVER

17

SALES Case Digest (Atty. Sarona)


Compiled by: Wigmore #wigmoreforever
frames, panels-as it ordinarily manufactured or was in a position
habitually to manufacture. The Oriental Sash Factory does
nothing more than sell the goods that it mass-produces or
habitually makes; sash, panels, mouldings, frames, cutting them
to such sizes and combining them in such forms as its customers
may desire.
Appellant invokes Article 1467 of the New Civil Code to bolster
its contention that in filing orders for windows and doors
according to specifications, it did not sell, but merely contracted
for particular pieces of work or merely sold its services.
Said article reads as follows:
Article 1467. A contract for the delivery at a certain price of an
article which the vendor in the ordinary course of his business
manufactures or procures for the general market, whether the
same is on hand at the time or not, is a contract of sale, but if the
goods are to be manufactured specially for the customer and
upon his special order, and not for the general market, it is
contract for a piece of work.
In our opinion when this Factory accepts a job that requires the
use of extraordinary or additional equipment, or involves services
not generally performed by it-it thereby contracts for a piece of
work filing special orders within the meaning of Article 1467.
The orders herein exhibited were not shown to be special. They
were merely orders for work nothing is shown to call them
special requiring extraordinary service of the factory. The thought
occurs to us that if, as alleged-all the work of appellant is only to
fill orders previously made, such orders should not be called
special work, but regular work. The Supreme Court affirms the
assailed decision by the CTA.
CIR VS ARNOLDUS CARPENTRY SHOP
GR NO. 71122
FACTS: Arnoldus Carpentry Shop, Inc. is a domestic corporation
which has been in existence since 1960 which has for its
purpose the preparing, processing, buying, selling, exporting,
importing, manufacturing, trading and dealing in cabinet shop
products, wood and metal home and office furniture, cabinets,
doors, windows, etc., including their component parts and
materials, of any and all nature and description. The company
kept samples or models of its woodwork on display from where
its customers may refer to when placing their orders.
On March 1979, the examiners from BIR who conducted an
investigation on the companys tax liabilities reported that subject
corporation should be considered a contractor and not a
manufacturer since the corporation renders service in the course
of an independent occupation representing the will of his
employer only as to the result of his work, and not as to the
means by which it is accomplished. Hence, in the computation of
the percentage tax, the 3% contractors tax should be imposed
instead of the 7% manufacturers tax. However, responded
company holds that the carpentry shop is a manufacturer and
therefore entitled to tax exemption on its gross export sales
under Section 202 (e) of the National Internal Revenue Code.
CIR rendered its decision classifying the respondent as
contractor which was in turn reversed by the CTA.
ISSUE: Whether or not the Court of Tax Appeals erred in holding
that private respondent is a manufacturer and not a contractor.

HELD: No. The Supreme Court holds that the private respondent
is a manufacturer as defined in the Tax Code and not a
contractor under Section 205(e) of the Tax Code.
Petitioner CIR wants to impress upon this Court that under Article
1467, the true test of whether or not the contract is a piece of
work (and thus classifying private respondent as a contractor) or
a contract of sale (which would classify private respondent as a
manufacturer) is the mere existence of the product at the time of
the perfection of the contract such that if the thing already exists,
the contract is of sale, if not, it is work. This is not the test
followed in this jurisdiction. Based on Art. 1467, what determines
whether the contract is one of work or of sale is whether the thing
has been manufactured specially for the customer and upon his
special order. Thus, if the thing is specially done at the order of
another, this is a contract for a piece of work. If, on the other
hand, the thing is manufactured or procured for the general
market in the ordinary course of ones business, it is a contract of
sale.
The distinction between a contract of sale and one for work,
labor and materials is tested by the inquiry whether the thing
transferred is one not in existence and which never would have
existed but for the order of the party desiring to acquire it, or a
thing which would have existed and has been the subject of sale
to some other persons even if the order had not been given. The
one who has ready for the sale to the general public finished
furniture is a manufacturer, and the mere fact that he did not
have on hand a particular piece or pieces of furniture ordered
does not make him a contractor only.
A contract for the delivery at a certain price of an article which
the vendor in the ordinary course of his business manufactures
or procures for the general market, whether the same is on
hand at the time or not, is a contract of sale, but if the goods are
to be manufactured specially for the customer and upon his
special order, and not for the general market, it is a contract for a
piece of work. The facts show that the company had a ready
stock of its shop products for sale to its foreign and local buyers.
As a matter of fact, the purchase orders from its foreign buyers
showed that they ordered by referring to the models designated
by petitioner. Even purchases by local buyers for television
cabinets were by orders for existing models except only for some
adjustments in sizes and accessories utilized.
The Court finds itself in agreement with CTA and as the CTA did
not err in holding that private respondent is a manufacturer,
then private respondent is entitled to the tax exemption under
See. 202 (d) and (e) now Sec. 167 (d) and (e)] of the Tax Code.
ENGINEERING & MACHINERY CORPORATION, petitioner,
vs. COURT OF APPEALS and PONCIANO L.
ALMEDA, respondents.
FACTS:
Almeda and Engineering signed a contract, wherein Engineering
undertook to fabricate, furnish and install the air-conditioning
system in the latters building along Buendia Avenue, Makati in
consideration of P210,000.00. Petitioner was to furnish the
materials, labor, tools and all services required in order to so
fabricate and install said system. The system was completed in
1963 and accepted by private respondent, who paid in full the
COMPILED BY: WIGMORE #WIGMOREFOREVER 18

SALES Case Digest (Atty. Sarona)


Compiled by: Wigmore #wigmoreforever
contract price.
Almeda learned from the employees of NIDC of the defects of
the air-conditioning system of the building. Almeda spent for the
repair of the air-conditioning system. He now sues Engineering
for the refund of the repair. Engineering contends that the
contract was of sale and the claim is barred by prescription since
the responsibility of a vendor for any hidden faults or defects in
the thing sold runs only for 6 months (Arts 1566, 1567, 1571).
Almeda contends that since it was a contract for a piece of work,
hence the prescription period was ten years (Hence Art 1144
should apply on written contracts).
RTC found that Engineering failed to install certain parts and
accessories called for by the contract, and deviated from the
plans of the system, thus reducing its operational effectiveness
to achieve a fairly desirable room temperature.
ISSUE:
Whether the contract for the fabrication and installation of a
central air-conditioning system in a building, one of sale or for
a piece of work?
RULING:
CONTRACT
FOR
PIECE
OF
WORK
A contract for a piece of work, labor and materials may be
distinguished from a contract of sale by the inquiry as to whether
the thing transferred is one not in existence and which would
never have existed but for the order, of the person desiring it. In
such case, the contract is one for a piece of work, not a sale. On
the other hand, if the thing subject of the contract would have
existed and been the subject of a sale to some other person
even if the order had not been given, then the contract is one of
sale.
A contract for the delivery at a certain price of an article which
the vendor in the ordinary course of his business manufactures
or procures for the general market, whether the same is on hand
at the time or not is a contract of sale, but if the goods are to be
manufactured specially for the customer and upon his special
order, and not for the general market, it is a contract for a piece
of work .
The contract in question is one for a piece of work. It is not
petitioners line of business to manufacture air-conditioning
systems to be sold off-the-shelf. Its business and particular field
of expertise is the fabrication and installation of such systems as
ordered by customers and in accordance with the particular
plans and specifications provided by the customers. Naturally,
the price or compensation for the system manufactured and
installed will depend greatly on the particular plans and
specifications agreed upon with the customers.
2)The original complaint is one for damages arising from breach
of a written contract and not a suit to enforce warranties
against hidden defects we here with declare that the
governing law is Article 1715 (supra). However, inasmuch as this
provision does not contain a specific prescriptive period, the
general law on prescription, which is Article 1144 of the Civil
Code, will apply. Said provision states, inter alia, that actions
upon a written contract prescribe in ten (10) years. Since the
governing contract was executed on September 10, 1962 and

the complaint was filed on May 8, 1971, it is clear that the action
has not prescribed.
INOCENCIA YU DINO and her HUSBAND doing business
under the trade name "CANDY CLAIRE FASHION
GARMENTS", petitioners, vs. COURT OF APPEALS and
ROMAN SIO, doing business under the name "UNIVERSAL
TOY MASTER MANUFACTURING", respondents.
FACTS:
Petitioners spouses Dino, doing business under the trade name
"Candy Claire Fashion Garment" are engaged in the business of
manufacturing and selling shirts. Respondent Sio is part owner
and general manager of a manufacturing corporation doing
business under the trade name "Universal Toy Master
Manufacturing." Petitioners and respondent Sio entered into a
contract whereby the latter would manufacture for the petitioners
20,000 pieces of vinyl frogs and 20,000 pieces of vinyl
mooseheads with the sample approved by the petitioners.
Respondent Sio delivered the 40,000 pieces of frogs and
mooseheads. Subsequently, petitioners returned to respondent
29,772 pieces of frogs and mooseheads for failing to comply with
the approved sample. Petitioners then demanded from the
respondent a refund of the purchase price of the returned goods
in the amount of P208,404.00. As respondent Sio refused to pay,
petitioners filed on July 24, 1989 an action for collection of a sum
of money in the Regional Trial Court of Manila, Branch 38.The
trial court ruled in favor of the petitioners. Respondent Sio sought
recourse in the Court of Appeals. On January 24, 1994, the
respondent court reversed its decision and dismissed petitioners'
Complaint for having been filed beyond the prescriptive period.
ISSUE:
Whether or not the contract between the petitioner and the
respondent was a contract for a piece of work.
RULING:
CONTRACT OF PIECE OF WORK
Art. 1467. A contract for the delivery at a certain price of an
article which the vendor in the ordinary course of his business
manufactures or procures for the general market, whether the
same is on hand at the time or not, is a contract of sale, but if the
goods are to be manufactured specially for the customer and
upon his special order, and not for the general market, it is a
contract for a piece of work.
"Art. 1713. By the contract for a piece of work the contractor
binds himself to execute a piece of work for the employer, in
consideration of a certain price or compensation. The contractor
may either employ only his labor or skill, or also furnish the
material."
It was stipulated in the contract that respondent would
manufacture upon order of the petitioners 20,000 pieces of vinyl
frogs and 20,000 pieces of vinyl mooseheads according to the
samples specified and approved by the petitioners. Respondent
Sio did not ordinarily manufacture these products, but only upon
order of the petitioners and at the price agreed upon. Clearly, the
contract executed by and between the petitioners and the
respondent was a contract for a piece of work. At any rate,
whether the agreement between the parties was one of a
contract of sale or a piece of work, the provisions on warranty of
COMPILED BY: WIGMORE #WIGMOREFOREVER 19

SALES Case Digest (Atty. Sarona)


Compiled by: Wigmore #wigmoreforever
title against hidden defects in a contract of sale apply to the case
at bar. Article 1567 provides for the remedies available to the
vendee in case of hidden defects:
"Art. 1567. In the cases of Articles 1561, 1562, 1564, 1565 and
1566, the vendee may elect between withdrawing from the
contract and demanding a proportionate reduction of the price,
with damages in either case."
By returning the 29,772 pieces of vinyl products to respondent
and asking for a return of their purchase price, petitioners were in
effect "withdrawing from the contract" as provided in Art.
1567. The prescriptive period for this kind of action is provided in
Art. 1571 of the New Civil Code which provides that the action
shall be barred after six months from the delivery of the thing
sold. Respondent made the last delivery of the vinyl products to
petitioners on September 28,1988 and the action to recover
the purchase price of the goods was filed on July 24, 1989, more
than nine months from the date of last delivery. Clearly,
Petitioners are barred from claiming a sum of money
from respondent.

sponsorship for unfunded IPC research projects from


international
organizations,
private
foundations
and
governmental agencies. However, such sponsorships are subject
to private respondents terms and conditions, among which are,
that the research is confined to topics consistent with the private
respondents academic agenda; that no proprietary or
commercial purpose research is done; and that private
respondent retains not only the absolute right to publish but also
the ownership of the results of the research conducted by the
IPC.
4.

Distinguished from Agency to Sell

ANDRES QUIROGA vs PARSONS HARDWARE CO.,


G.R. L-11491 August 23, 1918
FACTS: On January 24, 1911, a contract was entered into by
and between Andres Quiroga and J. Parsons, to whose rights
and obligations the present defendant later subrogated itself. The
contract stipulated that Don Andres Quiroga grants the exclusive
right to sell his beds in the Visayan Islands to J. Parsons.

CIR VS CA AND ATENEO (GR NO 115349 APRIL 18, 1997)


FACTS: ADMU Institute of Philippine Culture is engaged in
social science studies of Philippine society and culture.
Occasionally, it accepts sponsorships for its research activities
from international organizations, private foundations and
government agencies.
On July 1983, CIR sent a demand letter assessing the sum of
P174,043.97 for alleged deficiency contractors tax. Accdg to
CIR, ADMU falls under the purview of independent contractor
pursuant to Sec 205 of Tax Code and is also subject to 3%
contractors tax under Sec 205 of the same code. (Independent
Contractor means any person whose activity consists essentially
of the sale of all kinds of services for a fee regardless of whether
or not the performance of the service calls for the exercise or use
of the physical or mental faculties of such contractors or their
employees.)
ISSUE: WON the acceptance of research projects by the IPC of
ADMU a contract of sale or a contract for a piece of work?
NEITHER. Transactions of Ateneos Institute of Philippine Culture
cannot be deemed either as a contract of sale or a contract of a
piece of work.
HELD: Records do not show that Ateneos IPC in fact contracted
to sell its research services for a fee. In the first place, the
petitioner has presented no evidence to prove its bare contention
that, indeed, contracts for sale of services were ever entered into
by the private respondent. Funds received by the Ateneo de
Manila University are technically not a fee. They may however
fall as gifts or donations which are tax-exempt. Another fact that
supports this contention is that for about 30 years, IPC had
continuously operated at a loss, which means that sponsored
funds are less than actual expenses for its research projects.
In fact, private respondent is mandated by law to undertake
research activities to maintain its university status. In fact, the
research activities being carried out by the IPC is focused not on
business or profit but on social sciences studies of Philippine
society and culture. Since it can only finance a limited number of
IPCs research projects, private respondent occasionally accepts

Quiroga files a case against Parsons for allegedly violating the


following stipulations: not to sell the beds at higher prices than
those of the invoices; to have an open establishment in Iloilo;
itself to conduct the agency; to keep the beds on public
exhibition, and to pay for the advertisement expenses for the
same; and to order the beds by the dozen and in no other
manner. With the exception of the obligation on the part of the
defendant to order the beds by the dozen and in no other
manner, none of the obligations imputed to the defendant in the
two causes of action are expressly set forth in the contract. But
the plaintiff alleged that the defendant was his agent for the sale
of his beds in Iloilo, and that said obligations are implied in a
contract of commercial agency. The whole question, therefore,
reduced itself to a determination as to whether the defendant,
byreason of the contract hereinbefore transcribed, was a
purchaser or an agent of the plaintiff for the sale of his beds.
ISSUE: Whether the contract is a contract of agency or of sale.
RULING:
In order to classify a contract, due regard must be given to its
essential clauses. In the contract in question, what was essential,
as constituting its cause and subject matter, is that the plaintiff
was to furnish the defendant with the beds which the latter might
order, at the price stipulated, and that the defendant was to pay
the price in the manner stipulated. The price agreed upon was
the one determined by the plaintiff for the sale of these beds in
Manila, with a discount of from 20 to 25 per cent, according to
their class. Payment was to be made at the end of sixty days, or
before, at the plaintiff's request, or in cash, if the defendant so
preferred, and in these last two cases an additional discount was
to be allowed for prompt payment. These are precisely the
essential features of a contract of purchase and sale. There was
the obligation on the part of the plaintiff to supply the beds, and,
on the part of the defendant, to pay their price. These features
exclude the legal conception of an agency or order to sell
whereby the mandatory or agent received the thing to sell it,
and does not pay its price, but delivers to the principal the
price he obtains from the sale of the thing to a third person,
COMPILED BY: WIGMORE #WIGMOREFOREVER

20

SALES Case Digest (Atty. Sarona)


Compiled by: Wigmore #wigmoreforever
and if he does not succeed in selling it, he returns it. By
virtue of the contract between the plaintiff and the defendant, the
latter, on receiving the beds, was necessarily obliged to pay their
price within the term fixed, without any other consideration and
regardless as to whether he had or had not sold the beds.
It would be enough to hold, as we do, that the contract by and
between the defendant and the plaintiff is one of purchase and
sale, in order to show that it was not one made on the basis of a
commission on sales, as the plaintiff claims it was, for these
contracts are incompatible with each other. But, besides,
examining the clauses of this contract, none of them is found that
substantially supports the plaintiff's contention. Not a single one
of these clauses necessarily conveys the idea of an agency. The
words commission on sales used in clause (A) of article 1 mean
nothing else, as stated in the contract itself, than a mere discount
on the invoice price. The word agency, also used in articles 2 and
3, only expresses that the defendant was the only one that could
sell the plaintiff's beds in the Visayan Islands. With regard to the
remaining clauses, the least that can be said is that they are not
incompatible with the contract of purchase and sale.

PUYAT & SONS, INC. V. ARCO AMUSEMENT CO.


FACTS
Arco Amusement was engaged in the business of operating
cinematographs. Gonzalo Puyat & Sons Inc. (GPS) was the
exclusive agent in the Philippines for the Starr
Piano
Company. Desiring to equip its cinematograph with sound
reproducing devices, Arco approached GPS, through its
president, GIl Puyat, and an employee named Santos. After
some negotiations, it was agreed between the parties that
GPS would order sound reproducing equipment from Starr
Piano Company and that Arco would pay GPS, in
addition to the price of the equipment, a 10% commission, plus
all expenses such as freight, insurance, etc. When GPS inquired
Starr Piano the price (without discount) of the equipment, the
latter quoted such at $1,700 FOB Indiana. Being agreeable to
the price (plus 10%commission plus all other expenses),
Arco formally authorized the order. The following year,
both parties agreed for another order of sound reproducing
equipment on the same terms as the first at $1,600 plus 10%
plus all other expenses. Three years later, Arco discovered
that the prices quoted to them by GPS with regard to their first 2
orders mentioned were not the net prices, but rather the list
price, and that it had obtained a discount from Starr Piano.
Moreover, Arco alleged that the equipment were overpriced.
Thus, being its agent, GPS had to reimburse the excess
amount it received from Arco.
ISSUE
W/N there was a contract of agency, not of sale
HELD
NO. The letters containing Arco's acceptance of the prices
for the equipment are clear in their terms and admit no
other interpretation
that
the
prices
are
fixed
and
determinate. While the letters state that GPS was to receive a
10% commission, this does not necessarily mean that it is
an agent of Arco, as this provision is only an
additional price which it bound itself to pay,
and
which
stipulation is not incompatible with the contract of sale. It is GPS

that is the exclusive agent of Starr Piano in the Philippines, not


the agent of Arco. It is out of the ordinary for one to be the agent
of both the seller and the buyer. The facts and
circumstances show that Arco entered into a contract of sale
with GPS, the exclusive agent of Starr Piano. As such, it is not
duty bound to reveal the private arrangement it had with
Starr Piano relative to the 25% discount. Thus, GPS is not bound
to reimburse Arco for any difference between the cost price
and the sales price, which represents the profit realized by
GPS out of the transaction.
KER & CO, LTD. LINGAD
FACTS
Ker and Co, Ltd. was assessed by then Commissioner of Internal
Revenue Domingo the sum of P 20,272.33 as the commercial
brokers percentage tax surcharge and compromise penalty.
There was a request on the part of Ker for the cancellation of
such assessment which request was turned down. As a result, it
filed a petition for review with the Court of Tax Appeals. The CTA
held that Ker is taxable except as to the compromise penalty of P
500, the amount due from it being fixed at P 19, 772.33.
Such liability arose from a contract of Ker with the United States
Rubber International (USRI). The former being referred to as the
distributor and the latter specifically designated as the company.
The contract was to apply to transactions between the former
and Ker, as distributor from July 1, 1948 to continue in force until
terminated by either party giving to the other 60 days notice. The
shipments would cover products for consumption in Cebu, Bohol,
Leyte, Samar, Jolo, Negros Oriental and Mindanao except the
province of Davao, Ker as distributor being precluded from
disposing such products elsewhere than in the above places
unless written consent be obtained from the company. Ker as
distributor is required to exert every effort to have the shipment
of the products in the maximum quantity and to promote in every
way the sale thereof.
(Crucial stipulation: The company shall form time to time consign
to Ker and Ker will receive, accept and/or hold upon consignment
the products specified under the terms of this agreement in such
quantities as in the judgment of company may be necessary.
It is further agreed that this agreement does not constitute Ker
the agent or legal representative of the company for any purpose
whatsoever.)
ISSUE
W/N the contract between Ker and the USRI is a contract of sale.
HELD
No. By taking the contractual stipulations as a whole and not just
the disclaimer, it would seem that the contract between them is a
contract of agency. That the petitioner Ker & Co., Ltd. is, by
contractual stipulation, an agent of U.S. Rubber International is
borne out by the facts that:
1. petitioner can dispose of the products of the Company only to
certain persons or entities and within stipulated limits, unless
excepted by the contract or by the Rubber Company;
2. it merely receives, accepts and/or holds upon consignment the
products, which remain properties of the latter company;
COMPILED BY: WIGMORE #WIGMOREFOREVER 21

SALES Case Digest (Atty. Sarona)


Compiled by: Wigmore #wigmoreforever

3. every effort shall be made by petitioner to promote in every


way the sale of the products (Par. 3); that sales made by
petitioner are subject to approval by the company;
4. on dates determined by the rubber company, petitioner shall
render a detailed report showing sales during the month;
5. the rubber company shall invoice the sales as of the dates of
inventory and sales report (Par. 14); that the rubber company
agrees to keep the consigned goods fully insured under
insurance policies payable to it in case of loss;
6. upon request of the rubber company at any time, petitioner
shall render an inventory of the existing stock which may be
checked by an authorized representative of the former
7. upon termination or cancellation of the Agreement, all goods
held on consignment shall be held by petitioner for the account of
the rubber company until their disposition is provided for by the
latter
The National Internal Revenue Code defined Commercial
broker as all persons, other than importer, manufacturers,
producers or bona fide employees who, for compensation or
profit, sell or bring about sales or purchase of merchandise for
other persons or being proposed buyers and sellers together
and also includes commission merchants such as Ker in this
case.
The mere disclaimer in a contract that an entity like Ker is not
the agent or legal representative for any purpose whatsoever
does not suffice to yield the conclusion that it is an independent
merchant if the control over the goods for resale of goods
consigned is pervasive in character.
SCHMID & OBERLY, INC. vs. RJL MARTINEZ
G.R. No. 75198 October 18, 1988
FACTS:
RJL Martinez Fishing Corporation is engaged in deep-sea
fishing. In the course of its business, it needed electrical
generators for the operation of its business. Schmid and Oberly
sells electrical generators with the brand of Nagata, a Japanese
product. D. Nagata Co. Ltd. of Japan was Schmids supplier.
Schmid advertised the 12 Nagata generators for sale and RJL
purchased 12 brand new generators. Through an irrevocable line
of credit, Nagata shipped to the Schmid the generators and RJL
paid the amount of the purchase price. (First sale = 3 generators;
Second sale = 12 generators).
Later, the generators were found to be factory defective. RJL
informed the Schmid that it shall return the 12 generators. 3 were
returned. Schmid replaced the 3 generators subject of the first
sale with generators of a different brand. As to the second sale, 3
were shipped to Japan and the remaining 9 were not replaced.
RJL sued the defendant on the warranty, asking for rescission of
the contract and that Schmid be ordered to accept the
generators and be ordered to pay back the purchase money as
well as be liable for damages. Schmid opposes such liability
averring that it was merely the indentor in the sale between
Nagata Co., the exporter and RJL Martinez, the importer. As

mere indentor, it avers that is not liable for the sellers implied
warranty against hidden defects, Schmid not having personally
assumed any such warranty.
ISSUE:
1) WON the second transaction between the parties was a sale
or an indent transaction? INDENT TRANSACTION
2) Even is Schmid is merely an indentor, may it still be liable for
the warranty? YES, under its contractual obligations it may be
liable. But in this case, Schmid did not warrant the products.
HELD:
An indentor is a middlemen in the same class as commercial
brokers and commission merchants. A broker is generally
defined as one who is engaged, for others, on a commission,
negotiating contracts relative to property with the custody of
which he has no concern; the negotiator between other parties,
never acting in his own name but in the name of those who
employed him; he is strictly a middleman and for some purpose
the agent of both parties. There are 3 parties to an indent
transaction, (1) buyer, (2) indentor, and (3) supplier who is
usually a non-resident manufacturer residing in the country
where the goods are to be bought. The chief feature of a
commercial broker and a commercial merchant is that in effecting
a sale, they are merely intermediaries or middle-men, and act in
a certain sense as the agent of both parties to the transaction.
RJL MARTINEZ admitted that the generators were purchased
through indent order. RJL admitted in its demand letter
previously sent to SCHMID that 12 of 15 generators were
purchased through your company, by indent order and three (3)
by direct purchase. The evidence also show that RJL
MARTINEZ paid directly NAGATA CO, for the generators, and
that the latter company itself invoiced the sale and shipped the
generators directly to the former. The only participation of
Schmid was to act as an intermediary or middleman between
Nagata and RJL, by procuring an order from RJL and forwarding
the same to Nagata for which the company received a
commission from Nagata.
Sale vs. Indent Transaction:
The essence of the contract of sale is transfer of title or
agreement to transfer it for a price paid or promised. If such
transfer puts the transferee in the attitude or position of an owner
and makes him liable to the transferor as a debtor for the agreed
price, and not merely as an agent who must account for the
proceeds of a resale, the transaction is, a sale.
3 evidences pointing to fact that Schmid is merely an indentor:
a. the Quotation and the General Conditions of Sale on the
dorsal side thereof do not necessarily lead to the conclusion that
NAGATA CO., was the real seller of the 12 generators.
b. When RJL complained to SCHMID, it immediately asked RJL
to send the defective generators to its shop to determine what
was wrong. SCHMID informed NAGATA about the complaint of
RJL. After the generators were found to have factory defects,
SCHMID facilitated the shipment of three (3) generators to Japan
and, after their repair, back to the Philippines.
c. the letter from NAGATA CO. to SCHMID regarding the repair
of the generators indicated that the latter was within the purview
of a seller.
2) Even as SCHMID was merely an indentor, there was nothing
to prevent it from voluntarily warranting that twelve (12)
generators subject of the second transaction are free from any
COMPILED BY: WIGMORE #WIGMOREFOREVER 22

SALES Case Digest (Atty. Sarona)


Compiled by: Wigmore #wigmoreforever
hidden defects. In other words, SCHMID may be held
answerable for some other contractual obligation, if indeed it had
so bound itself. As stated above, an indentor is to some extent
an agent of both the vendor and the vendee. As such agent,
therefore, he may expressly obligate himself to undertake the
obligations of his principal.
Notably, nowhere in the Quotation is it stated therein that
SCHMID did bind itself to answer for the defects of the things
sold. Balagtas testified initially that the warranty was in the
receipts covering the sale. Nowhere is it stated in the invoice that
SCHMID warranted the generators against defects. He again
changed his mind and asserted that the warranty was given
verbally. Hence, RJL has failed to prove that SCHMID had given
a warranty on the 12 generators subject of the second
transaction.

5.

Distinguished from Dacion En Pago

PNB vs. PINEDA


G.R. No. L-46658, May 13, 1991
FACTS: In 1963, Ignacio and Lourdes Arroyo (spouses Arroyo)
obtained a loan of P580,000.00 from PNB to purchase 60% of
the subscribed capital stock, and thereby acquire the controlling
interest of private respondent Tayabas Cement Company, Inc.
(TCC). Spouses Arroyo executed a real estate mortgage over a
parcel of land (La Vista property) as security for the said loan.

VICTORIAS MILLING CO. vs CA and CONSOLIDATED


SUGAR CO.
[G.R. No. 117356. June 19, 2000]
FACTS: St. Therese Merchandising (STM), who regularly bought
sugar from Victorias Milling Co. (VMC), was issued Shipping
List/Delivery Receipts (SLDRs) by the latter as proof of
purchases for bags of sugar. Thereafter, STM sold to
Consolidated Sugar Co. (CSC) its rights in one of the SLDRs.
CSC communicated to VMC that it had been authorized by STM
to withdraw the sugar covered by SLDR. Enclosed in the letter
were a copy of SLDR and a letter of authority from STM
authorizing CSC "to withdraw for and in our behalf the refined
sugar covered by SLDR. CSC surrendered the SLDR to VMCs
warehouse and was allowed to withdraw sugar but after several
bags were released, it was later on refused to allow further
withdrawals of sugar. CSC communicated to VMC to allow it to
withdraw sugar because the SLDR had been sold and
endorsed to it by STM. VMC contended that it could not allow
any further withdrawals of sugar against SLDR because STM
had already withdrawn sugar covered by cleared checks. CSC
filed complaint against VMC. VMC contended that it had no
privity of contract with CSC, the dealings between it and STM
were part of a series of transactions involving only one account
or one general contract of sale because CSC was an agent of
STM. CSC countered that the sugar purchases involving SLDR
were separate and independent transactions.
ISSUE: Whether or not CSC was an agent of STM.
HELD: No. CSC was a buyer of the SLDR form, and not an
agent of STM.
CSC was not subject to STM's control. The question of whether a
contract is one of sale or agency depends on the intention of the
parties as gathered from the whole scope and effect of the
language employed. That the authorization given to CSC
contained the phrase "for and in our (STM's) behalf" did not
establish an agency. CSC communicated to VMC that the SLDR
had been sold and endorsed to it by STM. The use of the words
"sold and endorsed" means that STM and CSC intended a
contract of sale, and not an agency.
The basis of agency is representation. On the part of
principal, there must be an actual intention to appointor
intention naturally inferable from his words or actions; and on
part of the agent, there must be an intention to accept

appointment and act on it, and in the absence of such intent,


there is generally no agency. One factor which most clearly
distinguishes agency from other legal concepts is control; one
person - the agent - agrees to act under the control or direction
of another - the principal. Indeed, the very word "agency" has
come to connote control by the principal. The control factor, more
than any other, has caused the courts to put contracts between
principal and agent in a separate category.

the
an
the
the

Thereafter, TCC filed with PNB an application and agreement for


the establishment of an 8 year deferred letter of credit (L/C) for
$7,000,000.00 in favor of Toyo Menka Kaisha, Ltd. of Tokyo,
Japan, to cover the importation of a cement plant machinery and
equipment.
Upon approval of said application and opening of an L/C by PNB
in favor of Toyo Menka Kaisha, Ltd. for the account of TCC, the
Arroyo spouses executed documents (Surety Agreement and
Covenant) to secure the loan accommodation.
The imported cement plant machinery and equipment arrived
from Japan and were released to TCC under a trust receipt
agreement. Toyo Menka Kaisha made the corresponding
drawings against the L/C as scheduled. TCC, however, failed to
remit and/or pay the amount covered by the drawings. Thus,
PNB notified TCC of its intention to repossess the imported
machinery and equipment for failure of TCC to settle its
obligations under the L/C.
In the meantime, the personal accounts of the spouses Arroyo,
which included another loan of P160,000.00 secured by a real
estate mortgage over Hacienda Bacon located in Isabela,
Negros Occidental, had likewise become due. The spouses
Arroyo failed to satisfy their obligations with PNB and the latter
decided to foreclose the real estate mortgages.
At the auction sale of the La Vista Property, PNB was the highest
bidder with a bid price of P1,000,001.00. However, when said
property was about to be awarded to PNB, the representative of
the mortgagor-spouses objected and demanded from the PNB
the difference between the bid price of P1,000,001.00 and the
indebtedness of P499,060.25 of the Arroyo spouses on their
personal account. It was the contention of the spouses Arroyo's
representative that the foreclosure proceedings referred only to
the personal account of the mortgagor spouses without reference
to the account of TCC.
To remedy the situation, PNB filed a supplemental petition
requesting the Sheriff's Office to proceed with the sale of the
COMPILED BY: WIGMORE #WIGMOREFOREVER

23

SALES Case Digest (Atty. Sarona)


Compiled by: Wigmore #wigmoreforever
subject real properties to satisfy not only the amount of
P499,060.25 owed by the spouses Arroyos on their personal
account but also the amount of P35,019,901.49 owed by said
spouses as sureties of TCC. Said petition was opposed by the
spouses Arroyo and the other bidder Araneta.
PNB filed a petition for mandamus to compel the sheriff to
proceed with the foreclosure sale of the mortgaged properties
and the petition was granted. However, TCC filed a complaint
against PNB to restrain the foreclosure of the mortgages over the
said properties as well as a declaration that its obligation with
PNB had been fully paid by reason of the latter's repossession of
the imported machinery and equipment.
ISSUE: Whether or not TCC's liability has been extinguished by
the repossession of PNB of the imported cement plant machinery
and equipment and whether or not the repossession amounts to
dacion en pago.
HELD: No. It must be remembered that PNB took possession of
the imported cement plant machinery and equipment pursuant to
the trust receipt agreement executed by PNB and TCC giving the
former the unqualified right to the possession and disposal of all
property shipped under the Letter of Credit until such time as all
the liabilities and obligations under said letter had been
discharged.
PNB's possession of the subject machinery and equipment being
precisely as a form of security for the advances given to TCC
under the Letter of Credit, said possession by itself cannot be
considered payment of the loan secured thereby. Payment would
legally result only after PNB had foreclosed on said securities,
sold the same and applied the proceeds thereof to TCC's loan
obligation. Mere possession does not amount to foreclosure for
foreclosure denotes the procedure adopted by the mortgagee to
terminate the rights of the mortgagor on the property and
includes the sale itself.
Neither can said repossession amount to dacion en pago.
Dation in payment takes place when property is alienated to
the creditor in satisfaction of a debt in money and the same
is governed by sales. Dation in payment is the delivery and
transmission of ownership of a thing by the debtor to the
creditor as an accepted equivalent of the performance of the
obligation. As aforesaid, the repossession of the machinery
and equipment in question was merely to secure the
payment of TCC's loan obligation and not for the purpose of
transferring ownership thereof to PNB in satisfaction of said
loan. Thus, no dacion en pago was ever accomplished.
Proceeding from this finding, PNB has the right to foreclose the
mortgages executed by the spouses Arroyo as sureties of TCC.
A surety is considered in law as being the same party as the
debtor in relation to whatever is adjudged touching the obligation
of the latter, and their liabilities are interwoven as to be
inseparable. As sureties, the Arroyo spouses are primarily liable
as original promissors and are bound immediately to pay the
creditor the amount outstanding.

LO VS. KJS
G.R. No. 149420, October 8, 2003
FACTS: Respondent KJS ECO-FORMWORK System Phil., Inc.
is a corporation engaged in the sale of steel scaffoldings, while
petitioner Sonny L. Lo, doing business under Sans Enterprises,
is a building contractor. Lo ordered scaffolding equipment from
KJS worth P540,425.80. He paid a down payment in the amount
of P150,000.00. The balance was made payable in ten monthly
instalments.
KJS delivered the scaffoldings to Lo. Lo was able to pay the first
two monthly instalments. His business, however, encountered
financial difficulties and he was unable to settle his obligation to
KJS despite oral and written demands made against him.
Lo and KJS executed a Deed of Assignment whereby Lo
assigned to KJS his receivables in the amount of P335,462.14
from Jomero Realty Corporation.
However, when KJS tried to collect the said credit from Jomero
Realty Corporation, the latter refused to honor the Deed of
Assignment because it claimed that Lo was also indebted to it.
Subsequently, KJS sent a letter to Lo demanding payment of his
obligation, but he refused to pay claiming that his obligation had
been extinguished when they executed the Deed of Assignment.
Consequently, KJS filed an action for recovery of a sum of
money against Lo before the RTC. Lo argued that his obligation
was extinguished with the execution of the Deed of Assignment
of credit. KJS, for its part, presented the testimony of its
employee, Almeda Baaga, who testified that Jomero Realty
refused to honor the assignment of credit because it claimed that
Lo had an outstanding indebtedness to it.
RTC dismissed the complaint on the ground that the assignment
of credit extinguished the obligation. CA reversed the decision. In
finding that the Deed of Assignment did not extinguish the
obligation of the petitioner to the respondent, the CA held that (1)
petitioner failed to comply with his warranty under the Deed; (2)
the object of the Deed did not exist at the time of the transaction,
rendering it void pursuant to Article 1409 of the Civil Code; and
(3) petitioner violated the terms of the Deed of Assignment when
he failed to execute and do all acts and deeds as shall be
necessary to effectually enable the respondent to recover the
collectibles.
ISSUE: Whether or not the deed of assignment extinguished
Los obligations.
HELD: No. An assignment of credit is an agreement by virtue of
which the owner of a credit, known as the assignor, by a legal
cause, such as sale, dacion en pago, exchange or donation, and
without the consent of the debtor, transfers his credit and
accessory rights to another, known as the assignee, who
acquires the power to enforce it to the same extent as the
assignor could enforce it against the debtor.
Corollary thereto, in dacion en pago, as a special mode of
payment, the debtor offers another thing to the creditor who
accepts it as equivalent of payment of an outstanding debt. In
order that there be a valid dation in payment, the following are
the requisites:
COMPILED BY: WIGMORE #WIGMOREFOREVER

24

SALES Case Digest (Atty. Sarona)


Compiled by: Wigmore #wigmoreforever
1.) There must be the performance of the prestation in lieu of
payment (animo solvendi) which may consist in the delivery
of a corporeal thing or a real right or a credit against the third
person;
2.) There must be some difference between the prestation due
and that which is given in substitution (aliud pro alio)
3.) There must be an agreement between the creditor and
debtor that the obligation is immediately extinguished by
reason of the performance of a prestation different from that
due.
The undertaking really partakes in one sense of the nature of
sale, that is, the creditor is really buying the thing or property of
the debtor, payment for which is to be charged against the
debtors debt. As such, the vendor in good faith shall be
responsible, for the existence and legality of the credit at the time
of the sale but not for the solvency of the debtor, in specified
circumstances.
Hence, it may well be that the assignment of credit, which is in
the nature of a sale of personal property, produced the effects of
a dation in payment which may extinguish the obligation.
However, as in any other contract of sale, the vendor or assignor
is bound by certain warranties. More specifically, the first
paragraph of Article 1628 of the Civil Code provides:
The vendor in good faith shall be responsible for the existence
and legality of the credit at the time of the sale, unless it should
have been sold as doubtful; but not for the solvency of the
debtor, unless it has been so expressly stipulated or unless the
insolvency was prior to the sale and of common knowledge.
From the above provision, petitioner, as vendor or assignor, is
bound to warrant the existence and legality of the credit at the
time of the sale or assignment. When Jomero Realty claimed
that it was no longer indebted to Lo since the latter also had an
unpaid obligation to it, it essentially meant that its obligation to Lo
has been extinguished by compensation. In other words, KJS
alleged the non-existence of the credit and asserted its claim to
Los warranty under the assignment. Therefore, it behooved on
Lo to make good its warranty and paid the obligation.
Furthermore, we find that Lo breached his obligation under
Deed of Assignment. Indeed, by warranting the existence of
credit, petitioner should be deemed to have ensured
performance thereof in case the same is later found to
inexistent. He should be held liable to pay to respondent
amount of his indebtedness.

the
the
the
be
the

Hence, we affirm the decision of the Court of Appeals ordering


petitioner to pay respondent the sum of P335,462.14 with legal
interest thereon.
AQUINTEY V. SPOUSES TIBONG
G.R. NO. 166704,DECEMBER 20, 2006
FACTS: On May 6, 1999, petitioner Aquintey filed before RTC
Baguio, a complaint for sum of money and damages against
respondents. Agrifina alleged that Felicidad secured loans from
her on several occasions at monthly interest rates of 6% to 7%.
Despite demands, spouses Tibong failed to pay their outstanding
loans of P773,000,00 exclusive of interests. However, spouses
Tiong alleged that they had executed deeds of assignment in

favor of Agrifina amounting to P546,459 and that their debtors


had executed promissory notes in favor of Agrifina. Spouses
insisted that by virtue of these documents, Agrifina became the
new collector of their debts. Agrifina was able to collect the total
amount of P301,000 from Felicdads debtors. She tried to collect
the balance of Felicidad and when the latter reneged on her
promise, Agrifina filed a complaint in the office of the barangay
for the collection of P773,000.00. There was no settlement. RTC
favored Agrifina. Court of Appeals affirmed the decision with
modification ordering defendant to pay the balance of total
indebtedness in the amount of P51,341,00 plus 6% per month.
ISSUE: Whether or not the deeds of assignment in favor of
petitioner has the effect of payment of the original obligation that
would partially extinguish the same
RULING: YES. Substitution of the person of the debtor may be
affected by delegacion. Meaning, the debtor offers, the creditor
accepts a third person who consent of the substitution and
assumes the obligation. It is necessary that the old debtor be
released fro the obligation and the third person or new debtor
takes his place in the relation . Without such release, there is no
novation. Court of Appeals correctly found that the
respondents obligation to pay the balance of their account
with petitioner was extinguished pro tanto by the deeds of
credit. CA decision is affirmed with the modification that the
principal amount of the respondents is P33,841.
In its modern concept, what actually takes place in dacion en
pago is an objective novation of the obligation where the thing
offered as an accepted equivalent of the performance of an
obligation is considered as the object of the contract of sale,
while the debt is considered as the purchase price.

SSS V CA, 553 SCRA 677 (2008)


FACTS: AG&P and Semirara Coal Company proposed to pay its
arrears of premiums and loan amortization delinquencies through
dacion en pago which was subsequently accepted by SSS.
Thereafter, SSS directed herein defendant to submit certain
documents necessary for the agreement which AG&P
immediately complied with. SSS finally approved the dacion en
pago which as of March 2001 amounted to P29, 261,902.45. To
effect said transfer, a Deed of Assignment had to be executed
between the two parties which SSS failed to come up. On the
other hand, defendant continuously submitted drafts to SSS of
the needed Deed of Assignment. ON 2003, SSS sent to AG&P a
revised copy of the Deed of Assignment, however, the amount
went from P29, 261,902.45 to P40, 846,610.64 allegedly
because of the additional interest and penalties.
AG&P
requested for the deduction of these interests and penalties for
the delay of the Deed of Assignment was the fault of SSS.
Thus, AG&P filed a complaint for the specific performance and
damages against SSS. SSS contended that the court has no
jurisdiction over the case in accordance with R.A. 8282 which
provides that any dispute should be filed in the Commission.
RTC ruled in favor of AG&P. upon appeal, the CA held that the
court has jurisdiction and that the case be reverted back to the
Trial Court for actual proceedings. Thus, SSS appealed to the
Court.

COMPILED BY: WIGMORE #WIGMOREFOREVER

25

SALES Case Digest (Atty. Sarona)


Compiled by: Wigmore #wigmoreforever
ISSUE:
A. WON dacion en pago should be implemented
B. Which body has jurisdiction over non-implementation of a
dacion en pago agreed by the parties?
HELD:
1. Yes. The Supreme Court absolutely adopted the CA decision.
Dacion en pago is the delivery and transmission of ownership of
a thing by the debtor to the creditor as an accepted equivalent of
the performance of the obligation. It is a special mode of
payment where the debtor offers another thing to the creditor
who accepts it as equivalent of payment of an outstanding debt.
The undertaking really partakes in one sense of the nature of
sale, that is the creditor is really buying the thing or property of
the debtor, payment for which is to be charged against the
debtors debt. As such, the essential elements of a contract of
sale, namely, consent, object certain, and cause or consideration
must be present. In its modern concept, what actually takes
place in dacion en pago is an objective novation of the obligation
where the thing offered as an accepted equivalent of the
performance of an obligation is considered as the object of the
contract of sale, while the debt is considered as the purchase
price. In any case, common consent is an essential prerequisite,
be it sale or novation, to have the effect of totally extinguishing
the debt or obligation.
From the averments in their complaint, the appellate court
observed that private respondents are seeking to implement the
Deed of Assignment which they had drafted and submitted to
SSS pursuant to the approval by SSS. The appellate court thus
held that the subject of the complaint is no longer the payment of
the premium and loan amortization delinquencies, as well as the
penalties appurtenant thereto, but the enforcement of the dacion
en pago. Thus, the trial court was ordered to settle the
controversy.
From the allegations of respondents complaint, it readily appears
that there is no longer any dispute with respect to respondents
accountability to the SSS. Respondents had, in fact admitted
their delinquency and offered to settle them by way of dacion en
pago subsequently approved by the SSS in Resolution No. 270s. 2001. SSS stated in said resolution that the dacion en pago
proposal of AG&P Co. of Manila and Semirara Coals Corporation
to pay their liabilities in the total amount ofP30,652,710.71 as of
31 March 2001 by offering their 5.8 ha. property located in San
Pascual, Batangas, be, as it is hereby, approved.. This
statement unequivocally evinces its consent to the dacion en
pago.
2. The controversy, instead, lies in the non-implementation of the
approved and agreed dacion en pago on the part of the SSS. As
such, respondents filed a suit to obtain its enforcement which is,
doubtless, a suit for specific performance and one incapable of
pecuniary estimation beyond the competence of the
Commission. Pertinently, the Court ruled in Singson v. Isabela
Sawmill, as follows:
In determining whether an action is one the subject matter of
which is not capable of pecuniary estimation this Court has
adopted the criterion of first ascertaining the nature of the
principal action or remedy sought. If it is primarily for the
recovery of a sum of money, the claim is considered capable of
pecuniary
estimation,
and
whether
jurisdiction in the municipal courts or in the courts of first instance

would depend on the amount of the claim. However, where the


basic issue is something other than the right to recover a sum of
money, where the money claim is purely incidental to, or a
consequence of, the principal relief sought, this Court has
considered such actions as cases where the subject of the
litigation may not be estimated in terms of money, and are
cognizable exclusively by courts of first instance (now Regional
Trial Courts).
MAR YUSON vs ATTY. JEREMIAS R. VITAN
A.C. No. 6955, July 27, 2006
FACTS: In October 2002, Mar Yuson who was a taxi driver and
had 8 children, received a sum of money by way of inheritance.
He and his wife intended to use the money for several purposes.
When they were able to purchase a secondhand taxi, and Atty.
Vitan helped him with legal matters regarding the purchase.
Unfortunately, Yusons other plans were put on hold when Atty.
Vitan borrowed P100, 000 from them in December 2002. To
guarantee payment, Atty. Vitan executed in favor of Yuson
several postdated checks to over the loaned amount, but
however, these turned out to be worthless.
Yuson maintained that he had repeatedly tried to recover the
debt, but was unsuccessful every time. When no payment was
still made pursuant to the administrative case against Atty. Vitan,
Yuson demanded a collateral to secure the loan. Thus, in his
favor, Atty. Vitan executed a document denominated as a Deed
of Absolute Sale, covering Atty. Vitans parcel of land located in
Sta. Maria, Bulacan. According to Yuson, their intention was to
transfer the title of the property to him temporarily, so that he
could either sell or mortgage the said land. Further, if it was
mortgaged, Atty. Vitan would redeem it as partial or full payment
of the loan. Allegedly, the parties executed another Deed of
Absolute Sale in favor of Atty. Vitan wherein Yuson was vendor.
The purpose for this was not explained by either party.
Yuson was able to mortgage the property for P30,000 but
contrary to their earlier agreement, Atty. Vita did not redeem it
from the mortgage, sent a letter instead, promising Yuson to pay
on or before July 12, 2004.
In the IBP-NCLA, Atty. Vitan averred that he had settled his
obligation through a Deed of Absolute Sale over his residential
property. The purpose of such was for Yuson to use, mortgage,
or sell the property and return to him the excess of the proceeds
after obtaining his money. Additionally, he called the second
document as a Counter Deed of Sale, executed to be sort of a
collateral/security for the account of his liaison officer Estur,
whom he alleged that she was the one who incurred said debts.
ISSUE: W/N Atty. Vitans obligation was extinguished by
virtue of the first Deed of Absolute Sale
HELD: NO. Atty. Vitan contends that his obligation was already
extinguished, because he had allegedly sold his Bulacan
property to complainant. Basically, he is asserting that what had
transpired was a dation in payment. Governed by the law on
sales, it is a transaction that takes place when a piece of
property is alienated to the creditor in satisfaction of a debt in
money. It involves delivery and transmission of ownership of a
thing -- by the debtor to the creditor -- as an accepted equivalent
of the performance of the obligation.
COMPILED BY: WIGMORE #WIGMOREFOREVER 26

SALES Case Digest (Atty. Sarona)


Compiled by: Wigmore #wigmoreforever

However, the records reveal that he did not really intend to sell
and relinquish ownership over his property in Sta. Maria,
Bulacan, notwithstanding the execution of a Deed of Absolute
Sale in favor of Yuson. The second Deed of Absolute Sale, which
reconveyed the property to respondent, is proof that he had no
such intention. This second Deed, which he referred to as his
"safety net," betrays his intention to counteract the effects of the
first one.
Ergo, Atty. Vitan was taking back with his right hand what he had
given with his left. The second Deed of Absolute Sale returned
the parties right back where they started, as if there were no sale
in favor of complainant to begin with. In effect, on the basis of the
second Deed of Sale, respondent took back and asserted his
ownership over the property despite having allegedly sold it.
Thus, he fails to convince us that there was a bona fide dation in
payment or sale that took place between the parties; that is, that
there was an extinguishment of obligation.
It appears that the true intention of the parties was to use the
Bulacan property to facilitate payment. They only made it appear
that the title had been transferred to complainant to authorize
him to sell or mortgage the property.Atty. Vitan himself admitted
in his letter dated July 30, 2004, that their intention was to
convert the property into cash, so that payment could be
obtained by complainant and the excess returned to respondent.
The records, however, do not show that the proceeds derived
were sufficient to discharge the obligation of the lawyer fully;
thus, he is still liable to the extent of the deficiency.
FILINVEST CREDIT CORPORATION VS. PHILIPPINE
ACETYLENE, CO., INC.
G.R. No. L-50449, January 30, 1982
FACTS: On October 30, 1971, the Philippine Acetylene Co., Inc.,
purchased from Alexander Lim, as evidenced by a Deed of Sale,
a Chevrolet 1969 model motor vehicle payable under the terms
and conditions of the promissory note provided by PhilAcetylene.
As security for the payment, PhilAcetylene executed a chattel
mortgage over the same vehicle in favor of Lim. Subsequently,
Lim assigned to Filinvest Finance all his rights, title, and interests
in the promissory note and the chattel mortgage which
subsequently assigned it to Filinvest Credit.
PhilAcetylene failed to comply with the terms in the promissory
note and chattel mortgage. With the choice of paying the full
amount plus interest and charges or returning the mortgaged
property, PhilAcetylene informed Filinvest Credit that it was
returning the mortgaged property in full satisfaction of its
indebtedness pursuant to Art. 1484 of the New Civil Code. When
it was returned to Filinvest, it had with it a document
denominated as Voluntary Surrender with Special Power of
Attorney to Sell.
Filinvest could not however sell the vehicle since there were
unpaid taxes on said vehicle. Upon Filinvests offer to return the
vehicle to PhilAcetylene, the latter refused to accept it.
PhilAectylene contends that Filinvest has no cause of action
since its obligation was extinguished when it returned the
mortgaged property to Filinvest and assuming however that the
return of the property did not extinguish its obligation, it was

nonetheless justified in refusing payment since Filinvest is not


entitled to recover the same due to the breach of warranty
committed by the original vendor-assignor Alexander Lim.
Additionally, it argues that by virtue of the return, it extinguished
their obligation through dation in payment.
ISSUE: W/N the return of the mortgaged motor vehicle to
Filinvest by virtue of its voluntary surrender by Philippine
Acetylene totally extinguished and/or cancelled its
obligation to Filinvest
HELD: NO. The mere return of the mortgaged motor vehicle by
the mortgagor, PhilAcetylene, to the mortgagee, Filinvest, does
not constitute dation in payment or dacion en pago in the
absence, express or implied of the true intention of the parties.
Dacion en pago, according to Manresa, is the transmission of the
ownership of a thing by the debtor to the creditor as an accepted
equivalent of the performance of obligation.
The evidence on the record fails to show that the mortgagee,
consented, or at least intended, that the mere delivery to, and
acceptance by him, of the mortgaged motor vehicle be construed
as actual payment, more specifically dation in payment or dacion
en pago. The fact that the mortgaged motor vehicle was
delivered to him does not necessarily mean that ownership
thereof, as juridically contemplated by dacion en pago, was
transferred from PhilAcetylene to Filinvest. In the absence of
clear consent of Filinvest, there can be no transfer of ownership
of the mortgaged motor vehicle from appellant to appellee. Only
transfer of possession of the mortgaged motor vehicle took
place, for it is quite possible that Filinvest merely wanted to
secure possession to forestall the loss, destruction, fraudulent
transfer of the vehicle to third persons, or its being rendered
valueless if left in the hands of PhilAcetylene.
The true intention of the parties is furnished by the document
executed by appellant captioned "Voluntary Surrender with
Special Power of Attorney To Sell". The document reveals that
the possession of the mortgaged motor vehicle was voluntarily
surrendered by PhilAcetylene to Filinvest authorizing the latter to
look for a buyer and sell the vehicle in behalf of PhilAcetylene
who retains ownership thereof, and to apply the proceeds of the
sale to the mortgage indebtedness, with the undertaking of the
appellant to pay the difference, if any, between the selling price
and the mortgage obligation. With the stipulated conditions as
stated, Filinvest in essence was constituted as a mere agent to
sell the motor vehicle which was delivered to it, not as its
property, for if it were, he would have full power of disposition of
the property, not only to sell it as is the limited authority given him
in the special power of attorney. Had Filinvest intended to
completely release PhilAcetylene of its mortgage obligation,
there would be no necessity of executing the document.
Nowhere in the said document where it states, that the mere
surrender of the mortgaged motor vehicle to the appellee
extinguished appellants obligation for the unpaid price.
On PhilAcetylenes argument that by accepting the delivery of
the mortgaged motor vehicle, Filinvest is estopped from
demanding payment of the unpaid obligation, the same is without
merit. As clearly set forth above, Filinvest never accepted the
mortgaged motor vehicle in full satisfaction of the mortgaged
debt.
Under the law, the delivery of possession of the mortgaged
COMPILED BY: WIGMORE #WIGMOREFOREVER 27

SALES Case Digest (Atty. Sarona)


Compiled by: Wigmore #wigmoreforever
property to the mortgagee, can only operate to extinguish
PhilAcetylene liability if Filinvest had actually caused the
foreclosure sale of the mortgaged property when it recovered
possession thereof.

6.

Distinguished from Payment by Cession

7.

Distinguished from Lease

FILINVEST CREDIT COROPORATION VS CA, JOSE SY


BANG and ILUMINADA TAN SY BANG
G.R. No. 82508, September 29, 1989
FACTS: Spouses Jose and Iluminada Sy Bang were engaged in
the sale of gravel produced from crushed rocks and used for
construction purposes. They engaged the serviced of Mr. Ruben
Mercurio of Gemini Motor Sales, to look for a rock crusher. Mr.
Mercurio then referred them to Rizal Consolidated who had said
machinery for sale.
They applied for financial assistance with Filinvest Credit
regarding their purchase of the machine. Fiinvest agreed to
extend to the Spouses Sy Bang financial aid on the following
conditions: that the machinery be purchased in the Filinvest's
name; that it be leased (with option to purchase upon the
termination of the lease period) to the Spouses Sy Bang; and
that Spouses Sy Bang execute a real estate mortgage as
security for the amount advanced by Filinvest. Accordingly, on
May 18,1981, a contract of lease of machinery (with option to
purchase) was entered into by the parties whereby the spouses
agreed to lease from Filinvest the rock crusher for two years
starting from July 5, 1981 payable as follows: P10,000.00 first
3 months, P23,000.00 next 6 months, P24,800.00 next 15
months. It was likewise stipulated that at the end of the two-year
period, the machine would be owned by the spouses.

property of the private respondents. This form of agreement has


been criticized as a lease only in name.
Sellers desirous of making conditional sales of their goods, but
who do not wish openly to make a bargain in that form, for one
reason or another, have frequently resorted to the device of
making contracts in the form of leases either with options to the
buyer to purchase for a small consideration at the end of term,
provided the so-called rent has been duly paid, or with
stipulations that if the rent throughout the term is paid, title shall
thereupon vest in the lessee. It is obvious that such transactions
are leases only in name. The so-called rent must necessarily be
regarded as payment of the price in installments since the due
payment of the agreed amount results, by the terms of bargain,
in the transfer of title to the lessee.
2.) NO.
They are alternative. The seller of movable in installments, in
case the buyer fails to pay 2 or more installments, may elect to
pursue either of the following remedies: (1) exact fulfillment by
the purchaser of the obligation; (2) cancel the sale; or (3)
foreclose the mortgage on the purchased property if one was
constituted thereon. It is now settled that the said remedies are
alternative and not cumulative, and therefore, the exercise of one
bars the exercise of the others. Indubitably, the device contract
of lease with option to buy is at times resorted to as a means to
circumvent Article 1484, particularly paragraph (3) thereof.
Through the set-up, the vendor, by retaining ownership over the
property in the guise of being the lessor, retains, likewise the
right to repossess the same, without going through the process
of foreclosure, in the event the vendee-lessee defaults in the
payment of the installments. There arises therefore no need to
constitute a chattel mortgage over the movable sold. More
important, the vendor, after repossessing the property and, in
effect, canceling the contract of sale, gets to keep all the
installments-cum-rentals already paid.

The spouses then issued a check for P150,550 as initial rental,


and 24 postdated checks corresponding to 24 monthly rentals in
favor of Filinvest. They likewise executed a real estate mortgage
over two parcels of land to guarantee their compliance with the
lease contract. The rock crusher was then delivered to the
spouses. However, 3 months later, the souses stopped payment
when petitioner had not acted on the complaints of the spouses
about the machine. As a consequence, petitioner extra-judicially
foreclosed the real estate mortgage. To thwart the impending
auction, the spouses filed a complaint for rescission of the
contract of lease and annulment of the real estate mortgage.

8.

ISSUE:
1) W/N the nature of the contract is one of a contract
of sale.
2) W/N the remedies of the seller provided for in
Article 1484 are cumulative.

On July 5, 1984, the heirs of Julio Garcia and Federico Briones


entered into a Kasunduan ng Pagbibilhan over the 21,460 sq. m
portion for the sum of P150,000--- P65,000 was paid at the
execution of the contract and the P85,000 was made payable
within 6 months from the date of the execution of the instrument.
At the time of the execution of the document, Briones was
informed that the title over the property is with their cousin
Conchalina who owns the bigger portion of the land. This
notwithstanding, respondent willingly entered into the Kasunduan
provided that the full payment of the purchase price will be made
upon delivery to him of the title.

HELD:
1.) YES.
It is apparent here that the intent of the parties to the subject
contract is for the so-called rentals to be the installment
payments. Upon the completion of the payments, then the rock
crusher, subject matter of the contract, would become the

Distinguished from Contract to Sell

JUANA ALMIRA ET AL vs CA and FEDERICO BRIONES


G.R. No. 115966, March 20, 2003
FACTS: Almira Et al are the wife and children of the late Julio
Garcia who inherited from his mother, Maria Alibudbud, a portion
of a Lot 1642 in Sta. Rosa, Laguna. Lot 1642 was co-owned and
registered in the names of Vicente de Guzman, Enrique
Hemedes and Francsisco Alibudbud.

Briones took possession of the subject property and made


COMPILED BY: WIGMORE #WIGMOREFOREVER 28

SALES Case Digest (Atty. Sarona)


Compiled by: Wigmore #wigmoreforever
various payments amounting to P58,500.00 but because of the
failure of the heirs of Garcia to deliver to him a separate title to
the property, he refused to make further payments. This
prompted the heirs to file a case for rescission of the Kasunduan
and the return of the possession of the subject land.
The heirs alleged that they approached Briones several times to
deliver the required title but the latter refused saying that he did
not have the money to pay the balance of the purchase price.
The RTC decreed the rescission prayed for and the return of the
possession of the subject property. The CA however reversed the
lower courts decision.
ISSUES:
(1) W/N payment of the balance of the purchase price is
conditioned upon delivery of a separate title in the
name of Julio Garcia;
(2) W/N the heirs are entitled to rescind the Kasunduan
for failure of Briones to complete payment.
HELD:
(1) YES.
The tenor of the correspondence between the heirs and Briones
shows that the parties intended that a separate title to the
property in the name of Julio Garcia shall be delivered to Briones
as a condition for the latters payment of the balance of the
purchase price. As such, Briones signified his willingness to pay
but reminded the heirs of their obligation to deliver title to the
property.
If the parties intended that the heirs deliver TCT No. RT-1076
instead of a separate title in the name of Julio Garcia to Briones,
then there would have been no need for the heirs to ask for
partial sums on the ground that this would be used to pay for the
processing fee of the title to the property. The heirs only had to
present the existing title to Briones and demand the balance of
the purchase price, but this they did not do. There is likewise no
basis to conclude that insufficiency of funds rather than failure of
the heirs to deliver a separate title in the name of Julio Garcia
prevented Briones from completing payment of the purchase
price.
That the parties agreed on delivery of a separate title in the
name of Julio Garcia as a condition for respondents payment of
the balance of the purchase price is bolstered by the fact that
there was already an approved subdivision plan of the 21,460
square-meter lot years before petitioners filed an action in court
for rescission. Unfortunately, the heirs were not able to secure a
separate title in the name of Julio Garcia.
(2) NO
In order to determine if rescission is proper, the Court needed to
ascertain whether the Kasunduan was a Contract to Sell or a
Contract of Sale. In a contract to sell, ownership is, by
agreement, reserved to the vendor and is not to pass until full
payment of the purchase price; whereas, in contract of sale, title
to the property passes to the vendee upon delivery of the thing
sold. Non-payment by the vendee in a contract of sale entitles
the vendor to demand specific performance or rescission of the
contract, with damages, under Article 1191 of the Civil Code.

A careful reading of the Kasunduan reveals that it is a contract of


sale. There was a perfected contract of sale in this case. The
parties agreed on the sale of a determinate object which is the
subject property in this case in the name of Julio Garcia, and
also the price certain therefor, without any reservation of title on
the part of the heirs. Ownership was effectively conveyed by
petitioners to respondent, who was given possession of the
property. The delivery of a separate title in the name of Julio
Garcia was a condition imposed on respondents obligation to
pay the balance of the purchase price. It was not a condition
imposed on the perfection of the contract of sale.
As to the rescission prayed for, the Court rules in the negative.
The power to rescind is only given to the injured party. The
injured party is the party who has faithfully fulfilled his obligation
or is ready and willing to perform with his obligation. In the case
at bar, petitioners were not ready, willing and able to comply with
their obligation to deliver a separate title in the name of Julio
Garcia to respondent. Therefore, they are not in a position to ask
for rescission of the Kasunduan. Moreover, respondents
obligation to pay the balance of the purchase price was made
subject to delivery by petitioners of a separate title in the name of
Julio Garcia within six (6) months from the time of the execution
of the Kasunduan, a condition with which petitioners failed to
comply. Failure to comply with a condition imposed on the
performance of an obligation gives the other party the option
either to refuse to proceed with the sale or to waive that condition
under Article 1545 of the Civil Code. Hence, it is the respondent
who has the option either to refuse to proceed with the sale or to
waive the performance of the condition imposed on his obligation
to pay the balance of the purchase price.
SPOUSES SERRANO VS. CAGUIAT
517 SCRA 57
G.R. NO. 139173
FEBRUARY 28, 2007
FACTS: Spouses Serrano are registered owners of a lot located
in Las Pinas. On March 23, 1990, Caguiat offered to buy the lot
and the Serranos agreed to sell it at 1,500.00/sqm.
Caguiat then paid them a partial payment of 100,000.00 as
evidenced by a receipt indicating therein Caguiats promise to
pay the remaining balance.
Respondent, after making known his readiness to pay the
balance, requested from petitioners the preparation of the
necessary Deed of Sale.
Petitioners informed respondent in a letter that Amparo Herrera
would be leaving for abroad on or before April 15, 1990 and they
are canceling the transaction and that respondent may recover
the earnest money (100,000) anytime. Petitioners also wrote him
stating that they already delivered a managers check to his
counsel in said amount.
Respondent thus filed a complaint for specific performance and
damages with the RTC of Makati.
The trial court relying on Article 1482 of the Civil Code ruled that
the payment of 100,000.00 being an earnest money signified
perfection of the contract of sale and ordered the petitioners to
COMPILED BY: WIGMORE #WIGMOREFOREVER 29

SALES Case Digest (Atty. Sarona)


Compiled by: Wigmore #wigmoreforever
documents shall be executed by the Nabuses. The Pacsons
thereafter occupied the land and built an auto shop thereon.

execute a final deed of sale in favor of respondent.


The Court of Appeals denied petitioners motion
reconsideration in affirmation of the lower courts decision.

for

ISSUE: WON there was a contract of sale. NO.


HELD: The transaction was a contract to sell.
When petitioners declared in the Receipt of Partial Payment that
they
Received from Mr. Godofredo Caguiat the amount of one
hundred thousand pesos as Partial payment of our lot
situated in Las PinasMr. Caguiat promised to pay the
balance of the purchase price on or before March 23,
1990 And that we will execute and sign the final deed of
sale on this date,
-- there can be no other interpretation than that they agreed to a
conditional contract of sale, consummation of which is subject
only to the full payment of the purchase price.

In 1977, Bate Nabus died,leaving Julie Nabus and her daughter


Michelle to execute a Deed of Extrajudicial Settlement over the
property. Anew TCT was issued in their names in 1984.
By 1984, the Pacsons have made 364 payments, leaving a
balance of57k.
Sometime later, Julie Nabus approached Joaquin Pacson for the
remaining balance.
While the Pacsons claimed they were ready to pay the balance,
they asked Nabus to return in 4 days as they wanted to make
sure of the remaining balance; wanted to see the new deeds
issued in the Nabuses name; wanted to see the guardianship
papers of the Nabus child. However, Julie Nabus did not return.
The Pacsons later discovered that the entire lothad been sold to
a Betty Tolero, and that a new title had been issued in Toleros
name.

A contract to sell is akin to a conditional sale where the efficacy


or obligatory force of the vendors obligation to transfer title is
subordinated to the happening of a future and uncertain event,
so that if the suspensive condition does not take place, the
parties would stand as if the conditional obligation had never
existed. The suspensive condition is commonly full payment of
the purchase price.

Aggrieved they prayed to annul Toleros title, as well as of the


other documents issued to the Nabuses.

In this case, the Receipt of Partial Payment shows that the true
agreement between the parties is a contract to sell.

They also claimed that Pacsons signature in apage of the


contract was absent.

First, ownership of the parcel of land was retained by petitioners


and was not to pass to respondent until full payment of the
purchase price. Second, the agreement between the parties was
not embodied in a deed of sale. The absence of a formal deed
of conveyance is a strong indication that the parties did not
intend immediate transfer of ownership, but only a transfer after
full payment of the purchase price. Third, petitioners retained
possession of the certificate of the lot.

The RTC and the CA both ruled in favor of the Pacsons and
ordered the execution of a Deed of Absolute Sale in favor of the
Pacsons, upon their payment of the full purchase price.

It is true that Article 1482 provides that whenever earnest money


is given in a contract of sale, it shall be considered as part of the
price and proof of the perfection of the contract. However, this
article speaks of earnest money given in a contract of sale. In
this case, the earnest money forms part of the consideration only
if the sale is consummated upon full payment of the purchase
price.
Clearly, respondent cannot compel petitioners to transfer
ownership of the property to him.
NABUS VS.PACSON
G.R. NO. 161318
NOVEMBER 25, 2009
FACTS: In 1977, the spouses Nabus executed a Deed of
Conditional Sale (DOCS) in favor of the Spouses Pacson.
This deed covered 1,000sqm out of the 1,665sqm land owned by
the Nabuses. The deed states that after the Pacsons have paid
the full consideration of 170k, the corresponding transfer

Intheir defense, the Nabuses claimed that their DoCS was


converted in a contract of lease, as Joaquin Pacson had
misgivings about buying the land after knowing that it was in
dispute.

The Nabuses appealed.


ISSUES:
1. WON the DOCS was converted to a Contract of Lease? NO.
2. What is the nature of the contract between the Nabuses and
the Pacsons? CONTRACT TO SELL
HELD:
1.
The DOCS was not converted to a Contract of Lease. The
receipts issued to the Pacsons contained the phrase as partial
payment to lot evidencing the intentof sale, rather than of lease.
Further, as found by the trial court, Joaquin Pacsons non-signing
of the second page of a carbon copy of the Deed of Conditional
Sale was through sheer inadvertence, since the original contract
and the other copies of the contract were all signed by Joaquin
Pacson and the other parties to the contract.

2.

The DOCS was a contract to sell, and because the Pacsons


failed to pay the full price, no obligation arose on the part of
the Nabuses.

It is not the title of the contract, but its express terms or


stipulations that determine the kind of contract entered into by
the parties. In this case, the contract entitled "Deed of
COMPILED BY: WIGMORE #WIGMOREFOREVER

30

SALES Case Digest (Atty. Sarona)


Compiled by: Wigmore #wigmoreforever
Conditional Sale" is actually a contract to sell. The contract
stipulated that "as soon as the full consideration of the sale has
been paid by the vendee, the corresponding transfer documents
shall be executed by the vendor to the vendee for the portion
sold."
Where the vendor promises to execute a deed of absolute sale
upon the completion by the vendee of the payment of the price,
the contract is only a contract to sell." The aforecited stipulation
shows that the vendors reserved title to the subject property until
full payment of the purchase price.
Unfortunately for the Spouses Pacson, since the Deed of
Conditional Sale executed in their favor was merely a contract to
sell, the obligation of the seller to sell becomes demandable
only upon the happening of the suspensive condition. The
full payment of the purchase price is the positive suspensive
condition, the failure of which is not a breach of contract, but
simply an event that prevented the obligation of the vendor to
convey title from acquiring binding force.
Thus, for its non-fulfillment, there is no contract to speak of, the
obligor having failed to perform the suspensive condition which
enforces a juridical relation. With this circumstance, there can be
no rescission or fulfillment of an obligation that is still nonexistent, the suspensive condition not having occurred as yet.
Emphasis should be made that the breach contemplated in
Article 1191 of the New Civil Code is the obligors failure to
comply with an obligation already extant, not a failure of a
condition to render binding that obligation.

Petitioner now is seeking the rescission of her contract with


respondent for the breach of nonpayment.
RTC: the contract entered into by the parties is a contract to sell
but ruled that the remedy of rescission could not apply because
the respondents failure to pay the petitioner the balance of the
purchase price in the total amount of 805,000.00 was not a
breach of contract, but merely an event that prevented the seller
(petitioner) from conveying title to the purchaser (respondent).
CA: Affirmed.
ISSUE: W/N the contract entered into by the parties is a contract
to sell? YES.
HELD: The subject Deed of Conditional Sale with Assumption of
Mortgage entered into by and among the two parties and FSL
Bank is a contract to sell and not a contract of sale. The
nonpayment of the full purchase price cannot give the petitioner
the remedy for rescission since the obligation did not yet exist
since the suspensive condition of payment of the full purchase
price had not taken place.
A contract to sell may thus be defined as a bilateral contract
whereby the prospective seller, while expressly reserving the
ownership of the subject property despite delivery thereof to the
prospective buyer, binds himself to sell the said property
exclusively to the prospective buyer upon fulfillment of the
condition agreed upon, that is, full payment of the purchase
price.

Therefore, since the Pacsons failed to fulfill the suspensive


condition, the obligation on the part of the Nabuses to sell them
the land never arose, and the Nabuses were well within their
rights when they sold the land to Tolero. The Pacsons, of course,
are entitled to reimbursement.

Based on their contract, the title and ownership of the subject


properties remains with the petitioner until the respondent fully
pays the balance of the purchase price and the assumed
mortgage obligation. Thereafter, FSL Bank shall then issue the
corresponding deed of cancellation of mortgage and the
petitioner shall execute the corresponding deed of absolute sale
in favor of the respondent.

REYES VS.TUPARAN
G.R. NO. 188064
JUNE 1, 2011

Accordingly, the petitioners obligation to sell the subject


properties becomes demandable only upon the happening of the
positive suspensive condition, which is the respondents full
payment of the purchase price.

FACTS: Petitioner Mila Reyes owns a building which the


respondent Victoria Tuparan was leasing a space for her
pawnshop business.
Thereafter, petitioner mortgaged the building to Farmer Savings
and Loan Bank for P2M. However, the loan reached
P2,278,078.13. Petitioner then decided to sell her real properties
for P6.5M in order to pay the bank. As a gesture of friendship,
respondent verbally offered to conditionally buy the building for
P4.2M and to assume the bank loan.

Without respondents full payment, there can be no breach of


contract to speak of because petitioner has no obligation yet to
turn over the title. Respondents failure to pay in full the purchase
price is not the breach of contract contemplated under Article
1191 of the New Civil Code but rather just an event that prevents
the petitioner from being bound to convey title to the respondent.

It was stipulated however that title to the ownership of the subject


real properties shall remain with the petitioner until full payment
of respondent. And only upon payment of full balance will the
bank issue the Deed of Cancellation of Mortgage and the
petitioner to execute the corresponding Deed of Absolute Sale.
Respondent however defaulted, revealing a balance of
P805,000. She was already able to pay the amountP3.4M and
the bank loan.

COMPILED BY: WIGMORE #WIGMOREFOREVER

31

SALES Case Digest (Atty. Sarona)


Compiled by: Wigmore #wigmoreforever
PART II: ELEMENTS OF A CONTRACT OF SALE
2. CONSENT: PARTIES
1)

Minors, Insane or Demented Persons, Deaf-Mutes

since 2/3 belonged to his sisters. Petitioner could not have


given her consent to the contract, being a minor at the time.
Consent of the contracting parties is among the essential
requisites of a contract, including one of sale, absent which
there can be no valid contract. Moreover, petitioner admittedly
did not pay any centavo for the property, which makes the sale
void.

LABAGALA VS. SANTIAGO


371 SCRA 360
G.R. NO. 132305
DECEMBER 4, 2001

Art 1471: If the price is simulated, the sale is void, but the act
may be shown to have been in reality a donation, or some other
act or contract.

FACTS: Jose Santiago owned a parcel of land covered by TCT


64729. Alleging that Jose had fraudulently registered it in his
name alone, his sisters (Nicolasa and Amanda, herein
respondents) sued Jose for the recovery of 2/3 share of the
property. RTC ruled in favor of the sisters and their names were
included in the certificate of title. Jose died intestate.

Even assuming that the deed is genuine, it cannot be a valid


donation. It lacks the acceptance of the donee required by Art
725 of the Civil Code. Being a minor, the acceptance of the
donation should have been made by her father (Leon Labagala)
or mother, or legal representative. No one of those mentioned in
the law accepted the donation for Ida.

Respondents filed a complaint for the recovery of title, ownership


and possession against petitioner Labagala to recover from her
the 1/3 portion of the said property pertaining to Jose, but which
came into petitioners sole possession upon his death.

The Court also ruled that petitioner is not the child of Jose
Santiago, and cannot inherit from him through succession. No
birth certificate was shown, only a baptismal certificate, which is
not conclusive proof of filiation.

Respondents alleged that Joses share in the property belongs to


them by operation of law since they are the only legal heirs of
their brother.

PARAGAS vs. HEIRS OF DOMINADOR BALACANO

Respondents contentions:
The deed of sale was a forgery. The deed showed that Jose
affixed his thumb mark, but respondents averred that, having
been able to graduate from college, Jose never put his thumb
mark on documents and always signed his name in full.
Respondents also pointed out that it is highly improbable for
petitioner to have paid the supposed consideration of P150,000
for the sale of the property because petitioner was unemployed
and without any visible means of livelihood at the time of the
alleged sale.
It was quite unusual and questionable that petitioner registered
the deed of sale almost 8 years after the execution of the sale.
Petitioner claimed to be the daughter of Jose, and thus entitled to
his share in the property.
She argued that the sale was in fact a donation to her, and that
nothing could have precluded Jose from putting his thumb mark
on the deed of sale instead of his signature.
ISSUE: WON there was a valid sale. NO
HELD: The Court agreed with CA that:
This deed is shot through and through with so many intrinsic
defects that a reasonable mind is inevitably led to the conclusion
that it is fake. Why hide the nature of the contract in the faade of
a sale? Why did Santiago (fully aware that he owned only 1/3)
sell or donate the whole property to Ida? Why did Santiago affix
only his thumb mark to a deed that falsely stated that xxx Ida
was of legal age when she was then only 15 years old? Etc
Clearly, there is no valid sale in this case. Jose did not have the
right to transfer ownership of the entire property to petitioner

FACTS: Gregorio Balacano, married to Lorenza Sumigcay, was


the registered owner of Lot 1175-E and Lot 1175-F of the Subd.
Plan Psd-38042. Gregorio and Lorenza had three children,
namely: Domingo, Catalino and Alfredo, all surnamed Balacano.
Lorenza died on December 11, 1991. Gregorio, on the other
hand, died on July 28, 1996. Prior to his death, Gregorio was
admitted on June 28, 1996, transferred hospital in the afternoon
of July 19, 1996 until his death.
Gregorio purportedly sold on July 22, 1996, or barely a week
prior to his death, a portion of Lot 1175-E (15,925 square meters
out of total area of 22,341 square meters) and the whole Lot
1175-F to Spouses Paragas for the total consideration of
P500,000.00. This sale appeared in a deed of absolute sale and
was notarized by Atty. De Guzman. Gregorios certificates of title
were consequently cancelled and new certificates of title were
issued in favor of the Spouses Paragas.
The Spouses Paragas then sold on October 17, 1996 a portion
of Lot 1175-E consisting of 6,416 square meters to Catalino for
the total consideration of P60,000.00.
Domingos children filed on October 22, 1996 a complaint for
annulment of sale and partition against Catalino and the
Spouses Paragas. They essentially alleged in asking for the
nullification of the deed of sale that:
(1) their grandfather Gregorio could not have appeared before
the notary public on July 22, 1996 at Santiago City because he
was then confined at the Veterans Memorial Hospital in Quezon
City;
(2) at the time of the alleged execution of the deed of sale,
Gregorio was seriously ill, in fact dying at that time, which vitiated
his consent to the disposal of the property; and
(3) Catalino manipulated the execution of the deed and prevailed
upon the dying Gregorio to sign his name on a paper the
contents of which he never understood because of his serious
COMPILED BY: WIGMORE #WIGMOREFOREVER 32

SALES Case Digest (Atty. Sarona)


Compiled by: Wigmore #wigmoreforever
condition.
Alternatively, they alleged that assuming Gregorio was of sound
and disposing mind, he could only transfer a half portion of Lots
1175-E and 1175-F as the other half belongs to their
grandmother Lorenza who predeceased Gregorio they claimed
that Lots 1175-E and 1175-F form part of the conjugal
partnership properties of Gregorio and Lorenza. Finally, they
alleged that the sale to the Spouses Paragas covers only a 5hectare portion of Lots 1175-E and 1175-F leaving a portion of
6,416 square meters that Catalino is threatening to dispose.
They asked for the nullification of the deed of sale executed by
Gregorio and the partition of Lots 1175-E and 1175-F. They
likewise asked for damages.
Plaintiff-appellant Nanette Balacano testified to prove the
material allegations of their complaint. On Gregorios medical
condition, she declared that:
(1) Gregorio, who was then 81 years old, weak and sick, was
brought to the hospital in Bayombong, Nueva Vizcaya on June
28, 1996 and stayed there until the afternoon on July 19, 1996;
(2) thereafter, Gregorio, who by then was weak and could no
longer talk and whose condition had worsened, was transferred
in the afternoon of July 19, 1996 to the Veterans Memorial
Hospital in Quezon City where Gregorio died.
She claimed that Gregorio could not have signed a deed of sale
on July 19, 1996 because she stayed at the hospital the whole of
that day and saw no visitors. She likewise testified on their
agreement for attorneys fees with their counsel and the litigation
expenses they incurred.
Defendants posit that Gregorios consent to the sale should be
determined, not at the time Gregorio signed the deed of sale on
July 18, 1996, but at the time when he agreed to sell the property
in June 1996 or a month prior to the deeds signing; and in June
1996, Gregorio was of sound and disposing mind and his
consent to the sale was in no wise vitiated at that time. They
presented as witnesses Notary Public de Guzman and
instrumental witness Antonio to prove Gregorios execution of the
sale and the circumstances under the deed was executed. They
uniformly declared that:
(1) on July 18, 1996, they went to the hospital in Bayombong,
Nueva Vizcaya where Gregorio was confined with Rudy;
(2) Atty. De Guzman read and explained the contents of the deed
to Gregorio;
(3) Gregorio signed the deed after receiving the money from
Rudy;
(4) Julia and Antonio signed the deed as witnesses.
Additionally, Atty. De Guzman explained that the execution of the
deed was merely a confirmation of a previous agreement
between the Spouses Paragas and Gregorio that was concluded
at least a month prior to Gregorios death; that, in fact, Gregorio
had previously asked him to prepare a deed that Gregorio
eventually signed on July 18, 1996. He also explained that the
deed, which appeared to have been executed on July 22, 1996,
was actually executed on July 18, 1996; he notarized the deed
and entered it in his register only on July 22, 1996. He claimed
that he did not find it necessary to state the precise date and
place of execution (Bayombong, Nueva Vizcaya, instead of
Santiago City) of the deed of sale because the deed is merely a
confirmation of a previously agreed contract between Gregorio

and the Spouses Paragas. He likewise stated that of the stated


P500,000.00 consideration in the deed, Rudy paid Gregorio
P450,000.00 in the hospital because Rudy had previously paid
Gregorio P50,000.00. For his part, Antonio added that he was
asked by Rudy to take pictures of Gregorio signing the deed. He
also claimed that there was no entry on the date when he signed;
nor did he remember reading Santiago City as the place of
execution of the deed. He described Gregorio as still strong but
sickly, who got up from the bed with Julias help.
The lower court, after trial, rendered the decision declaring null
and void the deed of sale purportedly executed by Gregorio
Balacano in favor of the spouses Paragas, noting that at the time
Gregorio executed the deed, Gregorio was ill. Because of the
seriousness of his illness, it is not expected that Gregorio
Balacano would be negotiating a contract of sale. The lower
court also ruled that Lots 1175-E and 1175-F were Gregorios and
Lorenzas conjugal partnership properties.
The Court of Appeals affirmed the Decision of the trial court, with
the modification that Lots 1175-E and 1175-F were adjudged as
belonging to the estate of Gregorio Balacano.
ISSUE:

WON Gregorio give an intelligent consent to the sale of Lots


1175-E and 1175-F when he signed the deed of sale?

WON Deed of Sale purportedly executed between


petitioners and the late Gregorio Balacano was null and void
RULING: It is not disputed that when Gregorio signed the deed
of sale, Gregorio was seriously ill, as he in fact died a week after
the deeds signing. Gregorio died of complications caused by
cirrhosis of the liver. Gregorios death was neither sudden nor
immediate; he fought at least a month-long battle against the
disease until he succumbed to death on July 22, 1996. Given
that Gregorio purportedly executed a deed during the last stages
of his battle against his disease, the Court seriously doubt
whether Gregorio could have read, or fully understood, the
contents of the documents he signed or of the consequences of
his act. There was no conclusive evidence that the contents of
the deed were sufficiently explained to Gregorio before he affixed
his signature. The evidence the defendants-appellants offered to
prove Gregorios consent to the sale consists of the testimonies
of Atty. de Guzman and Antonio which the Court did not find
credible.
Additionally, the irregular and invalid notarization of the deed is a
falsity that raises doubts on the regularity of the transaction itself.
While the deed was indeed signed on July 18, 1996 at
Bayombong, Nueva Vizcaya, the deed states otherwise, as it
shows that the deed was executed on July 22, 1996 at Santiago
City.
Article 24 of the Civil Code tells us that in all contractual, property
or other relations, when one of the parties is at a disadvantage
on account of his moral dependence, ignorance, indigence,
mental weakness, tender age or other handicap, the courts must
be vigilant for his protection.
Gregorios consent to the sale of the lots was absent, making the
contract null and void. Consequently, the spouses Paragas could
not have made a subsequent transfer of the property to Catalino
Balacano.
COMPILED BY: WIGMORE #WIGMOREFOREVER

33

SALES Case Digest (Atty. Sarona)


Compiled by: Wigmore #wigmoreforever
time or by ratification. There are two types of void contracts:
In the case at bar, the Deed of Sale was allegedly signed by
Gregorio on his death bed in the hospital. Gregorio was an
octogenarian at the time of the alleged execution of the contract
and suffering from liver cirrhosis at that circumstances which
raise grave doubts on his physical and mental capacity to freely
consent to the contract. Adding to the dubiety of the purported
sale and further bolstering respondents claim that their uncle
Catalino, one of the children of the decedent, had a hand in the
execution of the deed is the fact that on 17 October 1996,
petitioners sold a portion of Lot 1175-E consisting of 6,416
square meters to Catalino for P60,000.00. One need not stretch
his imagination to surmise that Catalino was in cahoots with
petitioners in maneuvering the alleged sale.
FRANCISCO VS HERRERA
FACTS: Eligio Herrera, Sr., the father of respondent, was the
owner of two parcels of land, one consisting of 500 sq. m. and
another consisting of 451 sq. m. On January 3, 1991, petitioner
bought from said landowner the first parcel, for the price of
P1,000,000, paid in installments from November 30, 1990 to
August 10, 1991.On March 12, 1991, petitioner bought the
second parcel, for P750,000.
Contending that the contract price for the two parcels of land was
grossly inadequate, the children of Eligio, Sr., namely, Josefina
Cavestany, Eligio Herrera, Jr., and respondent Pastor Herrera,
tried to negotiate with petitioner to increase the purchase price.
When petitioner refused, herein respondent then filed a
complaint for annulment of sale. In his complaint, respondent
claimed ownership over the second parcel, allegedly by virtue of
a sale in his favor since 1973. He likewise claimed that the first
parcel was subject to the co-ownership of the surviving heirs of
Francisca A. Herrera, the wife of Eligio, Sr., considering that she
died intestate on April 2, 1990, before the alleged sale to
petitioner.
Finally, respondent also alleged that the sale of the two lots was
null and void on the ground that at the time of sale, Eligio, Sr.
was already incapacitated to give consent to a contract because
he was already afflicted with senile dementia, characterized by
deteriorating mental and physical condition including loss of
memory.
The Regional Trial Court declared the deed of sale null and void.
CA affirmed trial courts decision.
ISSUE: WON the assailed contracts of sale void or merely
voidable and hence capable of being ratified.
HELD: It was established that the vendor Eligio, Sr. entered into
an agreement with petitioner, but that the formers capacity to
consent was vitiated by senile dementia. Hence, the Court must
rule that the assailed contracts are not void or inexistent per
se; rather, these are contracts that are valid and binding
unless annulled through a proper action filed in court
seasonably.
A void or inexistent contract is one which has no force and effect
from the very beginning. Hence, it is as if it has never been
entered into and cannot be validated either by the passage of

I. Those where one of the essential requisites of a valid


contract as provided for by Art 1318(10) of the NCC is totally
wanting; and
II. Those declared to be so under Art 14092 (11) of the NCC. By
contrast, a voidable or annullable contract is one in which the
essential requisites for validity under Art 1318 are present,
but vitiated by want of capacity, error, violence, intimidation,
undue influence or deceit.
Article 1318 of the Civil Code states that no contract exists
unless there is a concurrence of consent of the parties, object
certain as subject matter, and cause of the obligation
established. Article 1327 provides that insane or demented
persons cannot give consent to a contract. But, if an insane or
demented person does enter into a contract, the legal effect is
that the contract is voidable or annullable as specifically provided
in Article 1390.
An annullable contract may be rendered perfectly valid by
ratification, which can be express or implied. Implied ratification
may take the form of accepting and retaining the benefits of a
contract. This is what happened in this case. Respondents
contention that he merely received payments on behalf of his
father merely to avoid their misuse and that he did not intend to
concur with the contracts is unconvincing. If he was not
agreeable with the contracts, he could have prevented petitioner
from delivering the payments, or if this was impossible, he could
have immediately instituted the action for reconveyance and
have the payments consigned with the court. None of these
happened. As found by the trial court and the Court of Appeals,
upon learning of the sale, respondent negotiated for the increase
of the purchase price while receiving the installment payments. It
was only when respondent failed to convince petitioner to
increase the price that the former instituted the complaint for
reconveyance of the properties. Clearly, respondent was
agreeable to the contracts, only he wanted to get more. Further,
there is no showing that respondent returned the payments or
made an offer to do so. This bolsters the view that indeed there
was ratification. One cannot negotiate for an increase in the price
in one breath and in the same breath contend that the contract of
sale is void.
2)

Sale by and Between Spouses

GUIANG VS CA
FACTS: Plaintiff Gilda Corpuz and defendant Judie Corpuz are
legally married spouses. The couple have three children, namely:
Junie 18 years old, Harriet 17 years of age, and Jodie or Joji, the
youngest, who was 15 years of age at the time their mother
testified in court.
Over the objection of private respondent and while she was in
Manila seeking employment, her husband sold to the petitionersspouses one half of their conjugal property, consisting of their
residence and the lot on which it stood.
Sometime on February 14, 1983, the couple Gilda and Judie
Corpuz, with plaintiff-wife Gilda Corpuz as vendee, bought a 421
COMPILED BY: WIGMORE #WIGMOREFOREVER

34

SALES Case Digest (Atty. Sarona)


Compiled by: Wigmore #wigmoreforever
sq. meter lot from Manuel Callejo who signed as vendor through
a conditional deed of sale for a total consideration of P14,735.00.
The consideration was payable in installment, with right of
cancellation in favor of vendor should vendee fail to pay three
successive installments. Sometime on April 22, 1988, the couple
Gilda and Judie Corpuz sold one-half portion of their Lot No. 9,
Block 8, to the defendants spouses Guiang. The latter have
since then occupied the one-half portion and built their house
thereon. They are thus adjoining neighbors of the Corpuzes.
Plaintiff Gilda Corpuz left for Manila sometime in June 1989. She
was trying to look for work abroad, in the Middle East.
Unfortunately, she became a victim of an unscrupulous illegal
recruiter. She was not able to go abroad. She stayed for
sometime in Manila. After his wifes departure for Manila,
defendant Judie Corpuz seldom went home to the conjugal
dwelling. Sometime in January 1990, Harriet Corpuz learned that
her father intended to sell the remaining one-half portion
including their house, of their home lot to defendants Guiangs.
She wrote a letter to her mother informing her. Gilda replied that
she was objecting to the sale. Harriet, however, did not inform
her father about this; but instead gave the letter to Mrs.
Luzviminda Guiang so that she Guiang would advise her father.
However, in the absence of his wife Gilda Corpuz, defendant
Judie Corpuz pushed through the sale of the remaining one-half
portion of Lot 9, Block 8. On March 1, 1990, he sold to defendant
Luzviminda Guiang thru a document known as Deed of Transfer
of Rights the remaining one-half portion of their lot and the house
standing thereon for a total consideration of P30,000.00 of which
P5,000.00 was to be paid in June , 1990.
Sometime on March 11, 1990, plaintiff returned home. She found
her children staying with other households. Only Junie was
staying in their house. Harriet and Joji were with Mr. Panes.
Gilda gathered her children together and stayed at their house.
Her husband was nowhere to be found. She was informed by her
children that their father had a wife already.
For staying in their house sold by her husband, plaintiff was
complained against by Guiang spouses before the Barangay
authorities for trespassing. On March 16, 1990, the parties
thereat signed a document known as amicable settlement.
Believing that she had received the shorter end of the bargain,
plaintiff went to the Barangay Captain for the annulment of the
settlement. Defendant-spouses Guiang followed thru the
amicable settlement with a motion for the execution of the
amicable settlement, filing the same with the Municipal Trial
Court.
The judgment was rendered for the plaintiff and against the
defendants. CA affirmed the trial courts decision.
ISSUE:
I. WON the contract of sale (Deed of Transfer of Rights) was
merely voidable, and
II. WON such contract was ratified by private respondent when
she entered into an amicable settlement with them
RULING:
(1) The Contract of Sale was not merely voidable but void.
The error in petitioners contention is evident. Article 1390, par. 2,
refers to contracts visited by vices of consent, i.e., contracts
which were entered into by a person whose consent was

obtained and vitiated through mistake, violence, intimidation,


undue influence or fraud. In this instance, private respondents
consent to the contract of sale of their conjugal property was
totally inexistent or absent. Gilda Corpuz, on direct examination,
testified thus:
ART. 124. The administration and enjoyment of the
conjugal partnership property shall belong to both
spouses jointly. In case of disagreement, the husbands
decision shall prevail, subject to recourse to the court
by the wife for proper remedy, which must be availed of
within five years from the date of the contract
implementing such decision.
In the event that one spouse is incapacitated or
otherwise unable to participate in the administration of
the conjugal properties, the other spouse may assume
sole powers of administration. These powers do not
include the powers of disposition or encumbrance
which must have the authority of the court or the
written consent of the other spouse. In the absence of
such authority or consent, the disposition or
encumbrance shall be void. However, the transaction
shall be construed as a continuing offer on the part of
the consenting spouse and the third person, and may
be perfected as a binding contract upon the
acceptance by the other spouse or authorization by the
court before the offer is withdrawn by either or both
offerors.
The legal provision is clear. The disposition or encumbrance is
void. It becomes still clearer if we compare the same with the
equivalent provision of the Civil Code of the Philippines. Under
Article 166 of the Civil Code, the husband cannot generally
alienate or encumber any real property of the conjugal
partnership without the wifes consent. The alienation or
encumbrance if so made however is not null and void. It is
merely voidable. The offended wife may bring an action to annul
the said alienation or encumbrance. Thus, the provision of Article
173 of the Civil Code of the Philippines, to wit:
Art. 173. The wife may, during the marriage
and within ten years from the transaction
questioned, ask the courts for the
annulment of any contract of the husband
entered into without her consent, when
such consent is required, or any act or
contract of the husband which tends to
defraud her or impair her interest in the
conjugal partnership property. Should the
wife fail to exercise this right, she or her
heirs after the dissolution of the marriage,
may demand the value of property
fraudulently alienated by the husband.(n)
This particular provision giving the wife ten (10) years during
the marriage to annul the alienation or encumbrance was not
carried over to the Family Code. It is thus clear that any
alienation or encumbrance made after August 3, 1988 when
the Family Code took effect by the husband of the conjugal
partnership property without the consent of the wife is null
and void.
In sum, the nullity of the contract of sale is premised on the
COMPILED BY: WIGMORE #WIGMOREFOREVER 35

SALES Case Digest (Atty. Sarona)


Compiled by: Wigmore #wigmoreforever
absence of private respondents consent. To constitute a valid
contract, the Civil Code requires the concurrence of the following
elements: (1) cause, (2) object, and (3) consent, the last element
being indubitably absent in the case at bar.
(2) The amicable settlement did not ratify the contract. Both
the Deed of Transfer of Rights and the amicable settlement
are null and void.
Art. 1422. A contract which is the direct result of a
previous illegal contract, is also void and
inexistent. (Civil Code of the Philippines).
The sale of a conjugal property requires the consent of both the
husband and the wife. The absence of the consent of one
renders the sale null and void, while the vitiation thereof makes it
merely voidable. Only in the latter case can ratification cure the
defect.
HEIRS OF REYES vs MIJARES
FACTS: The controversy stemmed from a dispute over Lot No.
4349-B-2, approximately 396 square meters and registered in
the name of Spouses Vicente Reyes and Ignacia Aguilar-Reyes.
Said lot and the apartments built thereon were part of the
spouses conjugal properties having been purchased using
conjugal funds from their garments business.
Vicente and Ignacia were married in 1960, but had been
separated de facto since 1974. Sometime in 1984, Ignacia
learned that on March 1, 1983, Vicente sold Lot No. 4349-B-2 to
respondent Spouses Mijares for P40,000.00. She likewise found
out that Vicente filed a petition for administration and
appointment of guardian.Vicente misrepresented therein that his
wife, Ignacia, died on March 22, 1982, and that he and their 5
minor children were her only heirs. On September 29, 1983, the
court appointed Vicente as the guardian of their minor children.
Subsequently, in its Order dated October 14, 1983, the court
authorized Vicente to sell the estate of Ignacia.
On August 9, 1984, Ignacia, through her counsel, sent a letter to
respondent spouses demanding the return of her share in the
lot. Failing to settle the matter amicably, Ignacia filed on June 4,
1996 a complaint for annulment of sale against respondent
spouses. The complaint was thereafter amended to include
Vicente Reyes as one of the defendants.
The court rendered a decision declaring the sale of Lot No. 4349B-2 void with respect to the share of Ignacia. It held that the
purchase price of the lot was P110,000.00 and ordered Vicente
to return thereof or P55,000.00 to respondent spouses.
Ignacia filed a motion for modification of the decision praying that
the sale be declared void in its entirety and that the respondents
be ordered to reimburse to her the rentals they collected on the
apartments built on Lot No. 4349-B-2 computed from March 1,
1983.
On May 31, 1990, the trial court modified its decision by
declaring the sale void in its entirety and ordering Vicente Reyes
to reimburse respondent spouses the purchase price of
P110,000.

Pending the appeal, Ignacia died and she was substituted by her
compulsory heirs. Petitioners contended that they are entitled to
reimbursement of the rentals collected on the apartment built on
Lot No. 4349-B-2, while respondent spouses claimed that they
are buyers in good faith.
Court of Appeals reversed and set aside the decision of the trial
court. It ruled that notwithstanding the absence of Ignacias
consent to the sale, the same must be held valid in favor of
respondents because they were innocent purchasers for value.
ISSUE:
(1) What is the status of the sale of Lot No. 4349-B-2 to
respondent spouses?
(2) Assuming that the sale is annullable, should it be annulled in
its entirety or only with respect to the share of Ignacia?
(3) Are respondent spouses purchasers in good faith?
RULING:
(1) The sale was voidable.
Under the regime of the Civil Code, the alienation or
encumbrance of a conjugal real property requires the consent of
the wife. The absence of such consent renders the entire
transaction merely voidable and not void. The wife may, during
the marriage and within ten years from the transaction
questioned, bring an action for the annulment of the contract
entered into by her husband without her consent.
Articles 166 and 173 of the Civil Code, the governing laws at the
time the assailed sale was contracted, provide:
Art.166. Unless the wife has been declared a non
compos mentis or a spendthrift, or is under civil
interdiction or is confined in a leprosarium, the husband
cannot alienate or encumber any real property of the
conjugal partnership without the wifes consent. If she
refuses unreasonably to give her consent, the court
may compel her to grant the same
Art. 173. The wife may, during the marriage and within
ten years from the transaction questioned, ask the
courts for the annulment of any contract of the husband
entered into without her consent, when such consent is
required, or any act or contract of the husband which
tends to defraud her or impair her interest in the
conjugal partnership property. Should the wife fail to
exercise this right, she or her heirs after the dissolution
of the marriage, may demand the value of property
fraudulently alienated by the husband.
Pursuant to the foregoing provisions, the husband could not
alienate or encumber any conjugal real property without the
consent, express or implied, of the wife otherwise, the contract is
voidable. This is consistent with Article 173 of the Civil Code
pursuant to which the wife could, during the marriage and within
10 years from the questioned transaction, seek its annulment.
In the case at bar, there is no dispute that Lot No. 4349-B-2, is a
conjugal property having been purchased using the conjugal
funds of the spouses during the subsistence of their marriage. It
is beyond cavil therefore that the sale of said lot to respondent
spouses without the knowledge and consent of Ignacia is
voidable. Her action to annul the March 1, 1983 sale which was
filed on June 4, 1986, before her demise is perfectly within the 10
COMPILED BY: WIGMORE #WIGMOREFOREVER 36

SALES Case Digest (Atty. Sarona)


Compiled by: Wigmore #wigmoreforever
year prescriptive period under Article 173 of the Civil Code. Even
if we reckon the period from November 25, 1978 which was the
date when Vicente and the respondent spouses entered into a
contract concerning Lot No. 4349-B-2, Ignacias action would still
be within the prescribed period.
(2) The trial court correctly annulled the voidable sale of Lot
No. 4349-B-2 in its entirety.
The plain meaning attached to the plain language of the law is
that the contract, in its entirety, executed by the husband without
the wife's consent, may be annulled by the wife. Had Congress
intended to limit such annulment in so far as the contract shall
"prejudice" the wife, such limitation should have been spelled out
in the statute.
To be underscored here is that upon the provisions of Articles
161, 162 and 163 of the Civil Code, the conjugal partnership is
liable for many obligations while the conjugal partnership exists.
Not only that. The conjugal property is even subject to the
payment of debts contracted by either spouse before the
marriage, as those for the payment of fines and indemnities
imposed upon them after the responsibilities in Article 161 have
been covered (Article 163, par. 3), if it turns out that the spouse
who is bound thereby, "should have no exclusive property or if it
should be insufficient." These are considerations that go beyond
the mere equitable share of the wife in the property.
(3) Spouses Mijares are not purchasers in good faith.
In the instant case, there existed circumstances that should have
placed respondent spouses on guard. The death certificate of
Ignacia, shows that she died on March 22, 1982. The same
death certificate, however, reveals that (1) it was issued by the
Office of the Civil Registrar of Lubao Pampanga on March 10,
1982; (2) the alleged death of Ignacia was reported to the Office
of the Civil Registrar on March 4, 1982; and (3) her burial or
cremation would be on March 8, 1982. These obvious flaws in
the death certificate should have prompted respondents to
investigate further, especially so that respondent Florentina
Mijares admitted on cross examination that she asked for the
death certificate of Ignacia because she was suspicious that
Ignacia was still alive.

CONCEPCION AINZA ET AL V. ANTONIO AND EUGENIA


PADUA
JUNE 30, 2005
FACTS: In April 1987, Ainza and her daughter Eugenia orally
agreed that Ainza pay P100k in exchange for half of the portion
of Eugenias undivided conjugal property (a lot located in QC).
No Deed of Absolute Sale was executed. There was physical
delivery of the land through Concepcions other daughter
(Natividad) acting as atty-in-fact. Concepcion thereafter allowed
Natividad and her husband occupy the purchased portion of the
land. In 1994, Antonio caused the division of the lot into three
(two were occupied by the spouses), necessarily displacing
Natividad. He also had each subdivision titled. Antonio requested
Natividad to vacate the premises. Antonio averred that his wife
only admitted of selling 1/3 of the property to Concepcion for
which a receipt was issued signed by Concepcion. The RTC
ruled in favor of Concepcion. The CA reversed the RTC ruling.
CA explained that the property is conjugal hence the sale should
have been with Antonios consent.
ISSUE: Whether or not the contract of sale between Ainza and
Eugenia is valid.
RULING: Yes it is valid until annulled (voidable). There was a
perfected contract of sale between Eugenia and Concepcion.
The records show that Eugenia offered to sell a portion of the
property to Concepcion, who accepted the offer and agreed to
pay P100,000.00 as consideration. The contract of sale was
consummated when both parties fully complied with their
respective obligations. Eugenia delivered the property to
Concepcion, who in turn, paid Eugenia the price of P100,000.00,
as evidenced by the receipt. Since the land was undivided when
it was sold, Concepcion is entitled to have half of it. Antonio
cannot, however, attack the validity of the sale b/n his wife and
his mom-in-law, either under the Family Code or the Old Civil
Code due to prescription. The sale came to his knowledge in
1987. He only filed the case is 1999. His right prescribed in 1993
(under the FC [5 years]) and 1997 (under OCC [10 years]).
FUENTES VS ROCA

Respondent spouses cannot deny knowledge that at the time of


the sale in 1978, Vicente was married to Ignacia and that the
latter did not give her conformity to the sale. This is so because
the 1978 "Agreement" described Vicente as "married" but the
conformity of his wife to the sale did not appear in the deed.
Obviously, the execution of another deed of sale in 1983 over the
same Lot No. 4349-B-2, after the alleged death of Ignacia on
March 22, 1982, as well as the institution of the special
proceedings were, intended to correct the absence of Ignacias
consent to the sale. Even assuming that respondent spouses
believed in good faith that Ignacia really died on March 22, 1982,
after they purchased the lot, the fact remains that the sale of Lot
No. 4349-B-2 prior to Ignacias alleged demise was without her
consent and therefore subject to annulment. The October 14,
1983 order authorizing the sale of the estate of Ignacia, could not
have validated the sale of Lot No. 4349-B-2 because said order
was issued on the assumption that Ignacia was already dead
and that the sale dated March 1, 1983 was never categorically
approved in the said order.

FACTS: Sabina Tarroza owned a titled 358-square meter lot.


She sold it to her son, Tarciano Roca. Tarciano did not meantime
have the registered title to his name. Six years later, Tarciano
offered to sell the lot to petitioners spouses Fuentes. They
arranged to meet at the office of Atty. Romulo Plagata whom they
asked to prepare the documents of sale. They later signed an
agreement to sell which agreement stated that it was to take
effect in six months.
The agreement required the Fuentes spouses to pay Tarciano a
downpayment of P60K for the transfer of the lots title to him.
Within six months, Tarciano was to secure the consent of his
estranged wife Rosario to the sale.
In working with the other requirements, Atty. Plagata said that he
went to see Rosario in one of his trips in Manila and had her sign
the affidavit of consent. As soon as Tarciano met the other
conditions, Atty. Plagata notarized Rosarios affidavit. A new title
was issued to the Fuentes spouses and the remaining payment
was given to Tarciano. Tarciano passed away, followed by his
wife.
COMPILED BY: WIGMORE #WIGMOREFOREVER 37

SALES Case Digest (Atty. Sarona)


Compiled by: Wigmore #wigmoreforever
Eight years later, in 1997, the children of Tarciano and Rosario
filed an action for annulment of sale and reconveyance on the
ground that it is void since Rosario did not give her consent to it
and her signature has been forged. The Fuentes spouses
presented Atty. Plagata who testified that he personally saw
Rosario sign the document although he admitted otarizing it only
4 months after. Besides , the 4-year prescription period for
annulling sale due to fraud or forgery already lapsed.
ISSUE:
1.
Whether Rosarios signature on the document of consent to
her husband Tarcianos sale of their conjugal land to
Fuentes spouses were forged? Yes
2.
Whether Rocas action for the declaration of nullity of that
sale to the spouses already prescribed? No, an action to
annul a void contract is imprescriptible
3.
Whether only Rosario, the wife, whose consent was not
had, could bring the action to annul that sale? No
RULING:
1. Atty. Palagata admittedly falsified the jurat of the affidavit of
consent. That jurat declared that Rosario swore to the document
signed in Zamboanga City in Jan 11, 1989 when, as he testified,
he swore she supposedly signed it about 4 months earlier at her
residence in Manila on Sept 15, 1988.
While a defective notarization will merely strip the document of
its public character and reduce it into a private instrument, that
falsified jurat, taken together with the marks of forgery in the
signature, dooms the document as proof of Rosarios consent to
the sale of the land. That the Fuentes spouses honestly relied on
the notarized affidavit as proof of Rosarios consent does not
matter. The sale is still void without an authentic consent.
2.) The law that applies to this case is the Family Code, not the
Civil Code. Although Tarciano and Rosario got married in 1950,
Tarciano sold the property in 1989, a few months after the Family
Code took effect.
Art 124. FC: In the event that one spouse is incapacitated or
otherwise unable to participate in the administration of the
conjugal properties, the other spouse may assume sole powers
of administration. These powers do not include the powers of
disposition or encumbrance which must have the authority of the
court or the written consent of the other spouse. In the absence
of such authority or consent, the disposition or encumbrance
shall be void.
Art 1410, Civil Code: The action or defense for the declaration of
the inexistence of a contract does not prescribe.
Hence, in case at bar, the passage of time did not erode the right
to bring such an action.
3.) No, the sale was void from the beginning. Consequently, the
land remained the property of Tarcianos heirs, namely, the
Rocas.

SPOUSES REX AND CONCEPCION AGGABAO versus


DIONISIO Z. PARULAN, JR. and MA. ELENA PARULAN
G.R. No. 165803
FACTS: Involved in this action are two parcels of land and their
improvements in Paraaque City and registered under the name

of Spouses Parulan, who have been estranged from one


another. Real estate broker Atanacio offered the property to
Spouses Aggabao who upon Atanacios insistence prevailed
upon them, so that they and Atanacio met with Ma. Elena
(Parulans wife) at the site of the property. During their meeting,
Spouses Aggabao paid Ma. Elena earnest money amounting to
P20,000 which she acknowledged with a handwritten receipt.
Then and there, they agreed on the terms of how the buyers will
pay the price of the property.
Spouses Aggabao complied with all the terms with regard to the
payment of the properties, but when Ma. Elena already needed
to turn over the owners duplicate copies for both lands, she was
able to turn over only one (which was successfully transferred to
the name of spouses Aggabao). For the other one, she said that
it is with a relative in HongKong but she promised to deliver it to
the spouses in a week. Needless to say, she failed to do so and
by doing their own verification, the spouses found out that said
copy of title was in the hands of Dionisios brother.
The spouses met with Dionisios brother, Atty. Parulan, who told
them that he is the one with the power to sell the property. He
demanded P800,000 for said property and gave the spouses
several days to decide. When Atty. Parulan did not hear back
from the spouses, he gave them a call, and was then informed
that they have already paid the full amount to Ma. Elena.
Subsequently, Dionisio, through Atty. Parulan, commenced an
action praying for the declaration of the nullity of the deed of
absolute sale executed by Ma. Elena, and the cancellation of the
title issued to the petitioners by virtue thereof.
ISSUE: Whether or not the sale of conjugal property made by
Ma. Elena, by presenting a special power of attorney to sell
(SPA) purportedly executed by respondent husband in her favor
was validly made to the vendees
RULING: No, the Court ruled that the sale of conjugal property
without the consent of the husband was not merely voidable but
void; hence, it could not be ratified. Spouses Aggabao also
cannot use the defense that they are buyers in good faith
because they did not exercise the necessary prudence to inquire
into the wifes authority to sell.
The relevant part of Article 124 of the Family Code provides that:
xxx In the event that one spouse is incapacitated or
otherwise unable to participate in the administration of the
conjugal properties, the other spouse may assume sole
powers of administration. These powers do not include
disposition or encumbrance without authority of the court or
the written consent of the other spouse. In the absence of
such authority or consent, the disposition or encumbrance
shall be void. xxx
Spouses Aggabao also failed to substantiate their contention that
Dionisio, while holding the administration over the property, had
delegated to his brother, Atty. Parulan, the administration of the
property, considering that they did not present in court the SPA
granting to Atty. Parulan the authority for the administration.
Nonetheless, the Court would like to stress that the power of
administration does not include acts of disposition or
encumbrance, which are acts of strict ownership. As such, an
authority to dispose cannot proceed from an authority to
COMPILED BY: WIGMORE #WIGMOREFOREVER 38

SALES Case Digest (Atty. Sarona)


Compiled by: Wigmore #wigmoreforever
administer, and vice versa, for the two powers may only be
exercised by an agent by following the provisions on agency of
the Civil Code (from Article 1876 to Article 1878). Specifically,
the apparent authority of Atty. Parulan, being a special agency,
was limited to the sale of the property in question, and did not
include or extend to the power to administer the property.
On the other hand, we agree with Dionisio that the void sale was
a continuing offer from the petitioners and Ma. Elena that
Dionisio had the option of accepting or rejecting before the offer
was withdrawn by either or both Ma. Elena and the petitioners.
The last sentence of the second paragraph of Article 124 of the
Family Code makes this clear, stating that in the absence of the
other spouses consent, the transaction should be construed as a
continuing offer on the part of the consenting spouse and the
third person, and may be perfected as a binding contract upon
the acceptance by the other spouse or upon authorization by the
court before the offer is withdrawn by either or both offerors.
PELAYO VS PEREZ
JUNE 8, 2005
FACTS: David Pelayo husband of Lorenza Pelayo executed on
Jan. 11, 1988 conveyed to Melki Perez two parcels of agricultural
rd
land. Lorenza howeer signed only on the 3 page in the space
provided for the witness on which, Perez failed to register the
deed on the registry of deeds. Perez there upon filed a case
asking specific performance of the spouse. Spouses Pelayo
responded that the cause of action of Perez is unenforceable
pursuant to RA 6656 which provides in Sec. 6 that contracts
executed prior to its effectivity shall be valid only when
registered with the Registry of Deeds within a period of 3
months. Being that Perez was unable to do the same then there
should not be any cause of action. They also said that the said
contract of sale was only to simulate a sale just so they can
intimidate the illegal occupants of the land since Perez is feared
by many. However, Perez replied that the lot was given to him by
spouses in consideration of his services as his attorney-in-fact to
make necessary representation. David Pelayo claimed as well
that the contract of sale was without his wifes consent. RTC
rendered the deed of sale null and void on the account that there
was no consent by the wife and that Perez did not possess nor
pay taxes on the lots and that defendant Pelayo was indedted to
Perez for services rendered.
ISSUE: Whether or not the deed of sale was null and void on the
ground for lack of marital consent.
RULING: Petitioners not having questioned the Decision of the
CA dated November 24, 1994 which then attained finality, the
ruling that the deed of sale subject of this case is not among the
transactions deemed as invalid under R.A. No. 6657, is now
immutable.
We agree with the CA ruling that petitioner Lorenza, by affixing
her signature to the Deed of Sale on the space provided for
witnesses, is deemed to have given her implied consent to the
contract of sale.
Moreover, under Article 173, in relation to Article 166, both of the
New Civil Code, which was still in effect on January 11, 1988
when the deed in question was executed, the lack of marital
consent to the disposition of conjugal property does not make the

contract void ab initio but merely voidable. Said provisions of law


provide:
Art. 166. Unless the wife has been declared a non compos
mentis or a spendthrift, or is under civil interdiction or is confined
in a leprosarium, the husband cannot alienate or encumber any
real property of the conjugal property without the wifes consent.
If she refuses unreasonably to give her consent, the court may
compel her to grant the same.

ABALOS vs. MACATANGAY (September 30, 2004)


FACTS: Spouses Arturo and Esther Abalos are the registered
owners of a parcel of land with improvements.
Armed with a Special Power of Attorney dated June 2, 1988,
purportedly issued by his wife, Arturo executed a Receipt and
Memorandum of Agreement (RMOA) dated October 17, 1989, in
favor of respondent, binding himself to sell to respondent the
subject property and not to offer the same to any other party
within thirty (30) days from date. Arturo acknowledged receipt of
a check from respondent in the amount of Five Thousand Pesos
(P5,000.00), representing earnest money for the subject
property, the amount of which would be deducted from the
purchase price of One Million Three Hundred Three Hundred
Thousand Pesos (P1,300,000.00). Further, the RMOA stated
that full payment would be effected as soon as possession of the
property shall have been turned over to respondent.
Subsequently, Arturos wife, Esther, executed a Special Power of
Attorney dated October 25, 1989, appointing her sister,
Bernadette Ramos, to act for and in her behalf relative to the
transfer of the property to respondent. Ostensibly, a marital
squabble was brewing between Arturo and Esther at the time
and to protect his interest, respondent caused the annotation of
his adverse claim on the title of the spouses to the property on
November 14, 1989.
Respondent sent a letter to Arturo and Esther informing them of
his readiness and willingness to pay the full amount of the
purchase price. The letter contained a demand upon the spouses
to comply with their obligation to turn over possession of the
property to him. On the same date, Esther, through her attorneyin-fact, executed in favor of respondent, a Contract to Sell the
property to the extent of her conjugal interest therein.
He reiterated his demand upon them to comply with their
obligation to turn over possession of the property. Arturo and
Esther failed to deliver the property which prompted respondent
to cause the respondent to file a complaint for specific
performance with damages against petitioners.
The court declared that the RMOA is a contract to sell because it
signifies a unilateral offer of Arturo to sell the property to
respondent for a price certain within a period of thirty days. The
RMOA does not impose upon respondent an obligation to buy
petitioners property, as in fact it does not even bear his
signature thereon. It is quite clear that after the lapse of the
thirty-day period, without respondent having exercised his option,
Arturo is free to sell the property to another. This shows that the
intent of Arturo is merely to grant respondent the privilege to buy
the property within the period therein stated. There is nothing in
the RMOA which indicates that Arturo agreed therein to transfer
COMPILED BY: WIGMORE #WIGMOREFOREVER 39

SALES Case Digest (Atty. Sarona)


Compiled by: Wigmore #wigmoreforever
ownership of the land which is an essential element in a contract
of sale.

husband or the wife to one-half of the conjugal assets does not


vest until the liquidation of the conjugal partnership.

ISSUE: Can the sale be declared valid based on the RMOA?

prior to the liquidation of the conjugal partnership, the interest of


each spouse in the conjugal assets is inchoate, a mere
expectancy. The right of the husband or wife to one-half of the
conjugal assets does not vest until the dissolution and liquidation
of the conjugal partnership, or after dissolution of the marriage,
when it is finally determined that, after settlement of conjugal
obligations, there are net assets left which can be divided
between the spouses or their respective heirs.

HELD: No. The sale would is not valid. Granting for the sake of
argument that the RMOA is a contract of sale, the same would
still be void. Quite glaring is the absence of the signature of
Esther in the RMOA, which proves that she did not give her
consent to the transaction initiated by Arturo. The husband
cannot alienate any real property of the conjugal partnership
without the wifes consent.
However, it was the Contract to Sell executed by Esther through
her attorney-in-fact which the Court of Appeals made full use of.
Holding that the contract is valid, the appellate court explained
that while Esther did not authorize Arturo to sell the property, her
execution of the SPA authorizing her sister to sell the land to
respondent clearly shows her intention to convey her interest in
favor of respondent. In effect, the court declared that the lack of
Esthers consent to the sale made by Arturo was cured by her
subsequent conveyance of her interest in the property through
her attorney-in-fact.
This ruling is erroneous.
The nullity of the RMOA as a contract of sale emanates not only
from lack of Esthers consent thereto but also from want of
consideration and absence of respondents signature thereon.
Such nullity cannot be obliterated by Esthers subsequent
confirmation of the putative transaction as expressed in the
Contract to Sell. Under the law, a void contract cannot be
ratified and the action or defense for the declaration of the
inexistence of a contract does not prescribe. A void contract
produces no effect either against or in favor of anyoneit cannot
create, modify or extinguish the juridical relation to which it
refers.
True, in the Contract to Sell, Esther made reference to the earlier
RMOA executed by Arturo in favor of respondent. However, the
RMOA which Arturo signed is different from the deed which
Esther executed through her attorney-in-fact. For one, the first is
sought to be enforced as a contract of sale while the second is
purportedly a contract to sell only. For another, the terms and
conditions as to the issuance of title and delivery of possession
are divergent.
The congruence of the wills of the spouses is essential for the
valid disposition of conjugal property. Where the conveyance is
contained in the same document which bears the conformity of
both husband and wife, there could be no question on the validity
of the transaction. But when there are two (2) documents on
which the signatures of the spouses separately appear, textual
concordance of the documents is indispensable. Hence, in this
case where the wifes putative consent to the sale of conjugal
property appears in a separate document which does not,
however, contain the same terms and conditions as in the first
document signed by the husband, a valid transaction could not
have arisen.
Even on the supposition that the parties only disposed of their
respective shares in the property, the sale, assuming that it
exists, is still void for as previously stated, the right of the

CALIMLIM- CANULLAS vs. FORTUN (June 22, 1984)


FACTS: Petitioner MERCEDES Calimlim-Canullas and
FERNANDO Canullas were married on December 19, 1962.
They begot five children. They lived in a small house on the
residential land in question located at Bacabac, Bugallon,
Pangasinan. After FERNANDO's father died in 1965,
FERNANDO inherited the land.
In 1978, FERNANDO abandoned his family and was living with
private respondent Corazon DAGUINES. During the pendency of
this appeal, they were convicted of concubinage.
On April 15, 1980, FERNANDO sold the subject property with the
house thereon to DAGUINES for the sum of P2,000.00. In the
document of sale, FERNANDO described the house as "also
inherited by me from my deceased parents."
Unable to take possession of the lot and house, DAGUINES
initiated a complaint on June 19, 1980 for quieting of title and
damages against MERCEDES. The latter resisted and claimed
that the house in dispute where she and her children were
residing, including the coconut trees on the land, were built and
planted with conjugal funds and through her industry; that the
sale of the land together with the house and improvements to
DAGUINES was null and void because they are conjugal
properties and she had not given her consent to the sale.
ISSUE: whether or not the sale of the lot together with the house
and improvements thereon was valid.
HELD: No. Not Valid. Second paragraph of Article 158 of the
Civil Code, provides:
xxx xxx xxx
Buildings constructed at the expense of the partnership during
the marriage on land belonging to one of the spouses also
pertain to the partnership, but the value of the land shall be
reimbursed to the spouse who owns the same.
We hold that pursuant to the foregoing provision both the land
and the building belong to the conjugal partnership but the
conjugal partnership is indebted to the husband for the value of
the land. The spouse owning the lot becomes a creditor of the
conjugal partnership for the value of the lot, which value would
be reimbursed at the liquidation of the conjugal partnership.
The foregoing premises considered, it follows that FERNANDO
could not have alienated the house and lot to DAGUINES since
4
MERCEDES had not given her consent to said sale.

COMPILED BY: WIGMORE #WIGMOREFOREVER

40

SALES Case Digest (Atty. Sarona)


Compiled by: Wigmore #wigmoreforever
Further, we find that the contract of sale was null and void for
being contrary to morals and public policy. The sale was made
by a husband in favor of a concubine after he had abandoned his
family and left the conjugal home where his wife and children
lived and from whence they derived their support. That sale was
subversive of the stability of the family, a basic social institution
which public policy cherishes and protects.

3) Others Relatively Disqualified (Article 1491)


a. Guardians, Agents and Administrators

THE PHILIPPINE TRUST COMPANY vs. ROLDAN (May 31,


1956)
FACTS: These 17 parcels located in Guiguinto, Bulacan, were
part of the properties inherited by Mariano L. Bernardo from his
father, Marcelo Bernardo, deceased. In view of his minority,
guardianship proceedings were instituted, wherein Socorro
Roldan was appointed his guardian.
Socorro Roldan filed in said guardianship proceedings a motion
asking for authority to sell as guardian the 17 parcels for the sum
of P14,700 to Dr. Fidel C. Ramos, the purpose of the sale being
allegedly to invest the money in a residential house, which the
minor desired to have on Tindalo Street, Manila. The motion was
granted.
As guardian, Roldan executed the proper deed of sale in favor of
her brother-in-law Dr. Fidel C. Ramos, and on August 12, 1947
she asked for, and obtained, judicial confirmation of the sale. On
August 13, 1947, Dr. Fidel C. Ramos executed in favor of
Socorro Roldan, personally, a deed of conveyance covering the
same seventeen parcels, for the sum of P15,000. And on
October 21, 1947 Socorro Roldan sold four parcels out of the
seventeen to another party, reserving to herself the right to
repurchase.
The Philippine Trust Company replaced Socorro Roldan as
guardian and seeks to undo what the previous guardian had
done. The step-mother in effect, sold to herself, the properties of
her ward, contends the Plaintiff, and the sale should be annulled
because it violates Article 1459 of the Civil Code prohibiting the
guardian from purchasing either in person or through the
mediation of another the property of her ward.
ISSUE: Whether or not the sale should be annulled.
HELD: At first glance the resolutions of both courts accomplished
substantial justice the minor recovers his properties. But if the
conveyances are annulled as prayed for, the minor will obtain a
better deal he receives all the fruits of the lands from the year
1947 (Article 1303 Civil Code) and will return P14,700, not
P15,000.
To our minds the first two transactions herein described couldnt
be in a better juridical situation than if this guardian had
purchased the seventeen parcels on the day following the sale to
Dr. Ramos. Now, if she was willing to pay P15,000 why did she
sell the parcels for less? In one day (or actually one week) the
price could not have risen so suddenly. Obviously when, seeking

approval of the sale she represented the price to be the best


obtainable in the market, she was not entirely truthful. This is one
phase to consider.
Again, supposing she knew the parcels were actually worth
P17,000 then she agreed to sell them to Dr. Ramos at P14,700
and knowing the realtys value she offered him the next day
P15,000 or P15,500, and got it. Will there be any doubt that she
was recreant to her guardianship, and that her acquisition should
be nullified? Even without proof that she had connived with Dr.
Ramos. Remembering the general doctrine that guardianship is
a trust of the highest order, and the trustee cannot be allowed to
have any inducement to neglect his wards interest and in line
with the courts suspicion whenever the guardian acquires the
wards property 1 we have no hesitation to declare that in this
case, in the eyes of the law, Socorro Roldan took by purchase
her wards parcels thru Dr. Ramos, and that Article 1459 of the
Civil Code applies.
She acted it may be true without malice there may have been no
previous agreement between her and Dr. Ramos to the effect
that the latter would buy the lands for her. But the stubborn fact
remains that she acquired her proteges properties, through her
brother-in-law. That she planned to get them for herself at the
time of selling them to Dr. Ramos, may be deduced from the very
short time between the two sales (one week). The temptation
which naturally besets a guardian so circumstanced,
necessitates the annulment of the transaction, even if no actual
collusion is proved (so hard to prove) between such guardian
and the intermediate purchaser. This would uphold a sound
principle of equity and justice.
Hence, from both the legal and equitable standpoints these three
sales should not be sustained the first two for violation of article
1459 of the Civil Code and the third because Socorro Roldan
could pass no title to the third buyer.

DISTAJO vs. COURT OF APPEALS (August 25, 2000)


FACTS: During the lifetime of Iluminada Abiertas, she
designated one of her sons, Rufo Distajo, to be the administrator
of her parcels of land denoted as Lot Nos. 1018, 1046, 1047, and
1057.
Iluminada Abiertas certified to the sale of Lot Nos. 1018, 1046
and 1047 in favor of Rufo Distajo and other parcels of land to her
other kins.
After purchasing the above-mentioned parcels of land, Rufo
Distajo, together with his wife, Lagrimas, took possession of the
property and paid the corresponding real estate taxes thereon.
Consequently, on June 5, 1986, Ricardo Distajo, with the other
heirs of Iluminada Abiertas, filed a complaint for recovery of
possession and ownership of those lands sold alleging that Rufo
Distajo cannot acquire the subject parcels of land owned by
Iluminada Abiertas because the Civil Code prohibits the
administrator
from
acquiring
properties
under
his
administration. Rufo Distajo merely employed fraudulent
machinations in order to obtain the consent of his mother to the
sale, and may have even forged her signature on the deeds of
sale of the parcels of land.
COMPILED BY: WIGMORE #WIGMOREFOREVER

41

SALES Case Digest (Atty. Sarona)


Compiled by: Wigmore #wigmoreforever
ISSUE: Whether or not the sale transactions are void for having
been entered into by the administrator of the properties.1wphi1
HELD: The sale is valid.

Don Mariano in favor of Antonio Cui on March 2,1946, wherein


the former has constituted the latter as his "true and lawful
attorney" to perform in his name and that of the intestate heirs of
Doa Antonia Perales.

The pertinent Civil Code provision provides:


"Art. 1491. The following persons cannot acquire by purchase,
even at a public or judicial auction, either in person or through
the mediation of another:
xxx
(2) Agents, the property whose administration or sale
may have been entrusted to them, unless the consent
of the principal has been given;
xxx

ISSUE: WON the sale of the property to Antonio was valid.

Under the above article, the prohibition against agents


purchasing property in their hands for sale or management is not
absolute. It does not apply if the principal consents to the sale of
the property in the hands of the agent or administrator. In this
case, the deeds of sale signed by Iluminada Abiertas shows that
she gave consent to the sale of the properties in favor of her son,
Rufo, who was the administrator of the properties. Thus, the
consent of the principal Iluminada Abiertas removes the
transaction out of the prohibition contained in Article 1491(2).

The prohibition of the law is contained in article 1459 of the


old Civil Code, but this prohibition has already been
removed.

Petitioner also alleges that Rufo Distajo employed fraudulent


machinations to obtain the consent of Iluminada Abiertas to the
sale of the parcels of land. However, petitioner failed to adduce
convincing evidence to substantiate his allegations.

JESUS MA. CUI, ET AL., Plaintiff-Appellant, v. ANTONIO MA.


CUI, ET AL., Defendants-Appellees.
[G.R. No. L-7041. February 21, 1957.]

HELD: YES. While under article 1459 of the old Civil Code an
agent or administrator is disqualified from purchasing property in
his hands for sale or management, and, in this case, the property
in question was sold to Antonio Cui while he was already the
agent or administrator of the properties of Don Mariano Cui, we
however believe that this question cannot now be raised
or invoked.

Under the provisions of article 1491, section 2, of the new Civil


Code, an agent may now buy property placed in his hands for
sale or administration, provided that the principal gives his
consent thereto. While the new Code came into effect only on
August 30, 1950, however, since this is a right that is declared for
the first time, the same may be given retroactive effect if no
vested or acquired right is impaired (Article 2253, new Civil
Code). During the lifetime Don Mariano,and particularly on March
8, 1946, the herein appellants could not claim any vested or
acquired right in these properties, for,as heirs, the most they had
was a mere expentancy. We may, therefore, invoke now this
practical and liberal provision of our new Civil Code even if the
sale had taken place before its effectivity.
b. Attorneys

FACTS: Jesus and Antonio are the legitimate children of Don


Mariano Cui and Doa Antonia Perales who died intestate
in1939. Jesus alleged that during the marriage of Don Mariano
and Dona Antonia, their parents acquired certain properties in
the City of Cebu, namely, Lots Nos. 2312, 2313 and 2319. Upon
the death of their mother, the properties were placed under the
administration of their dad.that while the latter was 84 years of
age, Antonio by means of deceit, secured the transfer to
themselves the said lots without any pecuniary consideration;
that in the deed of sale executed on March 8, 1946, Rosario Cui
appeared as one of the vendees, but on learning of this fact she
subsequently renounced her rights under the sale and returned
her portion to Don Mariano Cui by executing a deed of resale in
his favor on October 11, 1946; that defendants, fraudulently and
with the desire of enriching themselves unjustly at the expense
of their father, Don Mariano Cui, and of their brothers and coheirs,secured a loan of P130,000 from the Rehabilitation
properties, and with the loan thus obtained, defendants
constructed thereon an apartment building of strong materials
consisting of 14 doors, valued at approximately P130,000 and
another building on the same parcels of land, which buildings
were leased to some Chinese commercial firms a monthly rental
of P7,600, which defendants have collected and will continue to
collect to the prejudice of the plaintiffs;Jesus alleged that the sale
should be invalidated so far as the portion of the property sold
to Antonio Cui is concerned, for the reason that when that sale
was effected, Antonio was then acting as the agent or
administrator of the properties of Don Mariano Cui.Jesus lays
stress on the power of attorney Exhibit L which was executed by

THE DIRECTOR OF LANDS, petitioner, vs. SILVERETRA


ABABA, ET AL., claimants, JUAN LARRAZABAL, MARTA C.
DE LARRAZABAL, MAXIMO ABAROQUEZ and ANASTACIA
CABIGAS, petitioners-appellants, ALBERTO FERNANDEZ,
adverse claimant-appellee.
G.R. No. L-26096 February 27, 1979
FACTS: The adverse claimant Atty. Fernandez was retained as
counsel by petitioner (Abarquez) in a civil a case for the
annulment of a contract of sale with right of repurchase and for
the recovery of the land which was the subject matter thereof.
Unable to compensate his lawyer whom he also retained for his
appeal, the petitioner executed a document whereby he obliged
himself to give to his lawyer of whatever he might recover from
Lots 5600 and 5602 should the appeal prosper.
The real property sought to be recovered was actually the share
of petitioner in Lots 5600 and 5602 which were part of the estate
of his deceased parents and which were partitioned among the
heirs, which included petitioner and his sister.
The case having been resolved and title having been issued to
petitioner, adverse claimant waited for petitioner to comply with
his obligation under the document executed by him by delivering
the portion of the said parcels of land. Petitioner refused to
comply with his obligation and instead offered to sell the whole
parcels of land to spouses Larrazabal. Then, adverse claimant
COMPILED BY: WIGMORE #WIGMOREFOREVER

42

SALES Case Digest (Atty. Sarona)


Compiled by: Wigmore #wigmoreforever
immediately took steps to protect his interest by filing a motion to
annotate his attorneys lien and by notifying the prospective
buyers of his claim over the portion of the parcels of land.
The motion was granted. The annotation of adverse claim
appeared on the new transfer certificate of title. This adverse
claim became the subject of cancellation proceedings filed by
petitioner-spouses. The trial court resolved the case in favor of
the adverse claimant. On appeal, petitioners contended that a
contract for a contingent fee violates Article 1491 because it
involves an assignment of a property subject of litigation.
ISSUE: WON the contract for a contingent fee as basis of the
interest of Atty. Fernandez is prohibited by Article 1491 of the
Civil Code.
HELD: NO. The contention is without merit. Article 1491 prohibits
only the sale or assignment between the lawyer and his client of
property which is the subject of litigation. For the prohibition to
operate, the sale or assignment of the property must take
place during the pendency of the litigation involving the
property.
Likewise, under American Law, the prohibition does not apply to
cases where after completion of litigation the lawyer accepts on
account of his fee and interest in the assets realized by the
litigation. There is clear distinction between such cases and one
in which the lawyer speculates on the outcome of the matter in
which he is employed.
Further, a contract for a contingent fee is not covered by Article
1491 because the transfer or assignment of the property in
litigation takes effect only after the finality of a favorable
judgment. In the instant case, the attorneys fees of Atty.
Fernandez, consisting of of whatever the petitioner might
recover from his share in the lots in question is contingent upon
the success of the appeal. Hence, the payment of the attorneys
fees, that is, the transfer or assignment of of the property in
litigation will take place only if the appeal prospers. Therefore,
the transfer actually takes effect after the finality of a favorable
judgment rendered on appeal and not during the pendency of
litigation involving the property in question. Consequently, the
contract for a contingent fee is not covered by Article 1491 of the
Civil Code.
PAULINO VALENCIA vs. ATTY. ARSENIO FER CABANTING;
CONSTANCIA L. VALENCIA vs. ATTY. DIONISIO C. ANTINIW,
ATTY. EDUARDO U. JOVELLANOS and ATTY. ARSENIO FER.
CABANTING
A.M. Nos. 1302, 1391 and 1543
April 26, 1991
FACTS: In 1933, complainant Paulino Valencia and his wife
allegedly bought a parcel of land, where they built their house,
from a certain Serapia Raymundo, an heir of Pedro Raymundo
the original owner. However, they failed to register the sale or
secure a transfer certificate of title in their names. A conference
was held in the house of Atty. Eduardo Jovellanos to settle the
land dispute between Serapia and the Valencia spouses.
Serapia was willing to relinquish ownership if the Valencias could
show documents evidencing ownership. Paulino exhibited a
deed of sale written in the Ilocano dialect. However, Serapia
claimed that the deed covered a different property. Serapia,

assisted by Atty. Arsenio Fer. Cabanting, filed a complaint


against Paulino for the recovery of possession with damages.
The Valencias engaged the services of Atty. Dionisio Antiniw.
Atty. Antiniw advised them to present a notarized deed of sale in
lieu of the private document written in Ilocano. For this purpose,
Paulino gave Atty. Antiniw an amount of P200.00 to pay the
person who would falsify the signature of the alleged vendor. A
"Compraventa Definitiva" as a result thereof.
The Court of First Instance of Pangasinan, rendered a decision
in favor of Serapia. Paulino filed a Petition for Certiorari with
Preliminary Injunction before the CA. While the petition was
pending, the TC issued an order of execution stating that "the
decision in this case has already become final and executory".
On March 20, 1973, Serapia sold 40 square meters of the
litigated lot to Atty. Jovellanos and the remaining portion she sold
to her counsel, Atty. Arsenio Fer. Cabanting, on April 25, 1973.
Paulino filed a disbarment proceeding against Atty. Cabanting on
the ground that said counsel allegedly violated Article 1491 of the
New Civil Code as well as Article II of the Canons of Professional
Ethics, prohibiting the purchase of property under litigation by a
counsel. The appellate court dismissed the petition of Paulino.
Constancia Valencia, daughter of Paulino, also filed a
disbarment proceeding against Atty. Dionisio Antiniw for his
participation in the forgery and its subsequent introduction as
evidence for his client; and also, against Attys. Eduardo
Jovellanos and Arsenio Cabanting for purchasing a litigated
property allegedly in violation of Article 1491 of the New Civil
Code; and against the three lawyers, for allegedly rigging the
case against her parents.
ISSUES:
Whether or not Atty. Cabanting purchased the subject
property in violation of Art. 1491 of the New Civil Code.
Whether or not Attys. Antiniw and Jovellanos are guilty of
malpractice in falsifying notarial documents.
Whether or not the three lawyers connived in rigging the case
against spouses Valencia.
HELD: Under Article 1491 of the New Civil Code: The following
persons cannot acquire by purchase, even at a public of judicial
auction, either in person or through the mediation of another: (5)
. . . this prohibition includes the act of acquiring by assignment
and shall apply to lawyers, with respect to the property and rights
which may be the object of any litigation in which they make take
part by virtue of their profession. Public policy prohibits the
transactions in view of the fiduciary relationship involved. It is
intended to curtail any undue influence of the lawyer upon his
client. Greed may get the better of the sentiments of loyalty and
disinterestedness. Any violation of this prohibition would
constitute malpractice and is a ground for suspension. Art. 1491,
prohibiting the sale to the counsel concerned, applies only while
the litigation is pending.
In the case at bar, while it is true that Atty. Arsenio Fer.
Cabanting purchased the lot after finality of judgment, there was
still a pending certiorari proceeding. A thing is said to be in
litigation not only if there is some contest or litigation over it in
court, but also from the moment that it becomes subject to the
judicial
action
of
the
judge.
Logic
indicates,
COMPILED BY: WIGMORE #WIGMOREFOREVER

43

SALES Case Digest (Atty. Sarona)


Compiled by: Wigmore #wigmoreforever
in certiorari proceedings, that the appellate court may either
grant or dismiss the petition. Hence, it is not safe to conclude, for
purposes under Article 1491 that the litigation has terminated
when the judgment of the trial court become final while
a certiorari connected therewith is still in progress. Thus,
purchase of the property by Atty. Cabanting in this case
constitutes malpractice in violation of Art. 1491 and the Canons
of Professional Ethics. Clearly, this malpractice is a ground for
suspension. The sale in favor of Atty. Jovellanos does not
constitute malpractice. There was no attorney-client relationship
between Serapia and Atty. Jovellanos, considering that the latter
did not take part as her counsel. The transaction is not covered
by Art. 1491 nor by the Canons adverted to.
II
It is asserted by Paulino that Atty. Antiniw asked for and received
the sum of P200.00 in consideration of his executing the
document "Compraventa Definitiva". This charge, Atty. Antiniw
simply denied. It is settled jurisprudence that affirmative
testimony is given greater weight than negative testimony. When
an individual's integrity is challenged by evidence, it is not
enough that he deny the charges against him; he must meet the
issue and overcome the evidence for the relator and show proofs
that he still maintains the highest degree of morality and integrity
which at all time is expected of him. There is a clear
preponderant evidence that Atty. Antiniw committed falsification
of a deed of sale, and its subsequent introduction in court
prejudices his prime duty in the administration of justice as an
officer of the court.
III
There is no evidence on record that the three lawyers involved in
these administrative cases conspired in executing the falsified
"Compraventa Definitiva" and rigged the case against spouses
Valencia.
Besides, the camaraderie among lawyers is not proof of
conspiracy, but a sign of brotherhood among them.
WHEREFORE, judgment is hereby rendered declaring: 1.
Dionisio Antiniw DISBARRED from the practice of law, and his
name is ordered stricken off from the roll of attorneys; 2. Arsenio
Fer. Cabanting SUSPENDED; and 3. Admin case against
Attorney Eduardo Jovellanos DISMISSED.
REGALADO DAROY, complainant, vs. ATTY. ESTEBAN
ABECIA, respondent.
[A.C. No. 3046. October 26, 1998]
FACTS: (1971) Atty. Abecia was the counsel of Daroy in an
ejectment case in which they won. To satisfy the judgement of
damages, one of the properties of their opposition was sold by
the sheriff at an auction to Daroy as the highest bidder. (1984)
Daroy filed a complaint against Atty. Abecia for falsification for
allagedly forging his signature in order to transfer title to said
property first to Jose Gangay and subsequently to Atty. Abcias
wife, Nena Abecia.
He contended that he only knew of such transfer on that year.
Moreover, he filed a disbarment proceeding against Atty. Abecia
for unethical conduct.
ISSUE: WON the transfer was valid. WON the transfer of the

property to Atty. Abecia a violation of the prohibition set forth in


Art. 1491 of the NCC.
HELD: Evidence that Daroy has known of such transfer since
1971: 1. In the report of the sheriff who placed them in the actual
possession of the land he had acquired through action, the
sheriff referred to Nena Abecia as assignee of Daroy.
In a criminal action filed by the oppositors of Daroy in the
ejectment case, Nena Abecia was impleaded as co-defendant on
her capacity as assignee of Daroy. All these fact were never
rebutted by Daroy. The parties were mistaken in thinking that
respondent could not validly acquire the land. In Guevara v.
Calalang, on facts similar to those in this case, we held that the
prohibition in Art. 1491 does not apply to the sale of a parcel
of land, acquired by a client to satisfy a judgment in his
favor, to his attorney as long as the property was not the
subject of the litigation. For indeed, while judges, prosecuting
attorneys, and others connected with the administration of justice
are prohibited from acquiring property or rights in litigation or
levied upon in execution, the prohibition with respect to
attorneys in the case extends only to property and rights which
may be the object of any litigation in which they may take part by
virtue of their profession.
The point is, the parties in this case thought the transfer of the
land to respondent Abecia was prohibited and so they contrived
a way whereby the land would be sold to Jose Gangay, whose
wife Anita is the sister of Mrs. Nena Abecia, and then Gangay
would sell the land to Mrs. Abecia. As Jose Gangay stated in his
affidavit of March 6, 1985. The sale of the land to Gangay may
be fictitious and, therefore, void, but that complainant Regalado
Daroy intended to convey the land ultimately to respondent
Esteban Abecia appears to be the case.
FEDERICO N. RAMOS VS PATRICIO A. NGASEO
FACTS: Ramos went to Atty. Ngaseo to engage his services as
counsel in a case involving a piece of land. After the Court of
Appeals rendered a favorable judgment ordering the land to be
returned to Ramos and his siblings (such decision having been
final and executor), Atty. Ngaseo sent a demand letter to Ramos
asking for the delivery of a piece of land which the complainant
allegedly promised as payment for respondents appearance fee.
As a result, Ramos filed before the IBP a complaint charging Atty.
Ngaseo of violation of the CPR for demanding the delivery of a
parcel of land, which was the subject of litigation. The IBP found
Atty. Ngaseo guilty. Atty. Ngaseo argues that he did not violate
Article 1491 CC because when he demanded the delivery of the
piece of land, the case has been terminated, when the appellate
court ordered the return of the land to the family of Ramos.
ISSUE: Whether or not Atty. Ngaseo violated Art. 1491 CC.
HELD: NO. Under Par. (5), Art. 1491 of the Civil Code, lawyers
are prohibited from acquiring either by purchase or assignment
the property or rights involved which are the object of litigation in
which they intervene by virtue of their public/judicial sales. The
article provides:
Article 1491. The following persons cannot acquire by purchase,
even at a public or judicial action, either in person or through the
COMPILED BY: WIGMORE #WIGMOREFOREVER 44

SALES Case Digest (Atty. Sarona)


Compiled by: Wigmore #wigmoreforever
mediation of another: xxx xxx xxx
(5) Justices, judges, prosecuting attorneys, clerks of superior and
inferior courts, and other officers and employees connected with
the administration of justice, the property and rights in litigation or
levied upon an execution before the court within whose
jurisdiction or territory they exercise their respective functions;
this prohibition includes the act of acquiring by assignment and
shall apply to lawyers, with respect to the property and rights
which may be the object of any litigation in which they may take
part by virtue of their profession.
The prohibition in the aforesaid Article applies only to the sale or
assignment of the property which is the subject of litigation to the
persons disqualified therein. WE have already ruled that "... for
the prohibition to operate, the sale or assignment of the property
must take place during the pendency of the litigation involving
the property."
Since such prohibition applies only if the sale or assignment of
the property takes place during the pendency of the litigation
involving the clients property. Consequently, where the property
is acquired after the termination of the case, as in the instant
case, no violation of paragraph 5, article 1491 of Civil Code
attatches.
In the instant case, there was no actual acquisition of the
property in litigation since the respondent only made a written
demand for its delivery, which the complainant refused to comply.
Mere demand for delivery of the litigated property does not cause
the transfer of ownership, hence, not a prohibited transaction
within the contemplation of Article 1491.
Note: (Rationale for prohibition: Public policy disallows the
transactions in view of the fiduciary relationship involved)
c. Judges

BERNARDITA R. MACARIOLA vs. HONORABLE ELIAS B.


ASUNCION
FACTS: Civil Case No. 3010 of the Court of First Instance of
Leyte was a complaint for partition filed by Sinforosa R. Bales,
Luz R. Bakunawa, Anacorita Reyes, Ruperto Reyes, Adela
Reyes, and Priscilla Reyes, plaintiffs, against Bernardita R.
Macariola, defendant, concerning the properties left by the
deceased Francisco Reyes, the common father of the plaintiff
and defendant.
On June 8, 1963, a decision was rendered by respondent Judge
Asuncion in Civil Case 3010 which became final for lack of an
appeal, and on October 16, 1963, a project of partition was
submitted to Judge Asuncion.
One of the properties mentioned in the project of partition was
Lot 1184 and when the project of partition was approved by the
trial court the adjudicatees caused Lot 1184 to be subdivided into
five lots denominated as Lot 1184-A to 1184-E. Lot 1184-E was
sold on July 31, 1964 to Dr. Arcadio Galapon. On March 6, 1965,
Dr. Arcadio Galapon and his wife Sold a portion of Lot 1184-E to
Judge Asuncion and his wife, Victoria S. Asuncion. On August

31, 1966, spouses Asuncion and spouses Galapon conveyed


their respective shares and interest in Lot 1184-E to "The Traders
Manufacturing and Fishing Industries Inc." with Judge Asuncion
as the President and Mrs. Asuncion as the secretary.
Macariola filed on August 9, 1968 the instant complaint dated
August 6, 1968 alleging that respondent Judge Asuncion violated
Article 1491, paragraph 5, of the New Civil Code in acquiring by
purchase a portion of Lot No. 1184-E which was one of those
properties involved in Civil Case No. 3010 decided by him.
ISSUE: Whether or not the actuation of Judge Asuncion in
acquiring by purchase a portion of property in a Civil Case
previously handled by him violated the prohibition under the Civil
Code?
HELD: NO. There is no merit in the contention of Macariola that
respondent Judge Elias B. Asuncion violated Article 1491,
paragraph 5, of the New Civil Code in acquiring by purchase a
portion of Lot No. 1184-E which was one of those properties
involved in Civil Case No. 3010. 'That Article provides:
Article 1491. The following persons cannot acquire by purchase,
even at a public or judicial action, either in person or through the
mediation of another: xxx xxx xxx
(5) Justices, judges, prosecuting attorneys, clerks of superior and
inferior courts, and other officers and employees connected with
the administration of justice, the property and rights in litigation or
levied upon an execution before the court within whose
jurisdiction or territory they exercise their respective functions;
this prohibition includes the act of acquiring by assignment and
shall apply to lawyers, with respect to the property and rights
which may be the object of any litigation in which they may take
part by virtue of their profession.
The prohibition in the aforesaid Article applies only to the sale or
assignment of the property which is the subject of litigation to the
persons disqualified therein. WE have already ruled that "... for
the prohibition to operate, the sale or assignment of the property
must take place during the pendency of the litigation involving
the property."
In the case at bar, when the respondent Judge purchased on
March 6, 1965 a portion of Lot 1184-E, the decision in Civil Case
No. 3010 which he rendered on June 8, 1963 was already final
because none of the parties therein filed an appeal within the
reglementary period; hence, the lot in question was no longer
subject of the litigation. Moreover, at the time of the sale on
March 6, 1965, respondent's order dated October 23, 1963 and
the amended order dated November 11, 1963 approving the
October 16, 1963 project of partition made pursuant to the June
8, 1963 decision, had long become final for there was no appeal
from said orders.
Furthermore, respondent Judge did not buy the lot in question on
March 6, 1965 directly from the plaintiffs in Civil Case No. 3010
but from Dr. Arcadio Galapon who earlier purchased on July 31,
1964 Lot 1184-E from three of the plaintiffs, namely, Priscilla
Reyes, Adela Reyes, and Luz R. Bakunawa after the finality of
the decision in Civil Case No. 3010. It may be recalled that Lot
1184 or more specifically one-half thereof was adjudicated in
equal shares to Priscilla Reyes,
COMPILED BY: WIGMORE #WIGMOREFOREVER

45

SALES Case Digest (Atty. Sarona)


Compiled by: Wigmore #wigmoreforever
Adela Reyes, Luz Bakunawa, Ruperto Reyes and Anacorita
Reyes in the project of partition, and the same was subdivided
into five lots denominated as Lot 1184-A to 1184-E. As
aforestated, Lot 1184-E was sold on July 31, 1964 to Dr.
Galapon for which he was issued TCT No. 2338 by the Register
of Deeds of Tacloban City, and on March 6, 1965 he sold a
portion of said lot to respondent Judge and his wife who declared
the same for taxation purposes only. The subsequent sale on
August 31, 1966 by spouses Asuncion and spouses Galapon of
their respective shares and interest in said Lot 1184-E to the
Traders Manufacturing and Fishing Industries, Inc., in which
respondent was the president and his wife was the secretary,
took place long after thefinality of the decision in Civil Case No.
3010 and of the subsequent two aforesaid orders therein
approving the project of partition.
The fact remains that respondent Judge purchased on March 6,
1965 a portion of Lot 1184-E from Dr. Arcadio Galapon; hence,
after the finality of the decision which he rendered on June 8,
1963 in Civil Case No. 3010 and his two questioned orders dated
October 23, 1963 and November 11, 1963. Therefore, the
property was no longer subject of litigation.
Consequently, the sale of a portion of Lot 1184-E to respondent
Judge having taken place over one year after the finality of the
decision in Civil Case No. 3010 as well as the two orders
approving the project of partition, and not during the pendency of
the litigation, there was no violation of paragraph 5, Article 1491
of the New Civil Code.
Finally, while it is true that respondent Judge did not violate
paragraph 5, Article 1491 of the New Civil Code in acquiring by
purchase a portion of Lot 1184-E which was in litigation in his
court, it was, however, improper for him to have acquired the
same.
In conclusion, while respondent Judge Asuncion, now Associate
Justice of the Court of Appeals, did not violate any law in
acquiring by purchase a parcel of land which was in litigation in
his court and in engaging in business by joining a private
corporation during his incumbency as judge of the Court of First
Instance of Leyte, he should be reminded to be more discreet in
his private and business activities, because his conduct as a
member of the Judiciary must not only be characterized with
propriety but must always be above suspicion.

GAN TINGCO vs PABINGUIT


FACTS: Candida Acabo was the owner of six parcels of land, all
situated in the municipality of Jimalalud, Oriental Negros. These
lands were sold on June 12, 1911 by Candida Acabo, to one Gan
Tingco. But the purchaser Gan Tingco was unable to take
possession of the six parcels of land sold him by Acabo, for they
were in the possession of Silvino Pabinguit, who alleges certain
rights therein. He claims to have purchased them from Faustino
Abad. Abad had become the owner through purchase from
Henry Gardner.
Prior to the purchase made by Garnder, a judgment has been
rendered against Ancabo as a result of the complaint filed by
Silvestre Basaltos. Because of Ancabos failure to comply, her
fixtures and other chattels were levied upon the order of Gardner
being the justice of peace.

Public auction sale was executed and Gardner appeared to be


the highest bidder and was the purchaser of Candida Acabo's
lands and carabaos sold at public auction held in the barrio of
Martelo, Municipality of Tayasan on March 20, 1907.
As Gardner subsequently learned that he was forbidden to
purchase, he sold what he had purchased to Faustino Abad,
Candida Acabo's son.
On June 19, 1907, Faustino Abad, for the sum of P375 sold to
Silvino Pabinguit six parcels of land.
The Court of First Instance of Oriental NEgros rendered
judgment in behalf of the plaintiff, Gan Tingco, declaring him the
owner of the lands described in the complaint, and ordered the
defendant, Silvino Pabinguit, to restore the plaintiff to their
possession.
The defendant appealed, with the right to a review of the
evidence.
The appeal was heard by this court, it having been brought it by
bill of exceptions.
The appellant alleges that the trial court erred in holding that,
notwithstanding the sale of the lands in question at public
auction. Candida Acabo did not cease to be the owner of these
properties, because there were certain irregularities and defects
in the said auction.
ISSUE: WON Candida Acabo did not cease to be the owner of
the properties despite certain irregularities and defects in the
said auction.
HELD: The trial court was impressed by the circumstance that in
the public auction the purchaser was the justice of the peace
himself. This, in the judge's opinion, was unauthorized, because
article 1459, No. 5, of the Civil Code, prohibits judges from
acquring by purchase, even at pub;ic or judicial sale, either in
person or by an agent, any property or rights litigated in the court
in the jurisdiction or territory within which they exercise their
respective duties; this prohibition includes taking of property by
assignment.
The appellant alleges that the property purchased by justice of
the peace Gardner was not the subject of litigation in the justice
court; that the action was to recover a certain sum of money, and
that he had ordered the property sold on execution.
This raises, therefore, a question as to the true meaning of
paragraph 5 of article 1459 of the Civil Code.
The Ley de Bases, in accordance with which the Civil Code was
enacted, provides as follows, in Base No. 26:
The forms, requirements and conditions of each particular
contract shall be determined and defined subject to the general
list of obligations and their effects, with the understanding that
the legislation in force and the legal principles evolved therefrom
by judicial decisions, etc., etc., shall serve as basis.
One of the bodies of law which conastitute the legislation now in
force in the Novisima Recopilacion. In Law 4, Title 14, Book 5 of
the same is found the following provision: "We order that in
COMPILED BY: WIGMORE #WIGMOREFOREVER 46

SALES Case Digest (Atty. Sarona)


Compiled by: Wigmore #wigmoreforever
public auctions held by direction of our alcaldes, neither the latter
nor any person whomsoever in their name shall bid in anything
sold at such public auctions." The word alcaldes means judges.
The caption of Title 14 is " Alcaldes or Provincial Judges," and
the entire title deals with the exercise of judicial jurisdiction. Prior
to the enactment of the Civil Code, the Penal Code was also in
force. Article 400 of the latter prohibits, under penalty, any judge
from taking part, either directly, or indirectly, in any operation of
exchange, trade or porfit with respect to things not the product of
his own property, within the territory over which he exercises
jurisdiction. Judging from the legal precedents on which the Civil
Code is based, it would not seem too much to conclude that the
said article of the Civil Code does not make any distinction
between property in litigation. In effect, it appears to be as
delicate a matter for a judge to take part in the sale of property
that had been the subject of ligitgation in his court, as to
intervene in auction of property which, though not directly
litigated in his court, is nevertheless levied upon and sold as the
result of a writ of execution issued by him. What the law intends
to avoid is the improper interference with an interest of a judge in
a thing levied upon and sold by his order.
If under the law Gardner was prohibited from acquiring the
ownership of Acabo's lands, then he could not have transmitted
to Faustino Abad the right of ownership that he did not possess;
nor could Abad, to whom this alleged ownership had not been
transmitte, have conveyed the same to Pabinguit. What Gardner
should have done in view of the fact that the sale, as he finally
acknowledged, was void, was to claim the price that had been
deposited in court, and the justice of the peace of Guijulngan
should have declared the auction void and haveordered a new
sale to be held, besides correcting the errors that had been
committed in the proceedings. To the reasons already stated,
there is to be added the additional one, with respect to the sale
made by Faustino Abad to Silvino Pabinguit, that Abad was a
minor at the time - a circumstance that deprived him of capacity
to sell (Civil Code,art. 1263). Abad had no ownership to transmit
to anyone and, besides, he had no personality to enable him to
contract by himself, on account of his lack of legal age. This
court finds no reason whatever why it should not affirm the
judgment appealed from.

2. SUBJECT MATTER
4. Existing, Future and Contingent

LEON SIBAL vs. EMILIANO J. VALDEZ


FACTS: As a first cause of action the plaintiff alleged that the
defendant Vitaliano Mamawal, deputy sheriff of the Province of
Tarlac, by virtue of a writ of execution issued by the Court of First
Instance of Pampanga, attached and sold to the defendant
Emiliano J. Valdez the sugar cane planted by the plaintiff and his
tenants on seven parcels of land described in the complaint in
the third paragraph of the first cause of action; that within one
year from the date of the attachment and sale the plaintiff offered
to redeem said sugar cane and tendered to the defendant Valdez
the amount sufficient to cover the price paid by the latter, the
interest thereon and any assessments or taxes which he may
have paid thereon after the purchase, and the interest
corresponding thereto and that Valdez refused to accept the

money and to return the sugar cane to the plaintiff.


Plaintiff prayed that a writ of preliminary injunction be issued
against the defendant Emiliano J. Valdez his attorneys and
agents. The preliminary injunction was granted The defendant
Emiliano J. Valdez, in his amended answer, denied generally and
specifically each and every allegation of the complaint and step
up the following defenses:
(a) That the sugar cane in question had the nature of personal
property and was not, therefore, subject to redemption; The
defendant Emiliano J. Valdez filed a counter-claim.
ISSUE: Whether or not the sugar cane in question had the
nature of personal property.
HELD: The sugar cane in question had the nature of personal
property.
CIVIL CODE, JURISPRUDENCE
The first question raised by the appeal is, whether the sugar
cane in question is personal or real property. It is contended that
sugar cane comes under the classification of real property as
"ungathered products" in paragraph 2 of article 334 of the Civil
Code. Said paragraph 2 of article 334 enumerates as real
property the following: Trees, plants, and ungathered products,
while they are annexed to the land or form an integral part of any
immovable property." That article, however, has received in
recent years an interpretation by the Tribunal Supremo de
Espaa, which holds that, under certain conditions, growing
crops may be considered as personal property. (Decision of
March 18, 1904, vol. 97, Civil Jurisprudence of Spain.)
However, from the discussion of Manresa it appears (1) that,
under Spanish authorities, pending fruits and ungathered
products may be sold and transferred as personal property; (2)
that the Supreme Court of Spain, in a case of ejectment of a
lessee of an agricultural land, held that the lessee was entitled to
gather the products corresponding to the agricultural year,
because said fruits did not go with the land but belonged
separately to the lessee; and (3) that under the Spanish
Mortgage Law of 1909, as amended, the mortgage of a piece of
land does not include the fruits and products existing thereon,
unless the contract expressly provides otherwise.
An examination of the decisions of the Supreme Court of
Louisiana may give us some light on the question which we are
discussing. Article 465 of the Civil Code of Louisiana, which
corresponds to paragraph 2 of article 334 of our Civil Code,
provides:
"Standing crops and the fruits of trees not gathered, and trees
before they are cut down, are likewise immovable, and are
considered as part of the land to which they are attached."
The Supreme Court of Louisiana having occasion to interpret
that provision, held that in some cases "standing crops" may be
considered and dealt with as personal property. In the case of
Lumber Co. vs. Sheriff and Tax Collector (106 La., 418) the
Supreme Court said:
"True, by article 465 of the Civil Code it is provided that 'standing
crops and the fruits of trees not gathered and trees before they
COMPILED BY: WIGMORE #WIGMOREFOREVER 47

SALES Case Digest (Atty. Sarona)


Compiled by: Wigmore #wigmoreforever
are cut down . . . are considered as part of the land to which they
are attached, but the immovability provided for is only one in
abstracto and without reference to rights on or to the crop
acquired by others than the owners of the property to which the
crop is attached. . . .
The existence of a right on the growing crop is a mobilization by
anticipation, a gathering as it were in advance, rendering the
crop movable quoad the right acquired therein. Our
jurisprudence recognizes the possible mobilization of the growing
crop."
From an examination of the reports and codes of the State of
California and other states we find that the settle doctrine
followed in said states in connection with the attachment of
property and execution of judgment is, that growing crops raised
by yearly labor and cultivation are considered personal property.
Mr. Mechem says that a valid sale may be made of a thing,
which though not yet actually in existence, is reasonably certain
to come into existence as the natural increment or usual incident
of something already in existence, and then belonging to the
vendor, and then title will vest in the buyer the moment the thing
comes into existence.
Things of this nature are said to have a potential existence. A
man may sell property of which he is potentially and not actually
possessed. He may make a valid sale of the wine that a vineyard
is expected to produce; or the gain a field may grow in a given
time; or the milk a cow may yield during the coming year; or the
wool that shall thereafter grow upon sheep; or what may be
taken at the next cast of a fisherman's net; or fruits to grow; or
young animals not yet in existence; or the good will of a trade
and the like. The thing sold, however, must be specific and
identified. They must be also owned at the time by the vendor.
(Hull vs. Hull, 48 Conn., 250
[40 Am. Rep., 165].)
It is contended on the part of the appellee that paragraph 2 of
article 334 of the Civil Code has been modified by section 450 of
the Code of Civil Procedure as well as by Act No. 1508, the
Chattel Mortgage Law.
Said section 450 enumerates the property of a judgment debtor
which may be subjected to execution. The pertinent portion of
said section reads as follows: "All goods, chattels, moneys, and
other property, both real and personal, * * * shall be liable to
execution. Said section 450 and most of the other sections of the
Code of Civil Procedure relating to the execution of judgment
were taken from the Code of Civil Procedure of California. The
Supreme Court of California, under section 688 of the Code of
Civil Procedure of that state (Pomeroy, p. 424) has held, without
variation, that growing crops were personal property and subject
to execution.
CHATTEL MORTGAGE
Act No. 1508, the Chattel Mortgage Law, fully recognized that
growing crops are personal property. Section 2 of said Act
provides: "All personal property shall be subject to mortgage,
agreeably to the provisions of this Act, and a mortgage executed
in pursuance thereof shall be termed a chattel mortgage."
Section 7 in part provides: "If growing crops be mortgaged the

mortgage may contain an agreement stipulating that the


mortgagor binds himself properly to tend, care for and protect the
crop while growing.
It is clear from the foregoing provisions that Act No. 1508 was
enacted on the assumption that "growing crops" are personal
property. This consideration tends to support the conclusion
hereinbefore stated, that paragraph 2 of article 334 of the Civil
Code has been modified by section 450 of Act No. 190 and by
Act No. 1508 in the sense that "ungathered products" as
mentioned in said article of the Civil Code have the nature of
personal property. In other words, the phrase "personal property"
should be understood to include "ungathered products.
We may, therefore, conclude that paragraph 2 of article 334 of
the Civil Code has been modified by section 450 of the Code of
Civil Procedure and by Act No. 1508, in the sense that, for the
purpose of attachment and execution, and for the purposes of
the Chattel Mortgage Law, "ungathered products" have the
nature of personal property. The lower court, therefore,
committed no error in holding that the sugar cane in question
was personal property and, as such, was not subject to
redemption.
NOTA BENE: In Sibal v. Valdez, the Court held that pending
crops which have potential existence may be the valid subject
matter of sale, and may be dealt with separately from the land on
which they grow.
LUIS PICHEL VS PRUDENCIO ALONZO (G.R. No. L-36902
Jan. 30, 1982)
FACTS: Alonzo was awarded by the Government a parcel of
land in Basilan City in accordance with RA 477. The award was
cancelled by the Board of Liquidators on January 27, 1965 on
the ground that, previous thereto, plaintiff was proved to have
alienated the land to another, in violation of law. In 1972,
plaintiff's rights to the land were reinstated.
On August 14, 1968, Alonzo and his wife sold to Luis Pichel all
the fruits of the coconut trees which may be harvested in the land
in question for the period, September 15, 1968 to January 1,
1976, in consideration of P4,200.00. Even as of the date of sale,
however, the land was still under lease to Ramon Sua, and it was
the agreement that part of the consideration of the sale, in the
sum of P3,650.00, was to be paid by Pichel directly to Ramon
Sua so as to release the land from the clutches of the latter.
Pending said payment Alonzo refused to allow the Pichel to
make any harvest. Later, Pichel for the first time since the
execution of the deed of sale in his favor, caused the harvest of
the fruit of the coconut trees in the land.
Pichel filed for the annulment of the contract on the ground that it
violated the provisions of R.A. 477, which states that lands
awarded under the said law shall not be subject to encumbrance
or alienation, otherwise the awardee shall no longer be entitled to
apply for another piece of land. RTC ruled that although the
agreement in question is denominated by the parties as a deed
of sale of fruits of the coconut trees found in the vendor's land, it
actually is, for all legal intents and purposes, a contract of lease
of the land itself
COMPILED BY: WIGMORE #WIGMOREFOREVER

48

SALES Case Digest (Atty. Sarona)


Compiled by: Wigmore #wigmoreforever
ISSUE:
1. WON the subject matter of the sale is valid. (YES)
2. WON the sale of the coconut fruits violated RA 477. (NO)
HELD:

severed from the trees, to be used, enjoyed, sold or otherwise


disposed of by the owner of the land. Herein respondents, as the
grantee of Lot No. 21 from the Government, had the right and
prerogative to sell the coconut fruits of the trees growing on the
property.

st

1 issue: The Deed of Sale is precisely what it purports to be. It


is a document evidencing the agreement of herein parties for the
sale of coconut fruits of the lot, and not for the lease of the land
itself as found by the lower Court. In clear and express terms, the
document defines the object of the contract thus: "the herein sale
of the coconut fruits are for an the fruits on the aforementioned
parcel of land during the years ...(from) SEPTEMBER 15, 1968;
up to JANUARY 1, 1976."
Moreover, as petitioner correctly asserts, the document in
question expresses a valid contract of sale. It has the essential
elements of a contract of sale as defined under Article 1485 of
the New Civil Code which provides thus:
Art. 1458. By the contract of sale one of the contracting parties
obligates himself to transfer the ownership of and to deliver a
determinate thing, and the other to pay therefor a price certain in
money or its equivalent.A contract of sale may be absolute or
conditional.
The subject matter of the contract of sale in question are the
fruits of the coconut trees on the land during the years from
September 15, 1968 up to January 1, 1976, which subject matter
is a determinate thing. Under Article 1461 of the New Civil Code,
things having a potential existence may be the object of the
contract of sale. And in Sibal vs. Valdez, 50 Phil. 512, pending
crops which have potential existence may be the subject matter
of the sale.
According to Mechem, a valid sale may be made of a thing,
which though not yet actually in existence, is reasonably certain
to come into existence as the natural increment or usual incident
of something already in existence, and then belonging to the
vendor, and the title will vest in the buyer the moment the thing
comes into existence.
nd

2 issue: The contract was clearly a "sale of the coconut fruits."


The vendor sold, transferred and conveyed "by way of absolute
sale, all the coconut fruits of his land," thereby divesting himself
of all ownership or dominion over the fruits during the sevenyear period. The possession and enjoyment of the coconut trees
cannot be said to be the possession and enjoyment of the land
itself because these rights are distinct and separate from each
other, the first pertaining to the accessory or improvements
(coconut trees) while the second, to the principal (the land). A
transfer of the accessory or improvement is not a transfer of the
principal. It is the other way around, the accessory follows the
principal. Hence, the sale of the nuts cannot be interpreted nor
construed to be a lease of the trees, much less extended further
to include the lease of the land itself.
The grantee of a parcel of land under R.A. No. 477 is not
prohibited from alienating or disposing of the natural and/or
industrial fruits of the land awarded to him. What the law
expressly disallows is the encumbrance or alienation of the land
itself or any of the permanent improvements thereon. While
coconut trees are permanent improvements of a land, their nuts
are natural or industrial fruits which are meant to be gathered or

4. Licit

BELINDA TANEDO vs CA AND SPOUSES RICARDO AND


TERESITA TANEDO (G.R. No. 104482 January 22, 1996)
FACTS: On October 20, 1962, Lazardo Ta edo executed a
notarized deed of absolute sale in favor of his eldest brother,
Ricardo Ta edo, and the latter's wife, Teresita Barera, private
respondents herein, whereby he conveyed to the latter in
consideration of P1,500.00, "1 hectare of whatever share I shall
have over Lot No. 191 of the cadastral survey of Gerona,
Province of Tarlac and covered by Title T-13829 of the Register
of Deeds of Tarlac", the said property being his "future
inheritance" from his parents. Upon the death of his father
Matias, Lazaro executed an "Affidavit of Conformity" dated Feb
28, 1980 to "re-affirm, respect, acknowledge and validate the
sale I made in 1962."
On January 13, 1981, Lazaro executed another notarized deed
of sale in favor of private respondents covering his "undivided
1/12 of a parcel of land known as Lot 191". He acknowledged
therein his receipt of P10,000.00 as consideration therefor. In
Feb 1981, Ricardo learned that Lazaro sold the same property to
his children, petitioners herein, through a deed of sale dated Dec
29, 1980. Then, private respondents recorded the Deed of Sale
in their favor in the Registry of Deeds and the corresponding
entry was made in TCT No. 166451.
Petitioners on July 16, 1982 filed a complaint for rescission (plus
damages) of the deeds of sale executed by Lazaro in favor of
private respondents covering the property inherited by Lazaro
from his father. Petitioners claimed that their father, Lazaro,
executed an "Absolute Deed of Sale" conveying to his 10
children his allotted portion tinder the extrajudicial partition
executed by the heirs of Matias, which deed included the land in
litigation (Lot 191).
Petitioners also presented in evidence: (1) a private writing
purportedly prepared and signed by Matias stating that it was his
desire that whatever inheritance Lazaro would receive from him
should be given to his (Lazaro's) children; (2) a typewritten
document signed by Lazaro in the presence of 2 witnesses,
wherein he confirmed that he would voluntarily abide by the
wishes of his father, Matias, to give to his (Lazaro's) children all
the property he would inherit from the latter; and (3) a letter of
Lazaro to his daughter, Carmela, stating that his share in the
extrajudicial settlement of the estate of his father was intended
for his children, petitioners herein.
Private respondents presented in evidence a "Deed of
Revocation of a Deed of Sale", wherein Lazaro revoked the sale
in favor of petitioners for the reason that it was "simulated or
fictitious without any consideration whatsoever".
Lazaro executed a sworn statement which virtually repudiated
COMPILED BY: WIGMORE #WIGMOREFOREVER

49

SALES Case Digest (Atty. Sarona)


Compiled by: Wigmore #wigmoreforever
the contents of the Deed of Revocation of a Deed of Sale and
the Deed of Sale in favor of private respondents. However,
Lazaro testified that he sold the property to Ricardo, and that it
was a lawyer who induced him to execute a deed of sale in favor
of his children after giving him P5.00 to buy a "drink".
The trial court decided in favor of private respondents, holding
that petitioners failed "to adduce a preponderance of evidence to
support (their) claim." On appeal, the CA affirmed the decision of
the trial court, ruling that the Deed of Sale was valid and that its
registration in good faith vested title in said respondents.
ISSUE: Is the sale of a future inheritance valid? NO.
HELD: The Court ruled that pursuant to Article 1347 of the Civil
Code, "no contract may be entered into upon a future inheritance
except in cases expressly authorized by law." Consequently, said
contract made in 1962 is not valid and cannot be the source of
any right nor the creator of any obligation between the parties.
Hence, the "affidavit of conformity" dated Feb 28, 1980, insofar
as it sought to validate or ratify the 1962 sale, is also useless
and, in the words of the respondent Court, "suffers from the
same infirmity." Even private respondents in their memorandum
concede this.
However, the documents that are critical to the resolution of this
case are: (a) the deed of sale of January 13, 1981 in favor of
private respondents covering Lazaro's undivided inheritance of
(1/12) share in Lot No. 191, which was subsequently registered
on June 7, 1982; and (b) the deed of sale dated Dec 29, 1980 in
favor of petitioners covering the same property. These two
documents were executed after the death of Matias and after a
deed of extra-judicial settlement of his (Matias') estate was
executed, thus vesting in Lazaro actual title over said property. In
other words, these dispositions, though conflicting, were no
longer infected with the infirmities of the 1962 sale.
Petitioners contend that what was sold on Jan 13, 1981 was only
one- half hectare out of Lot No. 191, citing as authority the trial
court's decision. As earlier pointed out, what is on review in these
proceedings by this Court is the CA's decision which correctly
identified the subject matter of the Jan 13, 1981 sale to be the
entire undivided 1/12 share of Lazaro in Lot No. 191 and which is
the same property disposed of on Dec 29, 1980 in favor of
petitioners.
Article 1544 of the CC governs the preferential rights of vendees
in cases of multiple sales, as follows: Art. 1544. If the same thing
should have been sold to different vendees, the ownership shall
be transferred to the person who may have first taken
possession thereof in good faith, if it should be movable property.
Should it be immovable property, the ownership shall belong to
the person acquiring it who in good faith first recorded it in the
Registry of Property. Should there be no inscription, the
ownership shall pertain to the person who in good faith was first
in the possession; and, in the absence thereof, to the person
who presents the oldest title, provided there is good faith.
The property in question is land, an immovable, and following the
law, ownership shall belong to the buyer who in good faith
registers it first in the registry of property. Thus, although the
deed of sale in favor of private respondents was later than the

one in favor of petitioners, ownership would vest in the former


because of the undisputed fact of registration. On the other hand,
petitioners have not registered the sale to them at all.
Petitioners contend that they were in possession of the property
and that private respondents never took possession thereof. As
between two purchasers, the one who registered the sale in his
favor has a preferred right over the other who has not registered
his title, even if the latter is in actual possession of the
immovable property.
MARTINEZ VS CA (G.R. L-31271 April 29, 1974)
FACTS: The spouses Romeo Martinez and Leonor Suarez, are
the registered owners of two (2) parcels of land located in Lubao,
Pampanga. Both parcels of land are fishponds and the property
nd
involved in this case is the 2 parcel of land.
The disputed property was originally owned by one Paulino
Montemayor, who secured a "titulo real" over it way back in
1883. After the death of Paulino Montemayor the said property
passed to his successors-in-interest, Maria Montemayor and
Donata Montemayor, who in turn, sold it, as well as the first
parcel, to a certain Potenciano Garcia.
Because Potenciano Garcia was prevented by the then
municipal president of Lubao from restoring the dikes
constructed on the contested property, the former, filed with the
CFI against municipal president to restrain the latter in his official
capacity from molesting him in the possession of the lot.
The Court, by decision promulgated the preliminary injunction
and the dikes around the property in question remained closed
until a portion thereof was again opened just before the outbreak
of the Pacific War. On April 17, 1925. Potenciano Garcia applied
for the registration of both parcels of land in his name and the
CFI granted the registration. Thereafter, the ownership of these
properties changed hands until eventually they were acquired by
the herein appellee spouses.
To avoid any untoward incident, the petitioners agreed to refer
the matter to the Committee on Rivers and Streams. Said SubCommittee submitted its report, which said that the subject
property was not a public river but a private fishpond owned by
the herein spouses.
ISSUE: W/N the subject property belongs to the petitioners. (NO)
HELD: IT IS A PROPERTY OF PUBLIC DOMINION
The ruling of the Court of Appeals that the lot of the petitionersappellants is a public stream and that said title should be
cancelled and the river covered reverted to public domain, is
assailed by the petitioners-appellants as being a collateral attack
on the indefeasibility of the torrens title originally issued in 1925
in favor of the petitioners-appellants' predecessor-in-interest,
Potenciano Garcia, which is violative of the rule of res judicata. It
is argued that as the decree of registration issued by the Land
Registration Court was not re-opened through a petition for
review filed within one (1) year from the entry of the decree of
title, the certificate of title issued pursuant thereto in favor of the
appellants for the land covered thereby is no longer open to
attack under Section 38 of the Land Registration Act (Act 496)
and the jurisprudence on the matter established by this Tribunal.
COMPILED BY: WIGMORE #WIGMOREFOREVER 50

SALES Case Digest (Atty. Sarona)


Compiled by: Wigmore #wigmoreforever
Section 38 of the Land Registration Act cited by appellants
expressly makes a decree of registration, which ordinarily makes
the title absolute and indefeasible.
At the time of the enactment of Section 496, one right recognized
or existing under the law is that provided for in Article 339 of the
old Civil Code which reads as follows:
Property of public ownership is:
1. That destined to the public use, such as roads, canals, rivers,
torrents, ports, and bridges constructed by the State, and banks
shores, roadsteads, and that of a similar character. (Par. 1)
The above-mentioned properties are parts of the public domain
intended for public use, are outside the commerce of men and,
therefore, not subject to private appropriation.
A simple possession of a certificate of title under the Torrens
system does not necessarily make the possessor a true owner of
all the property described therein. If a person obtains title under
the Torrens system which includes by mistake or oversight, lands
which cannot be registered under the Torrens system, he does
not by virtue of said certificate alone become the owner of the
land illegally included.
It is useless for the appellant now to allege that she has obtained
certificate of title No. 329 in her favor because the said certificate
does not confer upon her any right to the creek in question,
inasmuch as the said creek, being of the public domain, is
included among the various exceptions enumerated in Section
39 of Act 496 to which the said certificate is subject by express
provision of the law.
Torrens certificate of title does not operate when the land
covered thereby is not capable of registration.
It is, therefore, clear that the authorities cited by the appellants
as to the conclusiveness and incontestability of a Torrens
certificate of title do not apply here. The Land Registration Court
has no jurisdiction over non-registerable properties, such as
public navigable rivers which are parts of the public domain, and
cannot validly adjudge the registration of title in favor of a private
applicant.
HEIRS OF ARTURO REYES VS SOCCO-BELTRAN (G.R. No.
176474 November 27, 2008)
FACTS: The subject property in this case is a parcel of land
originally identified as Lot No. 6-B (with an area of 360 square
meters). It was originally part of a larger parcel of land,
measuring 1,022 square metersallocated to the Spouses
Marcelo Laquian and ConstanciaSocco (Spouses Laquian).
Upon their death, they left the original parcel of landto
Constanciassiblings (Filomena, Isabel, Miguel R. Socco, and
Elena Socco-Beltran). Pursuant to an unnotarized document
entitled Extrajudicial Settlement of the Estate of the Deceased
Constancia R. Socco, executed by Constancias heirs sometime
in 1965, the parcel of land was partitioned into three lotsLot
No. 6-A, Lot No.6-B, and Lot No. 6-C. The subject property, Lot
No. 6-B, was adjudicated to respondent Elena Socco-Beltran, but
no title had been issued in her name.

So respondent Socco-Beltran filed an application for the


purchase of Lot No. 6-B before the DAR, alleging that it was
adjudicated in her favor in the extra-judicial settlement of
ConstanciaSoccos estate.
Now, petitioners, the heirs of the late Arturo Reyes, filed their
protest to respondents petition before the DAR on the ground
that the subject property was sold by respondents brother,
Miguel R. Socco, in favor of their father, Arturo Reyes, as
evidenced by the Contract to Sell, dated 5 September 1954.
Petitioners averred that they took physical possession of the
subject property in 1954 and had been uninterrupted in their
possession of the said property since then.
Investigation was conducted by the legal officer Pinlac, and in
the end, the legal officer recommended the approval of
respondents petition for issuance of title over the subject
property, ruling that respondent was qualified to own the subject
property pursuant to Article 1091 of the New Civil Code.
However, DAR Regional Director Mr. Acosta, dismissed
respondents petition for issuance of title over the subject
property on the ground that respondent was not an actual tiller
and had abandoned the said property for 40 years. It went up to
the Department Secretary, then to the OP.
Aggrieved, the petitioners went the to the Court of Appeals but it
promulgated its decision, affirming the that of the Office of the
President. It held that petitioners could not have been actual
occupants of the subject property, since actual occupancy
requires the positive act of occupying and tilling the land, not just
the introduction of an unfinished skeletal structure thereon. The
Contract to Sell on which petitioners based their claim over the
subject property was executed by Miguel Socco, who was not
the owner of the said property and, therefore, had no right to
transfer the same.
ISSUES:
1. WON title to the property was transferred to petitioners by
virtue of the Contract to Sell executed by Miguel Socco. (NO)
2. WON petitioners are in OCEAN possession of the property
since 1954 (more than 30 years). (NO)
HELD: Petitioners claim over the subject property is anchored
on the Contract to Sell executed between Miguel Socco and
Arturo Reyes. Petitioners additionally allege that they and their
predecessor-in- interest, Arturo Reyes, have been in possession
of the subject lot since 1954 for an uninterrupted period of more
than 40 years.
Petitioners cannot derive title to the subject property by virtue of
the Contract to Sell. It was unmistakably stated in the Contract
and made clear to both parties thereto that the vendor, Miguel R.
Socco, was not yet the owner of the subject property and was
merely expecting to inherit the same as his share as a co-heir of
Constancias estate. It was also declared in the Contract itself
that Miguel R. Soccos conveyance of the subject to the buyer,
Arturo Reyes, was a conditional sale. It is, therefore, apparent
that the sale of the subject property in favor of Arturo Reyes was
conditioned upon the event that Miguel Socco would actually
inherit and become the owner of the said property. Absent such
occurrence, Miguel R. Socco never acquired ownership of the
subject property which he could validly transfer to Arturo Reyes.
Under Article 1459 of the Civil Code on contracts of sale, The
COMPILED BY: WIGMORE #WIGMOREFOREVER 51

SALES Case Digest (Atty. Sarona)


Compiled by: Wigmore #wigmoreforever
thing must be licit and the vendor must have a right to
transfer ownership thereof at the time it is delivered. The
law specifically requires that the vendor must have ownership of
the property at the time it is delivered.

Ederlina went to Germany to file a divorce however Ederlina had


not been able to secure a divorce from Klaus. The latter could
charge her for bigamy and could even involve Alfred, who himself
was still married.

Petitioners claim that the property was constructively delivered to


them in 1954 by virtue of the Contract to Sell. However, as
already pointed out by this Court, it was explicit in the Contract
itself that, at the time it was executed, Miguel R. Socco was not
yet the owner of the property and was only expecting to inherit it.
Hence, there was no valid sale from which ownership of the
subject property could have transferred from Miguel Socco to
Arturo Reyes. Without acquiring ownership of the subject
property, Arturo Reyes also could not have conveyed the same
to his heirs, herein petitioners.

Alfred and Ederlinas relationship started deteriorating. They lived


separately.

Petitioners, nevertheless, insist that they physically occupied the


subject lot for more than 30 years and, thus, they gained
ownership of the property through acquisitive prescription.

In the meantime, on November 7, 1985, Alfred also filed a


complaint against Ederlina with the Regional Trial Court, Davao
City, for specific performance, declaration of ownership of real
and personal properties, sum of money, and damages.

In the case of San Miguel Corporation it was underscored that


open, continuous, exclusive, and notorious occupation of
property for more than 30 years must be no less than conclusive,
such quantum of proof being necessary to avoid the erroneous
validation of actual fictitious claims of possession over the
property that is being claimed. In the present case, the evidence
presented by the petitioners falls short of being conclusive. Apart
from their self-serving statement that they took possession of the
subject property, the only proof offered to support their claim was
a general statement made Barangay Captain Carlos Gapero,
certifying that Arturo Reyes was the occupant of the subject
property since peace time and at present.
In contrast, respondents claim over the subject property is
backed by sufficient evidence. Her predecessors-in-interest, the
spouses Laquian, have been identified as the original allocatees
who have fully paid for the subject property. The subject property
was allocated to respondent in the extrajudicial settlement by the
heirs of Constancias estate which its authenticity or legality was
never put into question. Moreover, respondent has continuously
paid for the realty tax due on the subject property, a fact which,
though not conclusive, served to strengthen her claim over the
property.

Alfred filed a Complaint dated October 28, 1985, against


Ederlina, with the Regional Trial Court of Quezon City, for
recovery of real and personal properties located in Quezon City
and Manila. Alfred alleged, inter alia, that Ederlina, without his
knowledge and consent, managed to transfer funds from their
joint account in HSBC Hong Kong, to her own account with the
same bank.

Quezon City Trial Court decided in favor of Alfred but the Davao
Trial Court is in favor of Ederlina. The trial court ruled that based
on documentary evidence, the purchaser of the three parcels of
land subject of the complaint was Ederlina. The court further
stated that even if Alfred was the buyer of the properties, he had
no cause of action against Ederlina for the recovery of the same
because as an alien, he was disqualified from acquiring and
owning lands in the Philippines. The sale of the three parcels of
land to the petitioner was null and void ab initio. Applying the pari
delicto doctrine, the petitioner was precluded from recovering the
properties from the respondent.
CA affirmed the decision of Davao City Court.
ISSUE: W/n the lower court erred in applying the in pari delicto
rule in the case at bar.
HELD: No. Section 14, Article XIV of the 1973 Constitution
provides, as follows:
Save in cases of hereditary succession, no private land shall be
transferred or conveyed except to individuals, corporations, or
associations qualified to acquire or hold lands in the public
domain.

FRENZEL V. CATITO
FACTS: Petitioner Alfred Fritz Frenzel is an Australian citizen of
German descent. He arrived in the Philippines and engaged in
businesses. After two years, he married Teresita Santos, a
Filipino citizen. In 1981, Alfred and Teresita separated from bed
and board without obtaining a divorce.
Sometime in 1983 he arrived in Sydney and met Ederlina Catito,
a Filipina and a native of Bajada, Davao City. Unknown to Alfred,
she was married to Klaus Muller when she was in Germany.
Alfred was so enamored with Ederlina that he persuaded her to
stop working, move to the Philippines and get married.
They bought several properties in Manila and Davao using the
money of Alfred. He also sold all his properties in Australia
before moving in the country. They also opened an HSBC
Savings Account in Hong Kong in the name of Ederlina.

Lands of the public domain, which include private lands, may be


transferred or conveyed only to individuals or entities qualified to
acquire or hold private lands or lands of the public domain.
Aliens, whether individuals or corporations, have been
disqualified from acquiring lands of the public domain. Hence,
they have also been disqualified from acquiring private lands.
Even if, as claimed by the petitioner, the sales in question were
entered into by him as the real vendee, the said transactions are
in violation of the Constitution; hence, are null and void ab initio.
A contract that violates the Constitution and the law, is null and
void and vests no rights and creates no obligations. It produces
no legal effect at all. The petitioner, being a party to an illegal
contract, cannot come into a court of law and ask to have his
illegal objective carried out. One who loses his money or
property by knowingly engaging in a contract or transaction
which involves his own moral turpitude may not maintain an
action for his losses. To him who moves in deliberation and
COMPILED BY: WIGMORE #WIGMOREFOREVER 52

SALES Case Digest (Atty. Sarona)


Compiled by: Wigmore #wigmoreforever
premeditation, the law is unyielding. The law will not aid either
party to an illegal contract or agreement; it leaves the parties
where it finds them.
Under Article 1412 of the New Civil Code, the petitioner cannot
have the subject properties deeded to him or allow him to
recover the money he had spent for the purchase thereof. Equity
as a rule will follow the law and will not permit that to be
done indirectly which, because of public policy, cannot be done
directly. Where the wrong of one party equals that of the other,
the defendant is in the stronger position it signifies that in such
a situation, neither a court of equity nor a court of law will
administer a remedy. The rule is expressed in the maxims: EX
DOLO MALO NON ORITUR ACTIO and IN PARI DELICTO
POTIOR EST CONDITIO DEFENDENTIS.

3. Determinate or At Least Determinable

HEIRS OF JUAN SAN ANDRES V. RODRIGUEZ


FACTS: Juan San Andres was the owner of the lot situated in
Liboton, Naga city. The sale was evidenced by a deed of sale.
Upon the death of Juan Andres, Ramon San Andres was
appointed as administrator of the estate, and hired geodetic
engineer. Jose Panero prepared a consolidated plan of the
estate and also prepared a sketch plan of the lot sold to
respondent. It was found out that respondent had enlarged the
area which he purchased from Juan. The administrator sent a
letter to the respondent to vacate the said portion in which the
latter refused to do.
Respondent alleged that apart from the original lot, which had
been sold to him, the latter likewise sold to him the following day
the remaining portion of the lot. He alleged that the payment for
such would be affected in 5 years from the execution of the
formal deed of sale after a survey is conducted. He also alleged
that under the consent of Juan, he took possession of the same
and introduced improvements thereon. Respondent deposited in
court the balance of the purchase price amounting to P7,035.00
for the aforesaid 509-square meter lot.
On September 20, 1994, the trial court rendered judgment in
favor of petitioner. It ruled that there was no contract of sale to
speak of for lack of a valid object because there was no sufficient
indication to identify the property subject of the sale, hence, the
need to execute a new contract.
Respondent appealed to the Court of Appeals, which on April 21,
1998 rendered a decision reversing the decision of the trial court.
The appellate court held that the object of the contract was
determinable, and that there was a conditional sale with the
balance of the purchase price payable within five years from the
execution of the deed of sale.
ISSUE: Whether or not there was a valid sale.
HELD: YES. The Civil Code provides that By the contract of sale
one of the contracting parties obligates himself to transfer the
ownership of and to deliver a determinate thing, and the other to
pay therefor a price certain in money or its equivalent.

A contract of sale may be absolute or conditional. As thus


defined, the essential elements of sale are the following:
a) Consent or meeting of the minds, that is, consent to transfer
ownership in exchange for the price;
b) Determinate subject matter; and
c) Price certain in money or its equivalent.
As shown in the receipt, dated September 29, 1964, the late
Juan San Andres received P500.00 from respondent as
"advance payment for the residential lot adjoining his previously
paid lot on three sides excepting on the frontage; the agreed
purchase price was P15.00 per square meter; and the full
amount of the purchase price was to be based on the results of a
survey and would be due and payable in five (5) years from the
execution of a deed of sale.
Petitioner's contention is without merit. There is no dispute that
respondent purchased a portion of Lot 1914-B-2 consisting of
345 square meters. This portion is located in the middle of Lot
1914-B-2, which has a total area of 854 square meters, and is
clearly what was referred to in the receipt as the "previously paid
lot." Since the lot subsequently sold to respondent is said to
adjoin the "previously paid lot" on three sides thereof, the subject
lot is capable of being determined without the need of any new
contract. The fact that the exact area of these adjoining
residential lots is subject to the result of a survey does not
detract from the fact that they are determinate or determinable.
As the Court of Appeals explained:
Concomitantly, the object of the sale is certain and determinate.
Under Article 1460 of the New Civil Code, a thing sold is
determinate if at the time the contract is entered into, the thing is
capable of being determinate without necessity of a new or
further agreement between the parties. Here, this definition finds
realization.
Thus, all of the essential elements of a contract of sale are
present, i.e., that there was a meeting of the minds between the
parties, by virtue of which the late Juan San Andres undertook to
transfer ownership of and to deliver a determinate thing for a
price certain in money.
As Art. 1475 of the Civil Code provides:
The contract of sale is perfected at the moment there is a
meeting of minds upon the thing which is the object of the
contract and upon the price. . . .That the contract of sale is
perfected was confirmed by the former administrator of the
estates, Ramon San Andres, who wrote a letter to respondent on
March 30, 1966 asking for P300.00 as partial payment for the
subject lot.
As the Court of Appeals observed:
Without any doubt, the receipt profoundly speaks of a meeting of
the mind between San Andres and Rodriguez for the sale.
Evidently, this is a perfected contract of sale on a deferred
payment of the purchase price. All the pre-requisite elements for
a valid purchase transaction are present.
There is a need, however, to clarify what the Court of Appeals
said is a conditional contract of sale. Apparently, the appellate
court considered as a "condition" the stipulation of the parties
that the full consideration, based on a survey of the lot, would be
due and payable within five (5) years from the execution of a
COMPILED BY: WIGMORE #WIGMOREFOREVER 53

SALES Case Digest (Atty. Sarona)


Compiled by: Wigmore #wigmoreforever
formal deed of sale. It is evident from the stipulations in the
receipt that the vendor Juan San Andres sold the residential lot in
question to respondent and undertook to transfer the ownership
thereof to respondent without any qualification, reservation or
condition.
A deed of sale is considered absolute in nature where there is
neither a stipulation in the deed that title to the property sold is
reserved in the seller until full payment of the price, nor one
giving the vendor the right to unilaterally resolve the contract the
moment the buyer fails to pay within a fixed period.
Applying these principles to this case, it cannot be gainsaid that
the contract of sale between the parties is absolute, not
conditional. There is no reservation of ownership nor a stipulation
providing for a unilateral rescission by either party. In fact, the
sale was consummated upon the delivery of the lot to
respondent.
Thus, Art. 1477 provides that the ownership of the thing sold
shall be transferred to the vendee upon the actual or constructive
delivery thereof.
The stipulation that the "payment of the full consideration based
on a survey shall be due and payable in five (5) years from the
execution of a formal deed of sale" is not a condition which
affects the efficacy of the contract of sale. It merely provides the
manner by which the full consideration is to be computed and the
time within which the same is to be paid. But it does not affect in
any manner the effectivity of the contract. Consequently, the
contention that the absence of a formal deed of sale stipulated in
the receipt prevents the happening of a sale has no merit.
The claim of petitioners that the price of P7,035.00 is iniquitous is
untenable. The amount is based on the agreement of the parties
as evidenced by the receipt (Exh. 2). Time and again, we have
stressed the rule that a contract is the law between the parties,
and courts have no choice but to enforce such contract so long
as they are not contrary to law, morals, good customs or public
policy. Otherwise, court would be interfering with the freedom of
contract of the parties. Simply put, courts cannot stipulate for the
parties nor amend the latter's agreement, for to do so would be
to alter the real intentions of the contracting parties when the
contrary function of courts is to give force and effect to the
intentions of the parties.

ATILANO V. ATILANO
FACTS: In 1916, Eulogio Atilano I acquired lot No. 535 by
purchase. In 1920, he had the land subdivided into five parts,
identified as lots Nos. 535-A, 535-B, 535-C, 535-D and 535-E,
respectively. After the subdivision had been effected, Eulogio I
executed a deed of sale covering lot No. 535-E in favor of his
brother Eulogio II. Three other portions, namely, lots Nos. 535-B,
535-C, and 535-D, were likewise sold to other persons. Eulogio I
retained for himself the remaining portions of the land,
presumably covered by the title to lot No. 535-A. upon his death,
the title to this lot passed to Ladislao, in whose name the
corresponding certificate was issued.

deed of sale was lot No. 353-A and not lot 535-E, while the land
which remained in the possession of Eulogio I, and which was
passed to Ladislao was lot No. 353-E and not lot No. 535-A.
On 1960, the heirs of Eulogio II alleging, inter alia, that they
offered to surrender to the possession of lot No. 535-A and
demanded in return the possession of lot No. 535-E, but the
defendants refused to accept the exchange. The plaintiffs'
insistence is quite understandable, since lot No. 535-E has an
area of 2,612 square meters as compared to the 1,808 squaremeter area of lot No. 535-A.
In their answer to the complaint, the defendants alleged that the
reference to lot No. 535-E in the deed of sale was an involuntary
error; that the intention of the parties to that sale was to convey
the lot correctly identified as lot No. 535-A. On the basis of the
foregoing allegations the defendants interposed a counterclaim,
praying that the plaintiffs be ordered to execute in their favor the
corresponding deed of transfer with respect to Lot No. 535-E.
The trial court rendered judgment in favor of the plaintiffs.
ISSUE: Whether or not there has been a valid sale in view of the
real intention of the parties.
HELD: YES. When one sells or buys real property a piece of
land, for example one sells or buys the property as he sees it,
in its actual setting and by its physical metes and bounds, and
not by the mere lot number assigned to it in the certificate of title.
In the instant case, the portion correctly referred to as lot No.
535-A was already in the possession of the vendee, Eulogio
Atilano II, who had constructed his residence therein, even
before the sale in his favor even before the subdivision of the
entire lot No. 535 at the instance of its owner, Eulogio Atillano I.
In like manner the latter had his house on the portion correctly
identified, after the subdivision, as lot No. 535-E, even adding to
the area thereof by purchasing a portion of an adjoining property
belonging to a different owner. The two brothers continued in
possession of the respective portions the rest of their lives,
obviously ignorant of the initial mistake in the designation of the
lot subject of the 1920 until 1959, when the mistake was
discovered for the first time.
From the facts and circumstances, the object is lot No. 535-A
and its designation as lot No. 535-E in the deed of sale was a
simple mistake in the drafting of the document. The mistake did
not vitiate the consent of the parties, or affect the validity and
binding effect of the contract between them. The new Civil Code
provides a remedy by means of reformation of the instrument.
This remedy is available when, there having been a meeting of
the minds of the parties to a contract, their true intention is not
expressed in the instrument purporting to embody the agreement
by reason of mistake, fraud, inequitable conduct or accident
In this case, the deed of sale executed in 1920 need no longer
be reformed. The parties have retained possession of their
respective properties conformably to the real intention of the
parties to that sale, and all they should do is to execute mutual
deed of conveyance.

On 1959, Eulogio II and his children had the land resurveyed so


that it could be properly subdivided. However, they discovered
that the land they were actually occupying on the strength of the
COMPILED BY: WIGMORE #WIGMOREFOREVER

54

SALES Case Digest (Atty. Sarona)


Compiled by: Wigmore #wigmoreforever
MELLIZA V. CITY OF ILOILO
FACTS: Juliana Melliza during her lifetime owned, among other
properties, 3 parcels of residential land in Iloilo City (OCT 3462).
Said parcels of land were known as Lots Nos. 2, 5 and 1214.
The total area of Lot 1214 was 29,073 sq. m.
On 27 November 1931 she donated to the then Municipality of
Iloilo, 9,000 sq. m. of Lot 1214, to serve as site for the municipal
hall. The donation was however revoked by the parties for the
reason that the area donated was found inadequate to meet the
requirements of the development plan of the municipality, the socalled Arellano Plan.
Subsequently, Lot 1214 was divided by Certeza Surveying Co.,
Inc. into Lots 1214-A and 1214-B. And still later, Lot 1214-B was
further divided into Lots 1214-B-1, Lot 1214-B-2 and Lot 1214-B3. As approved by the Bureau of Lands, Lot 1214-B-1, with 4,562
sq. m., became known as Lot 1214-B; Lot 1214-B-2, with 6,653
sq. m., was designated as Lot 1214-C; and Lot 1214-B-3, with
4,135 sq. m., became Lot 1214-D.
On 15 November 1932, Juliana Melliza executed an instrument
without any caption providing for the absolute sale involving all of
lot 5, 7669 sq. m. of Lot 2 (sublots 2-B and 2-C), and a portion of
10,788 sq. m. of Lot 1214 (sublots 1214-B2 and 1214-B3) in
favor of the Municipal Government of Iloilo for the sum of P6,422;
these lots and portions being the ones needed by the municipal
government for the construction of avenues, parks and City hall
site according the Arellano plan.
On 14 January 1938, Melliza sold her remaining interest in Lot
1214 to Remedios Sian Villanueva (thereafter TCT 18178).
Remedios in turn on 4 November 1946 transferred her rights to
said portion of land to Pio Sian Melliza (thereafter TCT 2492).
Annotated at the back of Pio Sian Mellizas title certificate was
the following that a portion of 10,788 sq. m. of Lot 1214 now
designated as Lots 1412-B-2 and 1214-B-3 of the subdivision
plan belongs to the Municipality of Iloilo as per instrument dated
15 November 1932.
On 24 August 1949 the City of Iloilo, which succeeded to the
Municipality of Iloilo, donated the city hall site together with the
building thereon, to the University of the Philippines (Iloilo
branch). The site donated consisted of Lots 1214-B, 1214-C and
1214-D, with a total area of 15,350 sq. m., more or less.
Sometime in 1952, the University of the Philippines enclosed the
site donated with a wire fence. Pio Sian Melliza thereupon made
representations, thru his lawyer, with the city authorities for
payment of the value of the lot (Lot 1214-B). No recovery was
obtained, because as alleged by Pio Sian Melliza, the City did
not have funds. The University of the Philippines, meanwhile,
obtained Transfer Certificate of Title No. 7152 covering the three
lots, Nos. 1214-B, 1214-C and 1214-D.
On 10 December 1955 Pio Sian Melliza filed an action in the CFI
Iloilo against Iloilo City and the University of the Philippines for
recovery of Lot 1214-B or of its value. After stipulation of facts
and trial, the CFI rendered its decision on 15 August 1957,
dismissing the complaint. Said court ruled that the instrument
executed by Juliana Melliza in favor of Iloilo municipality included
in the conveyance Lot 1214-B, and thus it held that Iloilo City had
the right to donate Lot 1214-B to UP.

Pio Sian Melliza appealed to the Court of Appeals. On 19 May


1965, the CA affirmed the interpretation of the CFI that the
portion of Lot 1214 sold by Juliana Melliza was not limited to the
10,788 square meters specifically mentioned but included
whatever was needed for the construction of avenues, parks and
the city hall site. Nonetheless, it ordered the remand of the case
for reception of evidence to determine the area actually taken by
Iloilo City for the construction of avenues, parks and for city hall
site.
In the present petition, Melliza maintains that only Lots No. 1214C and 1214-D were included in the sale, and that the purpose of
the second paragraph of the deed of sale was only to better
identify the lots. Melliza also argues that the interpretation given
by the lower courts would render the sale invalid for it lacks an
essential element of a sale, a determinate (or determinable)
object. Respondents, however, maintain that the object of the
sale remains determinate, as it could be ascertained what lots
were needed by the Municipality of Iloilo for the Arellano Plan at
the time of the execution of the sale.
ISSUE: W/n there was a determinate object of the sale,
rendering the sale valid.
HELD: YES. The paramount intention of the parties was
to provide Iloilo municipality with lots sufficient or adequate in
area for the construction of the Iloilo City hall site, with its
avenues and parks. For this matter, a previous donation for this
purpose between the same parties was revoked by them,
because of inadequacy of the area of the lot donated. Said
instrument described 4 parcels of land by their lot numbers and
area; and then it goes on to further describe, not only those lots
already mentioned, but the lots object of the sale, by stating that
said lots were the ones needed for the construction of the city
hall site, avenues and parks according to the Arellano plan. If the
parties intended merely to cover the specified lots (Lots 2, 5,
1214-C and 1214-D), there would scarcely have been any need
for the next paragraph, since these lots were already plainly and
very clearly described by their respective lot number and areas.
Said next paragraph does not really add to the clear description
that was already given to them in the previous one. It is therefore
the more reasonable interpretation to view it as describing those
other portions of land contiguous to the lots that, by reference to
the Arellano plan, will be found needed for the purpose at hand,
the construction of the city hall site.
The requirement of the law that a sale must have for its object a
determinate thing, is fulfilled as long as, at the time the contract
is entered into, the object of the sale is capable of being made
determinate without the necessity of a new or further agreement
between the parties (Art. 1273, old Civil Code; Art. 1460, New
Civil Code). The specific mention of some of the lots plus the
statement that the lots object of the sale are the ones needed for
city hall site; avenues and parks, according to the Arellano plan,
sufficiently provides a basis, as of the time of the execution of the
contract, for rendering determinate said lots without the need of
a new and further agreement of the parties.
NATIONAL GRAINS AUTHORITY V. IAC
FACTS: On August 23, 1979, private respondent Leon Soriano
offered to sell palay grains to NFA through William Cabal, the
provincial manager in Tuguegarao. The documents submitted
COMPILED BY: WIGMORE #WIGMOREFOREVER 55

SALES Case Digest (Atty. Sarona)


Compiled by: Wigmore #wigmoreforever
were processed, and he was given a quota of 2,640 cavans,
which is the maximum number of cavans he may sell to NFA. On
the same day and on the following day, Soriano delivered 630
cavans, which were no rebagged, classified and weighed. When
he demanded payment, he was told that payment will be held in
abeyance since Mr. Cabal was still investigating on an
information received that Soriano was not a bona fide farmer.
Instead of withdrawing the palay, Soriano insisted that the palay
grains be delivered and paid. He filed a complaint for specific
performance. Petitioners contend that the delivery was merely
made for the purpose of offering it for sale because until the
grains were rebagged, classified and weighed, they are not
considered sold.
ISSUE: Whether there was a perfected sale.
HELD: Soriano initially offered to sell palay grains produced in
his farmland to NFA. When the latter accepted the offer by noting
in Soriano's Farmer's Information Sheet a quota of 2,640 cavans,
there was already a meeting of the minds between the parties.
The object of the contract, being the palay grains produced in
Soriano's farmland and the NFA was to pay the same depending
upon its quality. The fact that the exact number of cavans of
palay to be delivered has not been determined does not affect
the perfection of the contract. Article 1349 of the New Civil Code
provides: ". . .. The fact that the quantity is not determinate shall
not be an obstacle to the existence of the contract, provided it is
possible to determine the same, without the need of a new
contract between the parties." In this case, there was no need for
NFA and Soriano to enter into a new contract to determine the
exact number of cavans of palay to be sold. Soriano can deliver
so much of his produce as long as it does not exceed 2,640
cavans. From the moment the contract of sale is perfected, it is
incumbent upon the parties to comply with their mutual
obligations or "the parties may reciprocally demand
performance" thereof.

SCHUBACK & SONS VS. CA


FACTS: On October 16, 1981, defendant submitted to plaintiff
the list of bus spare parts he wanted to purchase to its
counterpart in Hamburg. Plaintiff sent an offer on the items listed.
On December 4, 1981, defendant informed plaintiff that he
preferred genuine to replacement parts, and requested a 15%
discount. On December 17, plaintiff submitted its formal offer. On
December 24, defendant submitted a purchase order, and
submitted the quantity on December 29. Plaintiff immediately
ordered the items from Schuback Hamburg, which thereafter
ordered the same from NDK, a supplier in Germany.
Plaintiff sent a pro-forma invoice to be used in applying for letter
of credit. On February 16, 1982, plaintiff reminded defendant to
open a letter of credit to avoid delay in shipment. Defendant
mentioned the difficulty he was encountering in procuring the
same. Plaintiff continued receiving invoices and partial deliveries
from NDK. On October 18, 1982, plaintiff again reminded the
defendant to open a letter of credit. Defendant replied that he did
not make a valid purchase order and that there was no definite
contract between him and the plaintiff. Plaintiff sent a rejoinder
explaining that there is a valid Purchase
Order and suggesting that defendant either proceed with the
order and open a letter of credit or cancel the order and pay the

cancellation fee of 30% of F.O.B. value, or plaintiff will endorse


the case to its lawyers. Demand letters sent to defendant by
plaintiff's counsel dated March 22, 1983 and June 9, 1983 were
to no avail. Consequently, petitioner filed a complaint for
recovery of actual or compensatory damages, unearned profits,
interest, attorney's fees and costs against private respondent.
ISSUE: Whether or not a contract of sale has been perfected
between the parties
HELD:
Article 1319 of the Civil Code states: "Consent is manifested by
the meeting of the offer and acceptance upon the thing and the
cause which are to constitute the contract. The offer must be
certain and the acceptance absolute. A qualified acceptance
constitutes a counter offer." The facts presented to us indicate
that consent on both sides has been manifested. The offer by
petitioner was manifested on December 17, 1981 when petitioner
submitted its proposal containing the item number, quantity, part
number, description, the unit price and total to private
respondent. On December 24, 1981, private respondent
informed petitioner of his desire to avail of the prices of the parts
at that time and simultaneously enclosed its Purchase Order. At
this stage, a meeting of the minds between vendor and vendee
has occurred, the object of the contract: being the spare parts
and the consideration, the price stated in petitioner's offer dated
December 17, 1981 and accepted by the respondent on
December 24, 1981.
4. Obligation to Transfer Ownership

ALCANTARA-DAUS v. SPOUSES DE LEON


FACTS: Spouses De Leon are the owners of a parcel of land
situated in the Municipality of San Manuel, Pangasinan with an
area of Four Thousand Two Hundred Twelve square meters
more or less. Respondent Hermoso De Leon inherited the said
lot from his father Marcelino De Leon by virtue of a Deed of
Extra-Judicial Partition. Said lot is covered by Original Certificate
of Title No. 22134 of the Land Records of Pangasinan.
Sometime 1960s, Spouses De Leon engaged the services of the
late Atty. Florencio Juan to take care of the documents of their
properties. They were asked to sign voluminous documents by
the latter. After the death of Atty. Juan, some documents
surfaced and most revealed that their properties had been
conveyed by sale or quitclaim to Hermosos brothers and sisters,
to Atty. Juan and his sisters, when in truth and in fact, no such
conveyances were ever intended by them. Furthermore,
respondent found out that his signature in the Deed of Extrajudicial Partition with Quitclaim made in favor of Rodolfo de Leon
was forged. They discovered that the land in question was sold
by Rodolfo de Leon to Aurora Alcantara.
Spouses De Leon demanded the annulment of the document
and re-conveyance but defendants refused. Petitioner, Aurora
Alcantara-Daus averred that she bought the land in question in
good faith and for value on December 1975 and that she has
been in continuous, public, peaceful, open possession over the
same and has been appropriating the produce thereof without
objection from anyone.
COMPILED BY: WIGMORE #WIGMOREFOREVER

56

SALES Case Digest (Atty. Sarona)


Compiled by: Wigmore #wigmoreforever
The RTC of Urdaneta, Pangasinan rendered its Decision in favor
of herein petitioner. It ruled that respondents claim was barred
by laches, because more than 18 years had passed since the
land was sold. It further ruled that since it was a notarial
document, the Deed of Extrajudicial Partition in favor of Rodolfo
de Leon was presumptively authentic.
ISSUES:
Whether or not the Deed of Absolute executed by Rodolfo De
Leon over the land in question in favor of petitioner was
perfected and binding upon the parties therein?
Whether or not the evidentiary weight of the Deed of
Extrajudicial Partition with Quitclaim, executed by respondent
Hermoso de Leon, Perlita de Leon and Carlota de Leon in
favor of Rodolfo de Leon was overcome by more than a
preponderance of evidence of respondents?
HELD:
First Issue:
NO. It is during the delivery that the law requires the seller to
have the right to transfer ownership of the thing sold. In general,
a perfected contract of sale cannot be challenged on the ground
of the sellers non-ownership of the thing sold at the time of the
perfection of the contract.
Further, even after the contract of sale has been perfected
between the parties, its consummation by delivery is yet another
matter. It is through tradition or delivery that the buyer acquires
the real right of ownership over the thing sold.

acts of the person signing.

CONCHITA NOOL and GAUDENCIO ALMOJERA vs. CA


FACTS: One lot formerly owned by Victorio Nool has an area of
1 hectare. Another lot previously owned by Francisco Nool has
an area of 3.0880 hectares. Spouses (plaintiffs) Conchita Nool
and Gaudencio Almojera alleged that they are the owners of the
subject lands. They are in dire need of money, they obtained a
loan DBP , secured by a real estate mortgage on said parcels of
land, which were still registered in the names of Victorino and
Francisco Nool, at the time, Since the plaintiffs failed to pay the
said loan, the mortgage was foreclosed; that within the period of
redemption, the plaintiffs contacted Anacleto Nool for the latter to
redeem the foreclosed properties from DBP, which the latter did;
and as a result, the titles of the 2 parcels of land in question were
transferred to Anacleto; that as part of their arrangement or
understanding, Anacleto agreed to buy from Conchita the 2
parcels of land , for a total price of P100,000.00, P30,000.00 of
which price was paid to Conchita, and upon payment of the
balance of P14,000.00, the plaintiffs were to regain possession
of the 2 hectares of land, which amounts spouses Anacleto Nool
and Emilia Nebre failed to pay. Anacleto Nool signed the private
writing, agreeing to return subject lands when plaintiffs have the
money to redeem the same; defendant Anacleto having been
made to believe, then, that his sister, Conchita, still had the right
to redeem the said properties.
ISSUE: Is the purchase of the subject lands to Anacleto valid?

Undisputed is the fact that at the time of the sale, Rodolfo De


Leon was not the owner of the land he delivered to petitioner.
Thus, the consummation of the contract and the consequent
transfer of ownership would depend on whether he subsequently
acquired ownership of the land in accordance with Article 1434 of
the Civil Code. Therefore, we need to resolve the issue of the
authenticity and the due execution of the Extrajudicial Partition
and Quitclaim in his favor.
Second Issue:
NO. As a general rule, the due execution and authenticity of a
document must be reasonably established before it may be
admitted in evidence. Notarial documents, however, may be
presented in evidence without further proof of their authenticity,
since the certificate of acknowledgment is prima facie evidence
of the execution of the instrument or document involved. To
contradict facts in a notarial document and the presumption of
regularity in its favor, the evidence must be clear, convincing and
more than merely preponderant.
The CA ruled that the signature of Hermoso De Leon on the
Extrajudicial Partition and Quitclaim was forged. However, this
factual finding is in conflict with that of the RTC. While normally
this Court does not review factual issues, this rule does not apply
when there is a conflict between the holdings of the CA and
those of the trial court, as in the present case.
After poring over the records, the SC finds no reason to reverse
the factual finding of the appellate court. A comparison of the
genuine signatures of Hermoso De Leon with his purported
signature on the Deed of Extrajudicial Partition with Quitclaim will
readily reveal that the latter is a forgery. As aptly held by the CA,
such variance cannot be attributed to the age or the mechanical

HELD: Nono dat quod non habet, No one can give what he does
not have; Contract of repurchase inoperative thus void.
Article 1505 of the Civil Code provides that where goods are
sold by a person who is not the owner thereof, and who does not
sell them under authority or with consent of the owner, the buyer
acquires no better title to the goods than the seller had, unless
the owner of the goods is by his conduct precluded from denying
the sellers authority to sell.
Jurisprudence, on the other hand, teaches us that a person can
sell only what he owns or is authorized to sell; the buyer can as a
consequence acquire no more than what the seller can legally
transfer. No one can give what he does not have nono dat
quod non habet. In the present case, there is no allegation at all
that petitioners were authorized by DBP to sell the property to
the private respondents.
Further, the contract of repurchase that the parties entered into
presupposes that petitioners could repurchase the property that
they sold to private respondents. As petitioners sold nothing, it
follows that they can also repurchase nothing. In this light, the
contract of repurchase is also inoperative and by the same
analogy, void.
HEIRS OF SEVERINA SAN MIGUEL VS. CA
FACTS: This case involves a parcel of land originally claimed by
Severina San Miguel (petitioners predecessor-in-interest,
hereafter, Severina). The land is situated in Panapan, Bacoor,
Cavite with an area of 632 sq. m., more or less. Without
Severina's knowledge, Dominador managed to cause the
COMPILED BY: WIGMORE #WIGMOREFOREVER 57

SALES Case Digest (Atty. Sarona)


Compiled by: Wigmore #wigmoreforever
subdivision of the land into three (3) lots, to wit:
LRC Psu 1312 - with an area of 108 square meters;
LRC Psu -1313 - Lot 1, with an area of 299 square meters;
LRC Psu -1313 - Lot 2, with an area of 225 square meters.
On September 25, 1974, Dominador, et al. filed a petition with
the CFI as a land registration court, to issue title over Lots 1 and
2 of LRC Psu-1313, in their names.
On July 19, 1977, the Land Registration Commission (hereafter
LRC) rendered a decision directing the issuance of OCT in the
names of Dominador, et al. Subsequently, Severina filed with the
CFI a petition for review of the decision alleging that the land
registration proceedings were fraudulently concealed by
Dominador from her., and the court declared the OCT as null and
void. The Register of Deeds of Cavite issued a TCT in the names
of Severina and her heirs.
The trial court issued an order in favor of Severinas heirs and
ordered that the writ of possession previously issued in favor of
Severina be implemented. However, the writ was returned
unsatisfied. Subsequently, the trial court ordered that an alias
writ of demolition be issued in favor of petitioners, Severina San
Miguel. Again, the writ was not satisfied.
Severinas heirs, decided not to pursue the writs of possession
and demolition and entered into a compromise (kasunduan) with
Dominador, et al. According to the compromise, Severinas heirs
were to sell the subject lots to Dominador, et al. for P1.5 M with
the delivery of TCT conditioned upon the purchase of another lot
which was not yet titled at an additional sum of P300k. On the
same day, on August 9, 1993 pursuant to the kasunduan,
Severinas heirs and Dominador, et al. executed a deed of sale
designated as kasulatan sa bilihan ng lupa.
Dominador, et al. filed with the trial court a motion praying that
Severinas heirs deliver the owners copy of the certificate of title
to them. Severinas heirs opposed the motion stressing that
under the kasunduan, the certificate of title would only be
surrendered upon Dominador, et al's payment of the amount of
P300k within two months from August 6, 1993, which was not
complied with.
Dominador, et al. admitted non-payment of P300k for the reason
that Severinas heirs have not presented any proof of ownership
over the untitled parcel of land. Apparently, the parcel of land is
declared in the name of a third party, a certain Emiliano Eugenio.
ISSUE: Whether Dominador, et al. may be compelled to pay the
P300k as agreed upon in the kasunduan (as a pre-requisite for
the release of the certificate of title), despite Severinas heirs lack
of evidence of ownership over the parcel of land.

Although a contract is the law between the parties, the provisions


of positive law which regulate contracts are deemed written
therein and shall limit and govern the relations between the
parties. The Civil Code provisions on sales state:
Article 1458. By the contract of sale one of the contracting
parties obligates himself to transfer the ownership of and to
deliver a determinate thing, and the other to pay a price certain in
money or its equivalent. xxx
Article 1459. The thing must be licit and the vendor must have a
right to transfer the ownership thereof at the time it is delivered.
Article 1495. The vendor is bound to transfer the ownership of
and deliver, as well as warrant the thing which is the object of
sale (underscoring ours).
True, in contracts of sale, the vendor need not possess title
to the thing sold at the perfection of the contract. However,
the vendor must possess title and must be able to transfer
title at the time of delivery. In a contract of sale, title only
passes to the vendee upon full payment of the stipulated
consideration, or upon delivery of the thing sold.
Severinas heirs are not in a position to transfer title. SC noted
that there is no proof of ownership in favor of Severinas heirs. In
fact, it is a certain Emiliano Eugenio who holds a tax declaration
over the said land in his name. Though tax declarations do not
prove ownership of the property, tax declarations and receipts
can be strong evidence of ownership of land when accompanied
by possession for a period sufficient for prescription. Severinas
heirs have nothing to counter this document.
Therefore, to insist that Dominador, et al. pay the price under
such circumstances would result in Severinas heirs unjust
enrichment. The essence of a sale is the transfer of title or an
agreement to transfer it for a price actually paid or promised.
Severinas heirs insist that delivery of the certificate of title is
predicated on a condition - payment of P300k to cover the sale of
Lot 3. The condition cannot be honored. Article 1183 of the Civil
Code provides that,
Impossible conditions, those contrary to good customs or public
policy and those prohibited by law shall annul the obligation,
which depends upon them. If the obligation is divisible, that part
thereof which is not affected by the impossible or unlawful
condition shall be valid. xxx
Hence, the non-payment of the P300k is not a valid justification
for refusal to deliver the certificate of title. Besides, the certificate
of title that covers Lots 1 and 2 were fully paid for by Dominador,
et al. Therefore, Severinas heirs are bound to deliver the
certificate of title covering the lots.

HELD: No. Severinas heirs anchor their claim on the kasunduan,


stressing on their freedom to stipulate and the binding effect of
contracts. This argument is misplaced. The Civil Code provides:
Article 1306. The contracting parties may establish such
stipulations, clauses, terms and conditions as they may deem
convenient provided they are not contrary to law, morals, good
customs, public order or public policy.
It is basic that the law is deemed written into every contract.
COMPILED BY: WIGMORE #WIGMOREFOREVER

58

SALES Case Digest (Atty. Sarona)


Compiled by: Wigmore #wigmoreforever
2. PRICE
1)

Price must be Real


a.

Price Simulated

MAPALO VS. MAPALO


FACTS: The spouses Miguel Mapalo and Candida Quiba were
the registered owners of a residential land located in
Pangasinan. (1,635 sq. m.) The spouses donated the eastern
half of the land to Miguels brother Maximo Mapalo who was
about to get married.
However, they were deceived into signing, on October 15, 1936,
a deed of absolute sale over the entire land in Maximos favor.
Their signatures were procured by fraud because they were
made to believe by Maximo and the lawyer who acted as notary
public who "translated" the document, that the same was a deed
of donation in Maximo's favor covering one-half of their land. (It
must be noted that the spouses are illiterate farmers). Although
the document of sale stated a consideration of Five Hundred
(P500.00) Pesos, the aforesaid spouses did not receive anything
of value for the land.
In 1938, Maximo Mapalo, without the consent of the spouse,
registered the sale in his favor. After thirteen years (1951), he
sold the land to the Narcisos. (Evaristo, Petronila Pacifico and
Miguel) who thereafter registered the sale and obtained a title in
their favor.
In 1952, the Narcisos filed a complaint with the CFI to be
declared owners of the entire land, for possession of its western
portion; for damages; and for rentals. The Mapalo spouses filed
a counterclaim seeking cancellation of the Narcisos titles as to
the western half of the land. They said that their signatures to
the deed of sale of 1936 was procured by fraud and that the
Narcisos were buyers in bad faith.
They also filed another complaint wherein they asked the court to
declare deeds of sale of 1936 and of 1951 over the land in
question be declared null and void as to the western half of said
land.
CFI ruled in favor of the Mapalo spouses. Upon appeal filed by
Narcisos, CA reversed the lower courts ruling solely on the
ground that the consent of the Mapalo spouses to the deed of
sale of 1936 having been obtained by fraud, the same was
voidable, not void ab initio, and, therefore, the action to annul the
same, within four years from notice of the fraud, had long
prescribed. (From March 15, 1938). Hence, this appeal.
ISSUES:
1. Whether or not the deed of sale executed in 1936 was null and
void. YES
2. Whether or not the Narcisos were purchasers in good faith.
NO
HELD:
1.) YES, the sale was void. The Civil Code governs the
transaction because it was executed in 1936. Accordingly, since
the deed of sale of 1936 is governed by the Old Civil Code, it

should be asked whether its case is one wherein there is no


consideration, or one with a statement of a false consideration. If
the former, it is void and inexistent; if the latter, only voidable,
under the Old Civil Code.
There is lack of consideration
As observed earlier, the deed of sale of 1936 stated that it had
for its consideration Five Hundred (P500.00) Pesos. In fact,
however, said consideration was totally absent. The problem,
therefore, is whether a deed which states a consideration that in
fact did not exist, is a contract without consideration, and
therefore void ab initio, or a contract with a false consideration,
and therefore, at least under the Old Civil Code, voidable.
When there is no consideration, the contract is null and void
According to Manresa, what is meant by a contract that states a
false consideration is one that has in fact a real consideration but
the same is not the one stated in the document.
In our view, therefore, the ruling of this Court in Ocejo, Perez &
Co. vs. Flores, 40 Phil. 921, is squarely applicable herein. In that
case we ruled that a contract of purchase and sale is null and
void and produces no effect whatsoever where the same is
without cause or consideration in that the purchase price which
appears thereon as paid has in fact never been paid by the
purchaser to the vendor.
2.) No, they were no purchasers in good faith. Aside from the fact
that all the parties in these cases are neighbors, except Maximo
Mapalo the foregoing facts are explicit enough and sufficiently
reveal that the Narcisos were aware of the nature and extent of
the interest of Maximo Mapalo their vendor, over the abovedescribed land before and at the time the deed of sale in their
favor was executed.
The Narcisos were purchaser-in-value but not purchasers in
good faith
What was the necessity, purpose and reason of Pacifico Narciso
in still going to the spouses Mapalo and asked them to permit
their brother Maximo to dispose of the above-described land? To
this question it is safe to state that this act of Pacifico Narciso is
a conclusive manifestation that they (the Narcisos) did not only
have prior knowledge of the ownership of said spouses over the
western half portion in question but that they also have
recognized said ownership. It also conclusively shows their prior
knowledge of the want of dominion on the part of their vendor
Maximo Mapalo over the whole land and also of the flaw of his
title thereto. Under this situation, the Narcisos may be
considered purchasers in value but certainly not as purchasers in
good faith.
MODINA VS. CA
FACTS: This case involves parcels of land registered under the
name of Ramon Chiang. Chiang theorized that the subject
properties were sold to him by his wife, Merlinda Plana Chiang
as evidenced by a Deed of Sale and were subsequently sold by
Chiang to the petitioner Serafin Modina. (Dates of sale: August 3,
1979 and August 24, 1979, respectively.)
Modina brought a Complaint for Recovery of Possession with
Damages against the private respondents before the RTC. Upon
learning the institution of the said case, Merlinda presented a
COMPILED BY: WIGMORE #WIGMOREFOREVER 59

SALES Case Digest (Atty. Sarona)


Compiled by: Wigmore #wigmoreforever
Complaint-in-intervention, seeking the declaration of nullity of the
Deed of Sale between her husband and MODINA on the ground
that the titles of the parcels of land in dispute were never legally
transferred to her husband. She contended that fraudulent acts
were allegedly employed by her husband to obtain a Torrens Title
in his favor. However, she confirmed the validity of the lease
contracts with the other private respondents.
MERLINDA also admitted that the said parcels of land were
those ordered sold by the CFI of Iloilo in Intestate Estate of
Nelson Plana where she was appointed as the administratix,
being the widow of the deceased, her first husband. An Authority
to Sell was issued by the said Probate Court for the sale of the
same properties.

Since one of the characteristics of a void or inexistent contract is


that it does not produce any effect, MERLINDA can recover the
property from petitioner who never acquired title thereover.
Records show that in the complaint-in-intervention of
MERLINDA, she did not aver the same as a ground to nullify
subject Deed of Sale. In fact, she denied the existence of the
Deed of Sale in favor of her husband. In the said Complaint, her
allegations referred to the want of consideration of such Deed of
Sale. She did not put up the defense under Article 1490, to
nullify her sale to her husband CHIANG because such a defense
would be inconsistent with her claim that the same sale was
inexistent.
2.) Modina was not a purchaser in good faith

RTC ruled in favor of the wife Merlinda declaring the two sales in
August 1979 as void and inexistent. Upon appeal, the CA
affirmed in toto the RTC ruling.
ISSUES:
1. Whether or not the sale of subject lots should be nullified. YES
2. Whether or not petitioner Modina was a purchaser in good
faith. NO
HELD:
1.)The sale of the subject lots should be nullified.
Prohibition of sale between spouses
Art. 1490. The husband and the wife cannot sell property to
each other, except:
(1) when a separation of property was agreed upon in the
marriage settlements; or
(2) when there has been a judicial separation of property under
Art. 191.
The sale between Chiang spouses was null and void. The
ownership of the lot did not transfer to Ramon Chiang. Hence,
the sale to Modina was null and void. The exception to the rule
laid down in Art. 1490 of the New Civil Code not having existed
with respect to the property relations of Ramon Chiang and
Merlinda Plana Chiang, the sale by the latter in favor of the
former of the properties in question is invalid for being prohibited
by law. Not being the owner of subject properties, Ramon
Chiang could not have validly sold the same to plaintiff Serafin
Modina. The sale by Ramon Chiang in favor of Serafin Modina
is, likewise, void and inexistent. Serafin Modina is, likewise, void
and inexistent.
A contract of sale without consideration is a void contract
Under Article 1409 of the New Civil Code, enumerating void
contracts, a contract without consideration is one such void
contract. One of the characteristics of a void or inexistent
contract is that it produces no effect. So also, inexistent
contracts can be invoked by any person whenever juridical
effects founded thereon are asserted against him. A transferor
can recover the object of such contract by accion reivindicatoria
and any possessor may refuse to deliver it to the transferee, who
cannot enforce the transfer.
Thus, Modinas insistence that Merlinda cannot attack subject
contract of sale as she was a guilty party thereto is equally
unavailing.
Merlinda can recover the property

There are circumstances which are indicia of bad faith on


Mondinas part:
(1) He asked his nephew, Placido Matta, to investigate the origin
of the property and the latter learned that the same formed part
of the properties of MERLINDAs first husband;
(2) that the said sale was between the spouses;
(3) that when the property was inspected, MODINA met all the
lessees who informed that subject lands belong to MERLINDA
and they had no knowledge that the same lots were sold to the
husband.
It is a well-settled rule that a purchaser cannot close his eyes to
facts which would put a reasonable man upon his guard to make
the necessary inquiries, and then claim that he acted in good
faith. His mere refusal to believe that such defect exists, or his
wilful closing of his eyes to the possibility of the existence of a
defect in his vendors title, will not make him an innocent
purchaser for value, if it afterwards develops that the title was in
fact defective, and it appears that he had such notice of the
defect as would have led to its discovery had he acted with that
measure of precaution which may reasonably be required of a
prudent man in a like situation.
VDA. DE CATINDIG VS. HEIRS OF ROQUE
FACTS: The subject property in this case is a fishpond which
was part of the Malolos Cadastre and has an area of more than
thirteen hectares. As shown in Original Certificate of Title, it is coowned or registered in the names of the different persons. (note:
there are 16/16 shares)
The co-owners of the fishpond leased it to Mrs. Catindig for a
term of ten years counted from October 1, 1941 for a total rental
of six thousand pesos. After the termination of the lease on
September 30, 1951, Mrs. Catindig remained in possession of
the fishpond because she was negotiating with the co-owners for
the purchase thereof. She wanted to buy it for P52,000.
On October 18, 1960 German Ramirez, one of the co-owners,
executed a deed wherein he sold his 2/16 share to Mrs. Catindig
for P6,500 The sale was annotated on the title on October 19,
1960. Two weeks later, Pedro Villanueva, one of the co-owners,
learned of the sale executed by German Ramirez. That sale
retroacted to April 13, 1950. In 1960 the respondents filed this
action against Mrs. Catindig to compel her to allow them to
redeem the portion sold by German Ramirez. The respondents
COMPILED BY: WIGMORE #WIGMOREFOREVER 60

SALES Case Digest (Atty. Sarona)


Compiled by: Wigmore #wigmoreforever
amended their complaint by including a prayer for the recovery
of the possession of the fishpond.
The RTC declared void certain documents of sale regarding
portions of the fishpond in litigation. It ordered Mrs. Catindig to
deliver to the respondents (except German Ramirez) the
possession of the said fishpond and to allow the respondents to
redeem from Mrs. Catindig the 2/16 portion of the fishpond which
German Ramirez had sold to her. CA affirmed in toto the RTC
ruling. CA said that Mrs. Catindig did not pay P52,000 (the
projected sale) and that it the contract was simulated. Hence,
this appeal.
ISSUE: Whether or not the sale by German Ramizer to Mrs.
Catindig was null and void.
HELD: YES. The alleged sales were null and void. The
conclusive factual finding of the Appellate Court that the alleged
sales on April 13 or 14, 1950 of respondents' shares are
simulated and void ab initio renders untenable appellant
Catindig's contentions that the remedies available to the
respondents, such as an action for annulment, rescission or
reformation, are barred by prescription or laches.
The alleged sales were absolutely simulated, fictitious or
inexistent contracts (Arts. 1346 and 1409(2)). "The action or
defense for the declaration of the inexistence of a contract does
not prescribe" (Art. 1410). Mere lapse of time cannot give
efficacy to a void contract.
The CAs finding that the price was not paid or that the statement
in the supposed contracts of sale as to the payment of the price
was simulated fortifies the view that the alleged sales were void.
"If the price is simulated, the sale is void..." (Art. 1471, Civil
Code).
A contract of sale with no consideration is void
A contract of sale is void and produces no effect whatsoever
where the price, which appears thereon as paid, has in fact
never been paid by the purchaser to the vendor. Such a sale is
non-existent or cannot be considered consummated.
Mrs. Catindig cannot demand
Mrs. Catindig is not entitled to demand the execution of a
notarized deed of sale for the 14/16 pro indiviso portion of the
fishpond. She is not entitled because, as already held, the
alleged sales in her favor are void.
Reasonable value of the use and occupation of the fishpond
should be limited
We hold that, as a matter of fairness and equity or to avoid unjust
enrichment, the liability of Mrs. Catindig for the reasonable value
of the use and occupation of the fishpond should be limited to the
period from October 1, 1951 up to the time in January, 1964
when she turned over the fishpond to the receiver, namely, the
deputy clerk of court of the Court of First Instance of Bulacan,
Malolos Branch I.
From the compensation of P6,000 per annum which Mrs.
Catindig is obligated to pay to the respondents, should be
deducted the 2/16 portion of said compensation, corresponding
to the share of German Ramirez, from October 1, 1951 to
January, 1964. Thereafter, Mrs. Catindig is entitled to demand
the 2/16 share in the net fruits or earnings of the fishpond from

the receiver until the said share is redeemed by the respondents.


Ruling by the Supreme Court:
- The receiver (not Asuncion Meneses Vda. de Catindig) should
deliver the possession of the fishpond to the respondents or their
duly authorized representative, together with 14/16 of the net
earnings of the fishpond from January 15, 1964 up to the time
the possession is delivered to the respondents.
- The receiver should deliver to Mrs. Catindig a 2/16 share of the
net earnings of the fishpond, corresponding to the share of
German Ramirez, from January 15, 1964 up to the time the said
share is redeemed from her.
SPOUSES LEQUIN VS. SPS. VIZCONDE
SUNDAY, AUGUST 24, 2014
FACTS: In 1995, spouses Ramon and Virginia Lequin, residents
bought the subject lot consisting of 10,115 sq. m. from one
Carlito de Leon. The sale was negotiated by respondent
Raymundo Vizconde.
In 1997, spouses Vizconde represented to spouses Lequin that
they had also bought from Carlito de Leon a 1,012 sq. m. lot
adjacent to the Lequins and built a house thereon.
As later confirmed by de Leon, however, the 1,012 sq. m. lot
claimed by the Vizcondes is part of the 10,115 sq. m. lot Lequin
bought from him.
With the consent of the Vizcondes, spouses Lequin then
constructed their house on the 500-square meter half-portion of
the lot claimed by respondents, as this was near the road.
Given this situation where the house of Lequins stood on a
portion of the lot allegedly owned by Vizcondes, the former
consulted a lawyer, who advised them that the 1,012 sq. m. lot
be segregated from the subject lot whose title they own and to
make it appear that they are selling to respondents 512 square
meters thereof.
This sale was embodied in the February 12, 2000 Kasulatan
where it was made to appear that the Vizcondes paid PhP
15,000 for the purchase of the 512-square meter portion of the
subject lot.
In July 2000, petitioners tried to develop the dried up canal
located between their 500-square meter lot and the public
road. However, the respondents objected, claiming ownership of
said dried up canal or sapang patay.
This prompted the Liquins to look into the ownership of the dried
up canal and the lot claimed by the respondents Carlito de Leon
told petitioners that what he had sold to respondents was the
dried up canal or sapang patay and that the 1,012-square meter
lot claimed by respondents really belongs to petitioners. In 2001,
petitioners filed a complaint praying for the Kasulatan to be
declared as null and void ab initio.
The RTC found the Kasulatan allegedly conveying 512 square
meters to respondents to be null and void due to: (1) the vitiated
consent of petitioners in the execution of the simulated contract
of sale; and (2) lack of consideration, since it was shown that
while petitioners were ostensibly conveying to respondents 512
COMPILED BY: WIGMORE #WIGMOREFOREVER 61

SALES Case Digest (Atty. Sarona)


Compiled by: Wigmore #wigmoreforever
square meters of their property, yet the consideration of PhP
15,000 was not paid to them and, in fact,they were the ones who
paid respondents PhP 50,000.

a loan nor a forbearance of credit. After finality of decision, the


amount of PhP 50,000 shall earn interest of 12% per annum until
fully paid.

Upon appeal by the respondent-spouses, CA reversed the


ruling.

There was vitiated consent on the part of Spouses Lequin.


There was fraud in the execution of the contract used on
petitioners which affected their consent. Petitioners reliance and
belief on the wrongful claim by respondents operated as a
concealment of a material fact in their agreeing to and in readily
executing the contract of sale, as advised and proposed by a
notary public.

ISSUE: WON the Kasulatan was null and void.


But take note, on the issue of consent, the SC said that the
Kasulatan was merely voidable. But on the issue of
consideration, it was void. Final ruling - void.
HELD: YES
Re: Lack of Consideration
The contract of sale or Kasulatan states that respondents paid
petitioners PhP 15,000 for the 512-square meter portion. On its
face, the above contract of sale appears to be supported by a
valuable consideration. We, however, agree with the trial courts
finding that this is a simulated sale and unsupported by any
consideration, for respondents never paid the PhP 15,000
purported purchase price.
The kasulatan did not express the true intent of the parties
Lack of consideration was proved by petitioners evidence
aliunde showing that the Kasulatan did not express the true
intent and agreement of the parties. As explained above, said
sale contract was fraudulently entered into through the
misrepresentations of respondents causing petitioners vitiated
consent.
There can be no doubt that the contract of sale or Kasulatan
lacked the essential element of consideration.
It is a well-entrenched rule that where the deed of sale states
that the purchase price has been paid but in fact has never been
paid, the deed of sale is null and void ab initio for lack of
consideration. Moreover, Art. 1471 of the Civil Code, which
provides that if the price is simulated, the sale is void, also
applies to the instant case, since the price purportedly paid as
indicated in the contract of sale was simulated for no payment
was actually made.
The contract is void ab intio
Consideration and consent are essential elements in a contract
of sale. Where a partys consent to a contract of sale is vitiated
or where there is lack of consideration due to a simulated price,
the contract is null and void ab initio.
The PhP 50,000 paid by petitioners to respondents as
consideration for the transfer of the 500-square meter lot to
petitioners must be restored to the latter.
Otherwise, an unjust enrichment situation ensues. The facts
clearly show that the 500-square meter lot is legally owned by
petitioners as shown by the testimony of de Leon; therefore, they
have no legal obligation to pay PhP 50,000 therefor.
Considering that the 512 square-meter lot on which respondents
house is located is clearly owned by petitioners, then the Court
declares petitioners legal ownership over said 512 square-meter
lot. The amount of PhP 50,000 should only earn interest at the
legal rate of 6% per annum from the date of filing of complaint up
to finality of judgment and not 12% since such payment is neither

Believing that Carlito de Leon indeed sold a 1,012-square meter


portion of the subject property to respondents, petitioners signed
the contract of sale based on respondents representations. Had
petitioners known, as they eventually would sometime in late
2000 or early 2001 when they made the necessary inquiry from
Carlito de Leon, they would not have entered or signed the
contract of sale, much less pay PhP 50,000 for a portion of the
subject lot which they fully own. Thus, petitioners consent was
vitiated by fraud or fraudulent machinations of Raymundo. In the
eyes of the law, petitioners are the rightful and legal owners of
the subject 512 square-meter lot anchored on their purchase
thereof from de Leon. This right must be upheld and protected.
HEIRS OF INTAC VS. CA
TUESDAY, AUGUST 19, 2014
FACTS: Ireneo Mendoza, married to Salvacion Fermin, was the
owner of the subject property located in Quezon city which he
purchased in 1954. (TCT No. 242655). Ireneo had two children:
respondents Josefina and Martina (respondents), Salvacion
being their stepmother. When he was still alive, Ireneo, also took
care of his niece, Angelina, since she was three years old until
she got married.
On October 25, 1977, Ireneo, with the consent of Salvacion,
executed a deed of absolute sale of the property in favor of
Angelina and her husband, Mario (Spouses Intac). Despite the
sale, Ireneo and his family, including the respondents, continued
staying in the premises and paying the realty taxes. After Ireneo
died intestate in 1982, his widow and the respondents remained
in the premises. After Salvacion died, respondents still
maintained their residence there. Up to the present, they are in
the premises, paying the real estate taxes thereon, leasing out
portions of the property, and collecting the rentals.
The controversy arose when respondents sought the
cancellation of TCT No. 242655, claiming that the sale was only
simulated and, therefore, void. The heirs of Ireneo, the
respondents in this case, alleged that:
1. When Ireneo was still alive, Spouses Intac borrowed the title
of the property (TCT No. 106530) from him to be used as
collateral for a loan from a financing institution;
2. They objected because the title would be placed in the names
of said spouses and it would then appear that the couple owned
the property; that Ireneo, however, tried to appease them, telling
them not to worry because Angelina would not take advantage of
the situation considering that he took care of her for a very long
time; that during his lifetime, he informed them that the subject
property would be equally divided among them after his death;
and
COMPILED BY: WIGMORE #WIGMOREFOREVER 62

SALES Case Digest (Atty. Sarona)


Compiled by: Wigmore #wigmoreforever
3. That respondents were the ones paying the real estate taxes
over said property.

vendees, Spouses Intac. There was simply no consideration and


no intent to sell it.

Spouses Intac countered, among others, that the subject


property had been transferred to them based on a valid deed of
absolute sale and for a valuable consideration; that the action to
annul the deed of absolute sale had already prescribed; that the
stay of respondents in the subject premises was only by
tolerance during Ireneos lifetime because they were not yet in
need of it at that time; and that despite respondents knowledge
about the sale that took place on October 25, 1977, respondents
still filed an action against them.

Evidences to prove that there was no absolute deed of sale


between the parties
Critical is the testimony of Marietto, a witness to the execution of
the subject absolute deed of sale. He testified that Ireneo
personally told him that he was going to execute a document of
sale because Spouses Intac needed to borrow the title to the
property and use it as collateral for their loan application. Ireneo
and Salvacion never intended to sell or permanently transfer the
full ownership of the subject property to Spouses Intac. Marietto
was characterized by the RTC as a credible witness.

RTC ruled in favor of the respondents saying that the sale to the
spouses Intac was null and void. The CA also ruled that there
was no consideration in the sale to the spouses Intac and that
the contract was one for equitable mortgage.
ISSUES:
WON the Deed of Absolute Sale was a simulated contract or a
valid agreement.
WON the Deed of Absolute Sale, dated October 25, 1977,
involving the subject real property in Pagasa, Quezon City, was a
simulated contract or a valid agreement.
HELD:
The deed of sale executed by Ireneo and Salvacion was
absolutely simulated for lack of consideration and cause and,
therefore, void.
Articles 1345 and 1346 of the Civil Code provide:
Art. 1345. Simulation of a contract may be absolute or relative.
The former takes place when the parties do not intend to be
bound at all; the latter, when the parties conceal their true
agreement.
Art. 1346. An absolutely simulated or fictitious contract is void. A
relative simulation, when it does not prejudice a third person and
is not intended for any purpose contrary to law, morals, good
customs, public order or public policy binds the parties to their
real agreement.
Relatively simulated agreement vs. Absolute simulation
If the parties state a false cause in the contract to conceal their
real agreement, the contract is only relatively simulated and the
parties are still bound by their real agreement. Hence, where the
essential requisites of a contract are present and the simulation
refers only to the content or terms of the contract, the agreement
is absolutely binding and enforceable between the parties and
their successors in interest
In absolute simulation, there is a colorable contract but it has no
substance as the parties have no intention to be bound by it.
"The main characteristic of an absolute simulation is that the
apparent contract is not really desired or intended to produce
legal effect or in any way alter the juridical situation of the
parties." "As a result, an absolutely simulated or fictitious
contract is void, and the parties may recover from each other
what they may have given under the contract."
No valid sale took place between Ireneo and Spouses Intac
In the case at bench, the Court is one with the courts below that
no valid sale of the subject property actually took place between
the alleged vendors, Ireneo and Salvacion; and the alleged

Aside from their plain denial, the heirs of Intac failed to present
any concrete evidence to disprove Mariettos testimony. They
claimed that they actually paid P150,000.00 for the subject
property. They, however, failed to adduce proof, even by
circumstantial evidence, that they did, in fact, pay it. Even for the
consideration of P60,000.00 as stated in the contract, petitioners
could not show any tangible evidence of any payment therefor.
Their failure to prove their payment only strengthened Mariettos
story that there was no payment made because Ireneo had no
intention to sell the subject property.
Angelinas story, except on the consideration, was consistent
with that of Marietto. Angelina testified that she and her husband
mortgaged the subject property sometime in July 1978 to finance
the construction of a small hospital in Sta. Cruz, Laguna.
Angelina claimed that Ireneo offered the property as he was in
deep financial need.
The contract of sale was only for the purpose of lending the title
of the property to Spouses Intac to enable them to secure a
loan.
Their arrangement was only temporary and could not give rise to
a valid sale. Where there is no consideration, the sale is null and
void ab initio. The case of Lequin vs. VIzconde was cited in this
case.
The fact that Ireneo was still in physical possession of the
subject property after the sale is a strong evidence to prove that
there was no valid sale between the parties.
More importantly, Ireneo and his family continued to be in
physical possession of the subject property after the sale in 1977
and up to the present. They even went as far as leasing the
same and collecting rentals. If Spouses Intac really purchased
the subject property and claimed to be its true owners, why did
they not assert their ownership immediately after the alleged sale
took place? Why did they have to assert their ownership of it only
after the death of Ireneo and Salvacion? One of the most striking
badges of absolute simulation is the complete absence of any
attempt on the part of a vendee to assert his right of dominion
over the property.
As heretofore shown, the contemporaneous and subsequent
acts of both parties in this case, point to the fact that the intention
of Ireneo was just to lend the title to the Spouses Intac to enable
them to borrow money and put up a hospital in Sta. Cruz,
Laguna. Clearly, the subject contract was absolutely simulated
and, therefore, void.
The Spouses Intac never became the owners of the property
despite its registration in their names.
COMPILED BY: WIGMORE #WIGMOREFOREVER 63

SALES Case Digest (Atty. Sarona)


Compiled by: Wigmore #wigmoreforever
It is also of no moment that TCT No. 106530 covering the subject
property was cancelled and a new TCT (TCT No. 242655)21 was
issued in their names. After all, registration does not vest title. As
a logical consequence, petitioners did not become the owners of
the subject property even after a TCT had been issued in their
names.
BUENAVENTURA VS. CA
TUESDAY, JULY 1, 2014
FACTS: Defendant spouses Leonardo Joaquin and Feliciana
Landrito are the parents of plaintiffs Consolacion, Nora, Emma
and Natividad as well as of defendants Fidel, Tomas, Artemio,
Clarita,
Felicitas,
Fe,
and
Gavino,
all
surnamed
JOAQUIN. (Note: So there are two sets of children here.)
Sought to be declared null and void ab initio are certain deeds of
sale of real property executed by Leonardo Joaquin and
Feliciana Landrito in favor of their co-defendant children and the
corresponding certificates of title issued in their names. The
plaintiffs in this case sought for the declaration of nullity of the six
deeds of sale and certificates of title in favor of the defendants.
They alleged that certain deed of sale were null and void ab initio
because they are simulated.
They said that:
a. Firstly, there was no actual valid consideration for the deeds of
sale xxx over the properties in litis;
b. Secondly, assuming that there was consideration in the sums
reflected in the questioned deeds, the properties are more than
three-fold times more valuable than the measly sums appearing
therein;
c. Thirdly, the deeds of sale do not reflect and express the true
intent of the parties (vendors and vendees); and
d. Fourthly, the purported sale of the properties in litis was the
result of a deliberate conspiracy designed to unjustly deprive the
rest of the compulsory heirs (plaintiffs herein) of their legitime.
Defendants, on the other hand aver:
(1) That plaintiffs do not have a cause of action against them as
well as the requisite standing and interest to assail their titles
over the properties in litis;
(2) That the sales were with sufficient considerations and made
by defendants parents voluntarily, in good faith, and with full
knowledge of the consequences of their deeds of sale; and
(3) That the certificates of title were issued with sufficient factual
and legal basis.
RTC ruled in favor of the defendants (respondents in this case)
and dismissed the complaint. Upon appeal, the CA upheld RTCs
ruling.
ISSUES:
1. Whether the Deeds of Sale are void for lack of consideration.
NO
2. Whether the Deeds of Sale are void for gross inadequacy of
price. NO
HELD:
1st issue: There was a consideration.

If there is a meeting of the minds of the parties as to the price,


the contract of sale is valid, despite the manner of payment, or
even the breach of that manner of payment. If the real price is
not stated in the contract, then the contract of sale is valid but
subject to reformation. If there is no meeting of the minds of the
parties as to the price, because the price stipulated in the
contract is simulated, then the contract is void. Article 1471 of the
Civil Code states that if the price in a contract of sale is
simulated, the sale is void.
It is not the act of payment of price that determines the validity of
a contract of sale.
Payment of the price has nothing to do with the perfection of the
contract. Payment of the price goes into the performance of the
contract. Failure to pay the consideration is different from lack of
consideration. The former results in a right to demand the
fulfillment or cancellation of the obligation under an existing valid
contract while the latter prevents the existence of a valid
contract.
Petitioners failed to show that the prices in the Deeds of Sale
were absolutely simulated.
To prove simulation, petitioners presented Emma Joaquin
Valdozs testimony stating that their father, respondent Leonardo
Joaquin, told her that he would transfer a lot to her through a
deed of sale without need for her payment of the purchase price.
The trial court did not find the allegation of absolute simulation of
price credible.
Petitioners failure to prove absolute simulation of price is
magnified by their lack of knowledge of their respondent siblings
financial capacity to buy the questioned lots. On the other hand,
the Deeds of Sale which petitioners presented as evidence
plainly showed the cost of each lot sold. Not only did
respondents minds meet as to the purchase price, but the real
price was also stated in the Deeds of Sale. As of the filing of the
complaint, respondent siblings have also fully paid the price to
their respondent father.
2nd issue: The general rule is that inadequacy of
consideration shall not invalidate a contract.
Articles 1355 of the Civil Code states:
Art. 1355. Except in cases specified by law, lesion or
inadequacy of cause shall not invalidate a contract, unless there
has been fraud, mistake or undue influence. (Emphasis
supplied)
Article 1470 of the Civil Code further provides:
Art. 1470. Gross inadequacy of price does not affect a contract
of sale, except as may indicate a defect in the consent, or that
the parties really intended a donation or some other act or
contract. (Emphasis supplied)
Petitioners failed to prove any of the instances mentioned in
Articles 1355 and 1470 of the Civil Code which would invalidate,
or even affect, the Deeds of Sale. Indeed, there is no
requirement that the price be equal to the exact value of the
subject matter of sale. All the respondents believed that they
received the commutative value of what they gave.
Ruling: In the instant case, the trial court found that the lots were
sold for a valid consideration, and that the defendant children
actually paid the purchase price stipulated in their respective
COMPILED BY: WIGMORE #WIGMOREFOREVER 64

SALES Case Digest (Atty. Sarona)


Compiled by: Wigmore #wigmoreforever
Deeds of Sale. Actual payment of the purchase price by the
buyer to the seller is a factual finding that is now conclusive upon
us. WHEREFORE, we AFFIRM the decision of the Court of
Appeals in toto.
b.

Price is False
MACAPAGAL vs. CATALINA O. REMORIN, CORAZON
CALUZA BAMRUNGCHEEP, and LAURELIA CALUZAVALENCIANO

FACTS: Lots 24 and 25 were registered in the name of Candido


Caluza under Transfer Certificate of Title (TCT) No. 160544.
Purificacion Arce-Caluza (Purificacion) is his second wife.
Corazon Caluza-Bamrungcheep (Corazon) is his legally adopted
daughter during his first marriage. After Candido died in 1981,
Corazon and Purificacion executed a Deed of Extrajudicial
Settlement adjudicating between themselves the properties of
Candido, as the latter's surviving heirs.
Lots 24 and 25, together with Lot 23, which was registered in
Candido's name, were adjudicated to Corazon. Purificacion got
Candido's land in Bulacan. However, administration of Lots 23,
24 and 25 were entrusted to Purificacion by Corazon as she had
to leave for Thailand after her marriage to a Thai.
Unknown to Corazon, Purificacion executed an Affidavit of Loss
alleging that the TCTs of Lots No 23, 24 and 25 were lost and
could no longer be found. She filed a petition for the issuance of
new owner's duplicates of title alleging that she was her
deceased husband's sole heir. The petition was granted and new
TCTs were issued in Purificacion's name. Purificacion then sold
the lots to Catalina Remorin (Catalina) and Catalina mortgaged
Lots 24 and 25 to L & R Lending Corp.
Corazon filed a complaint for reconveyance and damages
against Purificacion and Catalina upon discovery of sale. Plaintiff
alleged that the two defendants connived with each other in
transferring the three lots in their names through simulated sales.
Corazon likewise filed a criminal complaint for falsification and
perjury against the two.
Catalina executed a Deed of Transfer, signed by Purificacion as
witness, admitting the wrong they did in illegally transferring the
lots in their names and acknowledging Corazon to be the rightful
owner under the Deed of Extrajudicial Settlement. Corazon
presented the Deed of Transfer before the Register of Deeds of
Quezon City and Catalina's TCT over Lots 24 and 25 was
cancelled and a TCT was issued in Corazon's name.
Prior thereto, however, Catalina mortgaged Lots 24 and 25 to
respondent Laurelia Caluza-Valenciano (Laurelia) to pay off her
mortgage indebtedness to L & R Lending Corporation. The
inscription of the mortgage in favor of Laurelia was carried over
to Corazon's TCT.
-Corazon, Purificacion, Catalina, and Laurelia executed a
Memorandum of Agreement to settle Civil Case. It stipulated that
Corazon cedes and grants unto and in favor of Purificacion full
ownership and other real rights over the southernmost apartment
as well as the portion of the lot occupied thereby subject to the
condition that Purificacion shall assume satisfaction of the

mortgage debt contracted by Catalina in favor of Laurelia and


shall cause transfer of said annotation to the title to be issued in
her (Purificacion's) name; and furthermore that any and all
expenses for segregation survey, re-titling and annotation of said
mortgage shall be shouldered by said Purificacion Arce-Caluza;
Before the agreement could be implemented, Purificacion died.
Consequently, another compromise agreement was executed
stating that Corazon and Catalina agreed that title to the
southernmost apartment as well as the portion of the lot
occupied thereby shall be transferred direct to its interested
buyer with defendant Catalina assuming and paying (from the
proceeds of the sale) her mortgage obligation with Laurelia; any
and all expenses for segregation survey, re-titling, capital gains
taxes and those connected with the annotation and/or release of
said mortgage should now be shouldered by defendant Catalina
O. Remorin. Corazon then sold the subject Lot to Laurelia by
virtue of a deed entitled "Sale of Unsegregated Portion of Land."
However, Catalina also sold the same lot to Macapagal claiming
to be authorized under the Compromise Agreement. Macapagal
sought to nullify the sale executed by Corazon in favor of
Laurelia and to declare valid the one executed by Catalina in her
favor.
RTC rendered judgment in favor of petitioner. Corazon and
Laurelia appealed to the Court of Appeals which reversed the
decision of the trial court.
Macapagals contention: the sale executed by Catalina in her
favor should prevail over the one executed by Corazon in favor
of Laurelia, as Catalina was the one authorized to sell the
disputed property under the Compromise Agreement
Respondents contention: Corazon, the registered owner of the
disputed property, did not give Catalina authority to sell the lot. It
was provided in the Agreement that Catalina shall pay off her
mortgage obligation and incidental expenses from the proceeds
of the sale only to reassure Catalina that her obligation would be
paid in the event that Corazon sells the property.
ISSUE: WON Catalina was authorized to sell the land as
provided by their Compromise Agreement.
RULING: The Compromise Agreement dated September 9, 1988
cannot be taken as a waiver of Corazon's authority to sell and
grant thereof to Catalina considering that the Agreement merely
provided that Catalina pay off her mortgage obligation and
incidental expenses from the proceeds of the sale. Although it
was imperative, as part of the compromise, that the money come
from the proceeds of the sale, it was not expressly stated, nor did
it necessarily mean, that Catalina herself be the one to directly
sell the property. Authority to sell must be couched in clear and
unmistakable language.
Moreover, intent to give Catalina authority to sell may not be
easily attributed to Corazon considering that the latter had to file
the reconveyance case as a result of Purificacion's and
Catalina's acts of transferring the disputed lot in their names.
In contract interpretation, analysis is not to be limited to the
words used in the contract, as they may not accurately reflect the
parties' true intent. If the words of the contract appear to be
contrary to the evident intention as revealed by the
circumstances, the latter shall prevail over the former.
COMPILED BY: WIGMORE #WIGMOREFOREVER

65

SALES Case Digest (Atty. Sarona)


Compiled by: Wigmore #wigmoreforever
The fact that the deed of sale between respondents Corazon and
Laurelia did not accurately reflect the true consideration thereof
is not cause for declaration of its nullity. When the parties
intended to be bound by the contract except that it did not reflect
the actual purchase price of the property, there is only a relative
simulation of the contract which remains valid and enforceable. It
cannot be declared null and void since it does not fall under the
category of an absolutely simulated or fictitious contract. The
contract of sale is valid but subject to reformation.
Petition denied.
c.

Non-Payment of Price

CLARA M. BALATBAT vs CA, SPS. REPUYAN


G.R. No. 109410, August 28, 1996
FACTS: A parcel of land was acquired by plaintiff Aurelio Roque
and Maria Mesina during their conjugal union. Maria died on
August 28, 1966.
On June 15, 1977, Aurelio filed a case for partition. The trial court
in that case for partition, held that Aurelio is entitled to the
portion of his share in the conjugal property, and the other half
which formed part of the estate of Maria Mesina, will be divided
equally between him and their 4 children receiving 1/5 each. The
decision having become final and executory, the Register of
Deeds of Manila issued a transfer certificate of title on October 5,
1979 according to the ruling of the court.
On April 1, 1980, Aurelio sold his 6/10 shares to spouses Aurora
Tuazon-Repuyan and Jose Repuyan, as evidenced by a deed of
absolute sale. On June 21, 1980, Aurora caused the annotation
of her affidavit of adverse claim. However, on August 20, 1980,
Aurelio filed a complaint for rescission of contract grounded on
the buyers failure to pay the balance of the purchase price.
Subsequently, on February 4, 1982, another deed of absolute
sale was executed between Aurelio and his children, and herein
petitioner Clara Balatbat, involving the entire lot. Balatbat filed a
motion for the issuance of writ of possession, which was granted
by the court on September 20, 1982, subject to valid rights and
interests of third persons. Balatbat filed a motion to intervene in
the rescission case, but did not file her complaint in intervention.
The court ruled that the sale between Aurelio and Aurora is valid.
On March 3, 1987 however, Balatbat filed a notice of lis pendens
before the Register of Deeds regarding the subject property.
ISSUES:
W/N the alleged sale to Spouses Repuyan was merely
executor

deed of sale and declared that the Sale dated April 1, 1980, as
valid and enforceable. No appeal having been made, the
decision became final and executory.
Examining the terms and conditions of the "Deed of Sale" dated
April 1, 1980, the P45,000.00 balance is payable only "after the
property covered by T.C.T. No. 135671 has been partitioned and
subdivided, and title issued in the name of the BUYER" hence,
vendor Roque cannot demand payment of the balance unless
and until the property has been subdivided and titled in the name
of private respondents. Devoid of any stipulation that ownership
in the thing shall not pass to the purchaser until he has fully paid
the price, ownership of the thing shall pass from the vendor to
the vendee upon actual or constructive delivery of the thing sold
even if the purchase price has not yet been fully paid. The failure
of the buyer to make good the price does not, in law, cause
ownership to evert to the seller unless the bilateral contract of
sale is first rescinded or resolved pursuant to Article 1191 of the
New Civil Code.
Non-payment only creates a right to demand the fulfillment of the
obligation or to rescind the contract.
With respect to the non-delivery of the possession of the subject
property to the private respondent, suffice it to say that
ownership of the thing sold is acquired only from the time of
delivery thereof, actual or constructive.
A contract of sale being consensual, it is perfected by the mere
consent of the parties. Delivery of the thing bought or payment of
the price is not necessary for the perfection of the contract; and
failure of the vendee to pay to price after the execution of the
contract does not make the sale null and void for lack of
consideration but results at most in default on the part of the
vendee, for which the vendor may exercise his legal remedies.

2) Must be in Money or its Equivalent

IMELDA ONG, ET AL. vs ALFREDO ON ET AL.


G.R. No. L-67888, October 8, 1985
FACTS: On February 25, 1976, Imelda Ong for and in
consideration of One (1.00) Peso and other valuable
considerations, executed in favor of Sandra Maruzzo, then a
minor, a Quitclaim Deed whereby she transferred, released, and
assigned all her rights and title over a parcel of land in Makati.
However, on November 19, 1980, Imelda Ong revoked the
aforesaid Quitclaim and donated the property to her son Rex.

HELD:
No. The sale was consummated, hence, valid and enforceable.

Subsequently, Sandra Maruzzo on June 20, 1983, through here


guardian ad litem Alfredo Ong, filed with the RTC an action for
the recovery of ownership/possession and nullification of the
Deed of Donation in favor of Rex.

Contrary to petitioner's contention that the sale dated April 1,


1980 in favor of Spouses Repuyan was merely executory for the
reason that there was no delivery of the subject property and that
consideration/price was not fully paid, the Court finds the sale as
consummated, hence, valid and enforceable. The Court
dismissed vendor's Aurelio Roque complaint for rescission of the

Petitioners claimed that the Quitclaim Deed is null and void


inasmuch as it is equivalent to a Deed of Donation, acceptance
of which by the donee is necessary to give it validity. Further, it is
averred that the donee, Sandra Maruzzo, being a minor, had no
legal personality and therefore incapable of accepting the
donation.
COMPILED BY: WIGMORE #WIGMOREFOREVER

66

SALES Case Digest (Atty. Sarona)


Compiled by: Wigmore #wigmoreforever

The trial court ruled in favor of Maruzzo and held that the
Quitclaim Deed is equivalent to a Deed of Sale and, hence, there
was a valid conveyance in favor of the latter.
Appealing to the IAC, petitioners additionally contends that the
One (1.00) Peso consideration is not a consideration at all to
sustain the ruling that the Quitclaim Deed is equivalent to a sale.
The IAC however affirmed the TC.
ISSUE: W/N the quitclaim is equivalent to a deed of sale or
to a deed of donation
HELD: The Quitclaim Deed is equivalent to a deed of sale. A
careful perusal of the subject deed reveals that the conveyance
of the one- half () undivided portion of the above-described
property was for and in consideration of the One (P 1.00) Peso
and the other valuable considerations (emphasis supplied) paid
by private respondent Sandra Maruzzo through her
representative, Alfredo Ong, to petitioner Imelda Ong. Stated
differently, the cause or consideration is not the One (P1.00)
Peso alone but also the other valuable considerations.
Although the cause is not stated in the contract it is presumed
that it is existing unless the debtor proves the contrary (Article
1354 of the Civil Code). One of the disputable presumptions is
that there is a sufficient cause of the contract. It is a legal
presumption of sufficient cause or consideration supporting a
contract even if such cause is not stated therein (Article 1354,
New Civil Code of the Philippines.) This presumption cannot be
overcome by a simple assertion of lack of consideration
especially when the contract itself states that consideration was
given, and the same has been reduced into a public instrument
with all due formalities and solemnities. To overcome the
presumption of consideration the alleged lack of consideration
must be shown by preponderance of evidence in a proper action.
The execution of a deed purporting to convey ownership of a
realty is in itself prima facie evidence of the existence of a
valuable consideration, the party alleging lack of consideration
has the burden of proving such allegation.
Even granting that the Quitclaim deed in question is a donation,
Article 741 of the Civil Code provides that the requirement of the
acceptance of the donation in favor of minor by parents of legal
representatives applies only to onerous and conditional
donations where the donation may have to assume certain
charges or burdens. Donation to an incapacitated donee does
not need the acceptance by the lawful representative if said
donation does not contain any condition. In simple and pure
donation, the formal acceptance is not important for the donor
requires no right to be protected and the donee neither
undertakes to do anything nor assumes any obligation. The
Quitclaim now in question does not impose any condition.
ISAAC BAGNAS ET AL. vs. CA
G.R. No. L-38498, August 10, 1989
FACTS: Hilario Mateum died without a will and was survived only
by collateral relatives. Bagnas et al., the petitioners, were his
nearest kin. The respondents Retonil et al. on the other hand
were relatives to a farther extent.
Retonil et al. claims ownership of 10 parcels of land from the

estate of Hilarion which they contend were sold by Hilario


through two deeds of sale where the consideration for the lands
was one (1.00) Peso and services rendered, being rendered and
to be rendered.
Bagnas et al. filed a case against respondents seeking
annulment of the deeds of sale as fictitious, fraudulent or
falsified, or alternatively, as donations void for want of
acceptance embodied in a public instrument. In answer to the
complaint, the respondents denied the alleged fictitious or
fraudulent character of the sales in their favor, asserting that the
said sales were made for good and valuable consideration.
ISSUE: W/N the said consideration is valid.
HELD: The Court ruled that the deeds of sale are void and are of
no force and effect.
Upon the consideration alone that the apparent gross, not to say
enormous, disproportion between the stipulated price (in each
deed) of P l.00 plus unspecified and unquantified services and
the undisputably valuable real estate allegedly sold worth at least
P10,500.00 going only by assessments for tax purposes which, it
is well-known, are notoriously low indicators of actual value
plainly and unquestionably demonstrates that they state a false
and fictitious consideration, and no other true and lawful cause
having been shown, the Court finds both said deeds, insofar as
they purport to be sales, not merely voidable, but void ab initio.
The transfers in question being void, it follows as a necessary
consequence that the properties purportedly conveyed remained
part of the estate of Hilario Mateum, said transfers
notwithstanding, recoverable by his intestate heirs, the
petitioners herein, whose status as such is not challenged.
Even if the contract of sale would be shown as a donation
(apparently, this was the intent of the donor), failure to conform to
the requirements would not make it a valid donation.
There is no Contract of Sale for lack of consideration. Likewise,
there is also no valid deed of donation for failure to conform to
the requirements of donation.
REPUBLIC vs PRDC and CA
G.R. No. L-10141, January 31, 1958
FACTS: The Republic brought an action against Apostol for the
collection of sums owing to it for his purchase of Palawan
Almaciga and other logs. His total debt amounted to some
P34,000. PRDC intervened claiming that Apostol, as President of
the company, without prior authority, took goods (steel sheets,
pipes, bars, etc) from PRDC warehouse and appropriated them
to settle his personal debts in favor of the government. The
Republic opposed the intervention of PRDC, arguing that price is
always paid in money and that payment in kind is no payment at
all; hence, money and not the goods of PRDC are under dispute.
ISSUE: W/N payment in kind is equivalent to price paid in
money.
HELD: Yes. The Government argues that "Price . . . is always
paid in terms of money and the supposed payment being in kind,
it is no payment at all, "citing Article 1458 of the new Civil Code.
COMPILED BY: WIGMORE #WIGMOREFOREVER 67

SALES Case Digest (Atty. Sarona)


Compiled by: Wigmore #wigmoreforever
However, the same Article provides that the purchaser may pay
"a price certain in money or its equivalent," which means that
they meant of the price need not be in money. Whether the G.I.
sheets, black sheets, M. S. Plates, round bars and G. I. pipes
claimed by the respondent corporation to belong to it and
delivered to the Bureau of Prison by Macario Apostol in payment
of his account is sufficient payment therefore, is for the court to
pass upon and decide after hearing all the parties in the case.
Should the trial court hold that it is as to credit Apostol with the
value or price of the materials delivered by him, certainly the
herein respondent corporation would be affected adversely if its
claim of ownership of such sheets, plates, bars and pipes is true.

3)

Must be Certain or Ascertainable at Perfection


VILLANUEVA VS. CA
267 SCRA 89
G.R. NO. 107624
JANUARY 28, 1997

FACTS: Petitioner Gamaliel Villanueva has been a tenantoccupant of a unit in an apartment building erected on a parcel of
land owned by private respondents dela Cruz. In 1986, Jose dela
Cruz offered said land with the apartment building for sale and
petitioners (Gamaliel and Irene) showed interest in the property.
As initial step, Jose gave Irene a letter of authority for her to
inspect the property. Since the property was in arrears for
payment of realty taxes, Jose approached Irene and asked for a
certain amount to pay for the taxes so that the property would be
cleared of any encumbrance. Irene gave 10k (5k on 2
occasions). It was agreed by them that the 10k would form part
of the sale price of 550k.
Thereafter, Jose went to Irene, bringing with him Mr. Sabio,
requesting her to allow Sabio to purchase of the property, to
which they consented, so they would just purchase the other half
(265k, having paid the 10k). Dela Cruz executed in favor of their
co-defendants (Guido and Felicitas Pile) a Deed of Assignment
of the other portion of the land, wherein Gamaliels apartment
unit is situated. This was purportedly as full payment and
satisfaction of an indebtedness obtained from the Piles. TCT was
later issued in the name of the Piles.
Soon, Gamaliel learned about the assignment and issuance of
new TCT. Petitioners elevated their complaint to the Court
(specific performance). They contend that a contract of sale has
been perfected and that the 10k formed part of the purchase
price (necessarily then, there must have been an agreement as
to the price). They cite Art 1482: Whenever earnest money is
given in a contract of sale, it shall be considered as part of the
price and proof of perfection of the contract. On the other hand,
private respondents claim that what was agreed upon was that
the 10k be primarily intended as payment for realty tax, and was
going to for part of the consideration of the sale if the transaction
would finally be consummated. They insist that there was no
clear agreement as to the true amount of consideration.
ISSUE: Was there a perfected contract of sale? NO
HELD: After a review of the evidence, SC found that there was
no agreement as to the price (based on the testimonies). To

settle the conflicting claims, petitioners could have presented the


contract of sale. However, it was not presented in evidence.
Petitioners aver that even if the (unsigned) deed was not
produced, Jose admitted preparing said deed in accordance
with their agreement.
We do not agree with petitioners. Assuming arguendo that such
draft deed existed, it does not necessarily follow that there
was already a definite agreement as to the price. If there was,
why then did private respondent Jose de la Cruz not sign it? If
indeed the draft deed of sale was that important to petitioners'
cause, they should have shown some effort to procure it. They
could have secured it through a subpoena ducestecum or thru
the use of one of the modes of discovery. But petitioners made
no such effort. And even if produced, it would not have
commanded any probative value as it was not signed.
The price of the leased land not having been fixed, the essential
elements which give life to the contract were lacking. It follows
that the lessee cannot compel the lessor to sell the leased land
to him.
The price must be certain; it must be real, not fictitious. It is not
necessary that the certainty of the price be actual or determined
at the time of executing the contract. The fact that the exact
amount to be paid therefor is not precisely fixed, is no bar to an
action to recover such compensation, provided the contract, by
its terms, furnishes a basis or measure for ascertaining the
amount agreed upon. The price could be made certain by the
application of known factors. A contract of sale is not void for
uncertainty when the price, though not directly stated in terms of
pesos and centavos, can be made certain by reference to
existing invoices identified in the agreement.
In the instant case, however, what is dramatically clear from the
evidence is that there was no meeting of mind as to the price,
expressly or impliedly, directly or indirectly.

MORENO, JR. VS. PRIVATE MANAGEMENT OFFICE


507 SCRA 63
G.R. NO. 159373
NOVEMBER 16, 2006
FACTS: The subject-matter in the civil case is the J. Moreno
Building (formerly known as the North Davao Mining
Building) or more specifically, the 2nd, 3rd, 4th, 5th and
6th floors of the building.
Moreno is the owner of the Ground Floor, the 7th Floor and
the Penthouse of the J. Moreno Building and the lot on which it
stands.
Private Management Office (formerly, Asset Privatization Trust
or APT) on the other hand, is the owner of the 2nd, 3rd, 4th,
5th and 6th floors of the building, the subject-matter of this suit.
On February 13, 1993, APT called for a conference for the
purpose of discussing Morenos right of first refusal over the
floors of the building owned by APT. At said meeting, APT
informed Moreno that the proposed purchase price for said floors
was P21 Million.

COMPILED BY: WIGMORE #WIGMOREFOREVER

68

SALES Case Digest (Atty. Sarona)


Compiled by: Wigmore #wigmoreforever
In a letter dated February 22, 1993, APT, informed Moreno
that the Board of Trustees (BOT) of APT "is in agreement
that Mr. Jose Moreno, Jr. has the right of first refusal" and
requested Moreno to deposit 10% of the "suggested indicative
price" of P21 million on or before February 26, 1993.
Moreno paid the P21 million on February 26, 1993. APT issued
an OR for the said payment.
But later, APT wrote Moreno that its Legal Department has
questioned the basis for the computation of the indicative price
for the said floors. Thus, on April 2, 1993, APT wrote Moreno that
the APT BOT has "tentatively agreed on a settlement price of
P42,274,702.17" for the said floors.
RTC ruled in favor of Moreno, declared that there was a
perfected contract of sale and ordered APT to sell the subject
floors at P21M.
CA reversed, hence the petition.
ISSUE: WON there was a perfected contract of sale over the
subject floors at the price of 21 Million. NO
HELD: A contract of sale is perfected at the moment there is a
meeting of minds upon the thing which is the object of the
contract and upon the price. Consent is manifested by the
meeting of the offer and the acceptance upon the thing and the
cause which are to constitute the contract. The offer must be
certain and the acceptance absolute.
To reach that moment of perfection, the parties must agree on
the same thing in the same sense, so that their minds meet as to
all the terms. They must have a distinct intention common to both
and without doubt or difference; until all understand alike, there
can be no assent, and therefore no contract. The minds of
parties must meet at every point; nothing can be left open for
further arrangement
So long as there is any uncertainty or indefiniteness, or future
negotiations or considerations to be had between the parties,
there is not a completed contract, and in fact, there is no contract
at all.
Once there is concurrence of the offer and acceptance of the
object and cause, the stage of negotiation is finished. This
situation does not obtain in the case at bar. The letter of
February 22, 1993 and the surrounding circumstances clearly
show that the parties are not past the stage of negotiation,
hence there could not have been a perfected contract of sale.
The letter is clear evidence that APT did not intend to sell the
subject floors at the price certainofP21M, viz.:
(This letter was addressed to Morenos Atty.)
xxx We are pleased to inform you that the Board is in
agreement that Mr. Jose Moreno, Jr. has the right of first
refusal. This will be confirmed by our Board during the next
board meeting on February 26, 1993. In the meantime,
please advise Mr. Moreno that the suggested indicative price
for APTs five (5) floors of the building in question is P21
Million.
If Mr. Moreno is in agreement, he should deposit with
APT the amount of P2.1 Million equivalent to 10% of the

price on or before February 26, 1993. The balance will be


due within fifteen (15) days after Mr. Moreno receives the
formal notice of approval of the indicative price. xxx
The letter clearly states that P21M is merely a "suggested
indicative price" of the subject floors as it was yet to be approved
by the BOT.
Before the Board could confirm the suggested indicative price,
the Committee on Privatization must first approve the terms of
the sale or disposition. The imposition of this suspensive
condition finds basis under Proclamation No. 5022 which
vests in the Committee the power to approve the sale of
government assets, including the price of the asset to be sold
(apparently government pala itong APT, and may procedure na
sinusunod sa law).
Other discussions that may be relevant:
On Morenos argument that the "suggested indicative price" of
P21M is not a proposed price, but the selling price indicative of
the value at which APT was willing to sell.
The trial court relied upon the definition of the word "indicative"
under the Webster Comprehensive Dictionary, International
Edition. According to Webster, "to indicate" is to point out; direct
attention; to indicate the correct page. "Indicative" is merely the
adjective of the verb to indicate. xxx
Under the Rules of Court, the terms are presumed to have been
used in their primary and general acceptation, but evidence is
admissible to show that they have a local, technical, or
otherwise peculiar signification, and were so used and
understood
in the particular instance, in which case the
agreement must be construed accordingly.
The reliance of the trial court in the Webster definition of the term
"indicative," as also adopted by Moreno, is misplaced. The
transaction at bar involves the sale of an asset under a
privatization scheme which attaches a peculiar meaning or
signification to the term "indicative price." Under No. 6.1 of the
General Bidding Procedures and Rules of respondent, "an
indicative price is a ballpark figure and [respondent] supplies
such a figure purely to define the ball-park." The plain contention
of Moreno that the transaction involves an "ordinary arms-length
sale of property" is unsubstantiated and leaves much to be
desired. This case sprung from a case of specific performance
initiated by Moreno who has the burden to prove that the case
should be spared from the application of the technical terms in
the sale and disposition of assets under privatization.
He failed to discharge the burden.
It appears in the case at bar that Morenos construction of
the letter of February 22, 1993 that his assent to the
"suggested indicative price" of P21M converted it as the price
certain, thus giving rise to a perfected contract of sale is his
own subjective understanding. As such, it is not shared by APT.
Under American jurisprudence, mutual assent is judged by an
objective standard, looking to the express words the parties used
in the contract. Under the objective theory of contract,
understandings and beliefs are effective only if shared. Based on
the objective manifestations of the parties in the case at bar,
there was no meeting of the minds.
COMPILED BY: WIGMORE #WIGMOREFOREVER

69

SALES Case Digest (Atty. Sarona)


Compiled by: Wigmore #wigmoreforever
4.

Manner of Payment of Price ESSENTIAL


NAVARRA VS. PLANTERS
527 SCRA 561
G.R. NO. 172674
JULY 12, 2007

FACTS: The Navarras obtained a loan of P1,200,000.00 from


Planters Bank and, by way of security therefor, executed a deed
of mortgage over their five (5) parcels of land. Unfortunately, the
couple failed to pay their loan obligation. Hence, Planters Bank
foreclosed on the mortgage. The one year redemption period
expired without the Navarras having redeemed the foreclosed
properties.
On the other hand, co-petitioner RRRC Development
Corporation (RRRC) is a real estate company owned by the
parents of Carmelita Bernardo Navarra. RRRC itself obtained a
loan from Planters Bank secured by a mortgage over another set
of properties owned by RRRC. The loan having been likewise
unpaid, Planters Bank similarly foreclosed the mortgaged assets
of RRRC.
Unlike the Navarras, however, RRRC was able to negotiate with
the Bank for the redemption of its foreclosed properties by way
of a concession whereby the Bank allowed RRRC to refer to it
would-be buyers of the foreclosed RRRC properties who would
remit their payments directly to the Bank, which payments would
then be considered as redemption price for RRRC. Eventually,
the foreclosed properties of RRRC were sold to third persons
whose payments therefor, directly made to the Bank, were in
excess by P300,000.00 for the redemption price.
In the meantime, Jorge Navarra sent a letter {*contents of
the letters are found in the ruling*} to Planters Bank,
proposing to repurchase the five (5) lots earlier auctioned to the
Bank, with a request that he be given until August 31, 1985 to
pay the down payment of P300,000.00. In response, Planters
Bank, thru its Vice-President Ma. Flordeliza Aguenza, wrote back
Navarra via a letter dated August 16, 1985. Then, on January 21,
1987, Planters Bank sent a letter to Jorge Navarra informing
him that it could not proceed with the documentation of the
proposed repurchase of the foreclosed properties on account
of
his
non-compliance with the Banks request for the
submission of the needed board resolution of RRRC, thus,
demanding that they surrender and vacate the properties in
question for their failure to exercise their right of redemption.
The Navarras filed their complaint for Specific Performance
with Injunction against Planters Bank, alleging that a perfected
contract of sale was made between them and Planters Bank
whereby they would repurchase the subject properties for
P1,800,000.00 with a down payment of P300,000.00. In its
Answer, Planters Bank asserted that there was no perfected
contract of sale because the terms and conditions for the
repurchase have not yet been agreed upon.
The RTC ruled that there was a perfected contract of sale
between the Navarras and Planters Bank. The CA reversed
the decision citing Article 1319 as basis, declaring that the
acceptance of the offer was not absolute.

ISSUES:
WON there was a perfected contract to repurchase the
foreclosed properties between the petitioners and the
private respondent Planters Development Bank. NO
WON the parties never got past the negotiation stage. YES
HELD:
In general, contracts undergo three distinct stages: negotiation,
perfection or birth, and consummation. Negotiation begins from
the time the prospective contracting parties manifest their
interest in the contract and ends at the moment of their
agreement. Perfection or birth of the contract takes place when
the parties agree upon the essential elements of the contract,
i.e., consent, object and price. Consummation occurs when
the parties fulfill or perform the terms agreed upon in the
contract, culminating in the extinguishment thereof.
A negotiation is formally initiated by an offer which should
be certain with respect to both the object and the cause or
consideration of the envisioned contract. In order to produce a
contract, here must be acceptance, which may be express or
implied, but it must not qualify the terms of the offer. In other
words, it must be identical in all respects with that of the offer so
as to produce consent or meeting of the minds.
Here, the Navarras assert that the following exchange of
correspondence between them and Planters Bank constitutes
the offer and acceptance, thus:
Letter dated July 18, 1985 of Jorge Navarra:
This will formalize my request for your kind consideration
in allowing my brother and me to buy back my house and
lot and my restaurant building and lot together with the
adjacent road lot. Since my brother, who is working in
Saudi Arabia, has accepted this arrangement only
recently as a result of my urgent offer to him, perhaps it will
be safe for us to set August 31, 1985 as the last day for
the payment of a P300,000.00 down payment. I hope you
will grant us the opportunity to raise the funds within this
period, which includes an allowance for delays.
The purchase price, I understand, will be based on the
redemption value plus accrued interest at the prevailing
rate up to the date of our sales contract.
Maybe you can give us a long term payment scheme on the
basis of my brothers annual savings of roughly
US$30,000.00 everytime he comes home for his home
leave. I realize that this is not a regular transaction but I
am seeking your favor to give me a chance to reserve
whatever values I can still recover from the properties and
to avoid any legal complications that may arise as a
consequence of the total loss of the Balangay lot. I hope that
you will extend to me your favorable action on this grave
matter.
Letter dated August 16, 1985 of Planters Bank:
Regarding your letter dated July 18, 1985, requesting that
we give up to August 31, 1985 to buy back your house and
lot and restaurant and building subject to a P300,000.00
downpayment on the purchase price, please be advised that
the Collection Committee has agreed to your request.
COMPILED BY: WIGMORE #WIGMOREFOREVER 70

SALES Case Digest (Atty. Sarona)


Compiled by: Wigmore #wigmoreforever
Please see Mr. Rene Castillo, Head, Acquired Assets Unit,
as soon as possible for the details of the transaction so that
they may work on the necessary documentation.
Given the above, the basic question that comes to mind is:
Was the offer certain and the acceptance absolute enough
so as to engender a meeting of the minds between the
parties? Definitely not.
While the foregoing letters indicate the amount of P300,000.00
as down payment, they are, however, completely silent as to
how the succeeding instalment payments shall be made.
At most, the letters merely acknowledge that the down payment
of P300,000.00 was agreed upon by the parties. However, this
fact cannot lead to the conclusion that a contract of sale
had been perfected. Quite recently, this Court held that before
a valid and binding contract of sale can exist, the manner of
payment of the purchase price must first be established since
the agreement on the manner of payment goes into the price
such that a disagreement on the manner of payment is
tantamount to a failure to agree on the price.
The Navarras letter/offer failed to specify a definite amount of
the purchase price for the sale/repurchase of the subject
properties. It merely stated that the "purchase price will be based
on the redemption value plus accrued interest at the prevailing
rate up to the date of the sales contract."
The ambiguity of this statement only bolsters the uncertainty
of the Navarras so-called "offer" for it leaves much rooms for
such questions, as: what is the redemption value? What
prevailing rate of interest shall be followed: is it the rate
stipulated in the loan agreement or the legal rate? When will the
date of the contract of sale be based, shall it be upon the time of
the execution of the deed of sale or upon the time when the last
instalment payment shall have been made? To our mind, these
questions need first to be addressed, discussed and negotiated
upon by the parties before a definite purchase price can be
arrived at. Significantly, the Navarras wrote in the same letter the
following:
Maybe you can give us a long-term payment scheme on the
basis of my brothers annual savings of roughly
US$30,000.00 every time he comes home for his home
leave.
Again, the offer was not clear insofar as concerned the exact
number of years that will comprise the long-term payment
scheme. As we see it, the absence of a stipulated period within
which the repurchase price shall be paid all the more adds to the
indefiniteness of the Navarras offer. Clearly, then, the lack of a
definite offer on the part of the spouses could not possibly serve
as the basis of their claim that the sale/repurchase of their
foreclosed properties was perfected. The reason is obvious: one
essential element of a contract of sale is wanting: the price
certain. Here, what is dramatically clear is that there was no
meeting of minds vis-a-vis the price, expressly or impliedly,
directly or indirectly.
Further, the tenor of Planters Banks letter reply negates the
contention of the Navarras that the Bank fully accepted their
offer. The letter specifically stated that there is a need to
negotiate on the other details of the transaction before the sale

may be formalized.
Such statement in the Banks letter clearly manifests lack of
agreement between the parties as to the terms of the purported
contract of sale/repurchase, particularly the mode of payment of
the purchase price and the period for its payment. The law
requires acceptance to be absolute and unqualified.
As it is, the Banks letter is not the kind which would constitute
acceptance as contemplated by law for it does not evince any
categorical and unequivocal undertaking on the part of the Bank
to sell the subject properties to the Navarras.
The Navarras attempt to prove the existence of a perfected
contract of sale all the more becomes futile in the light of the
evidence that there was in the first place no acceptance of their
offer. It should be noted that aside from their first letter dated
July 18, 1985, the Navarras wrote another letter dated August
20, 1985, this time requesting the Bank that the down payment of
P300,000.00 be instead taken from the excess payment made by
the RRRC in redeeming its own foreclosed properties.
The very circumstance that the Navarras had to make this new
request is a clear indication that no definite agreement has yet
been reached at that point. As we see it, this request constitutes
a new offer on the part of the Navarras, which offer was again
conditionally accepted by the Bank as in fact it even required the
Navarras to submit a board resolution of RRRC before it could
proceed with the proposed sale/repurchase.
The eventual failure of the spouses to submit the required board
resolution precludes the perfection of a contract of
sale/repurchase between the parties.
Evidently, what transpired between the parties was only a
prolonged negotiation to buy and to sell, and, at the most, an
offer and a counter offer with no definite agreement having been
reached by them. With the hard reality that no perfected
contract of sale/repurchase exists in this case, any
independent transaction between the Planters Bank and a thirdparty, like the one involving the Gatchalian Realty, cannot be
affected.
AMADO VS. SALVADOR
G.R. NO. 171401 DECEMBER 13, 2007
FACTS: Judge Amado is the owner of a lot, a portion of which is
the subject of the present litigation. It was alleged that sometime
in 1979, Judge Amado and Salvador agreed that the latter
would sell the lot in favor of Salvador at P60/sqm. The
payment was to be made in cash or construction material,
whichever the Judge preferred and to whomever the latter
wished during his lifetime. The terms of payment, though, were
not stipulated.
Thereafter, Salvador undertook and the location of the squatters
in said land and eventually built several structures thereon for his
business. Salvador claims that by October 1980, he had already
given Judge Amado total cash advances of P30,310.93 and
delivered construction materials amounting to P36,904.45, the
total of which exceeded the agreed price for the subject
property.
COMPILED BY: WIGMORE #WIGMOREFOREVER

71

SALES Case Digest (Atty. Sarona)


Compiled by: Wigmore #wigmoreforever
Petitioner heirs averred that Judge Amado and Salvador were
co-borrowers from a bank. A loan agreement was executed by
them with Capitol City Devt bank as lender and the Lot of Judge
Amado was used as collateral. The loaned amount was released
to Salvador and Judge Amados share was paid to him in several
instalments. Salvador failed to pay his share in the amortization
of the lot so that Judge Amado had to pay the loan to avoid
foreclosure.
Thereafter, Judge Amado demanded Salvador to leave the
premises and an ejectment case was filed to that effect.
Salvador filed a case for specific performance contending that a
balance of P4,040.62 was not paid to Judge Amado because of
the latters failure to execute the deed of sale. Salvador
presented several documentary evidence.
RTC dismissed the complaint because Salvadors evidence does
not show that the money and construction materials were
intended as payment for the subject property. CA reversed the
decision on the finding that the construction materials delivered
were not paid for.
ISSUE: WON there was a perfected contract of sale. NO
HELD: No Convincing Proof as to Manner of Payment
In the present case, Salvador fails to allege the manner of
payment of the purchase price on which the parties should
have agreed. No period was set within which the payment must
be made. Of the purchase price of P66,360.00, which the
parties purportedly agreed upon, the amount which should be
paid in cash and the amount for construction materials was not
determined. This means that the parties had no exact notion of
the consideration for the contract to which they supposedly gave
their consent. Thus, such failure is fatal to Salvadors claim that a
sale had been agreed upon by the parties.
Furthermore, after carefully examining the records, serious
doubts became apparent as to whether cash advances and
deliveries of construction materials evidenced by numerous
statements of accounts and delivery receipts were actually
intended as payment for the land.
First of all, the statements of accounts and the delivery receipts
do not indicate that the construction materials or the cash
advances were made in connection with the sale of the subject
property. Any doubt as to the real meaning of the contract must
be resolved against the person who drafted the instrument and is
responsible for the ambiguity thereof. Since Salvador prepared
these statements of accounts and therefore caused the
ambiguity, he cannot benefit from the resulting ambiguity.
Salvador is hardly an ignorant and illiterate person; rather, he
is a businessman engaged in manufacturing and distributing
construction materials and operates no less than two branches. It
should have been noted in the statement of accounts, or even in
another document, that the cash advances and deliveries of
construction materials were made in connection with a
transaction as important as a sale of land. As they are, the
statements of accounts and especially the straightforward
delivery receipts are insufficient proof that Judge Amado sold his
property to Salvador.
Secondly, one of the delivery receipts presented by Salvador
was partially paid. If Judge Amado had already agreed that the

construction materials were payment for the subject property, the


act of partially paying for construction materials would be
incongruous to such intention.
Thirdly, Salvador himself gave conflicting statements on whether
he has completed payment. Other proofs presented gave no
weight to respondents allegations. The testimony of the witness
presented by Salvador was not given credence. Finally, the act of
Salvador in relocating the squatters is not substantial proof of
ownership.
BANK OF COMMERCE VS MANALO
FACTS: The Xavierville Estate, Inc. was the owner of parcels of
land in Quezon City, known as the Xavierville Estate Subdivision,
with an area of 42 hectares. XEI caused the subdivision of the
property into residential lots, which was then offered for sale to
individual lot buyers.
Sometime in 1972, then XEI president Emerito Ramos, Jr.
contracted the services of Engr. Carlos Manalo, Jr. who was in
business of drilling deep water wells and installing pumps under
the business name Hurricane Commercial, Inc. For P34,887.66,
Manalo, Jr. installed a water pump at Ramos residence at the
corner of Aurora Boulevard and Katipunan Avenue, Quezon City.
Manalo, Jr. then proposed to XEI, through Ramos, to purchase a
lot in the Xavierville subdivision, and offered as part of the
downpayment the P34,887.66 Ramos owed him. XEI, through
Ramos, agreed. In a letter dated February 8, 1972, Ramos
requested Manalo, Jr. to choose which lots he wanted to buy so
that the price of the lots and the terms of payment could be fixed
and incorporated in the conditional sale. Manalo, Jr. met with
Ramos and informed him that he and his wife Perla had chosen
Lots 1 and 2 of Block 2 with a total area of 1,740.3 square
meters.
In a letter dated August 22, 1972 to Perla Manalo, Ramos
confirmed the reservation of the lots. He also pegged the price of
the lots at P200.00 per square meter, or a total of P348,060.00,
with a 20% down payment of the purchase price amounting to
P69,612.00 less the P34,887.66 owing from Ramos, payable on
or before December 31, 1972; the corresponding Contract of
Conditional Sale would then be signed on or before the same
date, but if the selling operations of XEI resumed after December
31, 1972, the balance of the downpayment would fall due then,
and the spouses would sign the aforesaid contract within 5 days
from receipt of the notice of resumption of such selling
operations. It was also stated in the letter that, in the meantime,
the spouses may introduce improvements thereon subject to the
rules and regulations imposed by XEI in the subdivision. Perla
Manalo conformed to the letter agreement.
The spouses Manalo took possession of the property on
September 2, 1972, constructed a house thereon, and installed a
fence around the perimeter of the lots.
The spouses Manalo were notified of the resumption of the
selling operations of XEI. However, they did not pay the balance
of the downpayment on the lots because Ramos failed to
prepare a contract of conditional sale and transmit the same to
Manalo for their signature. On August 14, 1973, Perla Manalo
went to the XEI office and requested that the payment of the
amount representing the balance of the downpayment be
COMPILED BY: WIGMORE #WIGMOREFOREVER 72

SALES Case Digest (Atty. Sarona)


Compiled by: Wigmore #wigmoreforever
deferred, which, however, XEI rejected. On August 10, 1973, XEI
furnished her with a statement of their account as of July 31,
1973, showing that they had a balance of P34,724.34 on the
downpayment of the two lots after deducting the account of
Ramos, plus P3,819.68 interest thereon from September 1, 1972
to July 31, 1973, and that the interests on the unpaid balance of
the purchase price of P278,448.00 from September 1, 1972 to
July 31, 1973 amounted to P30,629.28. The spouses were
informed that they were being billed for said unpaid interests.
Subsequently, XEI turned over its selling operations to OBM,
including the receivables for lots already contracted and those
yet to be sold. Subsequently, the Commercial Bank of Manila
(CBM) acquired the Xavierville Estate from OBM.
In a letter dated August 5, 1986, the CBM requested Perla
Manalo to stop any on-going construction on the property since it
(CBM) was the owner of the lot and she had no permission for
such construction. She agreed to have a conference meeting
with CBM officers where she informed them that her husband
had a contract with OBM, through XEI, to purchase the property.
When asked to prove her claim, she promised to send the
documents to CBM. However, she failed to do so. On September
5, 1986, CBM reiterated its demand that it be furnished with the
documents promised, but Perla Manalo did not respond.
On July 27, 1987, CBM filed a complaint for unlawful detainer
against the spouses with the MTC Court of Quezon City.
In the meantime, the CBM was renamed the Boston Bank of the
Philippines. After CBM filed its complaint against the spouses
Manalo, the latter filed a complaint for specific performance and
damages against the bank before the RTC of Quezon City.
Boston Bank, now petitioner, maintains that, as held by the CA,
the records do not reflect any schedule of payment of the 80%
balance of the purchase price, or P278,448.00. Petitioner insists
that unless the parties had agreed on the manner of payment of
the principal amount, including the other terms and conditions of
the contract, there would be no existing contract of sale or
contract to sell.
ISSUE: WON the manner of payment has been agreed upon and
WON it is essential for there to be an existing contract of sale or
contract to sell
RULING: No, it was not agreed upon thus, there was no contract
to sell. The Court agrees with petitioners contention that, for a
perfected contract of sale or contract to sell to exist in law, there
must be an agreement of the parties, not only on the price of the
property sold, but also on the manner the price is to be paid by
the vendee.
Under Article 1458 of the New Civil Code, in a contract of sale,
whether absolute or conditional, one of the contracting parties
obliges himself to transfer the ownership of and deliver a
determinate thing, and the other to pay therefor a price certain in
money or its equivalent. A contract of sale is perfected at the
moment there is a meeting of the minds upon the thing which is
the object of the contract and the price. From the averment of
perfection, the parties are bound, not only to the fulfillment of
what has been expressly stipulated, but also to all the
consequences which, according to their nature, may be in
keeping with good faith, usage and law. On the other hand, when

the contract of sale or to sell is not perfected, it cannot, as an


independent source of obligation, serve as a binding juridical
relation between the parties.
A definite agreement as to the price is an essential element of a
binding agreement to sell personal or real property because it
seriously affects the rights and obligations of the parties. Price is
an essential element in the formation of a binding and
enforceable contract of sale. The fixing of the price can never be
left to the decision of one of the contracting parties. But a price
fixed by one of the contracting parties, if accepted by the other,
gives rise to a perfected sale.
It is not enough for the parties to agree on the price of the
property. The parties must also agree on the manner of payment
of the price of the property to give rise to a binding and
enforceable contract of sale or contract to sell. This is so
because the agreement as to the manner of payment goes into
the price, such that a disagreement on the manner of payment is
tantamount to a failure to agree on the price.
In a contract to sell property by installments, it is not enough that
the parties agree on the price as well as the amount of
downpayment. The parties must, likewise, agree on the manner
of payment of the balance of the purchase price and on the other
terms and conditions relative to the sale. Even if the buyer
makes a downpayment or portion thereof, such payment cannot
be considered as sufficient proof of the perfection of any
purchase and sale between the parties.
There is no showing, in the records, of the schedule of payment
of the balance of the purchase price on the property amounting
to P278,448.00. The said parties confined themselves to
agreeing on the price of the property (P348,060.00), the 20%
downpayment of the purchase price (P69,612.00), and credited
respondents for the P34,887.00 owing from Ramos as part of the
20% downpayment. The determination of the terms of payment
of the P278,448.00 had yet to be agreed upon on or before
December 31, 1972, or even afterwards, when the parties sign
the corresponding contract of conditional sale.
Jurisprudence is that if a material element of a contemplated
contract is left for future negotiations, the same is too indefinite to
be enforceable. And when an essential element of a contract is
reserved for future agreement of the parties, no legal obligation
arises until such future agreement is concluded.
Indeed, the parties are in agreement that there had been no
contract of conditional sale ever executed by XEI, OBM or
petitioner, as vendor, and the respondents, as vendees.
Respondents failed to allege and prove, in the trial court, that, as
a matter of business usage, habit or pattern of conduct, XEI
granted all lot buyers the right to pay the balance of the purchase
price in installments of 120 months of fixed amounts with precomputed interests, and that XEI and the respondents had
intended to adopt such terms of payment relative to the sale of
the two lots in question. Habit, custom, usage or pattern of
conduct must be proved like any other facts.
As a consequence, respondents and XEI (or OBM for that
matter) failed to forge a perfected contract to sell the two lots;
hence, respondents have no cause of action for specific
performance against petitioner.
COMPILED BY: WIGMORE #WIGMOREFOREVER 73

SALES Case Digest (Atty. Sarona)


Compiled by: Wigmore #wigmoreforever
5.

Inadequacy of Price Does Not Affect Ordinary Sale

THE DIRECTOR OF LANDS, vs. ABARCA, ET AL


FACTS: About fourteen years, the lot now in question was the
subject of litigation between Datu Bualan and his co-claimants,
on the one hand, and Ciriaco Lizada, on the other. Juan A.
Sarenas and Domingo Braganza were the attorneys for Datu
Bualan and his co-claimants in that suit, wherein a judgment was
rendered declaring Datu Bualan and his co-claimants the owners
of the land involved in the litigation.
Subsequently, a controversy arose between the Bagobos and
their attorneys as to the amount of fees due the latter,
whereupon the attorneys took possession of the property now in
question. Action was brought by the Bagobos against their
former attorneys for the recovery of the land. In this action
judgment was rendered ordering the attorneys to return the
property seized by them, and requiring the Bagobos to pay their
former attorneys the sum of P6,000 as fees. As a result of this
judgment Datu Bualan and his co-claimants paid Sarenas and
Braganza the sum of P5,126.13. They also paid to the municipal
treasurer of Davao in the name of Sarenas and Braganza, for
taxes and penalties due on the property in the year 1926, while
the same was in the possession of the latter, the sum of
P1,035.87. The Bagobos assumed that, by these payments
which amounted in all to P6,162, the judgment rendered against
them for P6,000 together with interests due thereon, was fully
satisfied.
Claiming that the sum paid to the municipal treasurer of Davao
should not be credited on the amount of the judgment obtained
by them, Sarenas and Braganza caused the clerk of the court to
issue a writ of execution on the said judgment. By reason of the
writ of execution so issued, the sheriff levied on the property here
in question and sold it to Sarenas and Braganza for the sum of
P877.25. Upon the failure of the Bagobos to redeem the
property, they filed their claim in the present cadastral case,
alleging that they were the absolute owners of the lot in question.
In view of the evidence presented by the parties, the lower court
dismissed the claim of Sarenas and Braganza, and ordered the
registration of the lot now in question in the names of Datu
Bualan and his co-claimants, subject, however, to a lien in favor
of Sarenas and Braganza for the sum of P877.25, with interest at
the rate of 12 per cent per annum from April 27, 1927.
In dismissing the claim of Sarenas and Braganza, the lower court
held that the sale by the sheriff of the property in question in
favor of said claimants was null and void, because the amount of
P877.25 paid by Sarenas and Braganza was absolutely
inadequate.
ISSUE: WON a judicial sale of real property will be set aside
when price is adequate
RULING: Yes if the price is so inadequate as to shock the
conscience of the court.
The lower court was right in declaring the sheriff's sale null and
void on the ground of the inadequacy of the price paid. It appears
that in 1927 the assessed value of the contested property was

more than P60,000. A judicial sale of real property will be set


aside when the price is so inadequate as to shock the
conscience of the court. (National Bank vs. Gonzalez, 45 Phil.,
693.)
In the instant case there is another important consideration. In
fairness and equity, which after all are the true aims of the law,
the amount paid by Datu Bualan and his co-claimants for taxes
and penalties due on the contested property should be credited
on the judgment obtained by Sarenas and Braganza. Such taxes
and penalties accrued while the property was in that possession
under a claim of ownership. It follows that the error assigned by
Datu Bualan and his co-claimants against the judgment below, to
the effect that the lower court erred in subjecting the property
sought to be registered to a lien in favor of Sarenas and
Braganza for P877.25 with interests, must be sustained.

BRAVO-GUERRERO vs. EDWARD P. BRAVO


FACTS: Spouses Mauricio and Simona Bravo owned 2 parcels
of land measuring 287 and 291 square meters and located in
Makati City, Metro Manila. The Properties are registered under
TCT Nos. 58999 and 59000 issued by the Register of Deeds of
Rizal on 23 May 1958. The Properties contain a large residential
dwelling, a smaller house and other improvements.
Mauricio and Simona had three children - Roland, Cesar and
Lily, all surnamed Bravo. Cesar died without issue. Lily Bravo
married David Diaz, and had a son, David B. Diaz, Jr. ("David
Jr."). Roland had six children, namely, Elizabeth Bravo-Guerrero,
Edward, Roland, Senia, Benjamin, and their half-sister, Ofelia.
Simona executed a General Power of Attorney ("GPA") on 17
June 1966 appointing Mauricio as her attorney-in-fact. In the
GPA, Simona authorized Mauricio to "mortgage or otherwise
hypothecate, sell, assign and dispose of any and all of my
property, real, personal or mixed, of any kind whatsoever and
wheresoever situated, or any interest therein." Mauricio
subsequently mortgaged the Properties to the PNB and DBP for
P10,000 and P5,000, respectively.
On 25 October 1970, Mauricio executed a Deed of Sale with
Assumption of Real Estate Mortgage conveying the Properties to
vendees Roland A. Bravo, Ofelia A. Bravo and Elizabeth BravoGuerrero. The sale was conditioned on the payment of P1,000
and on the assumption by the vendees of the PNB and DBP
mortgages over the Properties.
As certified by the Clerk of Court of the Regional Trial Court of
Manila, the Deed of Sale was notarized by Atty. Victorio Q.
Guzman on 28 October 1970 and entered in his Notarial
Register. However, the Deed of Sale was not annotated on TCT
Nos. 58999 and 59000. Neither was it presented to PNB and
DBP. The mortage loans and the receipts for loan payments
issued by PNB and DBP continued to be in Mauricios name
even after his death on 20 November 1973. Simona died in
1977.
On 23 June 1997, Edward, represented by his wife, Fatima
Bravo, filed an action for the judicial partition of the Properties.
Edward claimed that he and the other grandchildren of Mauricio
and Simona are co-owners of the Properties by succession.
Despite this, petitioners refused to share with him the possession
COMPILED BY: WIGMORE #WIGMOREFOREVER 74

SALES Case Digest (Atty. Sarona)


Compiled by: Wigmore #wigmoreforever
and rental income of the Properties. Edward later amended his
complaint to include a prayer to annul the Deed of Sale, which he
claimed was merely simulated to prejudice the other heirs.
The trial court upheld Mauricios sale of the Properties to the
vendees. The trial court ruled that the sale did not prejudice the
compulsory heirs, as the Properties were conveyed for valuable
consideration.

Respondents argue that P16,000 is still far below the actual


value of the Properties The tax declarations placed the assessed
value of both Properties at P16,160. Compared to this, the price
of P16,000 cannot be considered grossly inadequate, much less
so shocking to the conscience as to justify the setting aside of
the Deed of Sale.
PART III: FORMATION OF CONTRACT OF SALE

Citing Article 166 of the Civil Code, the Court of Appeals


reversed trial courts decision and declared the Deed of Sale void
for lack of Simonas consent. It also found that there was
insufficient proof that the vendees made the mortgage payments
on the Properties, since the PNB and DBP receipts were issued
in Mauricios name. The appellate court opined that the rental
income of the Properties, which the vendees never shared with
respondents, was sufficient to cover the mortgage payments to
PNB and DBP.
ISSUE: WON the sale of the properties was simulated or void for
gross inadequacy of price
RULING: No, the sale of the properties is not void either for
being simulated or for inadequacy of price.
Respondents, however, contend that the sale of the Properties
was merely simulated. As proof, respondents point to the
consideration of P1,000 in the Deed of Sale, which respondents
claim is grossly inadequate compared to the actual value of the
Properties.
Simulation of contract and gross inadequacy of price are distinct
legal concepts, with different effects. When the parties to an
alleged contract do not really intend to be bound by it, the
contract is simulated and void. A simulated or fictitious contract
has no legal effect whatsoever because there is no real
agreement between the parties.
In contrast, a contract with inadequate consideration may
nevertheless embody a true agreement between the parties. A
contract of sale is a consensual contract, which becomes valid
and binding upon the meeting of minds of the parties on the price
and the object of the sale. The concept of a simulated sale is
thus incompatible with inadequacy of price. When the parties
agree on a price as the actual consideration, the sale is not
simulated despite the inadequacy of the price.
Gross inadequacy of price by itself will not result in a void
contract. Gross inadequacy of price does not even affect the
validity of a contract of sale, unless it signifies a defect in the
consent or that the parties actually intended a donation or some
other contract. Inadequacy of cause will not invalidate a contract
unless there has been fraud, mistake or undue influence. In this
case, respondents have not proved any of the instances that
would invalidate the Deed of Sale.
Respondents even failed to establish that the consideration paid
by the vendees for the Properties was grossly inadequate. As the
trial court pointed out, the Deed of Sale stipulates that, in
addition to the payment of P1,000, the vendees should assume
the mortgage loans from PNB and DBP. The consideration for
the sale of the Properties was thus P1,000 in cash and the
assumption of the P15,000 mortgage.

I. POLICITATION
1. Option Contract

TAYAG vs. LACSON


FACTS: Respondents Angelica Tiotuyco Vda. de Lacson, and
her children were the registered owners of three parcels of land
located in Mabalacat, Pampanga. The properties were tenanted
agricultural lands.
On March 17, 1996, a group of original farmers/tillers, Tiamson,
et al., individually executed in favor of the petitioner separate
Deeds of Assignment in which the assignees assigned to the
petitioner their respective rights as tenants/tillers of the
landholdings possessed and tilled by them for and in
consideration of P50.00 per square meter. The said amount was
made payable "when the legal impediments to the sale of the
property to the petitioner no longer existed." The petitioner was
also granted the exclusive right to buy the property if and when
the respondents, with the concurrence of the defendants-tenants,
agreed to sell the property. In the interim, the petitioner gave
varied sums of money to the tenants as partial payments, and
the latter issued receipts for the said amounts.
On July 24, 1996, the petitioner called a meeting of the
defendants-tenants to work out the implementation of the terms
of their separate agreements.
However, on August 8, 1996, the defendants-tenants, through
Joven Mariano, wrote the petitioner stating that they were not
attending the meeting and instead gave notice of their collective
decision to sell all their rights and interests, as tenants/lessees,
over the landholding to the respondents Lacson.
On August 19, 1996, the petitioner filed a complaint against the
defendants-tenants, as well as the respondents, for the court to
fix a period within which to pay the agreed purchase price of
P50.00 per square meter to the defendants, as provided for in
the Deeds of Assignment.
Respondents as defendants asserted that they never induced
the defendants Tiamson to violate their contracts with the
petitioner; and, being merely tenants-tillers, the defendantstenants had no right to enter into any transactions involving their
properties without their knowledge and consent. They also
averred that the transfers or assignments of leasehold rights
made by the defendants-tenants to the petitioner is contrary to
Presidential Decree (P.D.) No. 27 and Republic Act No. 6657, the
Comprehensive Agrarian Reform Program (CARP).
The defendants-tenants Tiamson, et al., alleged in their answer
COMPILED BY: WIGMORE #WIGMOREFOREVER

75

SALES Case Digest (Atty. Sarona)


Compiled by: Wigmore #wigmoreforever
with counterclaim for damages, that the money each of them
received from the petitioner were in the form of loans, and that
they were deceived into signing the deeds of assignment. What
they knew was that they were made to sign a document that will
serve as a receipt for the loan granted to them by the plaintiff.
ISSUE: WON the Deeds of Assignment are perfected option
contracts
RULING: No, there is no perfected option contract.
The Court does not agree with the contention of the petitioner
that the deeds of assignment executed by the defendantstenants are perfected option contracts. An option is a contract by
which the owner of the property agrees with another person that
he shall have the right to buy his property at a fixed price within a
certain time. It is a condition offered or contract by which the
owner stipulates with another that the latter shall have the right to
buy the property at a fixed price within a certain time, or under, or
in compliance with certain terms and conditions, or which gives
to the owner of the property the right to sell or demand a sale. It
imposes no binding obligation on the person holding the option,
aside from the consideration for the offer. Until accepted, it is not,
properly speaking, treated as a contract. The second party gets
not lands, not an agreement that he shall have the lands, but the
right to call for and receive lands if he elects. An option contract
is a separate and distinct contract from which the parties may
enter into upon the conjunction of the option.
In this case, the defendants-tenants-subtenants, under the deeds
of assignment, granted to the petitioner not only an option but the
exclusive right to buy the landholding. But the grantors were
merely the defendants-tenants, and not the respondents, the
registered owners of the property. Not being the registered
owners of the property, the defendants-tenants could not legally
grant to the petitioner the option, much less the "exclusive right"
to buy the property. As the Latin saying goes, "NEMO DAT
QUOD NON HABET."
ADELFA PROPERTIES VS. CA
[G.R. NO. 111238. JANUARY 25, 1995.]
PARTIES: Roasrio and Salud Jimenez Seller
Adelfa Properties Buyer
Subject:: western portion of a parcel of land 8855 sq. ms.
Covered by TCT 309773 situated in Barrio Culasi, Las Pinas,
Metro Manila
FACTS: Rosario Jimenez-Castaneda, Salud Jimenez and their
brothers, Jose and Dominador Jimenez, were the registered coowners of a parcel of land consisting of 17,710 sq. ms
(TCT 309773) situated in Barrio Culasi, Las Pias, Metro
Manila. On 28 July 1988, Jose and Dominador Jimenez sold
their share consisting of 1/2 of said parcel of land, specifically the
eastern portion thereof, to Adelfa Properties pursuant to a
Kasulatan sa Bilihan ng Lupa. Subsequently, a Confirmatory
Extrajudicial Partition Agreement was executed by the
Jimenezes, wherein the eastern portion of the subject lot, with an
area of 8,855 sq. ms. was ADJUDICATED TO JOSE AND
DOMINADOR JIMENEZ, WHILE THE WESTERN PORTION
WAS ALLOCATED TO ROSARIO AND SALUD Jimenez.

Thereafter, Adelfa Properties expressed interest in buying the


western portion of the property from Rosario and Salud.
Accordingly, on 25 November 1989, an Exclusive Option to
Purchase was executed between the parties, with the condition
that the selling price shall be P2,856,150, that the option money
of P50,000 shall be credited as partial payment upon the
consummation of sale, that the balance is to be paid on or before
30 November 1989, and that in case of default by Adelfa
Properties to pay the balance, the option is cancelled and 50% of
the option money shall be forfeited and the other 50% refunded
upon the sale of the property to a third party.
Before Adelfa Properties could make payment, it received
summons on 29 November 1989, together with a copy of a
complaint filed by the nephews and nieces of Rosario and Salud
against the latter, Jose and Dominador Jimenez, and Adelfa
Properties in the RTC Makati (Civil Case 89-5541), for
annulment of the deed of sale in favor of Household Corporation
and recovery of ownership of the property covered by TCT
309773.
As a consequence, in a letter dated 29 November 1989, Adelfa
Properties informed Rosario and Salud that it would hold
payment of the full purchase price and suggested that the latter
settle the case with their nephews and nieces. . Salud Jimenez
refused to heed the suggestion of Adelfa Properties and
attributed the suspension of payment of the purchase price to
lack of word of honor.
On 14 December 1989, Rosario and Salud sent Francisca
Jimenez to see Atty. Bernardo, in his capacity as Adelfa
Properties counsel, and to inform the latter that they were
cancelling the transaction. In turn, Atty. Bernardo offered to pay
the purchase price provided that P500,000.00 be deducted
therefrom for the settlement of the civil case. This was rejected
by Rosario and Salud. On 22 December 1989, Atty. Bernardo
wrote Rosario and Salud on the same matter but this time
reducing the amount from P500,000.00 to P300,000.00, and this
was also rejected by the latter. On 23 February 1990, the RTC
dismissed Civil Case 89-5541.
On 16 April 1990, Atty. Bernardo wrote Rosario and Salud
informing the latter that in view of the dismissal of the case
against them, Adelfa Properties was willing to pay the purchase
price, and he requested that the corresponding deed of absolute
sale be executed. This was ignored by Rosario and Salud. On 27
July 1990, Jimenez counsel sent a letter to Adelfa Properties
enclosing therein a check for P25,000.00 representing the refund
of 50% of the option money paid under the exclusive option to
purchase. Rosario and Salud then requested Adelfa Properties
to return the owners duplicate copy of the certificate of title of
Salud Jimenez. Adelfa Properties failed to surrender the
certificate of title.
Rosario and Salud Jimenez filed Civil Case 7532 in the RTC
Pasay City (Branch 113) for annulment of contract with
damages, praying, among others, that the exclusive option to
purchase be declared null and void; that Adelfa Properties be
ordered to return the owners duplicate certificate of title; and that
the annotation of the option contract on TCT 309773 be
cancelled.
RTC: On 5 September 1991, the trial court rendered judgment
holding that the agreement entered into by the parties was
COMPILED BY: WIGMORE #WIGMOREFOREVER 76

SALES Case Digest (Atty. Sarona)


Compiled by: Wigmore #wigmoreforever
merely an option contract, and declaring that the suspension of
payment by Adelfa Properties constituted a counter-offer which,
therefore, was tantamount to a rejection of the option. It likewise
ruled that Adelfa Properties could not validly suspend payment in
favor of Rosario and Salud on the ground that the vindicatory
action filed by the latters kin did not involve the western portion
of the land covered by the contract between the parties, but the
eastern portion thereof which was the subject of the sale
between Adelfa Properties and the brothers Jose and Dominador
Jimenez. The trial court then directed the cancellation of the
exclusive option to purchase.
On appeal,
RTC: the Court of appeals affirmed in toto the decision of the
court a quo. That Article 1590 of the Civil Code on suspension of
payments applies only to a contract of sale or a contract to sell,
but not to an option contract which it opined was the nature of
the document subject of the case at bar.
Hence, the petition for review on certiorari.
Adelfa properties posits that the contract is a Contract of Sale
and not an Option Contract or Contract to Sell, making the
suspension of payment applicable in the case.
ISSUE: Whether or not the contract is a Contract of Sale , Option
Contract or Contract to Sell.
SC: The Supreme Court affirmed the assailed judgment of the
Court of Appeals in CA-GR CV 34767, with modificatory
premises.
Agreement between parties a contract to sell and not an
option contract or a contract of sale
The alleged option contract is a contract to sell, rather than a
contract of sale. The distinction between the two is important for
in contract of sale, the title passes to the vendee upon the
delivery of the thing sold; whereas in a contract to sell, by
agreement the ownership is reserved in the vendor and is not to
pass until the full payment of the price. In a contract of sale, the
vendor has lost and cannot recover ownership until and unless
the contract is resolved or rescinded; whereas in a contract to
sell, title is retained by the vendor until the full payment of the
price Thus, a deed of sale is considered absolute in nature
where there is neither a stipulation in the deed that title to the
property sold is reserved in the seller until the full payment of the
price, nor one giving the vendor the right to unilaterally resolve
the contract the moment the buyer fails to pay within a fixed
period.
That the parties really intended to execute a contract to sell is
bolstered by the fact that the deed of absolute sale would have
been issued only upon the payment of the balance of the
purchase price, as may be gleaned from Adelfa Properties letter
dated 16 April 1990 wherein it informed the vendors that it is
now ready and willing to pay you simultaneously with the
execution of the corresponding deed of absolute sale.
Contract interpreted to ascertain intent of parties; Title not
controlling if text shows otherwise. The important task in
contract interpretation is always the ascertainment of the
intention of the contracting parties and that task is to be
discharged by looking to the words they used to project that
intention in their contract, all the words not just a particular word

or two, and words in context not words standing alone.


Moreover, judging from the subsequent acts of the parties which
will hereinafter be discussed, it is undeniable that the intention of
the parties was to enter into a contract to sell. In addition, the title
of a contract does not necessarily determine its true nature.
Hence, the fact that the document under discussion is entitled
Exclusive Option to Purchase is not controlling where the text
thereof shows that it is a contract to sell.
Test to determine contract as a contract of sale or
purchase or mere option. The test in determining whether a
contract is a contract of sale or purchase or a mere option is
whether or not the agreement could be specifically enforced.
There is no doubt that Adelfas obligation to pay the purchase
price is specific, definite and certain, and consequently binding
and enforceable. Had the Jimenezes chosen to enforce the
contract, they could have specifically compelled Adelfa to pay the
balance of P2,806,150.00. This is distinctly made manifest in the
contract itself as an integral stipulation, compliance with which
could legally and definitely be demanded from petitioner as a
consequence.
Adelfa Properties justified in suspending payment of
balance by reason of vindicatory action filed against it. In
Civil Case 89-5541, it is easily discernible that, although the
complaint prayed for the annulment only of the contract of sale
executed between Adelfa Properties and the Jimenez brothers,
the same likewise prayed for the recovery of therein Jimenez
share in that parcel of land specifically covered by TCT 309773.
In other words, the Jimenezes were claiming to be co-owners of
the entire parcel of land described in TCT 309773, and not only
of a portion thereof nor did their claim pertain exclusively to the
eastern half adjudicated to the Jimenez brothers. Therefore,
Adelfa Properties was justified in suspending payment of the
balance of the purchase price by reason of the aforesaid
vindicatory action filed against it. The assurance made by the
Jimenezes that Adelfa Properties did not have to worry about the
case because it was pure and simple harassment is not the kind
of guaranty contemplated under the exceptive clause in Article
1590 wherein the vendor is bound to make payment even with
the existence of a vindicatory action if the vendee should give a
security for the return of the price.
Jimenezes may no longer be compelled to sell and deliver
subject property. Be that as it may, and the validity of the
suspension of payment notwithstanding, the Jimenezes may no
longer be compelled to sell and deliver the subject property to
Adelfa Properties for two reasons, that is, Adelfas failure to duly
effect the consignation of the purchase price after the
disturbance had ceased; and, secondarily, the fact that the
contract to sell had been validly rescinded by the Jimenezes.
Rescission in a contract to sell. Article 1592 of the Civil Code
which requires rescission either by judicial action or notarial act
is not applicable to a contract to sell. Furthermore, judicial action
for rescission of a contract is not necessary where the contract
provides for automatic rescission in case of breach, as in the
contract involved in the present controversy. By Adelfas failure
to comply with its obligation, the Jimenezes elected to resort to
and did announce the rescission of the contract through its letter
to Adelfa dated 27 July 1990. That written notice of rescission is
deemed sufficient under the circumstances.
WHEREFORE, on the foregoing modificatory premises, and
COMPILED BY: WIGMORE #WIGMOREFOREVER 77

SALES Case Digest (Atty. Sarona)


Compiled by: Wigmore #wigmoreforever
considering that the same result has been reached by
respondent Court of Appeals with respect to the relief awarded to
private respondents by the court a quo which we find to be
correct, its assailed judgment in CA-G.R. CV No. 34767 is
hereby AFFIRMED.

i.

Meaning of Separate Consideration

SPOUSES JULIO D. VILLAMOR AND MARINA VILLAMOR vs


THE HON. COURT OF APPEALS AND SPOUSES MACARIA
LABINGISA REYES AND ROBERTO REYES
G.R. No. 97332 October 10, 1991
FACTS: Macaria sold 300 square meters from her 600 square
meter lot to the spouses Villamor which is located at Baesa,
Caloocan City, for the total amount of P21,000.00.
Earlier, Macaria borrowed P2,000.00 from the spouses which
amount was deducted from the total purchase price of the 300
square meter lot sold.
Macaria executed a "Deed of Option" in favor of Villamor in which
the remaining 300 square meter portion (TCT No. 39934) of the
lot would be sold to Villamor under the conditions stated therein.
Included in the Deed of Option is:
That the only reason why the Spouses-vendees Julio Villamor
and Marina V. Villamor, agreed to buy the said one-half portion at
the above-stated price of about P70.00 per square meter, is
because I, and my husband Roberto Reyes, have agreed to sell
and convey to them the remaining one-half portion still owned by
me and now covered by TCT No. 39935 of the Register of Deeds
for the City of Caloocan, whenever the need of such sale arises,
either on our part or on the part of the spouses (Julio) Villamor
and Marina V. Villamor, at the same price of P70.00 per square
meter, excluding whatever improvement may be found the
thereon;
In 1984, when the husband of Macaria retired, wanted to
repurchased the said 300 square meter of lot from the
petitioners. However, petitioners rejected the said offer and
expresses their desire to purchase the remaining half of the lot.
Trial Court rendered judgment in favor of Villamor.
Court of Appeals reversed the decision premised on the finding
of respondent court that the Deed of Option is void for lack of
consideration.
ISSUE: Whether there is a Separate Consideration for the
Option Contract? YES
RULING: As expressed in Gonzales v. Trinidad, 67 Phil. 682,
consideration is "the why of the contracts, the essential reason
which moves the contracting parties to enter into the contract."
The cause or the impelling reason on the part of private
respondent executing the deed of option as appearing in the
deed itself is the petitioner's having agreed to buy the 300 square
meter portion of private respondents' land at P70.00 per square
meter "which was greatly higher than the actual reasonable
prevailing price."

The respondent appellate court failed to give due consideration


to petitioners' evidence which shows that in 1969 the Villamor
spouses bough an adjacent lot from the brother of Macaria
Labing-isa for only P18.00 per square meter which the private
respondents did not rebut. Thus, expressed in terms of money,
the consideration for the deed of option is the difference
between the purchase price of the 300 square meter portion
of the lot in 1971 (P70.00 per sq.m.) and the prevailing
reasonable price of the same lot in 1971. Whatever it is,
(P25.00 or P18.00) though not specifically stated in the deed
of option, was ascertainable. Petitioner's allegedly paying
P52.00 per square meter for the option may, as opined by the
appellate court, be improbable but improbabilities does not
invalidate a contract freely entered into by the parties.
The "deed of option" entered into by the parties in this case had
unique features. Ordinarily, an optional contract is a privilege
existing in one person, for which he had paid a consideration and
which gives him the right to buy, for example, certain
merchandise or certain specified property, from another person,
if he chooses, at any time within the agreed period at a fixed
price (Enriquez de la Cavada v. Diaz, 37 Phil. 982). If We look
closely at the "deed of option" signed by the parties, We will
notice that the first part covered the statement on the sale of the
300 square meter portion of the lot to Spouses Villamor at the
price of P70.00 per square meter "which was higher than the
actual reasonable prevailing value of the lands in that place at
that time (of sale)."
The second part stated that the only reason why the Villamor
spouses agreed to buy the said lot at a much higher price is
because the vendor (Reyeses) also agreed to sell to the
Villamors the other half-portion of 300 square meters of the land.
Had the deed stopped there, there would be no dispute that the
deed is really an ordinary deed of option granting the Villamors
the option to buy the remaining 300 square meter-half portion of
the lot in consideration for their having agreed to buy the other
half of the land for a much higher price. But, the "deed of option"
went on and stated that the sale of the other half would be made
"whenever the need of such sale arises, either on our (Reyeses)
part or on the part of the Spouses Julio Villamor and Marina V.
Villamor. It appears that while the option to buy was granted to
the Villamors, the Reyeses were likewise granted an option to
sell. In other words, it was not only the Villamors who were
granted an option to buy for which they paid a consideration. The
Reyeses as well were granted an option to sell should the need
for such sale on their part arise.
A contract of sale is, under Article 1475 of the Civil Code,
"perfected at the moment there is a meeting of minds upon the
thing which is the object of the contract and upon the price. From
that moment, the parties may reciprocally demand perform of
contracts." Since there was, between the parties, a meeting of
minds upon the object and the price, there was already a
perfected contract of sale. What was, however, left to be done
was for either party to demand from the other their respective
undertakings under the contract. It may be demanded at any
time either by the private respondents, who may compel the
petitioners to pay for the property or the petitioners, who may
compel the private respondents to deliver the property.
However, the Deed of Option did not provide for the period within
which the parties may demand the performance of their
respective undertakings in the instrument. The parties could not
COMPILED BY: WIGMORE #WIGMOREFOREVER 78

SALES Case Digest (Atty. Sarona)


Compiled by: Wigmore #wigmoreforever
have contemplated that the delivery of the property and the
payment thereof could be made indefinitely and render uncertain
the status of the land. The failure of either parties to demand
performance of the obligation of the other for an unreasonable
length of time renders the contract ineffective.
Under Article 1144 (1) of the Civil Code, actions upon written
contract must be brought within ten (10) years. The Deed of
Option was executed on November 11, 1971. The acceptance,
as already mentioned, was also accepted in the same
instrument. The complaint in this case was filed by the petitioners
on July 13, 1987, seventeen (17) years from the time of the
execution of the contract. Hence, the right of action had
prescribed. There were allegations by the petitioners that they
demanded from the private respondents as early as 1984 the
enforcement of their rights under the contract. Still, it was beyond
the ten (10) years period prescribed by the Civil Code.

ii.

No Separate Consideration

NICOLAS SANCHEZ VS. SEVERINA RIGOS


45 SCRA 368 JUNE 1972
FACTS: In an instrument entitled "Option to Purchase,"
executed on April 3, 1961, defendant-appellant Severina Rigos
"agreed, promised and committed ... to sell" to plaintiff-appellee
Nicolas Sanchez for the sum of P1,510.00 within two (2) years
from said date, a parcel of land situated in the barrios of Abar
and Sibot, San Jose, Nueva Ecija. It was agreed that said option
shall be deemed "terminated and elapsed," if Sanchez shall fail
to exercise his right to buy the property" within the stipulated
period. On March 12, 1963, Sanchez deposited the sum of
Pl,510.00 with the CFI of Nueva Ecija and filed an action for
specific performance and damages against Rigos for the latters
refusal to accept several tenders of payment that Sanchez made
to purchase the subject land.
Defendant Rigos contended that the contract between them was
only a unilateral promise to sell, and the same being
unsupported by any valuable consideration, by force of the New
Civil Code, is null and void." Plaintiff Sanchez, on the other hand,
alleged in his compliant that, by virtue of the option under
consideration, "defendant agreed and committed to sell" and "the
plaintiff agreed and committed to buy" the land described in the
option. The lower court rendered judgment in favor of Sanchez
and ordered Rigos to accept the sum Sanchez judicially
consigned, and to execute in his favor the requisite deed of
conveyance. The Court of Appeals certified the case at bar to the
Supreme Court for it involves a question purely of law.
ISSUE: Was there a contract to buy and sell between the parties
or only a unilateral promise to sell?
RULING: The option did not impose upon plaintiff the obligation
to purchase defendant's property. Annex A is not a "contract to
buy and sell." It merely granted plaintiff an "option" to buy. And
both parties so understood it, as indicated by the caption, "Option
to Purchase," given by them to said instrument. Under the
provisions thereof, the defendant "agreed, promised and
committed" herself to sell the land therein described to the
plaintiff for P1,510.00, but there is nothing in the contract to

indicate that her aforementioned agreement, promise and


undertaking is supported by a consideration "distinct from the
price" stipulated for the sale of the land.
There is no question that under article 1479 of the new Civil
Code "an option to sell," or "a promise to buy or to sell," as used
in said article, to be valid must be "supported by a consideration
distinct from the price." This is clearly inferred from the context of
said article that a unilateral promise to buy or to sell, even if
accepted, is only binding if supported by consideration. In other
words, "an accepted unilateral promise can only have a binding
effect if supported by a consideration which means that the
option can still be withdrawn, even if accepted, if the same is not
supported by any consideration. It is not disputed that the option
is without consideration. It can therefore be withdrawn
notwithstanding the acceptance of it by appellee.
In other words, since there may be no valid contract without a
cause or consideration, the promisor is not bound by his promise
and may, accordingly, withdraw it. Pending notice of its
withdrawal, his accepted promise partakes, however, of the
nature of an offer to sell which, if accepted, results in a perfected
contract of sale.
Indeed, the presumption is that, in the process of drafting the
Code, its author has maintained a consistent philosophy or
position. Moreover, the decision in Southwestern Sugar &
Molasses Co. v. Atlantic Gulf & Pacific Co., holding that Art. 1324
is modified by Art. 1479 of the Civil Code, in effect, considers the
latter as an exception to the former, and exceptions are not
favored, unless the intention to the contrary is clear, and it is not
so, insofar as said two (2) articles are concerned. What is more,
the reference, in both the second paragraph of Art. 1479 and Art.
1324, to an option or promise supported by or founded upon a
consideration, strongly suggests that the two (2) provisions
intended to enforce or implement the same principle.
In the present case the trial court found that the "Plaintiff (Nicolas
Sanchez) had offered the sum of Pl,510.00 before any
withdrawal from the contract has been made by the Defendant
(Severina Rigos)." Since Rigos' offer sell was accepted by
Sanchez, before she could withdraw her offer, a bilateral
reciprocal contract to sell and to buy was generated.
PERCELINO DIAMANTE vs. HON. COURT OF APPEALS and
GERARDO DEYPALUBUS
FACTS: A fishery lot, encompassing an area of 9.4 hectares and
designated as Lot No. 518-A of the Cadastral Survey of
Dumangas, Iloilo, was previously covered by Fishpond Permit
No. F-2021 issued in the name of Anecita Dionio. Upon Anecita's
death, her heirs, petitioner Diamante and Primitivo Dafeliz,
inherited the property which they later divided between
themselves; petitioner got 4.4. hectares while Dafeliz got 5
hectares. It is the petitioner's share that is the subject of the
present controversy. Primitivo Dafeliz later sold his share to
private respondent.
On 21 May 1959, petitioner sold to private respondent his
leasehold rights over the property in question for P8,000.00 with
the right to repurchase the same within three (3) years from said
date.
COMPILED BY: WIGMORE #WIGMOREFOREVER

79

SALES Case Digest (Atty. Sarona)


Compiled by: Wigmore #wigmoreforever
On 16 August 1960, private respondent filed an application with
the Bureau of Fisheries, dated 12 July 1960, for a fishpond
permit and a fishpond lease agreement over the entire lot,
submitting therewith the deeds of sale executed by Dafeliz and
the petitioner.
Pressed by urgent financial needs, petitioner, on 17 October
1960, sold all his remaining rights over the property in question
to the private respondent for P4,000.00.
On 25 October 1960, private respondent, with his wife's consent,
executed in favor of the petitioner an Option to Repurchase the
property in question within ten (10) years from said date, with a
ten-year grace period.
Private respondent submitted to the Bureau of Fisheries the
definite deed of sale; he did not, however, submit the Option to
Repurchase.

contract to sell and to buy is created, and the offeree ipso


facto assumes the obligations of a purchaser; the offeror, on the
other hand, would be liable for damages if he fails to deliver the
thing he had offered for sale.
The contract of option is a separate and distinct contract from the
contract which the parties may enter into upon the
consummation of the option, and a consideration for an optional
contract is just as important as the consideration for any other
kind of contract. Thus, a distinction should be drawn between
the consideration for the option to repurchase, and the
consideration for the contract of repurchase itself.
Even if the promise was accepted, private respondent was not
bound thereby in the absence of a distinct consideration.

BIBLE BAPTIST CHURCH and PASTOR REUBEN


BELMONTE vs CA and SPOUSES VILLANUEVA
G.R. No. 126454
November 26, 2004

Thereafter, on 2 August 1961, the Bureau of Fisheries issued to


private respondent Fishpond Permit
On 11 December 1963, petitioner, contending that he has a valid
twenty-year option to repurchase the subject property, requested
the Bureau of Fisheries to respondents permit insofar as the
said property is concerned. On 18 December 1964, his lettercomplaint was dismissed. Petitioner then sought a
reconsideration of the dismissal; the same was denied on 29
April 1965. His appeal to the Secretary of the DANR was likewise
dismissed on 30 October 1968. Again, on 20 November 1968,
petitioner sought for a reconsideration; this time, however, he
was successful.

FACTS: On June 7, 1985, the Bible Baptist Church entered into


a contract of lease with Spouses Villanueva over a property
located in Malate, Manila. The pertinent portions of the contract
are:
xxx
2. That lease shall take effect on June 7, 1985 and shall be for a
period of 15 years.
xxx

ISSUE: Was there a valid Option? No


RULING: It is settled by this Court that "an agreement to
repurchase becomes a promise to sell when made after the sale,
because when the sale is made without such an agreement, the
purchaser acquires the thing sold absolutely, and if he afterwards
grants the vendor the right to repurchase, it is a new contract
entered into by the purchaser, as absolute owner already of the
object. In that case the vendor has not reserved to himself the
right to repurchase."
Hence, the Option to Repurchase executed by private
respondent in the present case, was merely a promise to sell,
which must be governed by Article 1479 of the Civil Code.
A copy of the so-called Option to Repurchase is neither attached
to the records nor quoted in any of the pleadings of the parties.
This Court cannot, therefore, properly rule on whether the
promise was accepted and a consideration distinct from the
price, supports the option. Undoubtedly, in the absence of either
or both acceptance and separate consideration, the promise to
sell is not binding upon the promissor (private respondent).
A unilateral promise to buy or sell is a mere offer, which is not
converted into a contract except at the moment it is accepted.
Acceptance is the act that gives life to a juridical obligation,
because, before the promise is accepted, the promissor may
withdraw it at any time. Upon acceptance, however, a bilateral

4. That upon signing of the LEASE AGREEMENT, the Baptist


Church shall pay the sum of P 84,000.00. Said sum shall be paid
directly to the Rural Bank of Bulacan for the purpose of
redemption of said property, which was mortgaged by the
Spouses.
xxx
8. That Bible Baptist has the option to buy the leased
property during the 15 years of the lease. If Baptist Church
decides to purchase the premises the terms will be:
1. A selling price of 1.8 M;
2. A down payment agreed upon by both parties;
3. The balance may be paid at the rate of P 120T per year.
These stipulations of the lease contract are the subject of the
present controversy for it is now the contention of Baptist Church
that the option contract is founded upon a separate consideration
that is the P 84 T paid by them upon the signing of the lease
agreement.
ISSUE: Whether or not the option to buy given to the Baptist
Church is founded upon a consideration.
HELD: No. Article 1479 of the Civil Code provides:
A promise to buy and sell a determinate thing for a price certain
is reciprocally demandable.
COMPILED BY: WIGMORE #WIGMOREFOREVER

80

SALES Case Digest (Atty. Sarona)


Compiled by: Wigmore #wigmoreforever
An accepted unilateral promise to buy or to sell a determinate
thing for a price certain is binding upon the promissor if the
promise is supported by a consideration distinct from the price.
Baptist Church cannot insist that the 84 T they paid in order to
release the Spouses property from the mortgage should be
deemed a separate consideration to support the option contract.
It must be pointed out that the said amount was in fact
apportioned into monthly rentals spread over a period of one
year, at 7 T per month. Thus for the entire period of June 1985 to
May 1986, Baptist Churchs monthly rent had already been paid
for, such that it only commenced paying rentals in June 1986.
Therefore, the amount of 84 T has been fully exhausted and
utilized by their occupation of the premises and there is no
separate consideration to speak of which could support the
option.
Baptist Church insists that a consideration need not be a
separate sum of money. They posit that their act of advancing
the money to rescue the property from the mortgage and
impending foreclosure should be enough consideration to
support the option. In Villamor vs CA the court defined
consideration as the why of the contracts, the essential reason
which moves the contracting parties to enter into a contract.
This would illustrate that the consideration need not be monetary.
Actual cash need not be exchanged for the prion. However, by
the very nature of an option contract the same is an onerous
contract for which the consideration must be something of value,
although its kind may vary. The Villamor case is distinct from this
case because:
The Court cannot find that the Baptist Church parted
with anything of value aside from the 84 T;
There is no document that contains an agreement
between the parties that Baptist Churchs supposed
rescue of the mortgaged property was the consideration
which the parties contemplated in support of the option
clause in the contract;
To summarize the rules, an option contract needs to be
supported by a separate consideration. The consideration need
not be monetary but could consist of other things or
undertakings. However, if the consideration is not monetary,
these must be things or undertakings of value, in view of the
onerous nature of the contract of option. Furthermore, when a
consideration for an option contract is not monetary, said
consideration must be clearly specified as such in the option
contract or clause.

two slipways within the first 10 years of the lease with a total
value of not less than 450 T.
On March 14, 1973 the property was mortgaged to China
Banking Corporation (CBC) as a security for a loan availed by
two of Carmens children, Mariano and Gabriel. The owners
duplicate copy was now with CBC.
On December 31, 1974 Carmen executed a Deed of Absolute
Sale with Assumption of Mortgage in which she as the vendor
conveyed the property to her children Serafin, Mariano, Rogelio,
Carmencita and Mary Carmela for the purchase price of 350 T.
Mariano wrote a letter to CBC requesting them to conform to the
sale however CBC refused.
On June 27, 1977 Mariano presented the deed to the ROD for
registration purposes. They requested the ROD to compel CBC
to transmit the owners duplicate copy of the title for annotation.
CBC informed them that they were just following the instruction
of Carmen not to surrender the owners duplicate.
In the meantime the balance of the loan was fully paid and on
June 29, 1977 CBC executed a Cancellation of Real Estate
Mortgage however the deed was not presented to the ROD for
registration. On the same date Mariano, on behalf of his siblings,
executed an Affidavit of Adverse claim asserting their rights as
vendees of the property.
On June 30, 1977 Carmen and NIC executed a Supplementary
Lease Agreement extending the lease period to October 2005.
NIC was also granted the option to buy the property for 1.6
M.
Mariano et. al. was able to have the sale registered and a new
title was issued in their name. Thereafter, they have informed
NIC to vacate the property, as they are now its new owners,
however, NIC refused.
Meanwhile, Carmen filed a case against NIC anent the
Supplementary Lease Agreement purportedly executed by her as
lessor and NIC as lessee. She averred that NIC took advantage
of the animosity between her and her children by inserting
therein blatantly unfair provisions.
On June 30, 1990 Mariano et. al informed NIC that they will be
no longer renew the contract and that as far as they were
concerned the Supplementary Lease Agreement was null and
void.
ISSUE: Whether or not there is a perfected option contract.

NAVOTAS INDUSTRIAL CORPORATION, represented by


DANIEL BAUTISTA vs GERMAN CRUZ, et. al.
G.R. No. 159212
September 12, 2005
FACTS: Carmen Vda. De Cruz (Carmen) was the owner of a
parcel of land in Navotas with an area of 13999 square meters.
On October 5, 1966, Carmen and Navotas Industrial Corporation
(NIC) entered into a contract of lease covering one half portion of
the property. The lease was for October 1, 1966 to October 1,
1990. The property was to be used for shipyard slipways and
NICs other allied businesses. The NIC obliged itself to construct

HELD: No. Article 1479, paragraph 1, provides that, A promise


to buy and sell a determinate thing for a price certain is
reciprocally demandable. NIC relies on this provision contending
that what was entered between them and Carmen was a mutual
promise to buy and sell. However, be it noted, that as early as
1977 they were already informed of the sale made by Carmen in
favor of her children and that by virtue of the annotation made by
Mariano on June 30, 1977 NIC was constructively notified
thereof. There is therefore a presumption of knowledge of the
sale between Carmen and her children.
Considering that Carmen was no longer the owner when the
Supplementary Lease Agreement was executed NICs claim that
COMPILED BY: WIGMORE #WIGMOREFOREVER 81

SALES Case Digest (Atty. Sarona)


Compiled by: Wigmore #wigmoreforever
it had the option to buy the property or to compel the heirs to sell
the property to it has no legal and factual basis.
Paragraphs 4 and o 5 of the Supplementary Lease Agreement
provides:
4. The LESSEE is hereby granted an exclusive option to buy the
property including all improvements already made by the
LESSEE (slipways and camarines) subject matter of this contract
comprising SIX THOUSAND NINE HUNDRED FORTY-NINE
Point FIVE Square Meters (6,949.5) which is one-half portion of
the area covered by TCT No. 81574 and same property subject
matter of this contract should also be equally divided with onehalf frontage along M. Naval Street and along the Navotas River
Bank shoreline during the period of the lease. The price of the
property is agreed to be fixed for the duration of the Option
to Buy at a flat sum of ONE MILLION SIX HUNDRED
THOUSAND PESOS (P1,600,000.00), Philippine Currency,
payable over a period to be mutually agreed upon. Should
the LESSEE exercise the option to buy during the lifetime of the
LESSOR, the LESSEE will continue to pay the monthly rental to
the LESSOR during her lifetime.
5. The LESSEE shall pay to the LESSOR the sum of FORTYTWO THOUSAND (P42,000.00) PESOS upon signing of this
contract as consideration thereof, to be applied as against the
rental for the period from October 1, 1990 to September 30,
1991.
It must be stressed that an option contract is a contract granting
a privilege to buy and sell within an agreed time and at a
determined price. Such contract is a separate and distinct
contract from the time the parties may enter into upon the
construction of the option. An option contract is a preparatory
contract in which one party grants to the other for a fixed period
and under specified conditions the power to decide, whether or
not to enter into a principal contract. Therefore, it is only when
the option is exercised may a sale be perfected. An option
NEEDS TO BE SUPPORTED by a separate consideration.
In the present case, there was no given period for the petitioner
to exercise its option; it had yet to be determined and fixed at a
future time by the parties, subsequent to the execution of the
Supplementary Lease Agreement. There was, likewise, no
consideration for the option. The amount of P42,000.00 paid by
the petitioner to Carmen Cruz on July 30, 1977 was payment for
rentals from October 1, 1990 to September 30, 1991, and not as
a consideration for the option granted to the petitioner.
iii.

There Must be Acceptance of Option Offer

SPOUSES CIPRIANO VASQUEZ and VALERIANA GAYANELO


vs CA and SPOUSES MARTIN VALLEJERA
G.R. No. 83759
JULY 12, 1991
FACTS: A certain property in Himamaylan, Negros Occidental
was registered in the name of Spouse Vallejera. On October
1959, they leased the property to the Spouses Vasquez. After the
execution of the lease, the Vasquez took possession of the lot
and devoted the same to the cultivation of sugar.

On September 21, 1964, the spouses Vallejera sold the lot the
spouses Vasquez for the amount of 9 T. On the same day and
along with the execution of the Deed of Sale, a separate
instrument, denominated as Right to Repurchase was executed
by the parties granting the Vallejeras the right to repurchase the
lot for 12 T.
By virtue of the Deed of Sale the spouses Vasquez secured a
title in their name. However, on January 2, 1969, the Vallejeras
sold the lot to Benito Derrama after securing the spouse
Vasquez title for 12 T. Upon the protestation of the spouses
Vasquez the sale was cancelled after payment of 12 T to
Derrama.
The spouses Vasquez resisted the action for redemption on the
premise that the deed of Right to Repurchase is just an option
to buy since it is not embodied in the same document of sale but
in a separate document and since such option is not supported
by a consideration distinct from the price, said deed is not
binding upon them.
The spouses Vazquez insist that they can not be compelled to
resell the subject property for the nature of the sale over the said
lot between them and the Vallejeras can only be either an option
to buy or a mere promise on their part to resell the property.
Spouses Vasquez opined that since the Right to Repurchase
was not supported by any consideration distinct from the
purchase price it is not valid and binding upon the spouses
Vasquez pursuant to Article 1479.
ISSUE: Whether or not the spouse Vallejera has a right to
repurchase under the contract.
HELD: No. The Court made reference to the earlier case of
Sanchez vs. Rigos (Sanchez doctrine), stating that an option
contract without a separate consideration from the purchase
price is void, as a contract, but would still constitute as a valid
offer; so that if the option is exercised prior to its withdrawal, that
is equivalent to an offer being accepted prior to withdrawal and
would give rise to a valid and binding sale.
The Sanchez doctrine also dictates that the burden of proof to
show that the option contract was supported by a separate
consideration is with the party seeking to show it. No reliance
can be placed upon the provisions of Article 1354 which
presumes the existence of a consideration in every contract,
since in the case of an option contract, Article 1479 being the
specific provision, requires such separate consideration for an
option to be valid.
In an option contract, the offeree has the burden of proving that
the option is supported by a separate consideration, it also held
that the Sanchez doctrine (That upon the option contract not
supported by a separate consideration; is void as contract, but
valid as an offer), can only apply if the option has been accepted
and such acceptance is communicated to the offeror. It held that
not even the annotation of the option contract on the title of the
property can be considered a proper acceptance of the option.
Neither can the signature of the spouses Vasquez in the
document called "right to repurchase" signify acceptance of the
right to repurchase. The Vallejeras did not sign the offer.
Acceptance should be made by the promisee, in this case, the
Vallejeras and not the promises, spouses Vasquez herein. It
COMPILED BY: WIGMORE #WIGMOREFOREVER 82

SALES Case Digest (Atty. Sarona)


Compiled by: Wigmore #wigmoreforever
would be absurd to require the promisor of an option to buy to
accept his own offer instead of the promisee to whom the option
to buy is given.
iv.

2.

Proper Exercise of Option Contract

Right of First Refusal

POLYTECHNIC UNIVERSITY OF THE PHILIPPINES vs


GOLDEN HORIZON REALTY CORP.
x-----------------------------------------------x
NATIONAL DEVELOPMENT AUTHORITY vs GOLDEN
HORIZON REALTY CORP.
G.R. No. 183612; G.R. No. 184260
MARCH 15, 2010
(This is case related to the case of PUP vs. CA and Firestone
Ceramics)
FACTS: National Development Corp. (NDC) had in its disposal a
10-hectare property located at Sta. Mesa, Manila. The estate
was popularly known as NDC Compound.
On September 7, 1977 NDC entered into a Contract of Lease
with Golden Horizon Realty Corp. (GHRC) over a portion of the
property with an area of 2,407 sq. m. for a period of 10 years,
renewable for another 10 years with mutual consent of the
parties.
On May 4, 1978, a second Contract of Lease was executed by
NDC and GHRC covering 3,222 sq. m., also renewable upon the
mutual consent after the expiration of the 10-year lease period.
In addition, GHRC was granted the option to purchase the area
leased, the price to be negotiated and determined at the time the
option to purchase is exercised.
On June 13, 1988, before the expiration of the 10-year period
under the second contract, GHRC wrote a letter to NDC
indicating its exercise of the option to renew the lease for another
10 years. NDC gave no response to the said letter.
In September of the same year, GHRC discovered that NDC had
decided to secretly dispose the property to a third party, PUP.
This led to the filing of cases before the trial court.
In the meantime President Aquino issued Memo. Order No. 214
dated January 6, 1989 ordering the transfer of the whole NDC
Compound to the National Government, which in turn would
convey the said property in favor of PUP at acquisition cost.
PUP then contended that GHRCs right to exercise the option to
purchase had expired with the termination of the original contract
of lease and was not carried over to the subsequent implied new
lease between GHRC and NDC. Moreover, the contracts clearly
state that GHRC is granted the option to renew for another 10
years with mutual consent of both parties. As regards the
continued receipt of rentals by NDC and possession by GHRC of
the leased premises, the impliedly renewed lease was only
month-to-month and not 10 years since the rentals are being
paid on monthly basis.

ISSUE: Whether or not GHRCs right of first refusal was violated.


HELD: Yes. The pertinent portion of the second contract of lease
provides that: Lessee shall also have the option to purchase the
area leased, the price to be negotiated and determined at the
time the option to purchase is exercised.
An option is a contract by which the owner of the property agrees
with another person that the latter shall have the right to buy the
formers property at a fixed price within a certain time. It is a
condition offered or contract by which the owner stipulates with
another that the latter shall have the right to buy the property at a
fixed price within a certain time, or under, or in compliance with
certain terms and conditions; or which gives to the owner of the
property the right to sell or demand a sale. It binds the party, who
has given the option, not to enter into the principal contract with
any other person during the period designated, and, within that
period, to enter into such contract with the one to whom the
option was granted, if the latter should decide to use the option.
Upon the other hand, a right of first refusal is a contractual grant,
not of the sale of a property, but of the first priority to buy the
property in the event the owner sells the same. As distinguished
from an option contract, in a right of first refusal, while the object
might be made determinate, the exercise of the right of first
refusal would be dependent not only on the owners eventual
intention to enter into a binding juridical relation with another but
also on terms, including the price, that are yet to be firmed up.
When a lease contract contains a right of first refusal, the lessor
has the legal duty to the lessee not to sell the leased property to
anyone at any price until after the lessor has made an offer to
sell the property to the lessee and the lessee had failed to accept
it. Only after the lessee has failed to exercise his right of first
priority could the lessor sell the property to other buyers under
the same terms and conditions offered to the lessee, or under
terms and conditions more favorable to the lessor.
NDC contended that the ruling of the Court in PUP vs CA and
Firestone cannot be applied in this case because the lease
contract of firestone had not yet expired while in this case
GHRCs lease contract have already expired. This is untenable.
The reckoning point of the offer of sale to a third party was not
the issuance of Memorandum Order No. 214 on January 6, 1989
but the commencement of such negotiations as early as July
1988 when GHRCs right of first refusal was still subsisting and
the lease contracts still in force. NDC did not bother to respond
to GHRCs letter of June 13, 1988 informing it of GHRCs
exercise of the option to renew and requesting to discuss further
the matter with NDC, nor to the subsequent letter of August 12,
1988 reiterating the request for renewing the lease for another
ten (10) years and also the exercise of the option to purchase
under the lease contract. NDC had dismissed these letters as
"mere informative in nature, and a request at its best."
GHRC is similarly situated with Firestone such that it was also
prejudiced by NDCs sale to PUP. Therefore, GHRC is entitled to
exercise its option to purchase until October 1988 in as much as
the May 4, 1978 contract embodied the option to renew the lease
contract for another 10 years upon mutual consent and giving
GHRC the option to purchase the leased premises for a price to
COMPILED BY: WIGMORE #WIGMOREFOREVER

83

SALES Case Digest (Atty. Sarona)


Compiled by: Wigmore #wigmoreforever
be negotiated and determined at the time such option was
exercised by GHRC. It to be noted that MO 214 itself declared
that the transfer is subject to such liens/leases existing on the
subject property.

considering the mercurial and uncertain forces in our market


economy today, the same right of first refusal to herein
plaintiffs/appellants in the event that the subject property is sold
for a price in excess of Eleven Million pesos or more had
become final to the effect

ANG YU ASUNCION vs. THE HON. COURT OF APPEALS


G.R. No. 109125 December 2, 1994

The owners were ordered to execute the necessary Deed of Sale


of the property in litigation in favor of plaintiffs Ang Yu Asuncion,
Keh Tiong and Arthur Go for the consideration of P15 Million
pesos in recognition of plaintiffs' right of first refusal and that a
new Transfer Certificate of Title be issued in favor of the buyer.
All previous transactions involving the same property
notwithstanding the issuance of another title to Buen Realty
Corporation, is hereby set aside as having been executed in bad
faith.

FACTS: Ang Yu Asuncion and Keh Tiong, et al., are tenants or


lessees of residential and commercial spaces owned by Cu
Unjieng, Rose Cu Unjieng and Jose Tan. They have occupied
said spaces since 1935 and have been religiously paying the
rental and complying with all the conditions of the lease contract.
On several occasions before October 9, 1986, the owners
informed Ang Yus party that they are offering to sell the
premises and are giving them priority to acquire the same.
During the negotiations, Bobby Cu Unjieng offered a price of P6million while Asuncion and Keh Tiong made a counter offer of
P5-million. They thereafter asked the owners to put their offer in
writing to which request they acceded; that in reply to
defendant's letter, plaintiffs wrote them on October 24, 1986
asking that they specify the terms and conditions of the offer to
sell; that when Asuncion did not receive any reply, they sent
another letter dated January 28, 1987 with the same request.
Since the owners failed to specify the terms and conditions of the
offer to sell and because of information received that the owners
were about to sell the property, Ang Yu Asuncion and Keh Tiong
were compelled to file the complaint to compel defendants to sell
the property to them.
The trial court found that Cu Unjieng, Rose Cu Unjieng and Jose
Tan offer to sell was never accepted by the Ang Yu Asuncion
and Keh Tiong, et al., for the reason that the parties did not
agree upon the terms and conditions of the proposed sale,
hence, there was no contract of sale at all. Nonetheless, the
lower court ruled that should the defendants subsequently offer
their property for sale at a price of P11-million or below, plaintiffs
will have the right of first refusal.
Aggrieved by the decision that there was no contract of sale at
all, the lessees brought a petition for review on certiorari to the
Supreme Court. The Supreme Court denied the appeal on May
6, 1991.
On November 15, 1990, while the case filed by the lessees was
pending consideration, the Cu Unjieng spouses executed a Deed
of Sale, transferring the property in question to Buen Realty and
Development Corporation for 15M.
As a consequence of the sale, TCT No. 105254/T-881 in the
name of the Cu Unjieng spouses was cancelled and, in lieu
thereof, TCT No. 195816 was issued in the name of petitioner on
December 3, 1990.
On July 1, 1991, petitioner as the new owner of the subject
property wrote a letter to the lessees demanding that the latter
vacate the premises.
The decision that should the the owners decide to offer the
property for sale for a price of P11 Million or lower, and

A writ of execution was subsequently issued.


ISSUE: Whether or not a Writ of Execution may be decreed on a
judgment recognizing the right of first refusal.
HELD: NO. Writ of Execution is not a remedy. In the law on
sales, the so-called "right of first refusal" is an innovative juridical
relation. Needless to point out, it cannot be deemed a perfected
contract of sale under Article 1458 of the Civil Code. Neither can
the right of first refusal, understood in its normal concept, per
se be brought within the purview of an option under the second
paragraph of Article 1479, aforequoted, or possibly of an offer
9
under Article 1319 of the same Code.
10

An option or an offer would require, among other things, a


clear certainty on both the object and the cause or consideration
of the envisioned contract. In a right of first refusal, while the
object might be made determinate, the exercise of the right,
however, would be dependent not only on the grantor's eventual
intention to enter into a binding juridical relation with another but
also on terms, including the price, that obviously are yet to be
later firmed up. Prior thereto, it can at best be so described as
merely belonging to a class of preparatory juridical relations
governed not by contracts (since the essential elements to
establish the vinculum juris would still be indefinite and
inconclusive) but by, among other laws of general application,
the pertinent scattered provisions of the Civil Code on human
conduct.
Even on the premise that such right of first refusal has been
decreed under a final judgment, like here, its breach cannot
justify correspondingly an issuance of a writ of execution under a
judgment that merely recognizes its existence, nor would it
sanction an action for specific performance without thereby
negating the indispensable element of consensuality in the
11
perfection of contracts. It is not to say, however, that the right
of first refusal would be inconsequential for, such as already
intimated above, an unjustified disregard thereof, given, for
12
instance, the circumstances expressed in Article 19 of the Civil
Code, can warrant a recovery for damages.
The final judgment in Civil Case No. 87-41058, it must be
stressed, has merely accorded a "right of first refusal" in favor of
petitioners. The consequence of such a declaration entails no
more than what has heretofore been said. In fine, if, as it is here
COMPILED BY: WIGMORE #WIGMOREFOREVER

84

SALES Case Digest (Atty. Sarona)


Compiled by: Wigmore #wigmoreforever
so conveyed to us, petitioners are aggrieved by the failure of
private respondents to honor the right of first refusal, the remedy
is not a writ of execution on the judgment, since there is none to
execute, but an action for damages in a proper forum for the
purpose.
Furthermore, whether private respondent Buen Realty
Development Corporation, the alleged purchaser of the property,
has acted in good faith or bad faith and whether or not it should,
in any case, be considered bound to respect the registration of
the lis pendens in Civil Case No. 87-41058 are matters that must
be independently addressed in appropriate proceedings. Buen
Realty, not having been impleaded in Civil Case No. 87-41058,
cannot be held subject to the writ of execution issued by
respondent Judge, let alone ousted from the ownership and
possession of the property, without first being duly afforded its
day in court.

PARAAQUE KINGS ENTERPRISES, INC. vs. COURT OF


APPEALS
G.R. No. 111538 February 26, 1997
Catalina L. Santos is the owner of eight (8) parcels of land
located at Paraaque, Metro Manila. On November 28, 1977, a
certain Frederick Chua leased the property from defendant
Catalina L. Santos. Subsequently, Chua assigned all his rights
and interest and participation in the leased property to Lee Ching
Bing, by virtue of a deed of assignment and with the conformity
of defendant Santos, the said assignment was also registered.
Ching Bing also assigned all his rights and interest in the leased
property to Paraaque Kings Enterprises, Incorporated by virtue
of a deed of assignment and with the conformity of defendant
Santos.
Paragraph 9 of the assigned leased contract provides among
others that:
"9. That in case the properties subject of the lease
agreement are sold or encumbered, Lessors shall impose
as a condition that the buyer or mortgagee thereof shall
recognize and be bound by all the terms and conditions of
this lease agreement and shall respect this Contract of
Lease as if they are the LESSORS thereof and in case of
sale, LESSEE shall have the first option or priority to buy
the properties subject of the lease;"
On September 21, 1988, defendant Santos sold the eight parcels
of land subject of the lease to defendant David Raymundo for a
consideration of FIVE MILLION PESOS. The said sale was in
contravention of the contract of lease, for the first option or
priority to buy was not offered by defendant Santos to Paraaque
Kings Enterprises, Incorporated (plaintiff).
Upon learning of this fact plaintiff's representative wrote a letter
to defendant Santos, requesting her to rectify the error and
consequently realizing the error, she had it reconveyed to her for
the same consideration of FIVE MILLION (P5,000,000.00)
PESOS.

Subsequently the property was offered for sale to plaintiff by the


defendant for the sum of FIFTEEN MILLION (P15,000,000.00)
PESOS. Plaintiff was given ten (10) days to make good of the
offer.
On May 8, 1989, before the period given in the letter offering the
properties for sale expired, plaintiff's counsel wrote counsel of
defendant Santos offering to buy the properties for FIVE
MILLION PESOS.
On May 15, 1989, before they replied to the offer to purchase,
another deed of sale was executed by defendant Santos (in favor
of) defendant Raymundo for a consideration of NINE MILLION
PESOS.
Defendant Santos violated again paragraph 9 of the contract of
lease by executing a second deed of sale to defendant
Raymundo.
ISSUE: Whether or not there is a violation on the right of first
refusal.
HELD. Yes. In order to have full compliance with the contractual
right granting petitioner the first option to purchase, the sale of
the properties for the amount of P9 million, the price for which
they were finally sold to respondent Raymundo, should have
likewise been first offered to petitioner.
The basis of the right of first refusal* must be the current offer to
sell of the seller or offer to purchase of any prospective buyer.
Only after the optionee fails to exercise its right of first priority
under the same terms and within the period contemplated, could
the owner validly offer to sell the property to a third person,
again, under the same terms as offered to the optionee.
Deed of Assignment include the option to purchase
On the contention of respondent Santos that the assignment of
the lease contract to petitioner did not include the option to
purchase. The provisions of the deeds of assignment with regard
to matters assigned were very clear. Under the first assignment
between Frederick Chua as assignor and Lee Ching Bing as
assignee, it was expressly stated that:
. . . . the ASSIGNOR hereby CEDES, TRANSFERS and
ASSIGNS to herein ASSIGNEE, all his rights, interest and
participation over said premises afore-described, . . . .
And under the subsequent assignment executed between Lee
Ching Bing as assignor and the petitioner, represented by its
Vice President Vicenta Lo Chiong, as assignee, it was likewise
expressly stipulated that;
. . . . the ASSIGNOR hereby sells, transfers and assigns all his
rights, interest and participation over said leased premises, . . . .
One of such rights included in the contract of lease and,
therefore, in the assignments of rights was the lessee's right of
first option or priority to buy the properties subject of the lease,
COMPILED BY: WIGMORE #WIGMOREFOREVER

85

SALES Case Digest (Atty. Sarona)


Compiled by: Wigmore #wigmoreforever
as provided in paragraph 9 of the assigned lease contract. The
deed of assignment need not be very specific as to which rights
and obligations were passed on to the assignee. It is understood
in the general provision aforequoted that all specific rights
and obligations contained in the contract of lease are those
referred to as being assigned. Needless to state, respondent
Santos gave her unqualified conformity to both assignments of
rights.

ROSENCOR DEVELOPMENT CORPORATION vs. PATERNO


INQUING
G.R. No. 140479 March 8, 2001
FACTS: This is a petition for review on certiorari under Rule 45
1
of the Rules of Court seeking reversal of the Decision of the
Court of Appeals dated June 25, 1999 in CA-G.R. CV No. 53963.
The Court of Appeals decision reversed and set aside the
2
Decision dated May 13, 1996 of Branch 217 of the Regional
Trial Court of Quezon City in Civil Case No. Q-93-18582.
The case was originally filed on December 10, 1993 by Paterno
Inquing, Irene Guillermo and Federico Bantugan, herein
respondents, against Rosencor Development Corporation
(hereinafter "Rosencor"), Rene Joaquin, and Eufrocina de Leon.
Originally, the complaint was one for annulment of absolute deed
of sale but was later amended to one for rescission of absolute
deed of sale. A complaint-for intervention was thereafter filed by
respondents Fernando Magbanua and Danna Lizza Tiangco.
The complaint-in-intervention was admitted by the trial court in
3
an Order dated May 4, 1994.
The facts of the case, as stated by the trial court and adopted by
the appellate court, are as follows:
"This action was originally for the annulment of the Deed of
Absolute Sale dated September 4, 1990 between defendants
Rosencor and Eufrocina de Leon but later amended (sic) praying
for the rescission of the deed of sale.
Paterno Inquing, Irene Guillermo and Federico Bantugan averred
that they are the lessees since 1971 of a two-story residential
apartment owned by spouses Faustino and Cresencia Tiangco.
The lease was not covered by any contract. The lessees were
renting the premises then for P150.00 a month and were
allegedly verbally granted by the lessors the pre-emptive right to
purchase the property if ever they decide to sell the same.
Upon the death of the spouses Tiangcos in 1975, the
management of the property was adjudicated to their heirs who
were represented by Eufrocina de Leon. The lessees were
allegedly promised the same pre-emptive right by the heirs of
Tiangcos since the latter had knowledge that this right was
extended to the former by the late spouses Tiangcos.
In June 1990, the lessees received a letter from Atty. Erlinda
Aguila demanding that they vacate the premises so that the
demolition of the building be undertaken. They refused to leave
the premises. In that same month, de Leon refused to accept the
lessees rental payment claiming that they have run out of
receipts and that a new collector has been assigned to receive

the payments. Thereafter, they received a letter from Eufrocina


de Leon offering to sell to them the property they were leasing for
P2,000,000.00. xxx.
The lessees offered to buy the property from de Leon for the
amount of P1,000,000.00. De Leon told them that she will be
submitting the offer to the other heirs. Since then, no answer was
given by de Leon as to their offer to buy the property. However,
in November 1990, Rene Joaquin came to the leased premises
introducing himself as its new owner.
In January 1991, the lessees again received another letter from
Atty. Aguila demanding that they vacate the premises. A month
thereafter, the lessees received a letter from de Leon advising
them that the heirs of the late spouses Tiangcos have already
sold the property to Rosencor. The following month Atty. Aguila
wrote them another letter demanding the rental payment and
introducing herself as counsel for Rosencor/Rene Joaquin, the
new owners of the premises.
The lessees requested from de Leon why she had disregarded
the pre-emptive right she and the late Tiangcos have promised
them. They also asked for a copy of the deed of sale between
her and the new owners thereof but she refused to heed their
request. In the same manner, when they asked Rene Joaquin a
copy of the deed of sale, the latter turned down their request and
instead Atty. Aguila wrote them several letters demanding that
they vacate the premises. The lessees offered to tender their
rental payment to de Leon but she refused to accept the same.
In April 1992 before the demolition can be undertaken by the
Building Official. It was at this instance that the lessees were
furnished with a copy of the Deed of Sale and discovered that
they were deceived by de Leon since the sale between her and
Rene Joaquin/Rosencor took place in September 4, 1990 while
de Leon made the offer to them only in October 1990 or after the
sale with Rosencor had been consummated. The lessees also
noted that the property was sold only for P726,000.00.
The lessees offered to reimburse de Leon the selling price of
P726,000.00 plus an additional P274,000.00 to complete their
P1,000.000.00 earlier offer. When their offer was refused, they
filed the present action praying for the following: a) rescission of
the Deed of Absolute Sale between de Leon and Rosencor dated
September 4, 1990; b) the defendants Rosencor/Rene Joaquin
be ordered to reconvey the property to de Leon; and c) de Leon
be ordered to reimburse the plaintiffs for the repairs of the
property, or apply the said amount as part of the price for the
4
purchase of the property in the sum of P100,000.00."
The trial court held that the right of redemption on which the
complaint was based was merely an oral one and as such, is
unenforceable under the law.
ISSUE: Whether or not a right of first refusal is indeed covered
by the provisions of the New Civil Code on the statute of frauds.
RULING: NO. It is not covered by the statute of frauds.

COMPILED BY: WIGMORE #WIGMOREFOREVER

86

SALES Case Digest (Atty. Sarona)


Compiled by: Wigmore #wigmoreforever
A right of first refusal is not among those listed as unenforceable
under the statute of frauds. Furthermore, the application of
Article 1403, par. 2(e) of the New Civil Code presupposes the
existence of a perfected, albeit unwritten, contract of sale. A right
of first refusal, such as the one involved in the instant case, is not
by any means a perfected contract of sale of real property. At
best, it is a contractual grant, not of the sale of the real property
involved, but of the right of first refusal over the property sought
to be sold.
It is thus evident that the statute of frauds does not contemplate
cases involving a right of first refusal. As such, a right of first
refusal need not be written to be enforceable and may be proven
by oral evidence.
The next question to be ascertained is whether or not
respondents have satisfactorily proven their right of first refusal
over the property subject of the Deed of Absolute Sale dated
September 4, 1990 between petitioner Rosencor and Eufrocina
de Leon.
Respondents have adequately proven the existence of their right
of first refusal. Federico Bantugan, Irene Guillermo, and Paterno
Inquing uniformly testified that they were promised by the late
spouses Faustino and Crescencia Tiangco and, later on, by their
heirs a right of first refusal over the property they were currently
leasing should they decide to sell the same. Moreover,
respondents presented a letter20 dated October 9, 1990 where
Eufrocina de Leon, the representative of the heirs of the spouses
Tiangco, informed them that they had received an offer to buy
the disputed property for P2,000,000.00 and offered to sell the
same to the respondents at the same price if they were
interested. Verily, if Eufrocina de Leon did not recognize
respondents right of first refusal over the property they were
leasing, then she would not have bothered to offer the property
for sale to the respondents. It must be noted that petitioners did
not present evidence before the trial court contradicting the
existence of the right of first refusal of respondents over the
disputed property.
The final question to be resolved is May a contract of sale
entered into in violation of a third partys right of first refusal be
rescinded in order that such third party can exercise said right?
The prevailing doctrine, as enunciated in some cited cases, is
that a contract of sale entered into in violation of a right of first
refusal of another person, while valid, is rescissible.
There is, however, a circumstance which prevents the
application of this doctrine in the case at bench. In some cases,
the Court ordered the rescission of sales made in violation of a
right of first refusal precisely because the vendees therein did not
act in good faith as they were aware or should have been aware
of the right of first refusal granted to another person by the
vendors therein.
In the instant case was an oral one given to respondents by the
deceased spouses Tiangco and subsequently recognized by
their heirs. As such, in order to hold that petitioners were in bad
faith, there must be clear and convincing proof that petitioners
were made aware of the said right of first refusal either by the

respondents or by the heirs of the spouses Tiangco. It is


axiomatic that good faith is always presumed unless contrary
evidence is adduced.
On this point, we hold that the evidence on record fails to show
that petitioners acted in bad faith in entering into the deed of sale
over the disputed property with the heirs of the spouses Tiangco.
Respondents failed to present any evidence that prior to the sale
of the property on September 4, 1990, petitioners were aware or
had notice of the oral right of first refusal.

VAZQUEZ vs. AYALA CORPORATION


G.R. No. 149734
November 19, 2004
FACTS: On April 23, 1981, spouses Daniel Vasquez and Ma.
Luisa M. Vasquez (hereafter, Vasquez spouses) entered into a
Memorandum of Agreement (MOA) with Ayala Corporation
(hereafter, AYALA) with AYALA buying from the Vazquez
spouses, all of the latter's shares of stock in Conduit
Development, Inc. (hereafter, Conduit). The main asset of
Conduit was a 49.9 hectare property in Ayala Alabang,
Muntinlupa, which was then being developed by Conduit under a
development plan where the land was divided into Villages 1, 2
and 3 of the "Don Vicente Village." The development was then
being undertaken for Conduit by G.P. Construction and
Development Corp.
Under the MOA, Ayala was to develop the entire property, less
what was defined as the "Retained Area" consisting of 18,736
square meters. This "Retained Area" was to be retained by the
Vazquez spouses. The area to be developed by Ayala was called
the "Remaining Area". In this "Remaining Area" were 4 lots
adjacent to the "Retained Area" and Ayala agreed to offer these
lots for sale to the Vazquez spouses at the prevailing price at the
time of purchase. Among the relevant provisions of the MOA on
this point is:
5.15. The BUYER agrees to give the SELLERS a first option to
purchase four developed lots next to the "Retained Area" at the
prevailing market price at the time of the purchase."
Taking the position that Ayala was obligated to sell the 4 lots
adjacent to the "Retained Area" within 3 years from the date of
the MOA, the Vasquez spouses sent several "reminder" letters of
the approaching so-called deadline. However, no demand after
April 23, 1984, was ever made by the Vasquez spouses for Ayala
to sell the 4 lots. On the contrary, one of the letters signed by
their authorized agent, Engr. Eduardo Turla, categorically stated
that they expected "development of Phase 1 to be completed by
February 19, 1990, three years from the settlement of the legal
problems with the previous contractor."
By early 1990 Ayala finished the development of the vicinity of
the 4 lots to be offered for sale. The four lots were then offered to
be sold to the Vasquez spouses at the prevailing price in 1990.
This was rejected by the Vasquez spouses who wanted to pay at
1984 prices, thereby leading to a suit.
The court ordered Ayala to sell to the Vazquez the relevant lots
described in the Complaint in the Ayala Alabang Village at the
price of P460.00 per square meter amounting to P1,349,540.00
COMPILED BY: WIGMORE #WIGMOREFOREVER

87

SALES Case Digest (Atty. Sarona)


Compiled by: Wigmore #wigmoreforever
In its decision, the court a quo concluded that the option to
purchase the 4 lots is valid because it was supported by
consideration as the option is incorporated in the MOA where the
parties had prestations to each other.
ISSUE: whether or not paragraph 5.15 of the MOA can properly
be construed as an option contract or a right of first refusal.
HELD: Paragraph 5.15 of the MOA is a mere right of first
refusal.
The Court has clearly distinguished between an option contract
and a right of first refusal. An option is a preparatory contract in
which one party grants to another, for a fixed period and at a
determined price, the privilege to buy or sell, or to decide
whether or not to enter into a principal contract. It binds the party
who has given the option not to enter into the principal contract
with any other person during the period designated, and within
that period, to enter into such contract with the one to whom the
option was granted, if the latter should decide to use the option.
It is a separate and distinct contract from that which the parties
may enter into upon the consummation of the option. It must be
supported by consideration.
In a right of first refusal, on the other hand, while the object might
be made determinate, the exercise of the right would be
dependent not only on the grantor's eventual intention to enter
into a binding juridical relation with another but also on terms,
including the price, that are yet to be firmed up.
Applied to the instant case, paragraph 5.15 is obviously a mere
right of first refusal and not an option contract. Although the
paragraph has a definite object, i.e., the sale of subject lots, the
period within which they will be offered for sale to petitioners and,
necessarily, the price for which the subject lots will be sold are
not specified. The phrase "at the prevailing market price at the
time of the purchase" connotes that there is no definite period
within which Ayala Corporation is bound to reserve the subject
lots for petitioners to exercise their privilege to purchase. Neither
is there a fixed or determinable price at which the subject lots will
be offered for sale. The price is considered certain if it may be
determined with reference to another thing certain or if the
determination thereof is left to the judgment of a specified person
or persons.
Further, paragraph 5.15 was inserted into the MOA to give
petitioners the first crack to buy the subject lots at the price which
Ayala Corporation would be willing to accept when it offers the
subject lots for sale. It is not supported by an independent
consideration. As such it is not governed by Articles 1324 and
1479 of the Civil Code, viz:
Art. 1324. When the offeror has allowed the offeree a certain
period to accept, the offer may be withdrawn at any time before
acceptance by communicating such withdrawal, except when the
option is founded upon a consideration, as something paid or
promised.
Art. 1479. A promise to buy and sell a determinate thing for a
price certain is reciprocally demandable.
An accepted unilateral promise to buy or to sell a determinate
thing for a price certain is binding upon the promissor if the
promise is supported by a consideration distinct from the price.

Consequently, the "offer" may be withdrawn anytime by


communicating the withdrawal to the other party.
In this case, Ayala Corporation offered the subject lots for sale to
petitioners at the price of P6,500.00/square meter, the prevailing
market price for the property when the offer was made on June
18, 1990. Insisting on paying for the lots at the prevailing market
price in 1984 of P460.00/square meter, petitioners rejected the
offer. Ayala Corporation reduced the price to P5,000.00/square
meter but again, petitioners rejected the offer and instead made
a counter-offer in the amount of P2,000.00/square meter. Ayala
Corporation rejected petitioners' counter-offer. With this rejection,
petitioners lost their right to purchase the subject lots.
It cannot, therefore, be said that Ayala Corporation breached
petitioners' right of first refusal and should be compelled by an
action for specific performance to sell the subject lots to
petitioners at the prevailing market price in 1984.
TANAY RECREATION CENTER AND DEVELOPMENT CORP.
vs. CATALINA MATIENZO FAUSTO
G.R. No. 140182. April 12, 2005
FACTS: Petitioner Tanay Recreation Center and Development
Corp. (TRCDC) is the lessee of a 3,090-square meter property
located in Sitio Gayas, Tanay, Rizal, owned by Catalina Matienzo
Fausto, under a Contract of Lease. On this property stands the
Tanay Coliseum Cockpit operated by petitioner. The lease
contract provided for a 20-year term, subject to renewal within
sixty days prior to its expiration. The contract also provided that
should Fausto decide to sell the property, petitioner shall have
the priority right to purchase the same.
On June 17, 1991, petitioner wrote Fausto informing her of its
intention to renew the lease. However, it was Faustos daughter,
respondent Anunciacion F. Pacunayen, who replied, asking that
petitioner remove the improvements built thereon, as she is now
the absolute owner of the property. It appears that Fausto had
earlier sold the property to Pacunayen and title has already been
transferred in her name. Petitioner filed an Amended Complaint
for Annulment of Deed of Sale, Specific Performance with
Damages, and Injunction.
In her Answer, respondent claimed that petitioner is estopped
from assailing the validity of the deed of sale as the latter
acknowledged her ownership when it merely asked for a renewal
of the lease. According to respondent, when they met to discuss
the matter, petitioner did not demand for the exercise of its option
to purchase the property, and it even asked for grace period to
vacate the premises.
ISSUE: The contention in this case refers to petitioners priority
right to purchase, also referred to as the right of first refusal.
HELD: When a lease contract contains a right of first refusal, the
lessor is under a legal duty to the lessee not to sell to anybody at
any price until after he has made an offer to sell to the latter at a
certain price and the lessee has failed to accept it. The lessee
has a right that the lessor's first offer shall be in his favor.
Petitioners right of first refusal is an integral and indivisible part
of the contract of lease and is inseparable from the whole
contract. The consideration for the lease includes the
consideration for the right of first refusal and is built into the
COMPILED BY: WIGMORE #WIGMOREFOREVER 88

SALES Case Digest (Atty. Sarona)


Compiled by: Wigmore #wigmoreforever
ISSUE: Whether or not the sale of the subject lot by Cornelio to
his sons is invalid for (1) violating the prohibitory clause in the
lease agreement between Cornelio, as lessor-owner, and
Orlando, as lessee; and (2) contravening the right of first refusal
of Orlando over the subject lot.

reciprocal obligations of the parties.


It was erroneous for the CA to rule that the right of first refusal
does not apply when the property is sold to Faustos relative.
When the terms of an agreement have been reduced to writing, it
is considered as containing all the terms agreed upon. As such,
there can be, between the parties and their successors in
interest, no evidence of such terms other than the contents of the
written agreement, except when it fails to express the true intent
and agreement of the parties. In this case, the wording of the
stipulation giving petitioner the right of first refusal is plain and
unambiguous, and leaves no room for interpretation. It simply
means that should Fausto decide to sell the leased property
during the term of the lease, such sale should first be offered to
petitioner. The stipulation does not provide for the qualification
that such right may be exercised only when the sale is made to
strangers or persons other than Faustos kin. Thus, under the
terms of petitioners right of first refusal, Fausto has the legal
duty to petitioner not to sell the property to anybody, even her
relatives, at any price until after she has made an offer to sell to
petitioner at a certain price and said offer was rejected by
petitioner.

HELD: No. Sale was valid. Under Article 1311 of the Civil Code,
the heirs are bound by the contracts entered into by their
predecessors-in-interest except when the rights and obligations
therein are not transmissible by their nature, by stipulation or by
provision of law. A contract of lease is generally transmissible to
the heirs of the lessor or lessee. It involves a property right and
the death of a party does not excuse non-performance of the
contract. The rights and obligations pass to the heirs of the
deceased and the heir is bound to respect the period of the
lease.
The parties expressly stipulated in the March 31, 1978
Agreement that Romeo, as lessee, shall transfer all his rights
and interests under the lease contract with option to renew in
favor of the party of Orlando, the latters heirs, successors and
assigns indicating the clear intent to allow the transmissibility of
all the rights and interests of Orlando under the lease contract
unto his heirs, successors or assigns. The rights and obligations
under the lease contract with option to renew were transmitted
from Orlando to his heirs upon his death. It does not follow,
however, that the lease subsisted at the time of the sale of the
subject.

ESTATE OF LLENADO VS EDUARDO LLENADO ET AL


MARCH 4, 2009
G.R. No. 145736
FACTS: The subject of this controversy is a parcel of land
denominated as Lot 249-D-1 (subject lot) registered in the names
of Eduardo and Jorge Llenado. The subject lot once formed part
of Lot 249-D owned by and registered in the name of their father,
Cornelio Llenado.

The election of the option to renew the lease in this case cannot
be inferred from petitioner Wenifredas continued possession of
the subject lot. It was incumbent upon Wenifreda with the burden
of proof during the trial below to establish by some positive act
that Orlando or his heirs exercised the option to renew the lease.
SC held that there was no evidence presented before the trial
court to prove that Orlando or his heirs exercised the option to
renew prior to or at the time of the expiration of the lease. As a
result, there was no obstacle to the sale of the subject lot by
Cornelio to respondents Eduardo and Jorge as the prohibitory
clause under the lease contract was no longer in force.

Cornelio leased Lot 249-D-1 to his nephew, Romeo. On March


31, 1978, Cornelio, Romeo and the latters cousin Orlando
executed an Agreement whereby Romeo assigned all his rights
to Orlando. The parties further agreed that Orlando shall have
the option to renew the lease contract and that during the period
that the agreement is enforced, the property cannot be sold,
transferred, alienated or conveyed in whatever manner to any
third party. Orlando died and his wife, Wenifreda, took over the
operation of the gasoline station. Cornelio sold Lot 249-D to his
children through a deed of sale, denominated as Kasulatan sa
Ganap Na Bilihan, for the sum of P160k. Lot 249-D-1 was sold
to Eduardo and Jorge.
Eduardo informed Wenifreda of his desire to take over the
subject lot, but the latter refused to vacate the premises despite
repeated demands. Thus, Eduardo filed a complaint for unlawful
detainer against Wenifreda. After Eduardo instituted the unlawful
detainer case, Wenifreda instituted a complaint for annulment of
deed of conveyance, title and damages against Eduardo and
Jorge.
Petitioner alleged that the transfer and conveyance of the subject
lot was fraudulent and in bad faith considering that the subject lot
was transferred and conveyed to his sons when the lease was in
full force and effect making the sale null and void; that Cornelio
verbally promised Orlando that Orlando or his heirs shall have
first priority or option to buy the subject lot. Respondents claimed
that they bought the subject lot from their father for value and in
good faith.

On the issue on the right of first refusal of Orlando and his heirs,
SC held that no testimonial evidence was presented to prove the
existence of said right. The claims based on this alleged right of
first refusal cannot be sustained for its existence has not been
duly established.

II.

PERFECTION STAGE

1.

Absolute Acceptance of a Certain Offer


HEIRS OF IGNACIO VS. HOME BANKERS SAVINGS AND
TRUST CO.
G.R. NO. 177783 , JANUARY 23, 2013

FACTS: The case sprang from a real estate mortgage of two


parcels of land in August 1981. Fausto C. Ignacio mortgaged the
properties to Home Bankers Savings and Trust Company (Bank)
as security for a loan extended by the Bank. After Ignacio
defaulted in the payment of the loan, the property was foreclosed
and subsequently sold to the Bank in a public auction.Ignacio
offered to repurchase the property. Universal Properties Inc.
COMPILED BY: WIGMORE #WIGMOREFOREVER

89

SALES Case Digest (Atty. Sarona)


Compiled by: Wigmore #wigmoreforever
(UPI), the banks collecting agent sent Ignacio a letter on March
22, 1984 which contained the terms of the repurchase. However,
Ignacio annotated in the letter new terms and conditions. He
claimed that these were verbal agreements between himself and
the Banks collection agent, UPI.No repurchase agreement was
finalized between Ignacio and the Bank. Thereafter the Bank
sold the property to third parties. Ignacio then filed an action for
specific performance against the Bank for the reconveyance of
the properties after payment of the balance of the purchase
price. He argued that there was implied acceptance of the
counter-offer of the sale through the receipt of the terms by
representatives of UPI. The Bank denied that it gave its consent
to the counter-offer of Ignacio. It countered that it did not approve
the unilateral amendments placed by Ignacio.

Cervantes entered into several negotiations with Villonco for sale


of the Buendia property. Cervantes made a written offer of
P400/sqm with a downpayment of P100,000 to serve as earnest
money. The offer also made the consummation of the sale
dependent upon the acquisition by Bormaheco of a Sta. Ana
property. Villonco made a counter-offer stating that the earnest
money was to earn 10% interest p.a. The check was enclosed
with the reply letter. Cervantes accepted and cashed the check.
The Sta. Ana Property was awarded to Bormaheco; the transfer
was also duly approved. However, Cervantes sent the check
back to Villonco with the interest thereonstating that he was no
longer interested in selling the property. He also claims that no
contract was perfected; Villonco sues for specific performance.
ISSUE: W/N there was a perfected contract of sale

ISSUE: Whether or not the negotiations between Ignacio and


UPI is binding on the Bank.
HELD: A contract of sale is consensual in nature and is perfected
upon mere meeting of the minds. When there is merely an offer
by one party without acceptance of the other, there is no
contract. When the contract of sale is not perfected, it cannot, as
an independent source of obligation, serve as a binding juridical
relation between the parties.
A contract of sale is perfected only when there is consent validly
given. There is no consent when a party merely negotiates a
qualified acceptance or a counter-offer. An acceptance must
reflect all aspects of the offer to amount to a meeting of the
minds between the parties.In this case, while it is apparent that
Ignacio proposed new terms and conditions to the repurchase
agreement, there was no showing that the Bank approved the
modified offer.
In the absence of conformity or acceptance by properly
authorized bank officers of petitioner's counter-proposal, no
perfected repurchase contract was born out of the talks or
negotiations between petitioner and Mr. Lazaro and Mr. Fajardo.
Petitioner therefore had no legal right to compel respondent bank
to accept the P600,000 being tendered by him as payment for
the supposed balance of repurchase price.

HELD: YES. There was a perfected contract of sale. The


contract of sale is perfected at the moment there is a meeting of
minds upon the thing which is the object of the contract and upon
the price. From that moment, the parties may reciprocally
demand performance, subject to the provisions of the law
governing the form of contracts. (Art. 1475 Ibid).
Consent is manifested by the meeting of the offer and the
acceptance upon the thing and the cause which are to constitute
the contract. The offer must be certain and the acceptance
absolute. A qualified acceptance constitutes a counter-offer (Art.
1319, Civil Code). An acceptance may be express or implied
(Art. 1320, Civil Code).
A contract is formed if offer is accepted, whether request for
changes in terms is granted or not; Change does not amount to
rejection of offer or a counter-offer. An acceptance may contain a
request for certain changes in the terms of the offer and yet be a
binding acceptance. So long as it is clear that the meaning of the
acceptance is positively and unequivocally to accept the offer,
whether such request is granted or not, a contract is formed.

A corporation may only give valid acceptance of an offer of sale


through its authorized officers or agents. Specifically, a counteroffer to repurchase a property will not bind a corporation by mere
acceptance of an agent in the absence of evidence of authority
from the corporations board of directors.

The vendors change in a phrase of the offer to purchase, which


change does not essentially change the terms of the offer, does
not amount to a rejection of the offer and the tender or a counteroffer. (The alleged changes made in the counter-offer are
immaterial and are mere clarifications. The changes of the words
Sta. Ana property to another property as well as the insertion of
the number 12 in the date, and the words per annum in the
interest are trivial. There is no incompatibility in the offer and
counter-offer. Cervantes assented to the interest and he, in fact,
paid the same. Also, earnest money constitutes prood of the
perfection of the contract of sale and forms part of the
consideration. The condition regarding the acquisition of the Sta.
Ana property was likewise fulfilled; there is thus no ground for the
refusal of Cervantes to consummate the sale.

2. When Deviation Allowed:

3. Sale by Auction

The negotiations between Ignacio and UPI, the collection agent,


were merely preparatory to the repurchase agreement and,
therefore, was not binding on the Bank. Ignacio could not compel
the Bank to accede to the repurchase of the property.

VILLONCO REALTY COMPANY vs BORMAHECO, INC.,


FRANCISCO N. CERVANTES and ROSARIO N. CERVANTES
G.R. No. L-26872 July 25, 1975

4. Earnest Money

FACTS: Cervantes and his wife owned 3 parcels of land along


Buendia where he buildings of Bormaheco Inc were situated.
Beside their property were lots owned by Villonco Realty.
COMPILED BY: WIGMORE #WIGMOREFOREVER

90

SALES Case Digest (Atty. Sarona)


Compiled by: Wigmore #wigmoreforever
5. Difference Between Earnest Money and Option Money
OESMER VS. PARAISO
FACTS: Petitioners Rizalino, Ernesto, Leonora, Bibiano, Jr.,
Librado, Enriqueta, Adolfo, and Jesus, all surnamed Oesmer
together with Adolfo Oesmer (Adolfo) and Jesus Oesmer
(Jesus), are brothers and sisters, and the co-owners of undivided
shares of two parcels of agricultural and tenanted land. Both lots
are unregistered and originally owned by their parents, Bibiano
Oesmer and Encarnacion Durumpili. When the spouses Oesmer
died, petitioners, together with Adolfo and Jesus, acquired the
lots as heirs of the former by right of succession.
Respondent Paraiso Development Corporation is engaged in the
real estate business. In March 1989, one Rogelio Paular, brought
along petitioner Ernesto to meet with a certain Sotero Lee,
President of respondent Paraiso Development Corporation. The
said meeting was for the purpose of brokering the sale of
petitioners properties to Respondent Corporation.
Pursuant to the said meeting, a Contract to Sell was drafted by
the Executive Assistant of Lee. On 1 April 1989, petitioners
Ernesto and Enriqueta signed the aforesaid Contract to Sell. A
check in the amount of P100,000.00, payable to Ernesto, was
given as option money.
Sometime thereafter, Rizalino, Leonora, Bibiano, Jr., and Librado
also signed the said Contract to Sell. However, two of the
brothers, Adolfo and Jesus, did not sign the document.
Petitioners, through a letter, informed the respondent company of
their intention to rescind the Contract to Sell and to return the
amount of P100,000.00 given by respondent as option money.
Respondent did not respond to the aforesaid letter.
Subsequently, the petitioners, together with Adolfo and Jesus,
filed a Complaint for Declaration of Nullity or for Annulment of
Option Agreement or Contract to Sell with Damages.
ISSUE:
(1) WON the supposed Contract to Sell is really a unilateral
promise to sell without consideration distinct from the price, and
hence, void. (NO, it is indeed a Contract to Sell.)
(2) WON the consideration of P100K paid is an option money. (It
is an earnest money.)
HELD: In the instant case, the consideration of P100,000.00 paid
by respondent to petitioners was referred to as "option money."
However, a careful examination of the words used in the contract
indicates that the money is not option money but earnest money.
"Earnest money" and "option money" are not the same but
distinguished thus: (a) earnest money is part of the purchase
price, while option money is the money given as a distinct
consideration for an option contract; (b) earnest money is given
only where there is already a sale, while option money applies to
a sale not yet perfected; and, (c) when earnest money is given,
the buyer is bound to pay the balance, while when the would-be
buyer gives option money, he is not required to buy, but may
even forfeit it depending on the terms of the option. The sum of
P100,000.00 was part of the purchase price. Although the same
was denominated as "option money," it is actually in the nature of
earnest money or down payment when considered with the other

terms of the contract. Doubtless, the agreement is not a mere


unilateral promise to sell, but, indeed, it is a Contract to Sell as
both the trial court and the appellate court declared in their
Decisions.
6. Sale Deemed Perfected Where Offer Was Made

FORMAL REQUIREMENTS OF SALE


1. Form not Important for Validity of Sale

NARANJA VS. CA
FACTS: Roque Naranja was the registered owner of a parcel of
land, Bacolod. Roque was also a co-owner of an adjacent lot (Lot
No. 2) which he co-owned with his brothers, Gabino and Placido
Naranja.
When Placido died, his one-third share was inherited by his
children, Nenita, Nazareto, Nilda, Naida and Neolanda, all
surnamed Naranja, herein petitioners. The adjacent lot is
covered by TCT No. T-18762 in the names of Roque, Gabino
and the said children of Placido. TCT No. T-18762 remained
even after Gabino died. The other petitioners Serafin Naranja,
Raul Naranja, and Amelia Naranja-Rubinos are the children of
Gabino.
The two lots were being leased by Esso Standard Eastern, Inc.
for 30 years from 1962-1992. For his properties, Roque was
being paid P200.00 per month by the company.
Roque had no other source of income except for the P200.00
monthly rental of his two properties. To show his gratitude to
Belardo, Roque sold Lot No. 4 and his one-third share in Lot No.
2 to Belardo on August 21, 1981, through a Deed of Sale of Real
Property which was duly notarized by Atty. Eugenio Sanicas.
Roques copies of TCT No. T-18764 and TCT No. T-18762 were
entrusted to Atty. Sanicas for registration of the deed of sale and
transfer of the titles to Belardo. But the deed of sale could not be
registered because Belardo did not have the money to pay for
the registration fees.
Belardos only source of income was her store and coffee shop.
Sometimes, her children would give her money to help with the
household expenses, including the expenses incurred for
Roques support. At times, she would also borrow money from
Margarita Dema-ala, a neighbor. When the amount of her loan
reached P15,000.00, Dema-ala required a security.
Roque executed a deed of sale in favor of Dema-ala, covering
his two properties in consideration of the P15,000.00 outstanding
loan and an additional P15,000.00, for a total ofP30,000.00.
Dema-ala explained that she wanted Roque to execute the deed
of sale himself since the properties were still in his name.
Belardo merely acted as a witness. The titles to the properties
were given to Dema-ala for safekeeping.
Three days later, Roque died of influenza. The proceeds of the
loan were used for his treatment while the rest was spent for his
COMPILED BY: WIGMORE #WIGMOREFOREVER

91

SALES Case Digest (Atty. Sarona)


Compiled by: Wigmore #wigmoreforever
burial.

are now the registered owners of the parcels of land

In 1985, Belardo fully paid the loan secured by the second deed
of sale. Dema-ala returned the certificates of title to Belardo,
who, in turn, gave them back to Atty. Sanicas.

The CA reversed the RTC Decision. The CA held that the


unregisterability of a deed of sale will not undermine its validity
and efficacy in transferring ownership of the properties to private
respondent. The CA noted that the records were devoid of any
proof evidencing the alleged vitiation of Roques consent to the
sale; hence, there is no reason to invalidate the sale.
Registration is only necessary to bind third parties, which
petitioners, being the heirs of Roque Naranja, are not. The trial
court erred in applying Article 1544 of the Civil Code to the case
at bar since petitioners are not purchasers of the said properties.
Hence, it is not significant that private respondent failed to
register the deed of sale before the extrajudicial settlement
among the heir.

Unknown to Belardo, petitioners, the children of Placido and


Gabino Naranja, executed an Extrajudicial Settlement Among
Heirs on October 11, 1985, adjudicating among themselves Lot
No. 4. On February 19, 1986, petitioner Amelia Naranja-Rubinos,
accompanied by Belardo, borrowed the two TCTs, together with
the lease agreement with Esso Standard Eastern, Inc., from Atty.
Sanicas on account of the loan being proposed by Belardo to
her. Thereafter, petitioners had the Extrajudicial Settlement
Among Heirs notarized on February 25, 1986. With Roques
copy of TCT No. T-18764 in their possession, they succeeded in
having it cancelled and a new certificate of title, TCT No. T140184, issued in their names.
In 1987, Belardo decided to register the Deed of Sale dated
August 21, 1981. With no title in hand, she was compelled to file
a petition with the RTC to direct the Register of Deeds to
annotate the deed of sale even without a copy of the TCTs. In an
Order dated June 18, 1987, the RTC granted the petition. But
she only succeeded in registering the deed of sale in TCT No. T18762 because TCT No. T-18764 had already been cancelled.
On December 11, 1989, Atty. Sanicas prepared a certificate of
authorization, giving Belardos daughter, Jennelyn P. Vargas, the
authority to collect the payments from Esso Standard Eastern,
Inc. But it appeared from the companys Advice of Fixed
Payment that payment of the lease rental had already been
transferred from Belardo to Amelia Naranja-Rubinos because of
the Extrajudicial Settlement Among Heirs.
On June 23, 1992, Belardo, through her daughter and attorneyin-fact, Rebecca Cordero, instituted a suit for reconveyance with
damages. The complaint prayed that judgment be rendered
declaring Belardo as the sole legal owner of Lot No. 4, declaring
null and void the Extrajudicial Settlement Among Heirs, and TCT
No. T-140184, and ordering petitioners to reconvey to her the
subject property and to pay damages.
Subsequently, petitioners also filed a case against respondent for
annulment of sale and quieting of title with damages, praying,
among others, that judgment be rendered nullifying the Deed of
Sale, and ordering the Register of Deeds of Bacolod City to
cancel the annotation of the Deed of Sale on TCT No. T-18762.
The RTC rendered a Decision in the consolidated cases in favor
of petitioners. The trial court noted that the Deed of Sale was
defective in form since it did not contain a technical description of
the subject properties but merely indicated that they were Lot No.
4, covered by TCT No. T-18764 consisting of 136 square meters,
and one-third portion of Lot No. 2 covered by TCT No. T-18762.
The trial court held that, being defective in form, the Deed of Sale
did not vest title in private respondent. Full and absolute
ownership did not pass to private respondent because she failed
to register the Deed of Sale. She was not a purchaser in good
faith since she acted as a witness to the second sale of the
property knowing that she had already purchased the property
from Roque. Whatever rights private respondent had over the
properties could not be superior to the rights of petitioners, who

ISSUE: Whether or not the deed of sale must contain a technical


description of the subject property in order to be valid
HELD: The Court does not agree with petitioners contention that
a deed of sale must contain a technical description of the subject
property in order to be valid. Petitioners anchor their theory on
Section 127 of Act No. 496, which provides a sample form of a
deed of sale that includes, in particular, a technical description of
the subject property.
To be valid, a contract of sale need not contain a technical
description of the subject property. Contracts of sale of real
property have no prescribed form for their validity; they follow the
general rule on contracts that they may be entered into in
whatever form, provided all the essential requisites for their
validity are present. The requisites of a valid contract of sale
under Article 1458 of the Civil Code are: (1) consent or meeting
of the minds; (2) determinate subject matter; and (3) price certain
in money or its equivalent.
The failure of the parties to specify with absolute clarity the
object of a contract by including its technical description is of no
moment. What is important is that there is, in fact, an object that
is determinate or at least determinable, as subject of the contract
of sale. The form of a deed of sale provided in Section 127 of Act
No. 496 is only a suggested form. It is not a mandatory form that
must be strictly followed by the parties to a contract.
In the instant case, the deed of sale clearly identifies the subject
properties by indicating their respective lot numbers, lot areas,
and the certificate of title covering them. Resort can always be
made to the technical description as stated in the certificates of
title covering the two properties.

DALION VS. CA
FACTS: This is a petition to annul and set aside the decision of
the Court of Appeals rendered on May 26, 1987, upholding the
validity of the sale of a parcel of land by petitioner Segundo
Dalion (hereafter, "Dalion") in favor of private respondent
Ruperto Sabesaje, Jr. (hereafter, "Sabesaje").
On May 28, 1973, Sabesaje sued to recover ownership of a
parcel of land, based on a private document of absolute sale,
dated July 1, 1965, allegedly executed by Dalion, who, however
denied the fact of sale, contending that the document sued upon
is fictitious, his signature thereon, a forgery, and that subject land
COMPILED BY: WIGMORE #WIGMOREFOREVER 92

SALES Case Digest (Atty. Sarona)


Compiled by: Wigmore #wigmoreforever
is conjugal property, which he and his wife acquired in 1960 from
Saturnina Sabesaje as evidenced by the "Escritura de Venta
Absoluta".
The spouses denied claims of Sabesaje that after executing a
deed of sale over the parcel of land, they had pleaded with
Sabesaje, their relative, to be allowed to administer the land
because Dalion did not have any means of livelihood. They
admitted, however, administering since 1958, five (5) parcels of
land in Sogod, Southern Leyte, which belonged to Leonardo
Sabesaje, grandfather of Sabesaje, who died in 1956.
They never received their agreed 10% and 15% commission on
the sales of copra and abaca, respectively. Sabesaje's suit, they
countered, was intended merely to harass, preempt and forestall
Dalion's threat to sue for these unpaid commissions. Dalion
nonetheless still impugns the validity of the sale on the ground
that the same is embodied in a private document, and did not
thus convey title or right to the lot in question since "acts and
contracts which have for their object the creation, transmission,
modification or extinction of real rights over immovable property
must appear in a public instrument."
ISSUE: Whether or not the sale is valid?
HELD: Yes. The provision of Art. 1358 on the necessity of a
public document is only for convenience, not for validity or
enforceability. It is not a requirement for the validity of a contract
of sale of a parcel of land that this be embodied in a public
instrument. A contract of sale is a consensual contract, which
means that the sale is perfected by mere consent. No particular
form is required for its validity. Upon perfection of the contract,
the parties may reciprocally demand performance (Art. 1475,
NCC), i.e., the vendee may compel transfer of ownership of the
object of the sale, and the vendor may require the vendee to pay
the thing sold (Art. 1458, NCC).
The trial court thus rightly and legally ordered Dalion to deliver to
Sabesaje the parcel of land and to execute corresponding formal
deed of conveyance in a public document. Under Art. 1498,
NCC, when the sale is made through a public instrument, the
execution thereof is equivalent to the delivery of the thing.
Delivery may either be actual (real) or constructive. Thus delivery
of a parcel of land may be done by placing the vendee in control
and possession of the land (real) or by embodying the sale in a
public instrument (constructive).

HEIRS OF BIONA VS. CA


FACTS: On October 23, 1953, the late Ernesto Biona, married to
plaintiff-appellee Soledad Biona, was awarded Homestead
Patent over the property subject of this suit, a parcel of
agricultural land, located in Bo. 3, Banga, Cotabato,
On June 3, 1954, Ernesto and Soledad Biona obtained a loan
from the then Rehabilitation Finance Corporation (now the
Development Bank of the Philippines) and put up as collateral
the subject property. On June 12, 1956, Ernesto Biona died
leaving as his heirs herein plaintiffs-appellees, namely, his wife,
Soledad Estrobillo Vda. De Biona, and five daughters, Editha B.
Blancaflor, Marianita B. de Jesus, Vilma B. Blancaflor, Elsie B.
Ramos and Perlita B. Carmen.

On March 1, 1960, plaintiff-appellee Soledad Biona obtained a


loan from defendant-appellant in the amount of P1,000 and as
security therefore, the subject property was mortgaged. It was
further agreed upon by the contracting parties that for a period of
two years until the debt is paid, defendant-appellant shall occupy
the land in dispute and enjoy the usufruct thereof.
The two-year period elapsed but Soledad Biona was not able to
pay her indebtedness. Defendant-appellant continued occupying
and cultivating the subject property without protest from plaintiffsappellees.
On July 3, 1962, defendant-appellant paid the sum of P1,400.00
to the Development Bank of the Philippines to cancel the
mortgage previously constituted by the Biona spouses on June
3, 1953.
Thereafter, and for a period of not less than twenty-five years,
defendant-appellant continued his peaceful and public
occupation of the property, declaring it in his name for taxation
purposes, paying real estate property taxes thereon, and causing
the same to be tenanted.
On June 19, 1985, plaintiffs-appellees, filed a complaint for
recovery of ownership, possession, accounting and damages,
with a prayer for a writ of preliminary mandatory injunction and/
or restraining order against defendant-appellant alleging, among
others, that the latter had unlawfully been depriving them of the
use, possession and enjoyment of the subject property; that the
entire parcel of land, which was devoted and highly suited to
palay and corn, was yielding three harvests annually, with an
average of one hundred twenty (120) sacks of corn and eighty
cavans of rice per hectare; that plaintiffs-appellees were deprived
of its total produce amounting to P150,000.00.
One of the claims of defendant-appellant was that by virtue of his
continuous and peaceful occupation of the property from the time
of its sale and for more than twenty- five years thereafter,
defendant possesses a better right thereto subject only to the
rights of the tenants whom he had allowed to cultivate the land
under the Land Reform Program of the government; and that
plaintiffs alleged right, if any, is barred by the statutes of fraud.
ISSUE: Whether or not the deed of sale was valid and if it
effectively conveyed to the private respondents the subject
property
HELD: YES but with regard only to Soledads share (7/12). But
since the daughters of Biona failed to assert their rights and
allowed defendant Hilajos to occupy the land in peace for more
than 30 years, they are now stopped due to laches.
All the requisites for a valid contract of sale are present in the
instant case. For a valuable consideration of P4,500.00, Soledad
Biona agreed to sell and actually conveyed the subject property
to private respondent. The fact that the deed of sale was not
notarized does not render the agreement null and void and
without any effect. The provision of Article 1358 of the Civil
Code9 on the necessity of a public document is only for
convenience, and not for validity or enforceability.10 The
observance of which is only necessary to insure its efficacy, so
that after the existence of said contract had been admitted, the
party bound may be compelled to execute the proper
document.11 Undeniably, a contract has been entered into by
Soledad Biona and the private respondent. Regardless of its
COMPILED BY: WIGMORE #WIGMOREFOREVER 93

SALES Case Digest (Atty. Sarona)


Compiled by: Wigmore #wigmoreforever
form, it was valid, binding and enforceable between the parties.
Under Art. 1356 of the Civil Code, contracts shall be obligatory in
whatever form they may have been entered into provided all the
essential requisites for their necessary elements for a valid
contract of sale were met when Soledad Biona agreed to sell and
actually conveyed Lot 177 to defendant-appellant who paid the
amount of P4,500.00 therefore. The deed of sale (Exh. 2) is not
made ineffective merely because it is not notarized or does not
appear in a public document.

2. When Form Important in Sale

a. To Bind Third Parties

b. For Enforceability between the Parties: Statute of Frauds

balance of P4,500,000.00 by imposing upon the private


respondents to pay same amount within thirty (30) days from
execution of the contract instead of the former term of ninety (90)
days.
Ruling of the lower court and the respondent judge: The statute
does not require a formal contract drawn up with technical
exactness for the language of Par. 2 of Art. 1403 of the Philippine
Civil Code is '... an agreement...or some note or memorandum
thereof,' thus recognizing a difference between the contract itself
and the written evidence which the statute require. ... The
contract of sale sued upon in this case is supported by letters
and telegrams annexed to the complaint. The private
respondents having alleged that the contract is backed up by
letters and telegrams, and the same being sufficient
memorandum, the complaint states a cause of action and they
should be given their day in court and allowed to substantiate
their allegations.
ISSUES: Whether or not there is a perfected contract of sale
between the parties. (NO) and

YUVIENGCO VS Hon. DACUYCUY and Dely Rodriguez,


Felipe Cruz, Constancia Nogar, et al. (GR No. L-55048 May
27, 1981)

Whether or not the claim for specific performance of respondents


is enforceable under the Statute of Frauds. (NO)

FACTS: Petitioners own a property in Tacloban City which they


intend to sell for 6.5M. They gave the private respondents the
right to purchase the property only until July 31, 1978. Private
respondents replied that they agree to buy the property and they
will negotiate for details. Petitioner sent another telegram
informing respondents that their proposal is accepted and a
contract will be prepared.

HELD:
1st issue: There was no perfected contract of sale yet because
both parties are still under negotiation and hence, no meeting of
the minds. Mr. Gamboa even went to the private respondents to
negotiate for the sale. Even though there was an agreement on
the terms of payment, there was no absolute acceptance
because respondents still insisted on further details.

Lawyer of the petitioners, Mr.Gamboa, arrived bringing a contact


with an altered mode of payment which says that the balance
payment should be paid within 30 days instead of the former 90
days. The original terms of the parties was: respondents will pay
2M upon execution, and the remaining 4.5m after 90 days.

2 issue: The conclusion is inescapable that the claim of private


respondents that petitioners have unjustifiably refused to
proceed with the sale to them of the property in question is
unenforceable under the Statute of Frauds.

In essence, the theory of petitioners is that while it is true that


they did express willingness to sell to private respondents the
subject property (land and building) for P6,500,000.00 provided
the latter made known their own decision to buy it not later than
July 31, 1978, the respondents' reply that they were agreeable
was not absolute, so much so that when ultimately petitioners'
representative went to Cebu City with a prepared and duly
signed contract for the purpose of perfecting and consummating
the transaction, respondents and said representative found
variance between the terms of payment stipulated in the
prepared document and what respondents had in mind, hence
the bank draft which respondents were delivering to the
representative was returned and the document remained
unsigned by respondents.
Hence, the action for specific performance filed by the private
respondents. However, the respondents, in their complaint,
contended That on August 1, 1978 Pedro Gamboa arrived
Tacloban City bringing with him the prepared contract to
purchase and to sell referred to in his telegram dated July 27,
1978 for the purpose of closing the transactions referred to in
paragraphs 8 and 9 hereof, however, to the complete surprise of
private respondents, the petitioner without giving notice to
plaintiffs, changed the mode of payment with respect to the

nd

It is nowhere alleged in said paragraphs 8 to 12 of the complaint


that there is any writing or memorandum, much less a duly
signed agreement to the effect that the price of P6,500,000 fixed
by petitioners for the real property herein involved was agreed to
be paid not in cash but in installments as alleged by
respondents.
The only documented indication of the non-wholly-cash payment
extant in the record is that stipulated, the deeds already signed
by the petitioners and taken to Tacloban by Atty. Gamboa for the
signatures of the respondents.
In other words, the 90-day term for the balance of P4.5 M
insisted upon by respondents choices not appear in any note,
writing or memorandum signed by either the petitioners or any of
them, not even by Atty. Gamboa. Hence, looking at the pose of
private respondents that there was a perfected agreement of
purchase and sale between them and petitioners under which
they would pay in installments of P2 M down and P4.5 M within
90 days afterwards, it is evident that such oral contract involving
the "sale of real property" comes squarely under the Statute of
Frauds.
Respondent judge assumed that the requirement of perfection of
such kind of contract under Article 1475 of the Civil Code which
provides that "the contract of sale is perfected at the moment
COMPILED BY: WIGMORE #WIGMOREFOREVER 94

SALES Case Digest (Atty. Sarona)


Compiled by: Wigmore #wigmoreforever
there is a meeting of the minds upon the thing which is the object
of the contract and upon the price", the Statute would no longer
apply as long as the total price or consideration is mentioned in
some note or memorandum and there is no need of any
indication of the manner in which such total price is to be paid.
Thus, the SC held that in any sale of real property on
installments, the Statute of Frauds read together with the
perfection requirements of Article 1475 of the Civil Code must be
understood and applied in the sense that the idea of payment on
installments must be in the requisite of a note or memorandum
therein contemplated. Stated otherwise, the inessential
elements" relied upon by respondent judge must be deemed to
include the requirement just discussed when it comes to
installment sales.
For the essence and thrust of the said monograph refers only to
the form of the note or memorandum which would comply with
the Statute, and no doubt, while such note or memorandum need
not be in one single document or writing and it can be in just
sufficiently implicit tenor, imperatively the separate notes must,
when put together', contain all the requisites of a perfected
contract of sale.
To put it the other way, under the Statute of Frauds, the contents
of the note or memorandum, whether in one writing or in
separate ones merely indicative for an adequate understanding
of all the essential elements of the entire agreement, may be
said to be the contract itself, except as to the form.

CLAUDEL VS CA and HEIRS OF MACARIO (GR No 85240


July 12, 1991)
FACTS: As early as December 28, 1922, Basilio also known as
"Cecilio" Claudel, acquired from the Bureau of Lands, Lot No.
1230 of the Muntinlupa Estate Subdivision; he secured Transfer
Certificate of Title (TCT) No. 7471 issued by the Registry of
Deeds for the Province of Rizal in 1923; he also declared the lot
in his name. He dutifully paid the real estate taxes thereon until
his death in 1937. Thereafter, his widow "Basilia" and later, her
son Jose, one of the herein petitioners, paid the taxes. The same
piece of land purchased by Cecilio would, however, become the
subject of protracted litigation thirty-nine years after his death.
Two branches of Cecilio's family contested the ownership over
the land-on one hand the children of Cecilio, namely, Modesto,
Loreta, Jose, et al. and on the other, the brother and sisters of
Cecilio, namely, Macario, Esperidiona, Raymunda, and Celestina
et. al. In 1972, the HEIRS OF CECILIO partitioned this lot among
themselves.
Four years later, on December 7, 1976, private respondents
SIBLINGS OF CECILIO, filed Civil Case No. 5276-P as already
adverted to at the outset, with the then Court of First Instance of
Rizal, a "Complaint for Cancellation of Titles and Reconveyance
with Damages," alleging that 46 years earlier, or sometime in
1930, their parents had purchased from the late Cecilio Claudel
several portions of Lot No. 1230 for the sum of P30.00. They
admitted that the transaction was verbal. However, as proof of
the sale, the SIBLINGS OF CECILIO presented a subdivision
plan of the said land, dated March 25, 1930, indicating the
portions allegedly sold to the SIBLINGS OF CECILIO. The Lower
Court dismissed the case. The Court of Appeals reversed the

decision of the trial court 7. Hence, this petition


ISSUE: Whether or not a contract of sale of land may be proven
orally. (NO)
HELD: The rule of thumb is that a sale of land, once
consummated, is valid regardless of the form it may have been
entered into. For nowhere does law or jurisprudence prescribe
that the contract of sale be put in writing before such contract
can validly cede or transmit rights over a certain real property
between the parties themselves.
However, in the event that a third party, as in this case, disputes
the ownership of the property, the person against whom that
claim is brought cannot present any proof of such sale and
hence has no means to enforce the contract. Thus the Statute of
Frauds was precisely devised to protect the parties in a contract
of sale of real property so that no such contract is enforceable
unless certain requisites, for purposes of proof, are met. The
provisions of the Statute of Frauds pertinent to the present
controversy, state:
Art. 1403 (Civil Code). The following
unenforceable, unless they are ratified:

contracts

are

xxx xxx xxx


2) Those that do not comply with the Statute of Frauds as set
forth in this number. In the following cases, an agreement
hereafter made shall be unenforceable by action unless the
same, or some note or memorandum thereof, be in writing, and
subscribed by the party charged, or by his agent; evidence,
therefore, of the agreement cannot be received without the
writing, or a secondary evidence of its contents:
xxx xxx xxx
e) An agreement for the leasing for a longer period than one
year, or for the sale of real property or of an interest therein;
xxx xxx xxx
(Emphasis supplied.)
The purpose of the Statute of Frauds is to prevent fraud and
perjury in the enforcement of obligations depending for their
evidence upon the unassisted memory of witnesses by requiring
certain enumerated contracts and transactions to be evidenced
in Writing.
The provisions of the Statute of Frauds originally appeared under
the old Rules of Evidence. However when the Civil Code was rewritten in 1949 (to take effect in 1950), the provisions of the
Statute of Frauds were taken out of the Rules of Evidence in
order to be included under the title on Unenforceable Contracts
in the Civil Code. The transfer was not only a matter of style but
to show that the Statute of Frauds is also a substantive law.
Therefore, except under the conditions provided by the Statute of
Frauds, the existence of the contract of sale made by Cecilio with
his siblings 13 cannot be proved.

COMPILED BY: WIGMORE #WIGMOREFOREVER

95

SALES Case Digest (Atty. Sarona)


Compiled by: Wigmore #wigmoreforever
SPOUSES ALFREDO vs SPOUSES BORRAS (GR No 144225
June 17, 2003)
FACTS: The Alfredo Spouses mortgaged the subject land
situated in Brgy. Culis, Mabiga, Hermosa, Bataan, to the DBP for
P7,000.00, and in order to pay their debt, the Alfredo Spouses
sold the subject land to the Borras Spouses for P15,000.00. The
Borras paid the loan and its interest and the balance is to be paid
by the Alfredos, and they (Alfredos) delivered the Owner's
Duplicate Copy of OCT No. 284 to them (Borras).
Later, Borras discovered that the Alfredos had re-sold portiions of
the land to several persons. Borras filed an adverse claim with
the Register of Deeds of Bataan, and later they found out that
the Alfredos had secured a duplicate copy of OCT No. 284, the
tax declaration and the receipts of the realty. The Alfredos filed a
complaint for Specific Performance, they claimed that the sale,
not being in writing, is unenforceable under the Statute of
Frauds.
ISSUE: W/N the contract of sale is unenforceable under the
Statute of Frauds. (NO)
HELD: NO. The Statute of Frauds provides that a contract for the
sale of real property shall be unenforceable unless the contract
or some note or memorandum of the sale is in writing and
subscribed by the party charged or his agent. The existence of
the receipt dated 11 March 1970, which is a memorandum of the
sale, removes the transaction from the provisions of the Statute
of Frauds.
The Statute of Frauds applies only to executory contracts and
not to contracts either partially or totally performed. Thus, where
one party has performed ones obligation, oral evidence will be
admitted to prove the agreement. In the instant case, the parties
have consummated the sale of the Subject Land, with both
sellers and buyers performing their respective obligations under
the contract of sale. In addition, a contract that violates the
Statute ofFrauds is ratified by the acceptance of benefits under
the contract.
Alfredo spouses benefited from the contract because they paid
their DBP loan and secured the cancellation of their mortgage
using the money given by Borras. Alfredo also accepted payment
of the balance of the purchase price.
Alfredo spouses cannot invoke the Statute of Frauds to deny the
existence of the verbal contract of sale because they have
performed their obligations, and have accepted benefits, under
the verbal contract. The Borras spouses have also performed
their obligations under the verbal contract. Clearly, both the
sellers and the buyers have consummated the verbal contract of
sale of the Subject Land. The Statute of Frauds was enacted to
prevent fraud. This law cannot be used to advance the very evil
the law seeks to prevent.

c. For Validity: Sale of Realty through Agent, Authority


must be in Writing

III. CONSUMMATION
A.

Obligations of Seller

SANTOS VS SANTOS (GR No 133895 October 2, 2001)


FACTS: Petitioner Zenaida M. Santos is the widow of Salvador
Santos. Salvador Santos is a brother of private respondents
Calixto, Alberto, Antonio, and Rosa Santos-Carreon. The
spouses Jesus and Rosalia Santos are the parents of the 5
siblings.
They owned a parcel of land registered under TCT No. 27571
with an area of 154 square meters, located at Sta. Cruz Manila.
On it was a four-door apartment administered by Rosalia who
rented them out. The spouses had five children, Salvador,
Calixto, Alberto, Antonio and Rosa.
On January 19, 1959 Jesus and Rosalia executed a deed of sale
of the properties in favor of their children Salvador and Rosa.
TCT No. 27571 became TCT No. 60819.
On November 20, 1973 Rosa in turn sold her share to Salvador
which resulted in the issuance of a new TCT No. 113221.
Despite the transfer of the property to Salvador, Rosalia, their
mother, continued to lease and receive rentals from the
apartment units.
November 1, 1979, Jesus died. January 9, 1985, Salvador died.
After a month (Feb 1985), Rosalia died.
Shortly after, petitioner Zenaida, claiming to be Salvador's heir
(specifically, as Salvadors widow), demanded the rent from
Antonio Hombrebueno, a tenant of Rosalia. When the latter
refused to pay, Zenaida filed and ejectment suit against him with
the Metropolitan Trial Court of Manila, Branch 24, which
eventually decided in Zenaida's favor.
On January 5, 1989 - private respondents instituted an action for
reconveyance of property with preliminary injunction against
petitioner in RTC of Manila, where they alleged that the two
deeds of sale executed on January 19, 1959 and November 20,
1973 were simulated for lack of consideration. They were
executed to accommodate Salvador in generation funds for his
business and providing him with greater business flexibility.
Zenaida argued that Salvador was the registered owner of the
property, which could only be subjected to encumbrances or
liens annotated on the title; that the respondents' right to
reconveyance was already barred by prescription and laches;
and that the complaint state no cause of action.
Ruling of the lower court: RTC decided in favour of private
respondents:
a) Declaring the deed of sale executed by Rosalia Santos and
Jesus Santos on January 19, 1959, as entirely null and void for
being fictitious or stimulated and inexistent
b) Declaringthe deed of sale executed by Rosa Santos in favor of
COMPILED BY: WIGMORE #WIGMOREFOREVER

96

SALES Case Digest (Atty. Sarona)


Compiled by: Wigmore #wigmoreforever
Salvador Santos on November 20, 1973, also as entirely null and
void for being likewise fictitious or stimulated and inexistent
c) Directing ROD of Manila to cancel TCT#113221 registered in
the name of Salvador Santos, as well as, TCT# 60819 in the
names of Salvador Santos, Rosa Santos and the Transfer
Certificate of Title No. T-27571 registered in the name of Rosalia
A. Santos, married to Jesus Santos, the same to be partitioned
by the heirs of the said registered owners in accordance with law.
The trial court reasoned that notwithstanding the deeds of sale
transferring the property to Salvador, the spouses Rosalia and
Jesus continued to possess the property and to exercise rights of
ownership not only by receiving the monthly rentals, but also by
paying the realty taxes. Also, Rosalia kept the owner's duplicate
copy of the title even after it was already in the name of
Salvador. Further, the spouses had no compelling reason in 1959
to sell the property and Salvador was not financially capable to
purchase it. The deeds of sale were therefore fictitious. Hence,
the action to assail the same does not prescribe.
The CA affirmed the decision of the RTC. It held that in order for
the execution of a public instrument to effect tradition, as
provided in Article 1498 of the Civil Code, the vendor shall have
had control over the thing sold, at the moment of sale. It was not
enough to confer upon the purchaser the ownership and the right
of possession. The thing sold must be placed in his control. The
subject deeds of sale did not confer upon Salvador the
ownership over the subject property, because even after the
sale, the original vendors remained in dominion, control, and
possession thereof.
ISSUE: WON there was DELIVERY by the Seller
HELD: Theres CONSTRUCTIVE DELIVERY but it was NOT
EFFECTED.
Petitioner in her memorandum invokes Article 1477 of the Civil
Code which provides that ownership of the thing sold is
transferred to the vendee upon its actual or constructive delivery.
Article 1498, in turn, provides that when the sale is made through
a public instrument, its execution is equivalent to the delivery of
the thing subject of the contract. Petitioner avers that applying
said provisions to the case, Salvador became the owner of the
subject property by virtue of the two deeds of sale executed in
his favor.
Nowhere in the Civil Code, however, does it provide that
execution of a deed of sale is a conclusive presumption of
delivery of possession. The Code merely said that the execution
shall be equivalent to delivery. The presumption can be rebutted
by clear and convincing evidence. Presumptive delivery can be
negated by the failure of the vendee to take actual possession of
the land sold.
In Danguilan vs. IAC, 168 SCRA 22, 32 (1988), we held that for
the execution of a public instrument to effect tradition, the
purchaser must be placed in control of the thing sold. When
there is no impediment to prevent the thing sold from converting
to tenancy of the purchaser by the sole will of the vendor,
symbolic delivery through the execution of a public instrument is
sufficient. But if, notwithstanding the execution of the instrument,
the purchaser cannot have the enjoyment and material tenancy
nor make use of it himself or through another in his name, then
delivery has not been effected.

Salvador was never placed in control of the property. The original


sellers retained their control and possession. Therefore, there
was no real transfer of ownership.
In Norkis Distributors, Inc. vs. CA, the SC held that the critical
factor in the different modes of effecting delivery, which gives
legal effect to the act is the actual intention of the vendor to
deliver, and its acceptance by the vendee. Without that intention,
there is no tradition. In the instant case, although the spouses
Jesus and Rosalia executed a deed of sale, they did not deliver
the possession and ownership of the property to Salvador and
Rosa. They agreed to execute a deed of sale merely to
accommodate Salvador to enable him to generate funds for his
business venture.

DY, JR. V. CA, GELAC TRADING INC., AND ANTONIO V.


GONZALES
FACTS: Wilfredo Dy purchased a truck and a farm tractor
through LIBRA which was also mortgaged with the latter, as a
security to the loan.
Petitioner, expresses his desire to purchase his brothers tractor
in a letter to LIBRA which also includes his intention to shoulder
its mortgaged. LIBRA approved the request. At the time that
Wilfredo Dy executed a deed of absolute sale in favor of
petitioner, the tractor and truck were in the possession of LIBRA
for his failure to pay the amortization.
When petitioner finally fulfilled its obligation to pay the tractor,
LIBRA would only release the same only if he would also pay for
the truck. In order to fulfill LIBRAs condition, petitioner convinced
his sister to pay for the remaining truck, to which she released a
check amounting to P22, 000. LIBRA however, insisted that the
check must be first cleared before it delivers the truck and
tractor.
Meanwhile, another case penned Gelac Trading Inc vs. Wilfredo
Dy was pending in Cebu as a case to recover for a sum of
money (P12, 269.80). By a writ of execution the court in Cebu
ordered to seize and levy the tractor which was in the premise of
LIBRA, it was sold in a public auction to which it was purchased
by GELAC. The latter then sold the tractor to Antonio Gonzales.
RTC rendered in favor of petitioner. CA dismissed the case,
alleging that it still belongs to Wilfredo Dy.
ISSUE: Whether or not there was a consummated sale between
Petitioner and LIBRA?
HELD: NO. The payment of the check was actually intended to
extinguish the mortgage obligation so that the tractor could be
released to the petitioner. It was never intended nor could it be
considered as payment of the purchase price because the
relationship between Libra and the petitioner is not one of sale
but still a mortgage. The clearing or encashment of the check
which produced the effect of payment determined the full
payment of the money obligation and the release of the chattel
mortgage. It was not determinative of the consummation of the
COMPILED BY: WIGMORE #WIGMOREFOREVER 97

SALES Case Digest (Atty. Sarona)


Compiled by: Wigmore #wigmoreforever
sale. The transaction between the brothers is distinct and apart
from the transaction between Libra and the petitioner. The
contention, therefore, that the consummation of the sale
depended upon the encashment of the check is untenable.
ADDISON V. FELIX
FACTS: The defendants-appellees spouses Maciana Felix and
Balbino Tioco purchased from plaintiff-appellant A.A. Addison
four parcels of land to which Felix paid, at the time of the
execution of the deed, the sum of P3,000 on account of the
purchase price. She likewise bound herself to the remainder in
installments, the first of P,2000 on July 15, 1914, the second of
P5,000 thirty days after the issuance to her of a certificate of title
under the Land Registration Act, and further, within ten years
from the date of such title, P10 for each cocoanut tree in bearing
and P5 for each such tree not in bearing that might be growing
on said parcels of land on the date of the issuance of title to her,
with the condition that the total price should not exceed P85,000.
It was further stipulated that Felix was to deliver to the Addison
25% of the value of the products that she might obtain from the
four parcels "from the moment she takes possession of them
until the Torrens certificate of title be issued in her favor," and
that within 1 year from the date of the certificate of title in her
favor, Marciana Felix may rescind the contract of purchase and
sale.
In January 1915, Addison, filed suit in the CFI of Manila to
compel Felix to pay the first installment of P2,000, demandable,
in accordance with the terms of the contract of sale. The
defendants Felix and her husband Tioco contended that Addison
had absolutely failed to deliver the lands that were the subject
matter of the sale, notwithstanding the demands they made upon
him for this purpose. The evidence adduced shows Addison was
able to designate only two of the four parcels, and more than
two-thirds of these two were found to be in the possession of one
Juan Villafuerte, who claimed to be the owner of the parts he so
occupied. The trial court held the contract of sale to be rescinded
and ordered Addison to return to Felix the P3,000 paid on
account of the price, together with interest thereon at the rate of
10% per annum.
ISSUE: Was there a delivery made and, therefore, a transfer of
ownership of the thing sold?
HELD: The Supreme Court affirmed the decision of the lower
court, with modification that the interest thereon will be at the rate
of 6% (instead of 10%) per annum from the date of the filing of
the complaint until payment.
The thing is considered to be delivered when it is placed "in the
hands and possession of the vendee." It is true that the same
article declares that the execution of a public instrument is
equivalent to the delivery of the thing which is the object of the
contract, but, in order that this symbolic delivery may produce the
effect of tradition, it is necessary that the vendor shall have had
such control over the thing sold that, at the moment of the sale,
its material delivery could have been made. Symbolic delivery
through the execution of a public instrument is sufficient when
there is no impediment whatever to prevent the thing sold
passing into the tenancy of the purchaser by the sole will of the
vendor. But if, notwithstanding the execution of the instrument,
the purchaser cannot have the enjoyment and material tenancy

of the thing and make use of it himself or through another in his


name, because such are opposed by a third persons will, then
the delivery has not been effected. In the case at bar, therefore, it
is evident, that the mere execution of the instrument was not a
fulfillment of the vendor's obligation to deliver the thing sold, and
that from such non-fulfillment arises the purchaser's right to
demand, as she has demanded, the rescission of the sale and
the return of the price.
SPOUSES SANTOS V. CA
FACTS: Spouses Santos owned the house and lot in Better
Living Subdivision, Paranaque, Metro Manila. The land together
with the house, was mortgaged with the Rural Bank of Salinas,
Inc., to secure a loan of P150K. The bank sent Rosalinda Santos
a letter demanding payment of P16K in unpaid interest and other
charges. Since the Santos couple had no funds, Rosalinda
offered to sell the house and lot to Carmen Caseda. After
inspecting the real property, Carmen and her husband agreed.
Carmen and Rosalinda signed a document, involving the sale of
the house P350K as full amount, P54K as downpayment.
Among other condition set is that Caseda will pay the balance of
the mortgage in the bank, real estate taxes and the electric and
water bills.
The Casedas complied with the bank mortgage and the bills. The
Santoses, seeing that the Casedas lacked the means to pay the
remaining installments and/or amortization of the loan,
repossessed the property. The Santoses then collected the
rentals from the tenants. Carmen approached petitioners and
offered to pay the balance of the purchase price for the house
and lot. The parties, however, could not agree, and the deal
could not push through because the Santoses wanted a higher
price.
Carmen is now praying that the Santoses execute the final deed
of conveyance over the property.
ISSUE: WON there was a perfected contract of sale? NO
HELD: A contract is what the law defines it to be, taking into
consideration its essential elements, and not what the
contracting parties call it. Article 1458 expressly obliges the
vendor to transfer ownership of the thing sold as an essential
element of a contract of sale. This is because the transfer of
ownership in exchange for a price paid or promised is the very
essence of a contract of sale.
There was no transfer of ownership simultaneously with the
delivery of the property purportedly sold. The records clearly
show that, notwithstanding the fact that the Casedas first took
then lost possession of the disputed house and lot, the title to the
property has remained always in the name of Rosalinda Santos.
Although the parties had agreed that the Casedas would assume
the mortgage, all amortization payments made by Carmen
Caseda to the bank were in the name of Rosalinda Santos. The
foregoing circumstances categorically and clearly show that no
valid transfer of ownership was made by the Santoses to the
Casedas. Absent this essential element, their agreement cannot
be deemed a contract of sale.
It was a contract to sell. Ownership is reserved by the vendor
and is not to pass until full payment of the purchase price. This
COMPILED BY: WIGMORE #WIGMOREFOREVER 98

SALES Case Digest (Atty. Sarona)


Compiled by: Wigmore #wigmoreforever
we find fully applicable and understandable in this case, given
that the property involved is a titled realty under mortgage to a
bank and would require notarial and other formalities of law
before transfer thereof could be validly effected.
The CA cannot order rescission. If the vendor should eject the
vendee for failure to meet the condition precedent, he is
enforcing the contract and not rescinding it. When the petitioners
in the instant case repossessed the disputed house and lot for
failure of private respondents to pay the purchase price in full,
they were merely enforcing the contract and not rescinding it.
SPOUSES SANTIAGO V. VILLAMOR
FACTS: Spouses Domingo Villamor, Sr. and Trinidad Villamor
(spouses Villamor, Sr.) executed a deed of sale covering a parcel
of land in favor of petitioners Spouses Erosto and Nelsie
Santiago (spouses Santiago). The land in dispute was occupied
by spouses Villamor, Sr.s children, herein respondents Mancer
Villamor, Carlos Villamor, and Domingo Villamor, Jr. (Villamor
children)
Spouses Santiago demanded the Villamor children to vacate the
property but the latter refused to do so. Villamor children argued
that they are the lawful owners of the land since they acquired
the same from San Jacinto Bank. Thus, spouses Santiago filed
an action for quieting of title before the RTC. The RTC ruled in
favor of spouses Santiago. On appeal, the CA reversed the
RTCs decision on the ground that spouses Santiago failed to
prove their legal or equitable title to the land.
ISSUE: Whether or not the action to quiet title filed by Spouses
Santiago should prosper?
HELD: The petition lacks merit.
CIVIL LAW: quieting of title; constructive delivery; buyer in good
faith
Quieting of title is a common law remedy for the removal of any
cloud, doubt or uncertainty affecting title to real property. The
plaintiffs must show not only that there is a cloud or contrary
interest over the subject real property, but that they have a valid
title to it.
Article 1477 of the Civil Code recognizes that the "ownership of
the thing sold shall be transferred to the vendee upon the actual
or constructive delivery thereof." Related to this article is Article
1497 which provides that "the thing sold shall be understood as
delivered, when it is placed in the control and possession of the
vendee."
With respect to incorporeal property, Article 1498 of the Civil
Code lays down the general rule: the execution of a public
instrument "shall be equivalent to the delivery of the thing which
is the object of the contract, if from the deed the contrary does
not appear or cannot clearly be inferred." However, the execution
of a public instrument gives rise only to a prima facie
presumption of delivery, which is negated by the failure of the
vendee to take actual possession of the land sold. A person who
does not have actual possession of the thing sold cannot transfer
constructive possession by the execution and delivery of a public
instrument.

In this case, no constructive delivery of the land transpired upon


the execution of the deed of sale since it was not the spouses
Villamor, Sr. but the respondents who had actual possession of
the land. The presumption of constructive delivery is inapplicable
and must yield to the reality that the petitioners were not placed
in possession and control of the land.
A purchaser in good faith is one who buys property without notice
that some other person has a right to or interest in such property
and pays its fair price before he has notice of the adverse claims
and interest of another person in the same property. However,
where the land sold is in the possession of a person other than
the vendor, the purchaser must be wary and must investigate the
rights of the actual possessor; without such inquiry, the buyer
cannot be said to be in good faith and cannot have any right over
the property.

LA FUERZA, INC., vs. THE HON. COURT OF APPEALS


FACTS: The plaintiff (Associated Engineering, Co., Inc.) is a
corporation engaged in the manufacture and installation of flat
belt conveyors. The defendant (La Fuerza, Inc.) is also a
corporation engaged in the manufacture of wines.
Sometime in the month of January, 1960, Antonio Co, the
manager of the plaintiff corporation called the office of the
defendant and offered his services to manufacture and install a
conveyor system which, according to him, would increase
production and efficiency of his business.
The president of the defendant corporation then expressed his
conformity to the offer made in Exhibit A by writing at the foot
thereof under the word "confirmation" his signature. He caused,
however, to be added to this offer at the foot a note which reads:
"All specifications shall be in strict accordance with the approved
plan made part of this agreement hereof."
A few days later, Antonio Co made the demand for the down
payment of P5,000.00 which was readily delivered by the
defendant in the form of a check for the said amount. After that
agreement, the plaintiff started to prepare the premises for the
installations of the conveyor system . It seems that the work was
completed during the month of May, 1960. Trial runs were made
in the presence of the president and general manager of the
defendant corporation, Antonio Co, the technical manager of the
plaintiff, and some other people.
As a result of this trial or experimental runs, it was discovered,
according to the defendant's general manager, that the conveyor
system did not function to their satisfaction as represented by the
technical manager of the plaintiff Antonio Co for the reason that,
when operated several bottles collided with each other, some
jumping off the conveyor belt and were broken, causing
considerable damage.
After the last trial run made in the month of July and defects
indicated by the said president and general manager of the
defendant had not been remedied with the result that when the
plaintiff billed the defendant for the balance of the contract price,
the latter refused to pay for the reason that according to the
defendant the conveyor system installed by the plaintiff did not
COMPILED BY: WIGMORE #WIGMOREFOREVER 99

SALES Case Digest (Atty. Sarona)


Compiled by: Wigmore #wigmoreforever
serve the purpose for which the same was manufactured and
installed at such a heavy expense.
On March 22, 1961, the contractor commenced the present
action to recover the sums of P8,250, balance of the stipulated
price of the aforementioned conveyors, and P2,000, as attorney's
fees, in addition to the costs.
La Fuerza maintains that plaintiff is deemed not to have
delivered the conveyors, within the purview of Art. 1571, until it
shall have complied with the conditions or requirements of the
contract between them that is to say, until the conveyors shall
meet La Fuerza's "need of a conveyor system that would
mechanically transport empty bottles from the storage room to
the bottle workers in the production room thus increasing the
production and efficiency" of its business-and La Fuerza had
accepted said conveyors.
ISSUE: WON there was delivery. YES

In the exercise of this right of election, La Fuerza had chosen to


withdraw from the contract, by praying for its rescission; but the
action therefor in the language of Art. 1571 "shall be barred
after six months, from the delivery of the thing sold." The period
of four (4) years, provided in Art. 1389 of said Code, for "the
action to claim rescission," applies to contracts, in general , and
must yields, in the instant case, to said Art. 1571, which refers to
sales in particular.
Indeed, in contracts of the latter type, especially when goods,
merchandise, machinery or parts or equipment thereof are
involved, it is obviously wise to require the parties to define their
position, in relation thereto, within the shortest possible time.
Public interest demands that the status of the relations between
the vendor and the vendee be not left in a condition of
uncertainty for an unreasonable length of time, which would be
the case, if the lifetime of the vendee's right of rescission were
four (4) years.

RULING: Upon the completion of the installation of the


conveyors, in May, 1960, particularly after the last trial run, in
July 1960, La Fuerza was in a position to decide whether or not it
was satisfied with said conveyors, and, hence, to state whether
the same were a accepted or rejected. The failure of La Fuerza
to express categorically whether they accepted or rejected the
conveyors does not detract from the fact that the same were
actually in its possession and control; that, accordingly, the
conveyors had already been delivered by the plaintiff; and that,
the period prescribed in said Art. 1571 had begun to run.

B.

With respect to the second point raised by La Fuerza, Art. 1571


of the Civil Code provides:

To solve its problem of power shortage affecting some areas


within its coverage, MOELCI expressed its intention to purchase
a 10 MVA power transformer from David. For this reason, its
General Manager, Engr. Reynaldo Rada (Engr. Rada), went to
meet David in the latters office in Quezon City. David agreed to
supply the power transformer provided that MOELCI would
secure a board resolution because the item would still have to be
imported.

Actions arising from the provisions of the preceding ten articles


shall be barred after six months, from the delivery of the thing
sold.
Xx
Among the "ten articles" referred to in this provision, are Articles
1566 and 1567, reading:
Art. 1566. The vendor is responsible to the vendee for any
hidden faults or defects in the thing sold, even though he was not
aware thereof. ."This provision shall not apply if the contrary has
been stipulated, and the vendor was not aware of the hidden
faults or defects in the thing sold.
Art. 1567. In the cases of articles 1561, 1562, 1564, 1565 and
1566, the vendee may elect between withdrawing from the
contract and demanding a proportionate reduction of the price,
with damages in either case.
xxx

xxx

xxx

Pursuant to these two (2) articles, if the thing sold has hidden
faults or defects as the conveyors are claimed to have the
vendor in the case at bar, the plaintiff shall be responsible
therefor and the vendee or La Fuerza, in the present case
"may elect between withdrawing from the contract and
demanding a proportional reduction of the price, with damages in
either case."

Delivery/Special Rules
DAVID VS MISAMIS OCCIDENTAL

FACTS: Petitioner Virgilio S. David (David) was the owner or


proprietor of VSD Electric Sales, a company engaged in the
business of supplying electrical hardware including transformers
for rural electric cooperatives like respondent Misamis Occidental
II Electric Cooperative, Inc. (MOELCI), with principal office
located in Ozamis City.

The board resolution was thereafter attached to the proposal. As


stated in the proposal, the subject transformer, together with the
basic accessories, was valued at P5,200,000.00. It was also
stipulated therein that 50% of the purchase price should be paid
as downpayment and the remaining balance to be paid upon
delivery. Freight handling, insurance, customs duties, and
incidental expenses were for the account of the buyer.
The Board Resolution, on the other hand, stated that the
purchase of the said transformer was to be financed through a
loan from the National Electrification Administration (NEA). As
there was no immediate action on the loan application, Engr.
Rada returned to Manila in early December 1992 and requested
David to deliver the transformer to them even without the
required downpayment. David granted the request provided that
MOELCI would pay interest at 24% per annum. Engr. Rada
acquiesced to the condition. On December 17, 1992, the goods
were shipped to Ozamiz City via William Lines. In the Bill of
Lading, a sales invoice was included which stated the agreed
interest rate of 24% per annum.
When no payment was made after several months, Medina was
constrained to send a demand letter, dated September 15, 1993,
COMPILED BY: WIGMORE #WIGMOREFOREVER

100

SALES Case Digest (Atty. Sarona)


Compiled by: Wigmore #wigmoreforever
which MOELCI duly received. Engr. Rada replied in writing that
the goods were still in the warehouse of William Lines again
reiterating that the loan had not been approved by NEA. This
prompted Medina to head back to Ozamiz City where he found
out that the goods had already been released to MOELCI
evidenced by the shipping companys copy of the Bill of Lading
which was stamped "Released," and with the notation that the
arrastre charges in the amount of P5,095.60 had been paid. This
was supported by a receipt of payment with the corresponding
cargo delivery receipt issued by the Integrated Port Services of
Ozamiz, Inc.
On February 17, 1994, David filed a complaint for specific
performance with damages with the RTC. In response, MOECLI
moved for its dismissal on the ground that there was lack of
cause of action as there was no contract of sale, to begin with, or
in the alternative, the said contract was unenforceable under the
Statute of Frauds. MOELCI argued that the quotation letter could
not be considered a binding contract because there was nothing
in the said document from which consent, on its part, to the
terms and conditions proposed by David could be inferred. David
knew that MOELCIs assent could only be obtained upon the
issuance of a purchase order in favor of the bidder chosen by the
Canvass and Awards Committee.
ISSUE:
Whether or not there was a perfected contract of sale.
Whether or not there was a delivery that consummated the
contract.

the purchase. Then, when the loan that MOELCI was relying
upon to finance the purchase was not forthcoming, MOELCI,
through Engr. Rada, convinced David to do away with the 50%
downpayment and deliver the unit so that it could already
address its acute power shortage predicament, to which David
acceded when it made the delivery, through the carrier William
Lines, as evidenced by a bill of lading.
Second, the document specified a determinate subject matter
which was one (1) Unit of 10 MVA Power Transformer with
corresponding KV Line Accessories. And third, the document
stated categorically the price certain in money which was
P5,200,000.00 for one (1) unit of 10 MVA Power Transformer and
P2,169,500.00 for the KV Line Accessories.
In sum, since there was a meeting of the minds, there was
consent on the part of David to transfer ownership of the power
transformer to MOELCI in exchange for the price, thereby
complying with the first element. Thus, the said document cannot
just be considered a contract to sell but rather a perfected
contract of sale.
Second issue: MOELCI, in denying that the power transformer
was delivered to it, argued that the Bill of Lading which David
was relying upon was not conclusive. It argued that although the
bill of lading was stamped "Released," there was nothing in it
that indicated that said power transformer was indeed released
to it or delivered to its possession. For this reason, it is its
position that it is not liable to pay the purchase price of the 10
MVA power transformer.

RULING: The Court finds merit in the petition.


First issue: The elements of a contract of sale are, to wit: a)
Consent or meeting of the minds, that is, consent to transfer
ownership in exchange for the price; b) Determinate subject
matter; and c) Price certain in money or its equivalent.9 It is the
absence of the first element which distinguishes a contract of
sale from that of a contract to sell.
An examination of the alleged contract to sell, "Exhibit A," despite
its unconventional form, would show that said document, with all
the stipulations therein and with the attendant circumstances
surrounding it, was actually a Contract of Sale. The rule is that it
is not the title of the contract, but its express terms or stipulations
that determine the kind of contract entered into by the parties.12
First, there was meeting of minds as to the transfer of ownership
of the subject matter. The letter (Exhibit A), though appearing to
be a mere price quotation/proposal, was not what it seemed. It
contained terms and conditions, so that, by the fact that Jimenez,
Chairman of the Committee on Management, and Engr. Rada,
General Manager of MOELCI, had signed their names under the
word "CONFORME," they, in effect, agreed with the terms and
conditions with respect to the purchase of the subject 10 MVA
Power Transformer. As correctly argued by David, if their
purpose was merely to acknowledge the receipt of the proposal,
they would not have signed their name under the word
"CONFORME."
Besides, the uncontroverted attending circumstances bolster the
fact that there was consent or meeting of minds in the transfer of
ownership. To begin with, a board resolution was issued
authorizing the purchase of the subject power transformer. Next,
armed with the said resolution, top officials of MOELCI visited
Davids office in Quezon City three times to discuss the terms of

To begin with, among the terms and conditions of the proposal to


which MOELCI agreed stated:
2. Delivery Ninety (90) working days upon receipt of your
purchase order and downpayment.
C&F Manila, freight, handling, insurance, custom duties and
incidental expenses shall be for the account of MOELCI II. 13
(Emphasis supplied)
On this score, it is clear that MOELCI agreed that the power
transformer would be delivered and that the freight, handling,
insurance, custom duties, and incidental expenses shall be
shouldered by it.
On the basis of this express agreement, Article 1523 of the Civil
Code becomes applicable.1wphi1 It provides:
Where, in pursuance of a contract of sale, the seller is authorized
or required to send the goods to the buyer delivery of the goods
to a carrier, whether named by the buyer or not, for the purpose
of transmission to the buyer is deemed to be a delivery of the
goods to the buyer, except in the cases provided for in Article
1503, first, second and third paragraphs, or unless a contrary
intent appears. (Emphasis supplied)
Thus, the delivery made by David to William Lines, Inc., as
evidenced by the Bill of Lading, was deemed to be a delivery to
MOELCI. David was authorized to send the power transformer to
the buyer pursuant to their agreement. When David sent the item
through the carrier, it amounted to a delivery to MOELCI.
Furthermore, in the case of Behn, Meyer & Co. (Ltd.) v.
COMPILED BY: WIGMORE #WIGMOREFOREVER 101

SALES Case Digest (Atty. Sarona)


Compiled by: Wigmore #wigmoreforever
Yangco,14 it was pointed out that a specification in a contract
relative to the payment of freight can be taken to indicate the
intention of the parties with regard to the place of delivery. So
that, if the buyer is to pay the freight, as in this case, it is
reasonable to suppose that the subject of the sale is transferred
to the buyer at the point of shipment. In other words, the title to
the goods transfers to the buyer upon shipment or delivery to the
carrier.
Of course, Article 1523 provides a mere presumption and in
order to overcome said presumption, MOELCI should have
presented evidence to the contrary. The burden of proof was
shifted to MOELCI, who had to show that the rule under Article
1523 was not applicable. In this regard, however, MOELCI failed.
There being delivery and release, said fact constitutes partial
performance which takes the case out of the protection of the
Statute of Frauds. It is elementary that the partial execution of a
contract of sale takes the transaction out of the provisions of the
Statute of Frauds so long as the essential requisites of consent
of the contracting parties, object and cause of the obligation
concur and are clearly established to be present.15

BEHN MEYER VS. YANGCO


FACTS: A sale of 80 drums of caustic soda was agreed between
Behn, Meyer & Co. and Teodoro Yanco. The merchandise was
shipped from New York to Manila.
However, the ship carrying the cargo was detained at Penang
and the 71 of the 80 drums were removed. Respondent Yangco
also refused to accept the 9 remaining and also refused to
accept the offer of Behn Meyer to have the products substituted
with other merchandise, which however were different from what
was ordered.
It must be noted that the contract provided for "c.i.f. Manila,
pagadero against delivery of documents."
Yanco filed an action seeking for damages for alleged breach of
contract.
ISSUE: WON Behn, Meyer & Co. should bear the burden of the
loss of the merchandise? YES
RULING: Rule as to delivery of goods by a vendor via a
common carrier (If contract is silent delivery of seller to
common carrier transfer ownership to buyer).
Determination of the place of delivery always resolves itself into
a question of act. If the contract be silent as to the person or
mode by which the goods are to be sent, delivery by the vendor
to a common carrier, in the usual and ordinary course of
business, transfers the property to the vendee.
Payment of freight by the buyer = acquires ownership at the
point of shipment.
A specification in a contact relative to the payment of freight can
be taken to indicate the intention of the parties in regard to the
place of delivery. If the buyer is to pay the freight, it is reasonable
to suppose that he does so because the goods become his at
the point of shipment.

On the other hand, if the seller is to pay the freight, the inference
is equally so strong that the duty of the seller is to have the
goods transported to their ultimate destination and that title to
property does not pass until the goods have reached their
destination.
c.i.f. means Cost, Insurance and Freight = CFI is paid by the
seller.
The letters "c.i.f." found in British contracts stand for cost,
insurance, and freight. They signify that the price fixed covers not
only the cost of the goods, but the expense of freight and
insurance to be paid by the seller.
F.O.B. stands for Free on Board = seller bear all expenses
until goods are delivered.
In this case, in addition to the letters "c.i.f.," has the word
following, "Manila." In mercantile contracts of American origin the
letters "F.O.B." standing for the words "Free on Board," are
frequently used. The meaning is that the seller shall bear all
expenses until the goods are delivered where they are to be
"F.O.B."
According as to whether the goods are to be delivered "F.O.B." at
the point of shipment or at the point of destination determines the
time when property passes. However, both the terms "c.i.f." and
"F.O.B." merely make rules of presumption which yield to proof of
contrary intention.
Delivery was to be made at Manila.
Hence, we believe that the word Manila in conjunction with the
letters "c.i.f." must mean that the contract price, covering costs,
insurance, and freight, signifies that delivery was to made at
Manila. If petitioner Behn Meyer has seriously thought that the
place of delivery was New York and Not Manila, it would not have
gone to the trouble of making fruitless attempts to substitute
goods for the merchandise named in the contract, but would
have permitted the entire loss of the shipment to fall upon the
defendant.
Behn Meyer failed to prove that it performed its part in the
contract.
In this case, the place of delivery was Manila and plaintiff (Behn
Meyer) has not legally excused default in delivery of the specified
merchandise at that place. In resume, we find that the plaintiff
has not proved the performance on its part of the conditions
precedent in the contract.
For breach of warranty, the buyer (Yanco) may demand
rescission of the contract of sale.
The warranty the material promise of the seller to the buyer
has not been complied with. The buyer may therefore rescind the
contract of sale because of a breach in substantial particulars
going to the essence of the contract. As contemplated by article
1451 of the Civil Code, the vendee can demand fulfillment of the
contract, and this being shown to be impossible, is relieved of his
obligation. There thus being sufficient ground for rescission, the
defendant is not liable.

Payment of freight by the seller = title of property does not


pass until the goods have reached their destination.
COMPILED BY: WIGMORE #WIGMOREFOREVER

102

SALES Case Digest (Atty. Sarona)


Compiled by: Wigmore #wigmoreforever
D.

Double Sales
CORONEL vs CA

FACTS: The petition involves a complaint for specific


performance to compel petitioners to consummate the sale of a
parcel of land with its improvements located along Roosevelt
Avenue in Quezon City entered into by the parties sometime in
January 1985 for the price of P1,240,000.00.
On January 19, 1985, defendants-appellants Romulo Coronel, et
al. (Coronels) executed a document entitled "Receipt of Down
Payment" in favor of plaintiff Ramona Patricia Alcaraz
(hereinafter referred to as Ramona)
Clearly, the conditions appurtenant to the sale are the following:
1. Ramona will make a down payment P50,000.00 upon
execution of the document aforestated; 2. The Coronels will
cause the transfer in their names of the title of the property
registered in the name of their deceased father upon receipt of
the P50,000.00 down payment; 3. Upon the transfer in their
names of the subject property, the Coronels will execute the
deed of absolute sale in favor of Ramona and the latter will pay
the former the whole balance of P1,190,000.00.
On the same date (January 15, 1985), Concepcion D. Alcaraz
(Concepcion), mother of Ramona, paid the down payment of
P50,000.00.
On February 6, 1985, the property originally registered in the
name of the Coronels' father was transferred in their names
under TCT No. 327043.
On February 18, 1985, the Coronels sold the property covered
by TCT No. 327043 to intervenor-appellant Catalina B. Mabanag
(Catalina) for P1,580,000.00
after the latter has paid
P300,000.00. For this reason, Coronels canceled and rescinded
the contract with Ramona by depositing the down payment paid
by Concepcion in the bank in trust for Ramona Patricia Alcaraz .
On February 22, 1985, Concepcion, et al., filed a complaint for
specific performance against the Coronels and caused the
annotation of a notice of lis pendens at the back of TCT No.
327403.
On April 2, 1985, Catalina caused the annotation of a notice of
adverse claim covering the same property with the Registry of
Deeds of Quezon City (Exh. "F"; Exh. "6").
On April 25, 1985, the Coronels executed a Deed of Absolute
Sale over the subject property in favor of Catalina to which a new
title over the subject property was issued in her name.
ISSUE: WON the "Receipt of Down Payment" embodied a
perfected contract of sale, which perforce, they seek to enforce
by means of an action for specific performance or signified only a
mere executory contract to sell, subject to certain suspensive
conditions/ WON double sale applies.
RULING: The parties (Coronel and Alcaraz) had agreed to a
conditional contract of sale, consummation of which is subject
only to the successful transfer of the certificate of title from the
name of petitioners' father, Constancio P. Coronel, to their
names.

It is essential to distinguish between a contract to sell and a


conditional contract of sale specially in cases where the subject
property is sold by the owner not to the party the seller
contracted with, but to a third person, as in the case at bench. In
a contract to sell, there being no previous sale of the property, a
third person buying such property despite the fulfillment of the
suspensive condition such as the full payment of the purchase
price, for instance, cannot be deemed a buyer in bad faith and
the prospective buyer cannot seek the relief of reconveyance of
the property. There is no double sale in such case. Title to the
property will transfer to the buyer after registration because there
is no defect in the owner-seller's title per se, but the latter, of
course, may be used for damages by the intending buyer.
In a conditional contract of sale, however, upon the fulfillment of
the suspensive condition, the sale becomes absolute and this will
definitely affect the seller's title thereto. In fact, if there had been
previous delivery of the subject property, the seller's ownership
or title to the property is automatically transferred to the buyer
such that, the seller will no longer have any title to transfer to any
third person. Applying Article 1544 of the Civil Code, such
second buyer of the property who may have had actual or
constructive knowledge of such defect in the seller's title, or at
least was charged with the obligation to discover such defect,
cannot be a registrant in good faith. Such second buyer cannot
defeat the first buyer's title. In case a title is issued to the second
buyer, the first buyer may seek reconveyance of the property
subject of the sale.
The agreement could not have been a contract to sell because
the sellers herein made no express reservation of ownership or
title to the subject parcel of land . Furthermore, the circumstance
which prevented the parties from entering into an absolute
contract of sale pertained to the sellers themselves (the
certificate of title was not in their names) and not the full payment
of the purchase price. Under the established facts and
circumstances of the case, the Court may safely presume that,
had the certificate of title been in the names of petitioners-sellers
at that time, there would have been no reason why an absolute
contract of sale could not have been executed and
consummated right there and then.
Thus, the parties did not merely enter into a contract to sell
where the sellers, after compliance by the buyer with certain
terms and conditions, promised to sell the property to the latter.
What may be perceived from the respective undertakings of the
parties to the contract is that petitioners had already agreed to
sell the house and lot they inherited from their father, completely
willing to transfer full ownership of the subject house and lot to
the buyer if the documents were then in order. It just happened,
however, that the transfer certificate of title was then still in the
name of their father. It was more expedient to first effect the
change in the certificate of title so as to bear their names. That is
why they undertook to cause the issuance of a new transfer of
the certificate of title in their names upon receipt of the down
payment in the amount of P50,000.00. As soon as the new
certificate of title is issued in their names, petitioners were
committed to immediately execute the deed of absolute sale.
Only then will the obligation of the buyer to pay the remainder of
the purchase price arise.
What is clearly established by the plain language of the subject
document is that when the said "Receipt of Down Payment" was
COMPILED BY: WIGMORE #WIGMOREFOREVER 103

SALES Case Digest (Atty. Sarona)


Compiled by: Wigmore #wigmoreforever
prepared and signed by petitioners Romeo A. Coronel, et al., the
parties had agreed to a conditional contract of sale,
consummation of which is subject only to the successful transfer
of the certificate of title from the name of petitioners' father,
Constancio P. Coronel, to their names.
The Court significantly notes this suspensive condition was, in
fact, fulfilled on February 6, 1985 (Exh. "D"; Exh. "4"). Thus, on
said date, the conditional contract of sale between petitioners
and private respondent Ramona P. Alcaraz became obligatory,
the only act required for the consummation thereof being the
delivery of the property by means of the execution of the deed of
absolute sale in a public instrument, which petitioners
unequivocally committed themselves to do as evidenced by the
"Receipt of Down Payment."
With the foregoing conclusions, the sale to the other petitioner,
Catalina B. Mabanag, gave rise to a case of double sale where
Article 1544 of the Civil Code will apply, to wit:
Art. 1544. If the same thing should have been sold to different
vendees, the ownership shall be transferred to the person who
may have first taken possession thereof in good faith, if it should
be movable property. Should if be immovable property, the
ownership shall belong to the person acquiring it who in good
faith first recorded it in Registry of Property. Should there be no
inscription, the ownership shall pertain to the person who in good
faith was first in the possession; and, in the absence thereof to
the person who presents the oldest title, provided there is good
faith.

1985. At the time of registration, therefore, petitioner Mabanag


knew that the same property had already been previously sold to
private respondents, or, at least, she was charged with
knowledge that a previous buyer is claiming title to the same
property. Petitioner Mabanag cannot close her eyes to the defect
in petitioners' title to the property at the time of the registration of
the property.
Thus, the sale of the subject parcel of land between petitioners
and Ramona P. Alcaraz, perfected on February 6, 1985, prior to
that between petitioners and Catalina B. Mabanag on February
18, 1985, was correctly upheld by both the courts below.
SAN LORENZO DEVELOPMENT CORPORATION VS. CA
G.R. NO. 124242, January 21, 2005
FACTS: On 20 August 1986, the Spouses Lu purportedly sold
the two parcels of land to respondent Pablo Babasanta for the
price of P15 per square meter. The latter made a downpayment
of P50,000.00 as evidenced by a memorandum receipt issued by
Pacita Lu of the same date. Several other payments totaling
P200,000.00 were made by Babasanta.
Babasanta demanded the execution of a Final Deed of Sale in
his favor so he may effect full payment of the purchase price and
notified the spouses about having received information that the
spouses sold the same property to another without his
knowledge and consent. He demanded that the second sale be
cancelled and that a final deed of sale be issued in his favor.

The above-cited provision on double sale presumes title or


ownership to pass to the first buyer, the exceptions being: (a)
when the second buyer, in good faith, registers the sale ahead of
the first buyer, and (b) should there be no inscription by either of
the two buyers, when the second buyer, in good faith, acquires
possession of the property ahead of the first buyer. Unless, the
second buyer satisfies these requirements, title or ownership will
not transfer to him to the prejudice of the first buyer.

In response, Pacita Lu wrote a letter to Babasanta wherein she


acknowledged having agreed to sell the property, but reminded
Babasanta that when the balance of the purchase price became
due, he requested for a reduction of the price and when she
refused, Babasanta backed out of the sale. Pacita added that
she returned P50,000.00 to Babasanta through Eugenio Oya.
Thus, Babasanta filed a case for Specific Performance and
Damages.

Petitioner point out that the notice of lis pendens in the case at
bar was annoted on the title of the subject property only on
February 22, 1985, whereas, the second sale between
petitioners Coronels and petitioner Mabanag was supposedly
perfected prior thereto or on February 18, 1985. The idea
conveyed is that at the time petitioner Mabanag, the second
buyer, bought the property under a clean title, she was unaware
of any adverse claim or previous sale, for which reason she is
buyer in good faith.

On 19 January 1990, herein petitioner San Lorenzo


Development Corporation (SLDC) filed a Motion for Intervention
and alleged that it had legal interest in the subject matter under
litigation because on 3 May 1989, the two parcels of land
involved had been sold to it in a Deed of Absolute Sale with
Mortgage. It alleged that it was a buyer in good faith and for
value and therefore it had a better right over the property in
litigation.

We are not persuaded by such argument.


In a case of double sale, what finds relevance and materiality is
not whether or not the second buyer was a buyer in good faith
but whether or not said second buyer registers such second sale
in good faith, that is, without knowledge of any defect in the title
of the property sold.
As clearly borne out by the evidence in this case, petitioner
Mabanag could not have in good faith, registered the sale
entered into on February 18, 1985 because as early as February
22, 1985, a notice of lis pendens had been annotated on the
transfer certificate of title in the names of petitioners, whereas
petitioner Mabanag registered the said sale sometime in April,

Respondent Babasanta argued that the latter had no legal


interest in the case because the two parcels of land involved had
already been conveyed to him by the Spouses Lu and hence, the
vendors were without legal capacity to transfer or dispose of the
two parcels of land to the intervenor.
ISSUES:
1. Who between SLDC and Babasanta has a better right
over the two parcels of land subject of the instant case
in view of the successive transactions executed by the
Spouses Lu. - SLDC
2. Whether or not the agreement between Babasanta and
Spouses Lu was a contract to sell or a contract of sale. Contract to Sell
COMPILED BY: WIGMORE #WIGMOREFOREVER

104

SALES Case Digest (Atty. Sarona)


Compiled by: Wigmore #wigmoreforever
3.

Whether or not there was a double sale. - No double


sale

HELD: An analysis of the facts obtaining in this case, as well as


the evidence presented by the parties, irresistibly leads to the
conclusion that the agreement between Babasanta and the
Spouses Lu is a contract to sell and not a contract of sale.
The receipt signed by Pacita Lu merely states that she accepted
the sum of fifty thousand pesos (P50,000.00) from Babasanta as
partial payment of 3.6 hectares of farm lot. While there is no
stipulation that the seller reserves the ownership of the property
until full payment of the price which is a distinguishing feature of
a contract to sell, the subsequent acts of the parties convince us
that the Spouses Lu never intended to transfer ownership to
Babasanta except upon full payment of the purchase price.
Babasantas letter dated 22 May 1989 was quite telling. He
stated therein that despite his repeated requests for the
execution of the final deed of sale in his favor so that he could
effect full payment of the price, Pacita Lu allegedly refused to do
so. In effect, Babasanta himself recognized that ownership of
the property would not be transferred to him until such time as he
shall have effected full payment of the price. Doubtlessly, the
receipt signed by Pacita Lu should legally be considered as a
perfected contract to sell.
The perfected contract to sell imposed upon Babasanta the
obligation to pay the balance of the purchase price. There being
an obligation to pay the price, Babasanta should have made the
proper tender of payment and consignation of the price in court
as required by law. Glaringly absent from the records is any
indication that Babasanta even attempted to make the proper
consignation of the amounts due, thus, the obligation on the part
of the sellers to convey title never acquired obligatory force.
There was no double sale in this case because the contract in
favor of Babasanta was a mere contract to sell; hence, Art. 1544
is not applicable. There was neither actual nor constructive
delivery as his title is based on a mere receipt. Based on this
alone, the right of SLDC must be preferred.
PAGADUAN VS. SPOUSES OCUMA
G.R. 176308, May 8, 2009
FACTS: The subject lot used to be part of a big parcel of land
that originally belonged to Nicolas Cleto. The big parcel of land
was the subject of two separate lines of dispositions. The first
line of disposition: Cleto sold land to Antonio Cereso on May 11,
1925. Cereso in turn sold the land to the siblings with the
surname Antipolo on September 23, 1943. The Antipolos sold the
property to Agaton Pagaduan, father of petitioners, on March 24,
1961. All the dispositions in this line were not registered and did
not result in the issuance of new certificates of title in the name
of the purchasers.
The second line of disposition: started on January 30, 1954, after
Cletos death, when his widow Ruperta Asuncion as his sole heir
and new owner of the entire tract, sold the same to Eugenia
Reyes. This resulted in the issuance Transfer Certificate of Title
(TCT) No. T-1221 in the name of Eugenia Reyes in lieu of TCT
No. T-1220 in the name of Ruperta Asuncion.
On November 26, 1961, Eugenia Reyes executed a unilateral

deed of sale where she sold the northern portion with an area of
32,325 square meters to respondents for P1,500.00 and the
southern portion consisting of 8,754 square meters to Agaton
Pagaduan for P500.00. (FIRST SALE)
Later, on June 5, 1962, Eugenia executed another deed of sale,
this time conveying the entire parcel of land, including the
southern portion, in respondents favor (SECOND SALE). Thus,
TCT No. T-1221 was cancelled and in lieu thereof TCT No. T5425 was issued in the name of respondents. On June 27, 1989,
respondents subdivided the land into two lots.
On July 26, 1989, petitioners instituted a complaint for
reconveyance of the southern portion with an area of 8,754
square meters, with damages, against respondents before the
RTC of Olongapo City.
RTC decided in petitioners favor; a constructive trust over the
property was created in petitioners favor.
CA reversed decision; while the registration of the southern
portion in the name of respondents had created an implied trust
in favor of Agaton Pagaduan, petitioners, however, failed to show
that they had taken possession of the said portion.
ISSUE: Whether or not there was a double sale.
HELD: In this case, there was a double sale. Article 1544 should
apply.
ART. 1544. If the same thing should have been sold to different
vendees, the ownership shall be transferred to the person who
may have first possession thereof in good faith, if it should be
movable property.
Should it be immovable property, the ownership shall belong to
the person acquiring it who in good faith first recorded it in the
Registry of Property.
Should there be no inscription, the ownership shall pertain to the
person who in good faith was first in possession; and, in the
absence thereof; to the person who presents the oldest title,
provided there is good faith.
Where it is an immovable property that is the subject of a double
sale, ownership shall be transferred: (1) to the person acquiring it
who in good faith first recorded it in the Registry of Property; (2)
in default thereof, to the person who in good faith was first in
possession; and (3) in default thereof, to the person who
presents the oldest title, provided there is good faith. The
requirement of the law then is two-fold: acquisition in good faith
and registration in good faith.
DOUBLE SALE: first sale by Eugenia Reyes to Agaton
Pagaduan and a second sale by Eugenia Reyes to the
respondents.
For a second buyer like the respondents to successfully invoke
the second paragraph, Article 1544 of the Civil Code, it must
possess goodvfaith from the time of the sale in its favor until the
registration of the same. Respondents sorely failed to meet this
requirement of good faith since they had actual knowledge of
Eugenias prior sale of the southern portion property to the
petitioners, a fact antithetical to good faith. This cannot be denied
by respondents since in the same deed of sale that Eugenia sold
COMPILED BY: WIGMORE #WIGMOREFOREVER 105

SALES Case Digest (Atty. Sarona)


Compiled by: Wigmore #wigmoreforever
them the northern portion to the respondents for P1,500.00,
Eugenia also sold the southern portion of the land to Agaton
Pagaduan for P500.00.
It is to be emphasized that the Agaton Pagaduan never parted
with the ownership and possession of that portion of Lot No. 785
which he had purchased from Eugenia Santos. Hence, the
registration of the deed of sale by respondents was ineffectual
and vested upon them no preferential rights to the property in
derogation of the rights of the petitioners.
Knowledge gained by respondents of the first sale defeats their
rights even if they were first to register the second sale.
Knowledge of the first sale blackens this prior registration with
bad faith. Good faith must concur with the registration. Therefore,
because the registration by the respondents was in bad faith, it
amounted to no registration at all. As the respondents gained no
rights over the land, it is petitioners who are the rightful owners,
having established that their successor-in-interest. Agaton
Pagaduan had purchased the property from Eugenia Reyes on
November 26, 1961 and in fact took possession of the said
property.

CARBONELL VS. CA
TUESDAY, SEPTEMBER 9, 2014
FACTS: Respondent Jose Poncio was the owner of the parcel of
land located in Rizal. (Area more or less 195 sq. m.) The said
lot was subject to mortgage in favor of the Republic Savings
Bank for the sum of P1,500.00.
Carbonell and respondent Emma Infante offered to buy the said
lot from Poncio. Poncio offered to sell his lot to Carbonell
excluding the house on which he and his family stayed.
Carbonell accepted the offer and proposed the price of P9.50/sq.
m.. Poncio accepted the price on the condition that from the
purchase pric would come the money to be paid to the bank.
January 27, 1995: The parties executed a document in the
Batanes dialect which is translated as: CONTRACT FOR ONE
HALF LOT WHICH I (Poncio) BOUGHT FROM.
Carbonell asked a lawyer to prepare the deed of sale and
delivered the document, together with the balance of P400, to
Jose Poncio. (Note: Carbonell already paid P200 for the
mortgage debt of Poncio + obligated herself to pay the remaining
installments.) However, when she went to Poncio, the latter
informed her that he could no longer proceed with the sale as the
lot was already sold to Emma Infante and that he could not
withdraw with the sale. Poncio admitted that on January 30,
1995, Mrs. Infante improved her offer and he agreed to sell the
land and its improvements to her for P3,535.00.
In a private memorandum agreement, Poncio bound to sell to
Infante the lot for the sum of P2,357.52, with Infante still
assuming the mortgage debt of P1,177.48. (Note: The full
amount of mortgage debt was already paid by the Infantes)
February 2, 1995: A deed of sale was executed between Poncio
and Infante.

February 8, 1995: Knowing that the sale to Infante has not been
registered, Carbonell filed an adverse claim.
February 12, 1995: The deed of sale was registered but it has an
annotation of the adverse claim of Carbonell.
Thereafter, Emma Infante took possession of the lot, built a
house and introduced some improvements.
In June 1995, Carbonell filed a complaint praying that she be
declared the lawful owner of the land, that the subsequent sale to
spouses Infante be declared null and void, and that Jose Poncio
be ordered to execute the corresponding deed of conveyance of
said land in her favor.
RTC ruled that the sale to spouses Infante was null and void.
After re-trial, it reversed its ruling. CA ruled in favor of Carbonell
but after a MfR, it reversed its ruling and ruled in favor of the
Infantes.
ISSUE: WON Carbonell has a superior right over Emma Infante.
YES
HELD: Article 1544 provides that for double sale of an
immovable property, the ownership shall belong to the person
who first acquired it in good faith and recorded it in the Registry
of Property
Article 1544, New Civil Code, which is decisive of this case,
recites:
If the same thing should have been sold to different vendees, the
ownership shall be transferred to the person who may have first
taken possession thereof in good faith, if it should movable
property.
Should it be immovable property, the ownership shall belong to
the person acquiring it who in good faith first recorded it in the
Registry of Property.
Should there be no inscription, the ownership shall pertain to the
person who in good faith was first in the possession; and, in the
absence thereof, to the person who presents the oldest title,
provided there is good faith.
The buyer must act in good faith in registering the deed of sale
It is essential that the buyer of realty must act in good faith in
registering his deed of sale to merit the protection of the second
paragraph of said Article 1544.
Unlike the first and third paragraphs of said Article 1544, which
accord preference to the one who first takes possession in good
faith of personal or real property, the second paragraph directs
that ownership of immovable property should be recognized in
favor of one "who in good faith first recorded" his right. Under the
first and third paragraph, good faith must characterize the act of
anterior registration.
Rule when there is inscription or not
If there is no inscription, what is decisive is prior possession in
good faith. If there is inscription, as in the case at bar, prior
registration in good faith is a pre-condition to superior title.
Carbonell was in good faith when she bought the lot
When Carbonell bought the lot from Poncio on January 27, 1955,
COMPILED BY: WIGMORE #WIGMOREFOREVER 106

SALES Case Digest (Atty. Sarona)


Compiled by: Wigmore #wigmoreforever
she was the only buyer thereof and the title of Poncio was still in
his name solely encumbered by bank mortgage duly annotated
thereon. Carbonell was not aware and she could not have
been aware of any sale of Infante as there was no such sale
to Infante then.
Hence, Carbonell's prior purchase of the land was made in good
faith. Her good faith subsisted and continued to exist when she
recorded her adverse claim four (4) days prior to the registration
of Infantes's deed of sale.
Carbonells good faith did not cease when she was informed by
Poncio about the sale to Emma Infante
After learning about the second sale, Carbonell tried to talk to the
Infantes but the latter refused.
(Exact words of the SC: With an aristocratic disdain unworthy of
the good breeding of a good Christian and good neighbor,
Infante snubbed Carbonell like a leper and refused to see her.)
So Carbonell did the next best thing to protect her right she
registered her adversed claim on February 8, 1955. Under the
circumstances, this recording of her adverse claim should be
deemed to have been done in good faith and should emphasize
Infante's bad faith when she registered her deed of sale four (4)
days later on February 12, 1955.
The Infantes were in bad faith (5 indications of bad faith listed
below)
Bad faith arising from previous knowledge by Infante of the prior
sale to Carbonell is shown by the following facts:
1. Mrs. Infante refused to see Carbonell.
Her refusal to talk to Carbonell could only mean that she did not
want to listen to Carbonell's story that she (Carbonell) had
previously bought the lot from Poncio.
2. Carbonell was already in possession of mortgage passbook
and copy of the mortgage contract. (Not Poncios saving deposit
passbook.)
Infante naturally must have demanded from Poncio the delivery
to her of his mortgage passbook and mortgage contract so that
the fact of full payment of his bank mortgage will be entered
therein; and Poncio, as well as the bank, must have inevitably
informed her that said mortgage passbook could not be given to
her because it was already delivered to Carbonell.
3. Emma Infante did not inquire why Poncio was no longer in
possession of the mortgage passbook and why it was in
Carbonells possession.
The fact that Poncio was no longer in possession of his
mortgage passbook and that the said mortgage passbook was
already in possession of Carbonell, should have compelled
Infante to inquire from Poncio why he was no longer in
possession of the mortgage passbook and from Carbonell why
she was in possession of the same.
4. Emma Infante registered the sale under her name after
Carbonell filed an adverse claim 4 days earlier.
Here she was again on notice of the prior sale to Carbonell. Such
registration of adverse claim is valid and effective.
5. Infante failed to inquire to Poncio WON he had already sold

the property to Carbonell especially that it can be shown that he


was aware of the offer made by Carbonell.
Poncio alleged in his answer that Mrs. Infante and Mrs.
Carbonell offered to buy the lot at P15/sq. m. which offers he
rejected as he believed that his lot is worth at least P20.00/sq. m.
It is therefore logical to presume that Infante was told by Poncio
and consequently knew of the offer of Carbonell which fact
likewise should have put her on her guard and should have
compelled her to inquire from Poncio whether or not he had
already sold the property to Carbonell
The existence of prior sale to Carbonell was duly established
From the terms of the memorandum, it tends to show that the
sale of the property in favor of Carbonell is already an
accomplished act. As found by the trial court, to repeat the said
memorandum states "that Poncio is allowed to stay in the
property which he had sold to the plaintiff ..., it tends to show that
the sale of the property in favor of the plaintiff is already an
accomplished act..."
There was an adequate consideration or price for the sale in
favor of Carbonell
Poncio agreed to sell the same to Carbonell at P9.50 per square
meter, on condition that Carbonell:
1. Should pay (a) the amount of P400.00 to Poncio and the
arrears in the amount of P247.26 to the bank
2. Should assume his mortgage indebtedness.
The bank president agreed to the said sale with assumption of
mortgage in favor of Carbonell an Carbonell accordingly paid the
arrears of P247.26.
It is evident therefore that there was ample consideration, and
not merely the sum of P200.00, for the sale of Poncio to
Carbonell of the lot in question.
The subject property was identified and described
The court has arrived at the conclusion that there is sufficient
description of the lot referred to in Exh. As none other than the
parcel of lot occupied by the defendant Poncio and where he has
his improvements erected. The Identity of the parcel of land
involved herein is sufficiently established by the contents of the
note Exh. 'A'.
SPOUSES ROQUE vs. AGUADO, et.al.
G.R. No. 193787
April 7, 2014
PONENTE: Perlas-Bernabe, J.
TOPIC: Contract of conditional sale, contract to sell, double sale
FACTS: On July 21, 1977, petitioners-spouses Roque and the
original owners of the then unregistered Lot 18089 namely,
Rivero, et al. executed the 1977 Deed of Conditional Sale over a
1,231-sq. m. portion of Lot 18089 for a consideration of
P30,775.00. The parties agreed that Sps. Roque shall make an
initial payment of P15,387.50 upon signing, while the remaining
balance of the purchase price shall be payable upon the
registration of Lot 18089, as well as the segregation and the
concomitant issuance of a separate title over the subject portion
in their names. After the deeds execution, Sps. Roque took
possession and introduced improvements on the subject portion
which they utilized as a balut factory.
COMPILED BY: WIGMORE #WIGMOREFOREVER 107

SALES Case Digest (Atty. Sarona)


Compiled by: Wigmore #wigmoreforever

Pertinent provision of the 1977 Deed of Conditional Sale:


DEED OF CONDITIONAL SALE OF REAL PROPERTY
KNOW ALL MEN BY THESE PRESENTS:
xxx
That for and in consideration of the sum of THIRTY THOUSAND
SEVEN HUNDRED SEVENTY FIVE PESOS (P30,775.00),
Philippine Currency, payable in the manner hereinbelow
specified, the VENDORS do hereby sell, transfer and convey
unto the VENDEE, or their heirs, executors, administrators, or
assignors, that unsegregated portion of the above lot, x x x.
That the aforesaid amount shall be paid in two installments, the
first installment which is in the amount of __________
(P15,387.50) and the balance in the amount of __________
(P15,387.50), shall be paid as soon as the described portion of
the property shall have been registered under the Land
Registration Act and a Certificate of Title issued accordingly;
That as soon as the total amount of the property has been paid
and the Certificate of Title has been issued, an absolute deed of
sale shall be executed accordingly;
xxx
On August 12, 1991, Sabug, Jr, applied for a free patent over the
entire Lot 18089 and was eventually issued OCT No. M-59558 in
his name on October 21, 1991. On June 24, 1993, Sabug, Jr.
and Rivero, in her personal capacity and in representation of
Rivero, et al., executed the 1993 Joint Affidavit, acknowledging
that the subject portion belongs to Sps. Roque and expressed
their willingness to segregate the same from the entire area of
Lot 18089.
On December 8, 1999, however, Sabug, Jr., through the 1999
Deed of Absolute Sale, sold Lot 18089 to Aguado for
P2,500,000.00, who, in turn, caused the cancellation of OCT No.
M-5955 and the issuance of TCT No. M-96692 dated December
17, 199911 in her name.
Thereafter, Aguado obtained an P8,000,000.00 loan from the
Land Bank secured by a mortgage over Lot 18089. When she
failed to pay her loan obligation, Land Bank commenced extrajudicial foreclosure proceedings and eventually tendered the
highest bid in the auction sale. Upon Aguados failure to redeem
the subject property, Land Bank consolidated its ownership, and
TCT No. M-11589513 was issued in its name on July 21, 2003.
On June 16, 2003, Sps. Roque filed a complaint for
reconveyance, annulment of sale, deed of real estate mortgage,
foreclosure, and certificate of sale, and damages before the
RTC.
Aguado: innocent purchaser for value
Landbank: no knowledge of Sps. Claim. At the time when the
loan was taken out, Lot 18089 was registered in Aguados name
and no lien was annotated on COT.
RTC: dismissed complaint of spouses roque and NCCP.
CA: affirmed RTC ruling.
ISSUE: Whether or not the 1977 Deed of Conditional Sale is a
conditional contract of sale or a contract to sell.

HELD: It is a CONTRACT TO SELL. The Court held that where


the seller promises to execute a deed of absolute sale upon the
completion by the buyer of the payment of the purchase price,
the contract is only a contract to sell even if their agreement is
denominated as a Deed of Conditional Sale, as in this case. This
treatment stems from the legal characterization of a contract to
sell, that is, a bilateral contract whereby the prospective seller,
while expressly reserving the ownership of the subject property
despite delivery thereof to the prospective buyer, binds himself to
sell the subject property exclusively to the prospective buyer
upon fulfillment of the condition agreed upon, such as, the full
payment of the purchase price. Elsewise stated, in a contract to
sell, ownership is retained by the vendor and is not to pass to the
vendee until full payment of the purchase price.
In contracts to sell the obligation of the seller to sell becomes
demandable only upon the happening of the suspensive
condition, that is, the full payment of the purchase price by the
buyer. It is only upon the existence of the contract of sale that the
seller becomes obligated to transfer the ownership of the thing
sold to the buyer. Prior to the existence of the contract of sale,
the seller is not obligated to transfer the ownership to the buyer,
even if there is a contract to sell between them.
Final installment not paid thus no perfected contract of sale
Here, it is undisputed that Sps. Roque have not paid the final
installment of the purchase price. As such, the condition which
would have triggered the parties obligation to enter into and
thereby perfect a contract of sale in order to effectively transfer
the ownership of the subject portion from the sellers (i.e., Rivero
et al.) to the buyers (Sps. Roque) cannot be deemed to have
been fulfilled. Consequently, the latter cannot validly claim
ownership over the subject portion even if they had made an
initial payment and even took possession of the same.
Conditional contract of sale and contract to sell in relation
to double sale
It is essential to distinguish between a contract to sell and a
conditional contract of sale specially in cases where the subject
property is sold by the owner not to the party the seller
contracted with, but to a third person, as in the case at bench.
In a contract to sell, there being no previous sale of the
property, a third person buying such property despite the
fulfillment of the suspensive condition such as the full
payment of the purchase price, for instance, cannot be
deemed a buyer in bad faith and the prospective buyer
cannot seek the relief of reconveyance of the property.
There is no double sale in such case. Title to the property will
transfer to the buyer after registration because there is no defect
in the owner-sellers title per se, but the latter, of course, may
be sued for damages by the intending buyer.
On the matter of double sales, suffice it to state that Sps.
64
65
Roques reliance on Article 1544 of the Civil Code has been
misplaced since the contract they base their claim of ownership
on is, as earlier stated, a contract to sell, and not one of sale. In
66
Cheng v. Genato, the Court stated the circumstances which
must concur in order to determine the applicability of Article
1544, none of which are obtaining in this case, viz.:
(a) The two (or more) sales transactions in issue must pertain to
exactly the same subject matter, and must be valid sales
COMPILED BY: WIGMORE #WIGMOREFOREVER 108

SALES Case Digest (Atty. Sarona)


Compiled by: Wigmore #wigmoreforever
transactions;
(b) The two (or more) buyers at odds over the rightful ownership
of the subject matter must each represent conflicting interests;
and
(c) The two (or more) buyers at odds over the rightful ownership
of the subject matter must each have bought from the same
seller.

obtained, seized and impounded the car, but it was delivered


back to Jimenez upon his filing of a counter-bond.
The lower court held that Jimenez had the right of ownership and
possession over the car.
ISSUE: W/N Jimenez was a purchaser in good faith and thus
entitled to the ownership and possession of the car. YES

The action for reconveyance shall fail.

D. Obligations of Buyer

PART IV: DOCUMENTS OF TITLE

A.

Types of Documents of Title and its Warranties

B.

Delivery through Carrier

PART V: SALE BY NON-OWNER OR BY ONE HAVING


VOIDABLE TITLE

TAGATAC VS JIMENEZ
53 OG 3792
FACTS: Trinidad Tagatac bought a car for $4,500 in the US. After
7 months, she brought the car to the Philippines. Warner Feist,
who pretended to be a wealthy man, offered to buy Trinidads car
for P15,000, and Tagatac was amenable to the idea. Hnece, a
deed of sale was exceuted.
Feist paid by means of a postdated check, and the car was
delivered to Feist. However, PNB refused to honor the checks
and told her that Feist had no account in said bank.
Tagatac notified the law enforcement agencies of the estafa
committed by Feist, but the latter was not apprehended and the
car disappeared.
Meanwhile, Feist managed succeeded in having the cars
registration certificate (RC) transferred in his name. He sold the
car to Sanchez, who was able to transfer the registration
certificate to his name.
Sanchez then offered to sell the car to defendant Liberato
Jimenez, who bought the car for P10,000 after investigating in
the Motor Vehicles Office.
Tagatac discovered that the car was in California Car Exchanges
(place where Jimenez displayed the car for sale), so she
demanded from the manager for the delivery of the car, but the
latter refused.
Tagatac filed a suit for the recovery of the cars possession, and
the sheriff, pursuant to a warrant of seizure that Tagatac

HELD: It must be noted that Tagatac was not unlawfully deprived


of his car
In this case, there is a valid transmission of ownership from true
owner [Tagatac] to the swindler [Feist], considering that they had
a contract of sale (note: but such sale is voidable for the fraud
and deceit by Feist).
The disputable presumption that a person found in possession of
a thing taken in the doing of a recent wrongful act is the taker
and the doer of the whole act does NOT apply in this case
because the car was not stolen from Tagatac, and Jimenez came
into possession of the car two months after Feist swindled
Tagatac.
Jimenez was a purchaser in good faith for he was not aware of
any flaw invalidating the title from the seller of the car
In addition, when Jimenez acquired the car, he had no
knowledge of any flaw in the title of the person from whom he
acquired it. It was only later that he became fully aware that there
were some questions regarding the car, when he filed a petition
to dissolve Tagatacs search warrant which had as its subject the
car in question.
The contract between Feist and Tagactac was a voidable
contract, it can be annulled or ratified
. . . The fraud and deceit practiced by Warner L. Feist earmarks
this sale as a voidable contract (Article 1390). Being a voidable
contract, it is susceptible of either ratification or annulment. (If
the contract is ratified, the action to annul it is extinguished Article 1392) and the contract is cleansed from all its defects
(Article 1396); if the contract is annulled, the contracting parties
are restored to their respective situations before the contract and
mutual restitution follows as a consequence (Article 1398).
Being a voidable contract, it remains valid and binding until
annulled.
However, as long as no action is taken by the party entitled,
either that of annulment or of ratification, the contract of sale
remains valid and binding. When plaintiff-appellant Trinidad C.
Tagatac delivered the car to Feist by virtue of said voidable
contract of sale, the title to the car passed to Feist. Of course,
the title that Feist acquired was defective and voidable.
Nevertheless, at the time he sold the car to Felix Sanchez, his
title thereto had not been avoided and he therefore conferred a
good title on the latter, provided he bought the car in good faith,
for value and without notice of the defect in Feist's title (Article
1506, N.C.C.). There being no proof on record that Felix
Sanchez acted in bad faith, it is safe to assume that he acted in
good faith.
NB: ART. 1506. Where the seller of goods has a voidable title
thereto, but his title has not been avoided at the time of the sale,
the buyer acquires a good title to the goods provided he buys
COMPILED BY: WIGMORE #WIGMOREFOREVER 109

SALES Case Digest (Atty. Sarona)


Compiled by: Wigmore #wigmoreforever
them in good faith, for value, and without notice of the sellers
defect of title.
EDCA PUBLISHING VS SPS. SANTOS
G.R. No. 80298, April 26, 1990
FACTS: On October 5, 1981, a person identifying himself as
Prof. Jose Cruz ordered 406 books from EDCA Publishing.
EDCA Subsequently prepared the corresponding invoice and
delivered the books as ordered, for which Cruz issued a personal
check covering the purchase price of said books. Subsequently
on October 7, 1981, Cruz sold 120 of the books to Leonor
Santos who, after verifying the sellers ownership from the
invoice he showed her, paid him P1,700.
Upon verification by EDCA, it was discovered that Cruz was not
employed as professor by De La Salle College and that he had
no more account or deposit with Phil. Amanah Bank, the bank
where he allegedly drawn the payment check. Upon arrest of
Cruz by the police, it was revealed that his real name was Tomas
dela Pena and that there was a further sale of 120 books to Sps.
Santos.
EDCA, through the assistance of the police forced their way into
the store of Sps. Santos and threatened Leonor with prosecution
for buying stolen property. The 120 books were seized and were
later turned over to EDCA.
This resulted to Sps. Santos filing a case for recovery of the
books after their demand for the return of the books was
rejected.
ISSUE: W/N EDCA may retrieve the books from Santos. NO
(W/N EDCA has been unlawfully deprived of the books because
the heck issued by Cruz in payment thereof was dishonored.
NO.)
HELD: EDCA argues that because Cruz, the impostor acquired
no title to the books, the latter could not have validly transferred
such to Sps. Santos. Its reason is that as the payment check
bounced for lack of funds, there was a failure of consideration
that nullified the contract of sale between it and Cruz.
However, upon perusal of the provisions on the Law on Sales, a
contract of sale is consensual and is perfected once agreement
is reached between the parties on the subject matter and the
consideration. As provided in Art. 1478- Ownership in the thing
sold shall not pass to the buyer until full payment of the purchase
only if there is a stipulation to that effect. Otherwise, the rule is
that such ownership shall pass from the vendor to the vendee
upon the actual or constructive delivery of the thing sold even if
the purchase price has not yet been paid.
Non-payment only creates a right to demand payment or to
rescind the contract, or to criminal prosecution in the case of
bouncing checks. But absent the stipulation above noted,
delivery of the thing sold will effectively transfer ownership to the
buyer who can in turn transfer it to another.
Actual delivery of the books having been made, Cruz acquired
ownership over the books which he could then validly transfer to
the private respondents. The fact that he had not yet paid for
them to EDCA was a matter between him and EDCA and did not

impair the title acquired by the private respondents to the books.


Leonor Santos took care to ascertain first that the books
belonged to Cruz before she agreed to purchase them. The
EDCA invoice Cruz showed her assured her that the books had
been paid for on delivery. By contrast, EDCA was less than
cautious in fact, too trusting in dealing with the impostor.
Although it had never transacted with him before, it readily
delivered the books he had ordered (by telephone) and as
readily accepted his personal check in payment. It did not verify
his identity although it was easy enough to do this. It did not wait
to clear the check of this unknown drawer. Worse, it indicated in
the sales invoice issued to him, by the printed terms thereon, that
the books had been paid for on delivery, thereby vesting
ownership in the buyer.
Santos did not need to go beyond that invoice to satisfy herself
that the books being offered for sale by Cruz actually belonged to
him; yet she still did. Although the title of Cruz was presumed
under Article 559 by his mere possession of the books, these
being movable property, Leonor Santos nevertheless demanded
more proof before deciding to buy them.
NB: Law on Property
Art. 559. The possession of movable property acquired in good
faith is equivalent to a title. Nevertheless, one who has lost any
movable or has been unlawfully deprived thereof may recover it
from the person in possession of the same.
If the possessor of a movable lost or of which the owner has
been unlawfully deprived has acquired it in good faith at a public
sale, the owner cannot obtain its return without reimbursing the
price paid therefor.
AZNAR VS. YAPDIANGCO
13 SCRA 486
G.R. No. L-18536
March 31, 1965
Remember this case was discussed already before. Yung may
nagbenta ng kanyang car tapos inutusan nya ang son nya na
sumama dun sa gusting bumili then pumunta sila sa isang house
tapos nung paglabas ng anak wala na yung kotse and yung
pamangkin ng bibili.
FACTS: Teodoro Santos was selling his FORD FAIRLANE 500.
One day, a certain L. De Dios, claiming to be a nephew of
Vicente Marella, said that his uncle Vicente wants to buy the car.
Marella agreed to buy the car for P14,700.00 on the
understanding that the price would be paid only after the car had
been registered in his name.
The DOS for the car was executed in Marella's favor. Afterwhich,
the car in was registered Marella's name. Up to this stage of the
transaction, the purchased price had not been paid.
Teodoro Santos gave the registration papers and a copy of the
DOS to his son Irineo and instructed him not to part with them
until Marella has given the full payment for the car. Irineo Santos
and L. De Dios then proceeded to meet Marella and Ireneo
COMPILED BY: WIGMORE #WIGMOREFOREVER 110

SALES Case Digest (Atty. Sarona)


Compiled by: Wigmore #wigmoreforever
demanded the payment. Marella said that the amount he had on
hand then was short by some P2,000.00 and begged off to be
allowed to secure the shortage from a sister supposedly living
somewhere on Azcarraga Street, also in Manila. Thereafter, he
ordered L. De Dios to go to the said sister and suggested that
Irineo go with him. At the same time, he requested the
registration papers and the DOS from Irineo on the pretext that
he would like to show them to his lawyer. Trusting the good faith
of Marella, Irineo handed over the same to the latter and
thereupon, in the company of L. De Dios and another
unidentified person, proceeded to the alleged house of Marella's
sister. It was there that the car, L. De Dios and the unidentified
companion disappeared. That very same day, Marella was able
to sell the car in question Jose B. Aznar, for P15,000.00.

of it, he has a right to recover it, not only from the finder,
thief or robber, but also from third persons who may have
acquired it in good faith from such finder, thief or robber.
The said article establishes two exceptions to the general rule
of IRREVINDICABILITY: when the owner (1) has lost the thing,
or (2) has been unlawfully deprived thereof. In these cases, the
possessor cannot retain the thing as against the owner, who may
recover it without paying any indemnity, except when the
possessor acquired it in a public sale.

Aznar claims ownership over the vehicle. Trial court awarded the
vehicle to Santos.

CHRYSLER VS. CA
133 SCRA 567
G.R. No. 55684
December 19, 1984

ISSUE: Between Santos and Aznar, who has a better right to the
possession of the disputed automobile? - SANTOS
HELD: Aznar accepts that the car in question originally belonged
to and was owned by Santos, and that the latter was unlawfully
deprived of the same by Marella. However, Aznar contends that
upon the facts of this case, the applicable provision of the CC is
Article 1506 and not Article 559 as was held by the decision
under review. Article 1506 provides:
ART. 1506. Where the seller of goods has a voidable title
thereto, but his, title has not been voided at the time of the
sale, the buyer acquires a good title to the goods, provided
he buys them in good faith, for value, and without notice of
the seller's defect of title.
The contention is clearly unmeritorious. Under the aforequoted
provision, it is essential that the seller should have a
voidable title at least. It is very clearly inapplicable where, as in
this case, the seller had no title at all.
Ownership is not transferred by contract merely but by tradition
or delivery. Contracts only constitute titles or rights to the
transfer or acquisition of ownership, while delivery or tradition
is the mode of accomplishing the same. For the legal acquisition
and transfer of ownership and other property rights, the thing
transferred must be delivered, inasmuch as, according to settled
jurisprudence, the tradition of the thing is a necessary and
indispensable requisite in the acquisition of said ownership by
virtue of contract. So long as property is not delivered, the
ownership over it is not transferred by contract merely but by
delivery. Contracts only constitute titles or rights to the transfer or
acquisition of ownership, while delivery or tradition is the method
of accomplishing the same, the title and the method of acquiring
it being different in our law.
Vicente Marella did not have any title to the property under
litigation because the same was never delivered to him. He
sought ownership or acquisition of it by virtue of the contract.
Vicente Marella could have acquired ownership or title to the
subject matter thereof only by the delivery or tradition of the car
to him.
The lower court was correct in applying Article 559 of the CC to
the case at bar, for under it, the rule is to the effect that if the
owner has lost a thing, or if he has been unlawfully deprived

PART VI: LOSS, DETERIORATION, FRUITS AND OTHER


BENEFITS

FACTS: Petitioner Chrysler is a domestic corporation engaged in


the assembling and sale of motor vehicles and other automotive
products. Respondent Sambok is a general partnership and was
a dealer for automotive products.
Chrysler filed with CFI a complaint for damages against Allied
Brokerage Corp, Negros Navigation Co, and Sambok, alleging
that:
1. On Oct 2, 1970, Sambok Bacolod ordered from petitioner
various automotive products worth P30,909.61 payable in 45
days,
2. That on Nov 25, 1970, Chrysler delivered said products to its
forwarding agent Allied Brokerage for delivery to Sambok
(Allied loaded the goods through vessel of Negros Nav),
3. That when Chrysler tried to collect from Sambok Bacolod
the amount of P31,037.56 (price of spare parts plus handling
charges), Sambok refused to pay, claiming that it had not
received the merchandise,
4. That Chrysler also demanded the return of the merchandise
or their value from Allied and Negros Navigation, but both
denied any liability.
Sambok Bacolod denied having received the automotive
products and professed no knowledge of having ordered from
petitioner the said articles.
Trial Court dismissed the complaint against Allied and Negros
Navigation, but found Sambok liable for damages in refusing to
take delivery of the shipment for no justifiable reason. The
decision was reversed by CA after finding that Chrysler had not
performed its part of the obligation under the contract by not
delivering the goods at Sambok, Iloilo, the place designated in
the Parts Order Form. In other words, CA found that there was
misdelivery.
ISSUE: W/N Sambok Bacolod should be liable for damages. NO
HELD: To our minds, the matter of misdelivery is not the decisive
factor for relieving Sambok, Bacolod, of liability herein. While it
may be that the Parts Order Form specifically indicated Iloilo as
the destination, as testified to by Ernesto Ordonez, Parts Sales
Representative of petitioner, Sambok, Bacolod, and Sambok,
Iloilo, are actually one. In fact, admittedly, the order for spare
COMPILED BY: WIGMORE #WIGMOREFOREVER

111

SALES Case Digest (Atty. Sarona)


Compiled by: Wigmore #wigmoreforever
parts was made by the President of Sambok, Pepito Ng,
through its marketing consultant.

token of goodwill it sent to Tabora free of charge volumes 75, 76,


77 and 78 of the Philippine Reports.

Notwithstanding, upon receipt of the Bill of Lading, Sambok,


Bacolod, initiated, but did not pursue, steps to take delivery as
they were advised by Negros Navigation that because some
parts were missing. They would just be informed as soon as
the missing parts were located.

As Tabora failed to pay monthly installments, the company


commenced the present action for the recovery of the balance of
the obligation.

It was only four years later however, or in 1974, when a


warehouseman of Negros Navigation, Severino Aguarte, found in
their off-shore bodega, parts of the shipment in question, but
already deteriorated and valueless.
Under the circumstances, Sambok, Bacolod, cannot be faulted
for not accepting or refusing to accept the shipment from
Negros Navigation four years after shipment. The evidence is
clear that Negros Navigation could not produce the merchandise
nor ascertain its whereabouts at the time Sambok, Bacolod, was
ready to take delivery. Where the seller delivers to the buyer a
quantity of goods less than he contracted to sell, the buyer may
reject them.
From the evidentiary record, Negros Navigation was the party
negligent in failing to deliver the complete shipment either to
Sambok, Bacolod, or to Sambok, Iloilo, but as the Trial Court
found, petitioner failed to comply with the conditions precedent to
the filing of a judicial action. Thus, in the last analysis, it is
petitioner that must shoulder the resulting loss. The general
rule that before, delivery, the risk of loss is home by the
seller who is still the owner, under the principle of "res perit
domino", is applicable in petitioner's case.
In sum, the judgment of respondent Appellate Court, will have to
be sustained not on the basis of misdelivery but on non-delivery
since the merchandise was never placed in the control and
possession of Sambok, Bacolod, the vendee. Decision of CA
affirmed.
LAWYERS COOP VS TABORA
FACTS: Perfecto Tabora bought from the Lawyers Cooperative
Publishing Company one complete set of American
Jurisprudence consisting of 48 volumes with 1954 pocket parts,
plus one set of American Jurisprudence, General Index,
consisting of 4 volumes, for a total price of P1,675.50 which, in
addition to the cost of freight of P6.90, makes a total of
P1,682.40.
Tabora made a partial payment of P300.00, leaving a balance of
P1,382.40. The books were delivered and receipted by Tabora in
his law office.

It was provided in the contract that "title to and ownership of the


books shall remain with the seller until the purchase price shall
have been fully paid. Loss or damage to the books after delivery
to the buyer shall be borne by the buyer." The total price of the
books, including the cost of freight, amounts to P1,682.40.
Appellant only made a down payment of P300.00 thereby leaving
a balance of P1,382.40.
Defendant, in his answer, pleaded force majeure as a defense,
saying that since the loss was due to force majeure he cannot be
held responsible for the loss.
ISSUE: WON Tabora is absolved from liability on the ground of
force majeure
RULING: No. He is liable for the loss. It is true that in the
contract entered into between the parties the seller agreed that
the ownership of the books shall remain with it until the purchase
price shall have been fully paid, but such stipulation cannot make
the seller liable in case of loss not only because such was
agreed merely to secure the performance by the buyer of his
obligation but in the very contract it was expressly agreed
that the "loss or damage to the books after delivery to the
buyer shall be borne by the buyer."
Article 1504 of our Civil Code, which in part provides:
(1) Where delivery of the goods has been made to the buyer
or to a bailee for the buyer, in pursuance of the contract and
the ownership in the goods has been retained by the seller
merely to secure performance by the buyer of his
obligations under the contract, the goods are at the buyer's
risk from the time of such delivery.
The rule that an obligor should be held exempt from liability when
the loss occurs thru a fortuitous event should not apply because
it only holds true when the obligation consists in the delivery of a
determinate thing and there is no stipulation holding him liable
even in case of fortuitous event. Here these qualifications are not
present. The obligation does not refer to a determinate thing, but
is pecuniary in nature, and the obligor bound himself to assume
the loss after the delivery of the goods to him. In other words, the
obligor agreed to assume any risk concerning the goods from the
time of their delivery, which is an exception to the rule provided
for in Article 1262 of our Civil Code.
PART ViI: REMEDIES IN CASE OF BREACH

In the midnight of the same date, however, a big fire broke out in
that locality which destroyed and burned all the buildings
standing on one whole block including at the law office and
library of Tabora As a result, the books bought from the company
as above stated, together with Tabora's important documents
and papers, were burned during the conflagration.
This unfortunate event was immediately reported by Tabora to
the company by sending a letter. The company replied and as a

A. Ordinary Remedies of Seller

B. Special Remedies of Unpaid Seller of Goods

COMPILED BY: WIGMORE #WIGMOREFOREVER

112

SALES Case Digest (Atty. Sarona)


Compiled by: Wigmore #wigmoreforever
1) Possessory Lien

2) Stoppage in Transitu

3) Special Right of Resale

4) Special Right to Rescind

however, one on installments, but on straight term, in which the


balance, after payment of the initial sum, should be paid in its
totality at the time specified in the promissory note. The
transaction is not, therefore, the one contemplated in Act No.
4122 and accordingly the mortgagee is not bound by the
prohibition therein contained as to the right to the recovery of the
unpaid balance.
The suggestion that the cash payment made in this case should
be considered as an installment in order to bring the contract
sued upon under the operation of the law, is completely
untenable. A cash payment cannot be considered as a payment
by installment, and even if it can be so considered, still the law
does not apply, for it requires non-payment of two or more
installments in order that its provisions may be invoked. Here,
only one installment was unpaid.

C. Remedies of Buyer
DELTA MOTOR SALES CORP. v NIU KIM DUAN
D. Sale of Movable on Installments Article 1484 (Recto La)
(also 1485, 1486)

LEVY VS. GERVACIO


FACTS: Levy Hermanos, Inc., sold to defendant Lazaro Blas
Gervacio, a Packard car. Defendant, after making the initial
payment, executed a promissory note for the balance of P2,400,
payable on or before June 15, 1937, with interest at 12 % per
annum, to secure the payment of the note, he mortgaged the car
to the plaintiff.

FACTS: Niu Kim Duan purchased from Delta Motors 3 air


conditioning units. Niu paid the down payment, the balance
payable in 24 instalments. Title to the property remained with
Delta until the payment of the full purchase price.
Under the agreement, failure to pay 2monthly instalments makes
the obligation entirely due and demandable. The units were
delivered, Niu failed to pay. Thus, Delta filed a complaint for
Replevin and applied the installments paid by Niu as rentals.
Niu contends
unconscionable.

that

the

contractual

stipulations

are

ISSUE: W/N the remedy Delta availed of was unconscionable


Defendant failed to pay the note it its maturity. Thus, Levy
foreclosed the mortgage and the car was sold at public auction,
at which plaintiff was the highest bidder for P1,800. The present
action is for the collection of the balance of P1,600 and interest.
ISSUE: WON plaintiff still may collect the balance and interest
after it has already foreclosed the mortgage and sold it at public
auction
RULING: Yes it can still collect the balance.
Article 1454-A of the Civil Code reads as follows:
In a contract for the sale of personal property payable in
installments shall confer upon the vendor the right to cancel
the sale or foreclose the mortgage if one has been given on
the property, without reimbursement to the purchaser of the
installments already paid, if there be an agreement to this
effect.
However, if the vendor has chosen to foreclose the
mortgage he shall have no further action against the
purchaser for the recovery of any unpaid balance owing by
the same and any agreement to the contrary shall be null
and void.
In Macondray and Co. vs. De Santos, the Court held that "in
order to apply the provisions of article 1454-A of the Civil
Code it must appear that there was a contract for the sale of
personal property payable in installments and that there has
been a failure to pay two or more installments." The contract,
in the instant case, while a sale of personal property, is not,

HELD: NO. A stipulation in the contract treating installments as


rentals in case of failure to pay is VALID so long as they are
not unconscionable. The provision in this case is reasonable.
An unpaid seller has 3 alternative (not cumulative) remedies:
(1) to exact fulfilment of the obligation;
(2) to cancel the sale for default in 2 installments; and
(3) to foreclose the chattel mortgage.
If the creditor chooses one remedy, he cannot avail himself of
the other two.
It is not disputed that the plaintiff-appellee had taken possession
of the three air-conditioners, through a writ of replevin when
defendants-appellants refused to extra-judicially surrender the
same. This was done pursuant to paragraphs 5 and 7 of its Deed
of Conditional Sale when defendants-appellants failed to pay at
least two (2) monthly installments, so much so that as of January
6, 1977, the total amount they owed plaintiff-appellee, inclusive
of interest, was P12,920.08. 12 The case plaintiff-appellee filed
was to seek a judicial declaration that it had validly rescinded the
Deed of Conditional Sale.
Clearly, plaintiff-appellee chose the second remedy of Article
1484 in seeking enforcement of its contract with defendantsappellants. This is shown from the fact that the computation of
the outstanding account of defendants-appellants as of October
3, 1977 took into account "the value of the units repossessed."
Having done so, it is barred from exacting payment from
COMPILED BY: WIGMORE #WIGMOREFOREVER 113

SALES Case Digest (Atty. Sarona)


Compiled by: Wigmore #wigmoreforever
defendants-appellants of the balance of the price of the three airconditioning units which it had already repossessed.

owner of the car and sentencing Elisco to pay for actual


damages caused to the private respondents, thus this petition.

TAJANLANGIT v SOUTHERN MOTORS

ISSUE: WON the lease with option to buy is in reality an


installment sale so as to apply the Recto Law under Art. 1484.

FACTS: Tajanlangit bought 2 tractors and a thresher from


Southern Motors. They executed a promissory note in payment
thereof; it contained an acceleration clause. Tajanlang it failed to
pay any of the stipulated installments. Thus, Southern Motors
sued him on the PN. The sheriff levied upon the properties of
Tajanlangit (same machineries) and sold them at a public auction
to satisfy the debt. Southern Motors now prayed for execution.
Tajanlangit sought to annul the writ of execution claiming that
since Southern Motors repossessed the machineries
(mortgaged), he was therefore relieved from liability on the
balance of the purchase price.
ISSUE: W/N Tajanlangit is relieved from his obligation topay
HELD: NO. While it is true that the foreclosure on the chattel
mortgage on the thing sold bars further action for the recovery of
the balance of the purchase price, this does not apply in this
case since Southern did not foreclose on the mortgage but
instead sued based on the PNs exclusively. That being the case,
it is not limited to the proceeds of the sale on execution of the
mortgaged goods and may claim the balance from Tajanlangit.

ELISCO TOOL MANUFACTURING vs. COURT OF APPEALS,


ROLANDO AND RINA LANTAN
G.R. No. 109966
May 31, 1999
FACTS: Private respondent Rolando Lantan was employed at
the Elisco Tool Mfg. Corp. On Jan. 1980, he entered into a car
plan with the company, which constitutes a lease with option to
buy for a period of 5 years. The agreement provides that Lantan
shall pay a monthly rental of P 1010.65 to be deducted from his
salary or a total of P60, 639.00 at the end of 5 years. The
agreement provides that at the 60th month of payment he may
exercise his option to buy and all monthly rentals shall be applied
to the payment of the full purchase price of the car. In 1981
Elisco Tool ceased operations, and Rolando Lantan was laid off.
Nonetheless, as of December 4, 1984, private respondent was
able to make payments for the car in the total amount of P61,
070.94.
On 1986 Elisco filed a complaint for replevin plus sum of money
against Rolando Lantan for the latters alleged failure to pay the
monthly rentals as of May 1986. Elisco prayed for the following:
1. The payment of Lantan of the sum of the monthly rentals due
as of May 1986 plus legal interest;
2. The issuance of writ of replevin to gain possession of the car;
and
3. On the alternative, should the delivery of the car not be
possible, that Lantan be ordered to pay the actual value of the
car in the amount of 60,000 plus the accrued monthly rentals
thereof with interest until fully paid. Both the trial court and the
CA decided in favor of Lantan, declaring the latter the lawful

WON Elisco is entitled to any of the remedies under Art. 1484.


HELD:
1. Yes.
The agreement between Elisco and the Lantans is in reality an
installment sale of personal property. However, the remedies
under Article 1484 are alternative, not cumulative.
2. No.
There was already full payment. In the case at bar, although the
agreement provides for the payment of monthly rentals, it also
provides the option to purchase upon the payment of the 60th
monthly rental and that all monthly rentals shall be applied to the
payment of the full purchase price of the car. Clearly the
transaction is a lease in name only and so Articles 1484 and
1485 apply.
It is noteworthy that the remedies provided for in Art. 1484 are
alternative, not cumulative. The exercise of one bars the exercise
of the others. It was held that in choosing to deprive the
defendant of possession of the leased vehicles, the plaintiff
waived its right to bring an action to recover unpaid rentals on
the said vehicles.
Furthermore, both the trial court and the CA correctly ruled that
Elisco is not entitled to any of the remedies under Art. 1484 as
there has already been full payment.
The agreement does not provide for the payment of interest on
unpaid monthly "rentals" or installments. The 2% surcharge is not
provided for in the agreement. Consequently, the total amount of
P 61, 070.94 already paid is more than sufficient to cover the full
purchase price of the car which only amounts to P 60, 639.

E. Remedies in case of Immovables

F. Sale of Real Estate on Installments RA 6552 (Maceda


Law)
ABELARDO VALARAO, GLORIOSA VALARAO, CARLOS
VALARAO vs. COURT OF APPEALS AND MEDEN
ARELLANO
G.R. No. 130347
March 3, 1999
FACTS: Spouses Valarao, thru their son, Carlos, sold to Arellano
a parcel of land situated in Diliman, Quezon City for the sum of
3.225 M embodied under a Deed of Conditional Sale.
It was further stipulated upon that should Arellano fail to pay
three (3) successive monthly installments or any one year-end
COMPILED BY: WIGMORE #WIGMOREFOREVER

114

SALES Case Digest (Atty. Sarona)


Compiled by: Wigmore #wigmoreforever
lump sum payment within the period stipulated, the sale shall be
considered automatically rescinded without the necessity of
judicial action and all payments made by Arellano shall be
forfeited in favor of the spouses by way of rental for the use and
occupancy of the property and as liquidated damages. All
improvements introduced by Arellano to the property shall belong
to the spouses without any right of reimbursement.
Arellano alleged that as of September 1990 he was already able
to pay the sum of 2.028 M although she admitted that she failed
to pay for the installments due in October and November 1990.
Arellano tried to pay but was turned down by the spouses thru
their maid. Arellano avers that the same maid was the on who
received payments tendered by her. It appears that the maid
refused to receive the payment allegedly on orders of her
employees who were not at home. This prompted Arellano to
seek the help of barangay officials. Efforts to settle before the
barangay was unavailing, as the spouses never appeared in
meetings.
Arellano sought judicial action by filing a petition for consignation
on January 4, 1991.
Spouses Valarao, thru counsel, sent Arellano a letter dated 4
January 1991 notifying her that they were enforcing the provision
on automatic rescission as a consequence of which the Deed of
Conditional Sale was deemed null and void, and xxx all
payments made, as well as the improvements introduced on the
property, were thereby forfeited. The letter also made a formal
demand on Arellano to vacate the property should she not heed
the demand of the spouses to sign a contract of lease for her
continued stay in the property.
The RTC ruled against Arellano but the Court of Appeals
reversed the decision of the trial court hence this petition.
ISSUE:
1. WON the automatic forfeiture clause is enforceable.
2. WON RA 6552 is applicable.
HELD:
1. Yes. As a general rule, a contract is the law between the
parties. Thus, "from the moment the contract is perfected, the
parties are bound not only to the fulfillment of what has been
expressly stipulated but also to all consequences which,
according to their nature, may be in keeping with good faith,
usage and law." Also, "the stipulations of the contract being the
law between the parties, courts have no alternative but to
enforce them as they were agreed [upon] and written, there
being no law or public policy against the stipulated forfeiture of
payments already made." However, it must be shown that
Arellano failed to perform her obligation, thereby giving spouses
the right to demand the enforcement of the contract.
We concede the validity of the automatic forfeiture clause, which
deems any previous payments forfeited and the contract
automatically rescinded upon the failure of the vendee to pay
three successive monthly installments or any one-yearend lump
sum payment. However, the spouses failed to prove the
conditions that would warrant the implementation of this clause.
Based on the facts of the case, the spouses were not justified in
refusing the tender of payment made by Arellano. Had the

spouses accepted the payment, she would have paid all three
monthly installments. In other words, there was no deliberate
failure on Arellanos part to meet her responsibility to pay.
2. Yes. Sec. 3, RA 6552 provides:
Sec. 3. In all transactions or contracts involving the sale or
financing of real estate on installment payments, including
residential condominium apartments but excluding
industrial lots, commercial buildings and sales to tenants
under Republic Act. Numbered Thirty-eight hundred Forty-four as
amended by Republic Act Numbered Sixty-three hundred eightynine, where the buyer has paid at least two years of
installments, the buyer is entitled to the following rights in case
he defaults in the payment of succeeding installments:
(a) To pay, without additional interest, the unpaid
installments due within the total grace period earned by him,
which is hereby fixed at the rate of one month grace period
for every year of installment payments made: Provided, That
this right shall be exercised by the buyer only once in every five
years of the life of the contract and its extensions, if any.
(b) If the contract is cancelled, the seller shall refund to the
buyer the cash surrender value on the payments on the
property equivalent to fifty percent of the total payments
made and, after five years of installments, an additional five
percent every year but not to exceed ninety percent of the
total payments made: Provided, That the actual cancellation of
the contract shall take place after thirty days from receipt by the
buyer of the notice of cancellation or the demand for rescission
of the contract by a notarial act and upon full payment of the
cash surrender value to the buyer.
Down payments, deposits or options on the contract shall be
included in the computation of the total number of installments
made.
Therefore, Arellano is entitled to a one-month grace period for
every year of installment paid, which means that she had a total
grace period of three months from December 31, 1990. Indeed,
to rule in favor of the spouses would result in patent injustice and
unjust enrichment.
PAGTALUNAN vs. VDA. DE MANZANO
G.R. No. 147695 September 13, 2007
FACTS: On July 19, 1974, Patricio Pagtalunan (Patricio),
petitioners stepfather and predecessor-in-interest, entered into a
Contract to Sell with respondent Rufina dela Cruz Vda. De
Manzano, whereby the former agreed to sell, and the latter to
buy, a house and lot which formed half of a parcel of land for a
consideration of P17,800. The parties agreed that it shall be paid
in the following manner: P1,500 as downpayment upon
execution of the Contract to Sell, and the balance to be paid in
equal monthly installments of P150 on or before the last day of
each month until fully paid.
It was also stipulated in the contract that respondent could
immediately occupy the house and lot; that in case of default in
the payment of any of the installments for 90 days after its due
date, the contract would be automatically rescinded without need
of judicial declaration, and that all payments made and all
COMPILED BY: WIGMORE #WIGMOREFOREVER 115

SALES Case Digest (Atty. Sarona)


Compiled by: Wigmore #wigmoreforever
improvements done on the premises by respondent would be
considered as rentals for the use and occupation of the property
or payment for damages suffered, and respondent was obliged
to peacefully vacate the premises and deliver the possession
thereof to the vendor.
Petitioner claimed that respondent paid only P12,950. She
allegedly stopped paying after December 1979 without any
justification or explanation. Petitioner asserted that when
respondent ceased paying her installments, her status of buyer
was automatically transformed to that of a lessee. Therefore, she
continued to possess the property by mere tolerance of Patricio
and, subsequently, of petitioner.
Respondent did not deny that she still owed Patricio P5,650, but
claimed that she did not resume paying her monthly installment
because of the unlawful acts committed by Patricio, as well as
the filing of the ejectment case against her.
Patricio and his wife died on September 17, 1992 and on
October 17, 1994, respectively. Petitioner became their sole
successor-in-interest pursuant to a waiver by the other heirs. On
March 5, 1997, respondent received a letter from petitioners
counsel dated February 24, 1997 demanding that she vacate the
premises within five days on the ground that her possession had
become unlawful. Respondent ignored the demand. The Punong
Barangayfailed to settle the dispute amicably.
On April 8, 1997, petitioner filed a Complaint for unlawful
detainer against respondent.
ISSUE: Whether or not the cancellation of the contract complied
with what is required under the Maceda law.
HELD: No. R.A. No. 6552, otherwise known as the "Realty
Installment Buyer Protection Act," recognizes in conditional sales
of all kinds of real estate (industrial, commercial, residential) the
right of the seller to cancel the contract upon non-payment of an
installment by the buyer, which is simply an event that prevents
the obligation of the vendor to convey title from acquiring binding
force. The Court agrees with petitioner that the cancellation of the
Contract to Sell may be done outside the court particularly when
the buyer agrees to such cancellation.
However, the cancellation of the contract by the seller must be in
accordance with Sec. 3 (b) of R.A. No. 6552, which requires a
notarial act of rescission and the refund to the buyer of the full
payment of the cash surrender value of the payments on the
property. Actual cancellation of the contract takes place after 30
days from receipt by the buyer of the notice of cancellation or the
demand for rescission of the contract by a notarial act and upon
full payment of the cash surrender value to the buyer.
Based on the records of the case, the Contract to Sell was not
validly cancelled or rescinded under Sec. 3 (b) of R.A. No. 6552.
First, Patricio, the vendor in the Contract to Sell, died on
September 17, 1992 without canceling the Contract to Sell.
Second, petitioner also failed to cancel the Contract to Sell in
accordance with law.

Petitioner contends that that his demand letter dated February


24, 1997 should be considered as the notice of cancellation or
demand for rescission by notarial act.
The Court, however, finds that the letter dated February 24,
1997, which was written by petitioners counsel, merely made
formal demand upon respondent to vacate the premises in
question.
Clearly, the demand letter is not the same as the notice of
cancellation or demand for rescission by a notarial act required
by R.A No. 6552. Petitioner cannot rely on Layug v. Intermediate
Appellate Court to support his contention that the demand letter
was sufficient compliance since the seller therein filed an action
for annulment of contract, which is a kindred concept of
rescission by notarial act. Evidently, the case of unlawful detainer
filed by petitioner does not exempt him from complying with the
said requirement.
In addition, Sec. 3 (b) of R.A. No. 6552 requires refund of the
cash surrender value of the payments on the property to the
buyer before cancellation of the contract. The provision does not
provide a different requirement for contracts to sell which allow
possession of the property by the buyer upon execution of the
contract like the instant case. Hence, petitioner cannot insist on
compliance with the requirement by assuming that the cash
surrender value payable to the buyer had been applied to rentals
of the property after respondent failed to pay the installments
due.
There being no valid cancellation of the Contract to Sell, the CA
correctly recognized respondents right to continue occupying the
property subject of the Contract to Sell and affirmed the
dismissal of the unlawful detainer case by the RTC.
SC DISPOSITION: Considering that the Contract to Sell was not
cancelled by the vendor, the Court agrees with the CA that it is
only right and just to allow respondent to pay her arrears and
settle the balance of the purchase price.

SPOUSES GARCIA, SPOUSES vs. COURT OF APPEALS


G.R. No. 172036 April 23, 2010
FACTS: On May 28, 1993, plaintiffs spouses Faustino and
Josefina Garcia and spouses Meliton and Helen Galvez (herein
appellees) and defendant Emerlita dela Cruz (herein appellant)
entered into a Contract to Sell wherein the latter agreed to sell to
the former, for P3,170,220.00, five (5) parcels of land. At the time
of the execution of the said contract, three of the subject lots,
were registered in the name of one Angel Abelida from whom
defendant allegedly acquired said properties by virtue of a Deed
of Absolute Sale dated March 31, 1989.
As agreed upon, plaintiffs shall make a down payment of
P500,000.00 upon signing of the contract. The balance of
P2,670,220.00 shall be paid in three installments.
On its due date, December 31, 1993, plaintiffs failed to pay the
last installment in the amount of One P1,670,220.00. Sometime
in July 1995, plaintiffs offered to pay the unpaid balance, which
had already been delayed by one and a half year, which
COMPILED BY: WIGMORE #WIGMOREFOREVER 116

SALES Case Digest (Atty. Sarona)


Compiled by: Wigmore #wigmoreforever
defendant refused to accept. On September 23, 1995, defendant
sold the same parcels of land to intervenor Diogenes G.
Bartolome for P7,793,000.00.
Plaintiffs filed before the RTC a complaint for specific
performance to compel defendant to accept plaintiffs payment
and, thereafter, execute the necessary document of transfer.
In their complaint, plaintiffs alleged that they discovered the
infirmity of the Deed of Absolute Sale covering those 3 lots,
between their former owner Angel Abelida and defendant, the
same being spurious because the signature of Angel Abelida and
his wife were falsified. Due to their apprehension regarding the
authenticity of the document, they withheld payment of the last
installment. They tendered payment of the unpaid balance
sometime in July 1995, after Angel Abelida ratified the sale made
in favor of defendant, but defendant refused to accept their
payment for no jusitifiable reason.
In her answer, defendant denied the allegation that the Deed of
Absolute Sale was spurious and argued that plaintiffs failed to
pay in full the agreed purchase price on its due date despite
repeated demands; that the Contract to Sell contains a proviso
that failure of plaintiffs to pay the purchase price in full shall
cause the rescission of the contract and forfeiture of 1/2 of the
total amount paid to defendant; that a notarized letter stating the
indended rescission of the contract to sell and forfeiture of
payments was sent to plaintiffs at their last known address but it
was returned with a notation "insufficient address."
ISSUE: Whether or not Maceda Law is applicable in this case.
HELD: No. Not applicable. It is clear from the above-quoted
provisions that the parties intended their agreement to be a
Contract to Sell: Dela Cruz retains ownership of the subject lands
and does not have the obligation to execute a Deed of Absolute
Sale until petitioners payment of the full purchase price.
The Maceda Law applies to contracts of sale of real estate on
installment payments, including residential condominium
apartments but excluding industrial lots, commercial buildings
and sales to tenants. The subject lands, comprising five (5)
parcels and aggregating 69,028 square meters, do not comprise
residential real estate within the contemplation of the Maceda
Law. Moreover, even if we apply the Maceda Law to the present
case, petitioners offer of payment to Dela Cruz was made a year
and a half after the stipulated date. This is beyond the sixty-day
grace period under Section 4 of the Maceda Law. Petitioners still
cannot use the second sentence of Section 4 of the Maceda Law
against Dela Cruz for Dela Cruzs alleged failure to give an
effective notice of cancellation or demand for rescission because
Dela Cruz merely sent the notice to the address supplied by
petitioners in the Contract to Sell.
The applicable provision of law in instant case is Article 1191 of
the New Civil Code which provides as follows:
Art. 1191. The power to rescind obligations is implied in
reciprocal ones, in case one of the obligors should not comply
with what is incumbent upon him.
The injured party may choose between the fulfillment and the
rescission of the obligation, with the payment of damages in

either case. He may also seek rescission, even after he has


chosen fulfillment, if the latter should become impossible.
The Court shall decree the rescission claimed, unless there be
just cause authorizing the fixing of a period.

G.R. NO. 195619, SEPTEMBER 5, 2012


PLANTERS DEVELOPMENT BANK, VS JULIE CHANDUMAL
FACTS: BF Homes and Julie Chandumal entered into a contract
to sell a parcel of land located in Las Pinas. Later, BF Homes
sold to PDB all its rights over the contract.
Chandumal paid her monthly amortizations until she defaulted in
her payments. So, PDB sent a notice to Chandumal with a
demand to vacate the land within 30days, otherwise all of her
rights will be extinguished and the contract will be terminated and
deemed rescinded. In spite of the demand, Chandumal failed to
settle her account.
PDB filed an action for judicial confirmation of notarial rescission
and delivery of possession but still Chandumal refused to do so.
Summons were then issued and served by deputy sheriff Galing
but its was unavailing as she was always out of her house on the
dates the summons were served.
RTC then issued an order granting the motion of PDB.
Chandumal filed an urgent motion to set aside order of default
and to admit attached answer. Chandumal said that she did not
receive the summons and was not notified of the same and her
failure to file an answer within the reglementary period was due
to fraud. RTC denied Chandumals motion to set aside the order
of default.
Chandumal appealed to the CA. CA nullified the RTCs decision.
ISSUE:
(1) Whether there was valid substituted service of summons?
(2) Whether Chandumal voluntarily submitted to the jurisdiction
of the RTC?
(3) Whether there was proper rescission by notarial act of the
contract to sell?
HELD:
(1) Correctly ruled that the sheriffs return failed to justify a resort
to substituted service of summons. According to the CA, the
Return of Summons does not specifically show or indicate in
detail the actual exertion of efforts or any positive step taken by
the officer or process server in attempting to serve the summons
personally to the defendant.
(2) The Court notes that aside from the allegation that she did
not receive any summons, Chandumals motion to set aside
order of default and to admit attached answer failed to positively
assert the trial court lack of jurisdiction. In fact, what was set forth
therein was the substantial claim that PDB failed to comply with
the requirements of R.A. No. 6552 on payment of cash surrender
value, which already delves into the merits of PDBs cause of
action. In addition, Chandumal even appealed the RTC decision
to the CA, an act which demonstrates her recognition of the trial
courts jurisdiction to render said judgment.

COMPILED BY: WIGMORE #WIGMOREFOREVER

117

SALES Case Digest (Atty. Sarona)


Compiled by: Wigmore #wigmoreforever
(3) R.A. No. 6552 recognizes the right of the seller to cancel the
contract but any such cancellation must be done in conformity
with the requirements therein prescribed. In addition to the
notarial act of rescission, the seller is required to refund to the
buyer the cash surrender value of the payments on the property.
The actual cancellation of the contract can only be deemed to
take place upon the expiry of a thirty (30)-day period following
the receipt by the buyer of the notice of cancellation or demand
for rescission by a notarial act and the full payment of the cash
surrender value.
Petition is denied.
OPTIMUM DEVELOPMENT BANK vs. SPOUSES BENIGNO V.
JOVELLANOS and LOURDES R. JOVELLANOS
G.R. No. 189145
December 4, 2013
FACTS: On April 26, 2005, Sps. Jovellanos entered into a
6
Contract to Sell with Palmera Homes, Inc. (Palmera Homes) for
the purchase of a residential house and lot situated in Block 3,
Lot 14, Villa Alegria Subdivision, Caloocan City (subject property)
for a total consideration of P1,015,000.00. Pursuant to the
contract, Sps. Jovellanos took possession of the subject property
upon a down payment of P91,500.00, undertaking to pay the
remaining balance of the contract price in equal monthly
installments of P13,107.00 for a period of 10 years starting June
12, 2005.
On August 22, 2006, Palmera Homes assigned all its rights, title
and interest in the Contract to Sell in favor of petitioner Optimum
Development Bank (Optimum) through a Deed of Assignment of
even date.
On April 10, 2006, Optimum issued a Notice of Delinquency and
Cancellation of Contract to Sell for Sps. Jovellanoss failure to
pay their monthly installments despite several written and verbal
notices.
In a final Demand Letter dated May 25, 2006, Optimum required
Sps. Jovellanos to vacate and deliver possession of the subject
property within seven (7) days which, however, remained
unheeded. Hence, Optimum filed, on November 3, 2006, a
complaint for unlawful detainer before the MeTC, docketed as
Civil Case No. 06-28830. Despite having been served with
summons, together with a copy of the complaint, Sps. Jovellanos
failed to file their answer within the prescribed reglementary
period, thus prompting Optimum to move for the rendition of
judgment.
Thereafter, Sps. Jovellanos filed their opposition with motion to
admit answer, questioning the jurisdiction of the court, among
others. Further, they filed a Motion to Reopen and Set the Case
for Preliminary Conference, which the MeTC denied.
ISSUE: w/n there was a valid and effective cancellation of the
Contract to Sell in accordance with Section 4 of RA 6552
RULING: YES. Verily, in a contract to sell, the prospective seller
binds himself to sell the property subject of the agreement
exclusively to the prospective buyer upon fulfillment of the
condition agreed upon which is the full payment of the purchase
price but reserving to himself the ownership of the subject
property despite delivery thereof to the prospective buyer.The full

payment of the purchase price in a contract to sell is a


suspensive condition, the non-fulfillment of which prevents the
prospective sellers obligation to convey title from becoming
effective, as in this case.
Further, it is significant to note that given that the Contract to Sell
in this case is one which has for its object real property to be sold
on an installment basis, the said contract is especially governed
by and thus, must be examined under the provisions of RA
6552, or the Realty Installment Buyer Protection Act, which
provides for the rights of the buyer in case of his default in the
payment of succeeding instalments.
Given the nature of the contract of the parties, the respondent
court correctly applied Republic Act No. 6552. Known as the
Maceda Law, R.A. No. 6552 recognizes in conditional sales of all
kinds of real estate (industrial, commercial, residential) the right
of the seller to cancel the contract upon non-payment of an
installment by the buyer, which is simply an event that prevents
the obligation of the vendor to convey title from acquiring binding
force. It also provides the right of the buyer on installments in
case he defaults in the payment of succeeding installments, viz.:
(1) Where he has paid at least two years of installments,
(a) To pay, without additional interest, the unpaid installments
due within the total grace period earned by him, which is hereby
fixed at the rate of one month grace period for every one year of
installment payments made:
Provided, That this right shall be exercised by the buyer only
once in every five years of the life of the contract and its
extensions, if any. (b) If the contract is cancelled, the seller shall
refund to the buyer the cash surrender value of the payments on
the property equivalent to fifty per cent of the total payments
made and, after five years of installments, an additional five per
cent every year but not to exceed ninety per cent of the total
payments made:
Provided, That the actual cancellation of the contract shall take
place after cancellation or the demand for rescission of the
contract by a notarial act and upon full payment of the cash
surrender value to the buyer.
Down payments, deposits or options on the contract shall be
included in the computation of the total number of installments
made.
(2) Where he has paid less than two years in installments, Sec.
4. x x x the seller shall give the buyer a grace period of not less
than sixty days from the date the installment became due. If the
buyer fails to pay the installments due at the expiration of the
grace period, the seller may cancel the contract after thirty days
from receipt by the buyer of the notice of cancellation or the
demand for rescission of the contract by a notarial act.
(Emphasis and underscoring supplied)
Pertinently, since Sps. Jovellanos failed to pay their stipulated
monthly installments as found by the MeTC, the Court examines
Optimums compliance with Section 4 of RA 6552, as abovequoted and highlighted, which is the provision applicable to
buyers who have paid less than two (2) years-worth of
installments. Essentially, the said provision provides for three (3)
requisites before the seller may actually cancel the subject
contract: first, the seller shall give the buyer a 60-day grace
COMPILED BY: WIGMORE #WIGMOREFOREVER 118

SALES Case Digest (Atty. Sarona)


Compiled by: Wigmore #wigmoreforever
period to be reckoned from the date the installment became
due; second, the seller must give the buyer a notice of
cancellation/demand for rescission by notarial act if the
buyer fails to pay the installments due at the expiration of the
said grace period; and third, the seller may actually cancel the
contract only after thirty (30) days from the buyers receipt of the
said notice of cancellation/demand for rescission by notarial act.
In the present case, the 60-day grace period automatically
operated in favor of the buyers, Sps. Jovellanos, and took effect
from the time that the maturity dates of the installment payments
lapsed. With the said grace period having expired bereft of any
installment payment on the part of Sps. Jovellanos, Optimum
then issued a notarized Notice of Delinquency and Cancellation
of Contract on April 10, 2006. Finally, in proceeding with the
actual cancellation of the contract to sell, Optimum gave Sps.
Jovellanos an additional thirty (30) days within which to settle
their arrears and reinstate the contract, or sell or assign their
rights to another.
It was only after the expiration of the thirty day (30) period did
Optimum treat the contract to sell as effectively cancelled
making as it did a final demand upon Sps. Jovellanos to vacate
the subject property only on May 25, 2006. Thus, based on the
foregoing, the Court finds that there was a valid and effective
cancellation of the Contract to Sell in accordance with Section 4
of RA 6552 and since Sps. Jovellanos had already lost their right
to retain possession of the subject property as a consequence of
such cancellation, their refusal to vacate and turn over
possession to Optimum makes out a valid case for unlawful
detainer as properly adjudged by the MeTC.
DIEGO VS. DIEGO
FACTS: In 1993, petitioner Nicolas P. Diego (Nicolas) and his
brother Rodolfo, respondent herein, entered into an oral contract
to sell covering Nicolass share, fixed at P500,000.00, as coowner of the familys Diego Building situated in Dagupan City.
Rodolfo made a downpayment of P250,000.00. It was agreed
that the deed of sale shall be executed upon payment of the
remaining balance of P250,000.00. However, Rodolfo failed to
pay the remaining balance.
Meanwhile, the building was leased out to third parties, but
Nicolass share in the rents were not remitted to him by herein
respondent Eduardo, another brother of Nicolas and designated
administrator of the Diego Building. Instead, Eduardo gave
Nicolass monthly share in the rents to Rodolfo. Despite
demands and protestations by Nicolas, Rodolfo and Eduardo
failed to render an accounting and remit his share in the rents
and fruits of the building, and Eduardo continued to hand them
over to Rodolfo.
Thus, on May 17, 1999, Nicolas filed a Complaint against
Rodolfo and Eduardo before the RTC of Dagupan CitY. Nicolas
prayed that Eduardo be ordered to render an accounting of all
the transactions over the Diego Building; that Eduardo and
Rodolfo be ordered to deliver to Nicolas his share in the rents;
and that Eduardo and Rodolfo be held solidarily liable for
attorneys fees and litigation expenses.
Rodolfo and Eduardo filed their Answer with Counterclaim for
damages and attorneys fees. They argued that Nicolas had no
more claim in the rents in the Diego Building since he had

already sold his share to Rodolfo. Rodolfo admitted having


remitted only P250,000.00 to Nicolas. He asserted that he would
pay the balance of the purchase price to Nicolas only after the
latter shall have executed a deed of absolute sale.
RTC:
After trial on the merits, or on April 19, 2005, the trial court
rendered its Decision dismissing Civil Case No. 99-02971-D for
lack of merit and ordering Nicolas to execute a deed of absolute
sale in favor of Rodolfo upon payment by the latter of
the P250,000.00 balance of the agreed purchase price.
CA:
Nicolas appealed to the CA which sustained the trial courts
Decision in toto. The CA held that since there was a perfected
contract of sale between Nicolas and Rodolfo, the latter may
compel the former to execute the proper sale document.
Besides, Nicolass insistence that he has since rescinded their
agreement in 1997 proved the existence of a perfected sale. It
added that Nicolas could not validly rescind the contract
because: "1) Rodolfo ha[d] already made a partial payment; 2)
Nicolas ha[d] already partially performed his part regarding the
contract; and 3) Rodolfo opposes the rescission.
The CA then proceeded to rule that since no period was
stipulated within which Rodolfo shall deliver the balance of the
purchase price, it was incumbent upon Nicolas to have filed a
civil case to fix the same. But because he failed to do so, Rodolfo
cannot be considered to be in delay or default.
Finally, the CA made another interesting pronouncement, that by
virtue of the agreement Nicolas entered into with Rodolfo, he had
already transferred his ownership over the subject property and
as a consequence, Rodolfo is legally entitled to collect the fruits
thereof in the form of rentals. Nicolas remaining right is to
demand payment of the balance of the purchase price, provided
that he first executes a deed of absolute sale in favor of Rodolfo.
ISSUE: WON there is a perfected contract of sale between a
petitioner and respondent over Nicolas share of the building.
HELD: The contract entered into by Nicolas and Rodolfo
was a contract to sell.
a) The stipulation to execute a deed of sale upon full
payment of the purchase price is a unique and
distinguishing characteristic of a contract to sell. It also
shows that the vendor reserved title to the property until full
payment.
There is no dispute that in 1993, Rodolfo agreed to buy Nicolass
share in the Diego Building for the price ofP500,000.00. There is
also no dispute that of the total purchase price, Rodolfo paid, and
Nicolas received,P250,000.00. Significantly, it is also not
disputed that the parties agreed that the remaining amount
of P250,000.00 would be paid after Nicolas shall have executed
a deed of sale.
This stipulation, i.e., to execute a deed of absolute sale upon full
payment of the purchase price, is a unique and distinguishing
characteristic of a contract to sell. In Reyes v. Tuparan, this
Court ruled that a stipulation in the contract, "[w]here the
vendor promises to execute a deed of absolute sale upon
COMPILED BY: WIGMORE #WIGMOREFOREVER 119

SALES Case Digest (Atty. Sarona)


Compiled by: Wigmore #wigmoreforever
the completion by the vendee of the payment of the
price," indicates that the parties entered into a contract to sell.
According to this Court, this particular provision is tantamount to
a reservation of ownership on the part of the vendor. Explicitly
stated, the Court ruled that the agreement to execute a deed of
sale upon full payment of the purchase price"shows that the
vendors reserved title to the subject property until full
payment of the purchase price."
In the instant case, records show that Nicolas signed a mere
receipt acknowledging partial payment ofP250,000.00 from
Rodolfo. It states:
July 8, 1993
Received the amount of [P250,000.00] for 1 share of Diego
Building as partial payment for Nicolas Diego.
(signed)
Nicolas Diego
The parties agreement was likewise embodied only in a receipt.
Also, Nicolas did not want to sign the deed of sale unless he is
fully paid. On the other hand, Rodolfo did not want to pay unless
a deed of sale is duly executed in his favor. We thus say,
pursuant to our ruling in Chua v. Court of Appeals that the
agreement between Nicolas and Rodolfo is a contract to sell.
This Court cannot subscribe to the appellate courts view that
Nicolas should first execute a deed of absolute sale in favor of
Rodolfo, before the latter can be compelled to pay the balance of
the price. This is patently ridiculous, and goes against every rule
in the book. This pronouncement virtually places the prospective
seller in a contract to sell at the mercy of the prospective buyer,
and sustaining this point of view would place all contracts to sell
in jeopardy of being rendered ineffective by the act of the
prospective buyers, who naturally would demand that the deeds
of absolute sale be first executed before they pay the balance of
the price. Surely, no prospective seller would accommodate.
In fine, "the need to execute a deed of absolute sale upon
completion of payment of the price generally indicates that
it is a contract to sell, as it implies the reservation of title in
the vendor until the vendee has completed the payment of
the price." In addition, "[a] stipulation reserving ownership in the
vendor until full payment of the price is x x x typical in a contract
to sell." Thus, contrary to the pronouncements of the trial and
appellate courts, the parties to this case only entered into a
contract to sell; as such title cannot legally pass to Rodolfo until
he makes full payment of the agreed purchase price.
Nicolas did not surrender or deliver title or possession to
Rodolfo.
Moreover, there could not even be a surrender or delivery of title
or possession to the prospective buyer Rodolfo. This was made
clear by the nature of the agreement, by Nicolass repeated
demands for the return of all rents unlawfully and unjustly
remitted to Rodolfo by Eduardo, and by Rodolfo and Eduardos
repeated demands for Nicolas to execute a deed of sale which,
as we said before, is a recognition on their part that ownership
over the subject property still remains with Nicolas.
Significantly, when Eduardo testified, he claimed to be
knowledgeable about the terms and conditions of the transaction
between Nicolas and Rodolfo. However, aside from stating that

out of the total consideration ofP500,000.00, the amount


of P250,000.00
had
already
been
paid
while
the
remaining P250,000.00 would be paid after the execution of the
Deed of Sale, he never testified that there was a stipulation as
regards delivery of title or possession.
It is also quite understandable why Nicolas belatedly demanded
the payment of the rentals. Records show that the structural
integrity of the Diego Building was severely compromised when
an earthquake struck Dagupan City in 1990. In order to
rehabilitate the building, the co-owners obtained a loan from a
bank. Starting May 1994, the property was leased to third parties
and the rentals received were used to pay off the loan. It was
only in 1996, or after payment of the loan that the co-owners
started receiving their share in the rentals. During this time,
Nicolas was in the USA but immediately upon his return, he
demanded for the payment of his share in the rentals which
Eduardo remitted to Rodolfo. Failing which, he filed the instant
Complaint. To us, this bolsters our findings that Nicolas did not
intend to immediately transfer title over the property.
It must be stressed that it is anathema in a contract to sell that
the prospective seller should deliver title to the property to the
prospective buyer pending the latters payment of the price in full.
It certainly is absurd to assume that in the absence of stipulation,
a buyer under a contract to sell is granted ownership of the
property even when he has not paid the seller in full. If this were
the case, then prospective sellers in a contract to sell would in all
likelihood not be paid the balance of the price.
This ponente has had occasion to rule that "[a] contract to sell is
one where the prospective seller reserves the transfer of title to
the prospective buyer until the happening of an event, such as
full payment of the purchase price. What the seller obliges
himself to do is to sell the subject property only when the entire
amount of the purchase price has already been delivered to him.
In other words, the full payment of the purchase price partakes
of a suspensive condition, the nonfulfillment of which prevents
the obligation to sell from arising and thus, ownership is retained
by the prospective seller without further remedies by the
prospective buyer. It does not, by itself, transfer ownership to the
buyer.
The contract to sell is terminated or cancelled.
Having established that the transaction was a contract to sell,
what happens now to the parties agreement?
The remedy of rescission is not available in contracts to sell. As
explained in Spouses Santos v. Court of Appeals:
In view of our finding in the present case that the agreement
between the parties is a contract to sell, it follows that the
appellate court erred when it decreed that a judicial rescission of
said agreement was necessary. This is because there was no
rescission to speak of in the first place. As we earlier pointed out,
in a contract to sell, title remains with the vendor and does not
pass on to the vendee until the purchase price is paid in full.
Thus, in a contract to sell, the payment of the purchase price is a
positive suspensive condition. Failure to pay the price agreed
upon is not a mere breach, casual or serious, but a situation that
prevents the obligation of the vendor to convey title from
acquiring an obligatory force. This is entirely different from the
COMPILED BY: WIGMORE #WIGMOREFOREVER 120

SALES Case Digest (Atty. Sarona)


Compiled by: Wigmore #wigmoreforever
situation in a contract of sale, where non-payment of the price is
a negative resolutory condition. The effects in law are not
identical. In a contract of sale, the vendor has lost ownership of
the thing sold and cannot recover it, unless the contract of sale is
rescinded and set aside. In a contract to sell, however, the
vendor remains the owner for as long as the vendee has not
complied fully with the condition of paying the purchase price. If
the vendor should eject the vendee for failure to meet the
condition precedent, he is enforcing the contract and not
rescinding it. When the petitioners in the instant case
repossessed the disputed house and lot for failure of private
respondents to pay the purchase price in full, they were merely
enforcing the contract and not rescinding it. As petitioners
correctly point out, the Court of Appeals erred when it ruled that
petitioners should have judicially rescinded the contract pursuant
to Articles 1592 and 1191 of the Civil Code. Article 1592 speaks
of non-payment of the purchase price as a resolutory condition. It
does not apply to a contract to sell. As to Article 1191, it is
subordinated to the provisions of Article 1592 when applied to
sales of immovable property. Neither provision is applicable in
the present case.
Similarly, we held in Chua v. Court of Appeals that "Article 1592
of the Civil Code permits the buyer to pay, even after the
expiration of the period, as long as no demand for rescission of
the contract has been made upon him either judicially or by
notarial act. However, Article 1592 does not apply to a contract to
sell where the seller reserves the ownership until full payment of
the price," as in this case.

for they certainly had free rein over Nicolass interest in the
Diego Building. Rodolfo put off payment of the balance of the
price, yet, with the aid of Eduardo, collected and appropriated for
himself the rents which belonged to Nicolas.
Eduardo is solidarily liable with Rodolfo as regards the
share of Nicolas in the rents.
For his complicity, bad faith and abuse of authority as the Diego
Building administrator, Eduardo must be held solidarily liable with
Rodolfo for all that Nicolas should be entitled to from 1993 up to
the present, or in respect of actual damages suffered in relation
to his interest in the Diego Building. Eduardo was the primary
cause of Nicolass loss, being directly responsible for making and
causing the wrongful payments to Rodolfo, who received them
under obligation to return them to Nicolas, the true recipient.
As such, Eduardo should be principally responsible to Nicolas as
well. Suffice it to state that every person must, in the exercise of
his rights and in the performance of his duties, act with justice,
give everyone his due, and observe honesty and good faith; and
every person who, contrary to law, wilfully or negligently causes
damage to another, shall indemnify the latter for the same.
WHEREFORE, premises considered, the Petition is GRANTED.

PART VIII: CONDITIONS AND WARRANTIES


A.

Applying the above jurisprudence, we hold that when Rodolfo


failed to fully pay the purchase price, the contract to sell was
deemed terminated or cancelled. As we have held in Chua v.
Court of Appeals,"[s]ince the agreement x x x is a mere contract
to sell, the full payment of the purchase price partakes of a
suspensive condition. The non-fulfillment of the condition
prevents the obligation to sell from arising and ownership is
retained by the seller without further remedies by the buyer."
Similarly, we held in Reyes v. Tuparan that "petitioners obligation
to sell the subject properties becomes demandable only upon the
happening of the positive suspensive condition, which is the
respondents full payment of the purchase price. Without
respondents full payment, there can be no breach of
contract to speak of because petitioner has no obligation
yet to turn over the title.

Conditions
CATUNGAL VS. RODRIGUEZ

FACTS: Agapita Catungal owned a parcel of land in Barrio


Talamban, Cebu City. On April 232, 1990, Agapita, with the
consent of her husband (Atty. Jose Catungal), entered a Contract
to Sell with respondent Angel Rodriguez. This Contract to Sell
was further upgraded into a Conditional Deed of Sale where it
was stipulated that the sum of P25 million will be payable as
follows:
a)
b)

P500, 000 down payment upon signing of the


agreement;
The balance of P24, 500, 000 will be payable in five
separate checks:

Respondents failure to pay in full the purchase price in full is not


the breach of contract contemplated under Article 1191 of the
New Civil Code but rather just an event that prevents the
petitioner from being bound to convey title to respondent."
Otherwise stated, Rodolfo has no right to compel Nicolas to
transfer ownership to him because he failed to pay in full the
purchase price. Correlatively, Nicolas has no obligation to
transfer his ownership over his share in the Diego Building to
Rodolfo.

First check shall be for P4, 500, 000 while the remaining balance
to be paid in four checks in the amount of P5 million each will be
payable only after Rodriguez (Vendee) has successfully
negotiated, secured, and provided a Road Right of Way. If
however the Road Right of Way could not be negotiated,
Rodriguez shall notify the Catungals for them to reassess and
solve the problem by taking other options and should the
situation ultimately prove futile, he shall take steps to rescind or
cancel the herein Conditional Deed of Sale.

On the other hand, the respondents additional submission that


Nicolas cheated them by "vanishing and hibernating" in the USA
after receiving Rodolfos P250,000.00 downpayment, only to
come back later and claim that the amount he received was a
mere loan cannot be believed. How the respondents could
have been cheated or disadvantaged by Nicolass leaving is
beyond comprehension. If there was anybody who benefited
from Nicolass perceived "hibernation", it was the respondents,

It was also stipulated that the access road or Road Right of Way
leading to the lot shall be the responsibility of the VENDEE to
secure and any or all cost relative to the acquisition thereof shall
be borne solely by the VENDEE. He shall, however, be accorded
with enough time necessary for the success of his endeavor,
granting him a free hand in negotiating for the passage.
Spouses Catungal requested an advance of P5 million on the
COMPILED BY: WIGMORE #WIGMOREFOREVER 121

SALES Case Digest (Atty. Sarona)


Compiled by: Wigmore #wigmoreforever
purchase price for personal reasons. However, Rodriguez
refused on the ground that the amount was not due under the
terms of their agreement. Further, he learned that the Catungals
were offering the property for sale to third parties who are willing
to pay a higher amount of money for a Road Right of Way than
what Rodriguez has initially negotiated. In other words, instead
of assisting Rodriguez in successfully negotiating, the Catungals
allegedly maliciously defeated his efforts so to justify the
rescission. Rodriguez then received letters signed by Atty. Jose
Catungal demanding him to make up his mind about buying the
land or exercising his option to buy because they needed money
to pay personal obligations or else the Catungals warned that
they would consider the contract cancelled.
RTC ruled in favor of Rodriguez finding that his obligation to pay
the balance arises only after successfully negotiating a Road
Right of Way. CA affirmed the RTCs decision but the defendants
filed a motion for reconsideration and raised for the first time the
contention that the court erred in not finding their stipulations null
for violating the principle of mutuality of contracts.
ISSUE: Whether or not the stipulations of their Conditional Deed
of Sale constitute a potestative condition (one that is subject to
the will of one of the parties either the debtor or creditor).
HELD: NO. the condition in their Conditional Deed of Sale
stating that respondent shall pay the balance of the purchase
price when he has successfully negotiated and secured a road
right of way, is not a condition on the perfection of the contract
nor on the validity of the entire contract or its compliance as
contemplated in Article 1308. It is a condition imposed only on
respondent's obligation to pay the remainder of the purchase
price. In our view and applying Article 1182, such a condition is
not purely potestative as petitioners contend. It is not dependent
on the sole will of the debtor but also on the will of third persons
who own the adjacent land and from whom the road right of way
shall be negotiated. Ina manner of speaking, such a condition is
likewise dependent on chance as there is no guarantee that
respondent and the third party-landowners would come to
an agreement regarding the road right of way. This type of mixed
condition is expressly allowed under Article 1182 of the Civil
Code.
IN RELATION TO ARTICLE 1197 The Catungals also argued
that Rodriguez has been given enough time to negotiate for the
Road Right of Way. However, no stipulation regarding specific
time can be found in their agreement. SC said that Even
assuming arguendo that the Catungals were correct that the
respondent's obligation to negotiate a road right of way was one
with an uncertain period, their rescission of the Conditional Deed
of Sale would still be unwarranted. What the Catungals should
have done was to first file an action in court to fix the period
within which Rodriguez should accomplish the successful
negotiation of the road right of way pursuant to the above quoted
provision. Thus, the Catungals' demand for Rodriguez to make
an additional payment of P5 million was premature and
Rodriguez's failure to accede to such demand did not justify the
rescission of the contract.
WHEREFORE, the Decision dated August 8, 2000 and the
Resolution dated January 30, 2001 of the Court of Appeals are
AFFIRMED with the following
MODIFICATION:

If still warranted, respondent Angel S. Rodriguez is given a


period of thirty (30) days from the finality of this Decision to
negotiate a road right of way. In the event no road right of way is
secured by respondent at the end of said period, the parties shall
reassess and discuss other options as stipulated in paragraph
1(b) of the Conditional Deed of Sale and, for this purpose, they
are given a period of thirty (30) days to agree on a course of
action. Should the discussions of the parties prove futile after the
said thirty (30)-day period, immediately upon the expiration of
said period for discussion, Rodriguez may (a) exercise his option
to rescind the contract, subject to the return of his down
payment, in accordance with the provisions of paragraphs 1(b)
and 5 of the Conditional Deed of Sale or (b) waive the road right
of way and pay the balance of the deducted purchase price as
determined in the RTC Decision dated May 30, 1992.

B.

Warranties

1. Express Warranties

HARRISON MOTORS CORPORATION vs. RACHEL A.


NAVARRO
FACTS: Sometime in June of 1987 Harrison Motors Corporation
through its president, Renato Claros, sold 2 Isuzu Elf trucks to
Rachel Navarro, owner of RN Freight Lines. Prior to the sale,
Renato Claros represented to Navarro that all the BIR taxes and
customs duties for the parts used on the two 2 trucks had been
paid for.
On 10 September 1987 the Bureau of Internal Revenue (BIR)
and the Land Transportation Office (LTO) entered into a
Memorandum of Agreement (MOA) which provided that prior to
registration in the LTO of any assembled or re-assembled motor
vehicle which used imported parts, a Certificate of Payment
should first be obtained from the BIR to prove payment of all
taxes required under existing laws.
On 16 June 1988 the BIR, BOC and LTO entered into a tripartite
MOA which provided that prior to the registration in the LTO of
any locally assembled motor vehicle using imported component
parts, a Certificate of Payment should first be obtained from the
BIR and the BOC to prove that all existing taxes and customs
duties have been paid.
In December of 1988 government agents seized and detained
the two (2) Elf trucks of Navarro after discovering that there were
still unpaid BIR taxes and customs duties thereon. The BIR and
the BOC ordered NAvarro to pay the proper assessments or her
trucks would be impounded. Navarro went to Claros to ask for
the receipts evidencing payment of BIR taxes and customs
duties; however, Claros refused to comply. Navarro then
demanded from Claros but her demands were again ignored.
But wanting to secure the immediate release of the trucks to
comply with her business commitments, Navarro paid the
assessed BIR taxes and customs duties amounting to
P32,943.00. Consequently, she returned to Harrisons office to
ask for reimbursement, but it again refused, prompting her to
send a demand letter through her lawyer. When Harrison still
COMPILED BY: WIGMORE #WIGMOREFOREVER 122

SALES Case Digest (Atty. Sarona)


Compiled by: Wigmore #wigmoreforever
ignored her letter, she filed a complaint for a sum of money on 24
September 1990 with the Regional Trial Court of Makati.
On 5 March 1992 the trial court rendered a decision ordering
Harrison to reimburse private respondent in the amount of
P32,943.00 for the customs duties and internal revenue taxes
the latter had to pay to discharge her 2 Elf trucks from
government custody.
Harrison argues that it was no longer obliged to pay for the
additional taxes and customs duties imposed on the imported
component parts by the Memorandum Orders and the two (2)
Memoranda of Agreement based on non-impairment clause of
the Constitution but also the principle of non-retroactivity of laws
provided in Art. 4 of the Civil Code.

would have been avoided had Harrison simply furnished private


respondent with the receipts evidencing payment of BIR taxes
and customs duties. If only Navarro had the receipts to prove
payment of such assessments then she would have easily
secured the release of her two (2) Elf trucks. But Harrison
arbitrarily and unjustly denied Navarros demands. Instead,
Harrison obstinately insisted that it was no longer concerned with
the problem involving the two (2) trucks since it no longer owned
the vehicles after the consummation of the sale.

The records however reveal that the Memorandum Orders and


Memoranda of Agreement do not impose any additional BIR
taxes or customs duties. The MOA mandated that prior to
registration in the LTO of any assembled automobile using
imported parts, a Certificate of Payment should first be obtained
from the BIR which would then transmit the Certificate to the LTO
to prove that all the BIR taxes required under existing laws have
been paid.

It is true that the ownership of the trucks shifted to private


respondent after the sale. But petitioner must remember that
prior to its consummation it expressly intimated to her that it had
already paid the taxes and customs duties. Such representation
shall be considered as a sellers express warranty under Art.
1546 of the Civil Code which covers any affirmation of fact or any
promise by the seller which induces the buyer to purchase the
thing and actually purchases it relying on such affirmation or
promise. It includes all warranties which are derived from
express language, whether the language is in the form of a
promise or representation. Presumably, therefore, private
respondent would not have purchased the two (2) Elf trucks were
it not for petitioners assertion and assurance that all taxes on its
imported parts were already settled.

The MOA provided that prior to registration with the LTO of any
assembled motor vehicle using imported component parts, a
Certificate of Payment should first be secured from the BIR or
the BOC which should then be duly forwarded to LTO. The
Certificate would serve as proof that all taxes and customs duties
required under existing laws, rules and regulations had already
been settled.

This express warranty was breached the moment petitioner


refused to furnish private respondent with the corresponding
receipts since such documents were the best evidence she could
present to the government to prove that all BIR taxes and
customs duties on the imported component parts were fully paid.
Without evidence of payment, she was powerless to prevent the
trucks from being impounded.

ISSUE: Who should pay the BIR taxes and customs duties which
the administrative regulations sought to enforce?

Under Art. 1599 of the Civil Code, once an express warranty is


breached the buyer can accept or keep the goods and maintain
an action against the seller for damages. This was what private
respondent did. She opted to keep the two (2) trucks which she
apparently needed for her business and filed a complaint for
damages, particularly seeking the reimbursement of the amount
she paid to secure the release of her vehicles.

HELD: Harrison contends that Navarro should be the one to pay


the internal revenue taxes and customs duties. It claims that at
the time the Memorandum Orders and the two (2) Memoranda of
Agreement took effect the two (2) Elf trucks were already sold to
Navarro, thus, it no longer owned the vehicles.
Thus, although the Whereas clause in the MOA provides that
private respondent is the one required by the administrative
regulations to secure the Certificate of Payment for the purpose
of registration, petitioner as the importer and the
assembler/manufacturer of the two (2) Elf trucks is still the one
liable for payment of revenue taxes and customs duties.
Harrisons obligation to pay does not arise from the
administrative regulations but from the tax laws existing at the
time of importation. Hence, even if Navarro already owned the
two (2) trucks when the Memorandum Orders and Memoranda of
Agreement took effect, the fact remains that Harrison was still
the one duty-bound to pay for the BIR taxes and customs duties.
It is also quite obvious that as between Harrison, who is the
importer- assembler/manufacturer, and Navarro, who is merely
the buyer, it is petitioner which has the obligation to pay taxes to
the BIR and the BOC. Harrison would be unjustly enriched if
private respondent should be denied reimbursement.
Besides, Harrisons allegation that it already paid the BIR taxes
and customs duties is highly doubtful. This entire controversy

MOLES vs IAC
FACTS: In 1977, petitioner needed a linotype printing machine
for his printing business, The LM Press at Bacolod City, and
applied for an industrial loan with the Development Bank of the
Philippines. (DBP) for the purchase thereof. An agent of Smith,
Bell and Co. who is a friend of petitioner introduced the latter to
private respondent, owner of the Diolosa Publishing House in
Iloilo City, who had two available machines. Thereafter, petitioner
went to Iloilo City to inspect the two machines offered for sale
and was informed that the same were secondhand but
functional.
Sometime between April and May, 1977, the machine was
delivered to petitioner's publishing house where it was installed
by an employee of Diolosa. Prior to the release of the loan, a
representative from the DBP, Bacolod, supposedly inspected the
machine but he merely looked at it to see that it was there . The
inspector's recommendation was favorable and, thereafter,
petitioner's loan of P50,000.00 was granted and released.
COMPILED BY: WIGMORE #WIGMOREFOREVER 123

SALES Case Digest (Atty. Sarona)


Compiled by: Wigmore #wigmoreforever

But on November 29, 1977, petitioner wrote private respondent


that the machine was not functioning properly as it needed a new
distributor bar. Private respondent made no reply to said letter,
so petitioner engaged the services of other technicians. Later,
after several telephone calls regarding the defects in the
machine, private respondent sent two technicians to make the
necessary repairs but they failed to put the machine in running
condition. In fact, since then petitioner was never able to use the
machine. Petitioner again wrote private respondent, this time
with the warning that he would be forced to seek legal remedies
to protect his interest.
Obviously in response to the foregoing letter, private respondent
decided to purchase a new distributor bar and private respondent
delivered this spare part to petitioner. However, when thereafter
petitioner asked private respondent to pay for the price of the
distributor bar, the latter asked petitioner to share the cost with
him. Petitioner thus finally decided to indorse the matter to his
lawyer.
An expert witness for the petitioner declared that he inspected
the linotype machine involved in this case at the instance of
petitioner. In his inspection thereof, he found several defects.
ISSUE/S:
W/N private respondent is bound by an express warranty.
W/N private respondents express warrantly was a mere dealers
talk.
W/N hidden defects in the machine is sufficient to warrant a
rescission of the contract between the parties.
HELD: Private respondent is bound by the express warranty
which he executed in favor of the petitioners.
When an article is sold as a secondhand item, a question arises
as to whether there is an implied warranty of its quality or fitness.
It is generally held that in the sale of a designated and specific
article sold as secondhand, there is no implied warranty as to its
quality or fitness for the purpose intended, at least where it is
subject to inspection at the time of the sale. On the other hand,
there is also authority to the effect that in a sale of a secondhand
articles there may be, under some circumstances, an implied
warranty of fitness for the ordinary purpose of the article sold or
for the particular purpose of the buyer.
In a line of decisions rendered by the United States Supreme
Court, it had theretofore been held that there is no implied
warranty as to the condition, adaptation, fitness, or suitability for
the purpose for which made, or the quality, of an article sold as
and for a secondhand article. Said general rule, however, is not
without exceptions. Article 1562 of our Civil Code, which was
taken from the Uniform Sales Act, provides:
Art. 1562. In a sale of goods, there is an implied warranty or
condition as to the quality or fitness of the goods, as follows:
Where the buyer, expressly or by implication, makes known to
the seller the particular purpose for which the goods are
acquired, and it appears that the buyer relies on the seller's skill
or judgment (whether he be the grower or manufacturer or not),
there is an implied warranty that the goods shall be reasonably fit
for such purpose;

To repeat, in the case before Us, a certification to the effect that


the linotype machine bought by petitioner was in A-1 condition
was issued by private respondent in favor of the former. This
cannot but be considered as an express warranty. However, it is
private respondent's submission, that the same is not binding on
him, not being a part of the contract of sale between them.
It must be remembered that the certification was a condition sine
qua non for the release of petitioner's loan which was to be used
as payment for the purchase price of the machine. Private
respondent failed to refute this material fact. Neither does he
explain why he made that express warranty on the condition of
the machine if he had not intended to be bound by it. In fact, the
respondent court, in declaring that petitioner should have availed
of the remedy of requiring repairs as provided for in said
certification, thereby considered the same as part and parcel of
the verbal contract between the parties.
Private respondents express warranty as to the A-1
condition of the machine was not merely dealer's talk.
Private respondent was not a dealer of printing or linotype
machines to whom could be ascribed the supposed resort to the
usual exaggerations of trade in said items. His certification as to
the condition of the machine was not made to induce petitioner to
purchase it but to confirm in writing for purposes of the financing
aspect of the transaction his representations thereon. Ordinarily,
what does not appear on the face of the written instrument
should be regarded as dealer's or trader's talk conversely, what
is specifically represented as true in said document, as in the
instant case, cannot be considered as mere dealer's talk.
On the question as to whether the hidden defects in the machine
is sufficient to warrant a rescission of the contract between the
parties, we have to consider the rule on redhibitory defects
contemplated in Article 1561 of the Civil Code. A redhibitory
defect must be an imperfection or defect of such nature as to
engender a certain degree of importance. An imperfection or
defect of little consequence does not come within the category of
being redhibitory.
As already narrated, an expert witness for the petitioner
categorically established that the machine required major repairs
before it could be used. This, plus the fact that petitioner never
made appropriate use of the machine from the time of purchase
until an action was filed, attest to the major defects in said
machine, by reason of which the rescission of the contract of
sale is sought. The factual finding, therefore, of the trial court that
the machine is not reasonably fit for the particular purpose for
which it was intended must be upheld, there being ample
evidence to sustain the same.
At a belated stage of this appeal, private respondent came up for
the first time with the contention that the action for rescission is
barred by prescription. While it is true that Article 1571 of the
Civil Code provides for a prescriptive period of six months for a
redhibitory action a cursory reading of the ten preceding articles
to which it refers will reveal that said rule may be applied only in
case of implied warranties. The present case involves one with
and express warranty. Consequently, the general rule on
27
rescission of contract, which is four years
shall apply.
Considering that the original case for rescission was filed only
one year after the delivery of the subject machine, the same is
well within the prescriptive period.
COMPILED BY: WIGMORE #WIGMOREFOREVER 124

SALES Case Digest (Atty. Sarona)


Compiled by: Wigmore #wigmoreforever
2. Implied Warranties

companys president was a close friend of Evangelista. The


various animal feeds were paid and covered by checks with due
dates from July 1993-September 1993.

ESCALER V. CA AND SPS. REYNOSO


FACTS: On March 7, 1958, the spouses Reynoso sold to
petitioners a parcel of land situated in Antipolo, Rizal with an
area of 239,479 sqm and covered by TCT No. 57400. However,
on April 21, 1961, the Register of Deeds of Rizal and A. Doronilla
Resources Development, Inc. filed a case (Case no. 4252) for
the cancellation of OCT No. 1526 issued in the name of Angelina
C. Reynoso (the predecessors-in-interest of the spouses
Reynoso) on the ground that the subject property is already
covered by TCT No. 42999 issued under A. Doronilla. The court
favored A. Doronilla in this case.
Thereafter, herein petitioners filed the present case against the
respondents, for the recovery of the value of the property sold to
them plus damages on the ground that the spouses violated the
vendors warranty against eviction. The lower court ruled in
favor of the petitioners, which was reversed in the CA. Hence,
the present petition.
ISSUE: W/n Articles 1558 and 1559 of the Civil Code are to be
strictly applied in this case.
HELD: YES. (See Articles 1548, 1558 and 1559). In order that a
vendors liability for eviction may be enforced, the following
requisites must concur:
a.
b.
c.
d.

There must be a final judgment;


The purchaser has been deprived of the whole or part
of the thing sold;
Said deprivation was by virtue of a right prior to the sale
made by the vendor; and
The vendor has been summoned and made codefendant in the suit for eviction at the instance of the
vendee.

In the case at bar, the fourth requisitethat of being summoned


in the suit for eviction (Case no. 4252) at the instance of the
vendeeis not present. All that petitioners did, per their very
admission, was to furnish respondents, by registered mail, with a
copy of the opposition they (petitioners) filed in the suit.
Decidedly, this is not the kind of notice prescribed by Articles
1558 and 1559 of the Civil Code.
The term unless he is summoned in the suit for eviction at the
instance of the vendee means that the respondents as vendor/s
should be made parties to the suit at the instance of petitionersvendees, either by way of asking that the former be made a codefendant or by the filing of a third-party complaint against said
vendors. Nothing of that sort appeared to have been done by the
petitioners in the instant case.

NUTRIMIX FEEDS CORP. V. CA


FACTS: In 1993, private respondent spouses Evangelista
procured various animal feeds from petitioner Nutrimix Feeds
Corp. the petitioner gave the respondents a credit period of 3045 days to postdate checks to be issued as payment for the
feeds. The accommodation was made apparently because the

Initially, the spouses were good paying customers. However,


there were instances when they failed to issue checks despite the
delivery of goods. Consequently, the respondents incurred an
aggregate unsettled account with Nutrimix amounting to
P766,151
When the checks were deposited by the petitioner, the same
were dishonored (closed account). Despite several demands from
the petitioner, the spouses refused to pay the remaining balance
Thereafter, Nutrimix filed a complaint against Evangelista for
collection of money with damages.
The respondents admitted their unpaid obligation but impugned
their liability. The nine checks issued were made to guarantee the
payment of the purchases, which was previously determined to
be procured from the expected proceeds in the sale of their
broilers and hogs. They contended that inasmuch as the sudden
and massive death of their animals was caused by the
contaminated products of the petitioner, the nonpayment of their
obligation was based on a just and legal ground.
The respondents also lodged a complaint for damages against
the petitioner, for the untimely and unforeseen death of their
animals supposedly effected by the adulterated animal feeds the
petitioner sold to them.
Nutrimix alleged that the death of the respondents animals was
due to the widespread pestilence in their farm. The petitioner,
likewise, maintained that it received information that the
respondents were in an unstable financial condition and even
sold their animals to settle their obligations from other enraged
and insistent creditors. It, moreover, theorized that it was the
respondents who mixed poison to its feeds to make it appear that
the feeds were contaminated.
The trial court held in favor of petitioner on the ground that it
cannot be held liable under Articles 1561 and 1566 of the Civil
Code governing hidden defects of commodities sold. The trial
court is predisposed to believe that the subject feeds were
contaminated sometime between their storage at the bodega of
the Evangelistas and their consumption by the poultry and hogs
fed therewith, and that the contamination was perpetrated by
unidentified or unidentifiable ill-meaning mischief-maker(s) over
whom Nutrimix had no control in whichever way.
CA modified the decision of the trial court, citing that respondents
were not obligated to pay their outstanding obligation to the
petitioner in view of its breach of warranty against hidden defects.
The CA gave much credence to the testimony of Dr. Rodrigo
Diaz, who attested that the sample feeds distributed to the
various governmental agencies for laboratory examination were
taken from a sealed sack bearing the brand name Nutrimix
ISSUE: WON Nutrimix is guilty of breach of warranty due to
hidden defects
HELD: NO. The provisions on warranty against hidden defects
are found in Articles 1561 and 1566 of the New Civil Code of the
Philippines. A hidden defect is one which is unknown or could not
COMPILED BY: WIGMORE #WIGMOREFOREVER 125

SALES Case Digest (Atty. Sarona)


Compiled by: Wigmore #wigmoreforever
have been known to the vendee. Under the law, the requisites to
recover on account of hidden defects are as follows:
a) the defect must be hidden;
b) the defect must exist at the time the sale was made;
c) the defect must ordinarily have been excluded from the
contract;
d) the defect, must be important (renders thing UNFIT or
considerably decreases FITNESS);
e) the action must be instituted within the statute of
limitations
In the sale of animal feeds, there is an implied warranty that it is
reasonably fit and suitable to be used for the purpose which both
parties contemplated. To be able to prove liability on the basis of
breach of implied warranty, three things must be established by
the respondents. The first is that they sustained injury because
of the product; the second is that the injury occurred because the
product was defective or unreasonably unsafe; and finally, the
defect existed when the product left the hands of the petitioner. A
manufacturer or seller of a product cannot be held liable for any
damage allegedly caused by the product in the absence of any
proof that the product in question was defective. The defect must
be present upon the delivery or manufacture of the product; or
when the product left the sellers or manufacturers control; or
when the product was sold to the purchaser; or the product must
have reached the user or consumer without substantial change
in the condition it was sold. Tracing the defect to the petitioner
requires some evidence that there was no tampering with, or
changing of the animal feeds. The nature of the animal feeds
makes it necessarily difficult for the respondents to prove that the
defect was existing when the product left the premises of the
petitioner.
A review of the facts of the case would reveal that the petitioner
delivered the animal feeds, allegedly containing rat poison, on
July 26, 1993; but it is astonishing that the respondents had the
animal feeds examined only on October 20, 1993, or barely three
months after their broilers and hogs had died. A difference of
approximately three months enfeebles the respondents theory
that the petitioner is guilty of breach of warranty by virtue of
hidden defects. In a span of three months, the feeds could have
already been contaminated by outside factors and subjected to
many conditions unquestionably beyond the control of the
petitioner.
Even more surprising is the fact that during the meeting with
Nutrimix President Mr. Bartolome, the respondents claimed that
their animals were plagued by disease, and that they needed
more time to settle their obligations with the petitioner. It was
only after a few months that the respondents changed their
justification for not paying their unsettled accounts, claiming
anew that their animals were poisoned with the animal feeds
supplied by the petitioner.

and peaceful possession in favor of the petitioners.


The property is mortgage to PNP and as such, petitioners filed a
request to assume responsibility of the mortgage. Because of
petitioners failure to produce the required papers, their petition
was denied.
Petitioners allege that the contract should be rescinded because
of failure of delivery.
ISSUE: WON the contract is recissible due to breach of contract.
HELD: There is no breach of contact in this case since there is
no provision in the contract that imposes the obligation to the
respondents to eject the people occupying the property.
There was also a constructive delivery because the deed of sale
was made in a public document. The contention of the petitioners
that there could be no constructive delivery because the
respondents are not in possession of the property is of no merit.
What matters in a constructive delivery is control and not
possession. Control was placed in the hands of the petitioners
that is why they were able to file an ejectment case. Prior
physical delivery or possession is not legally required and the
execution of the deed of sale is deemed equivalent to delivery.

SUPERCARS MANAGEMENT AND DEVELOPMENT


CORPORATION VS. FLORES
FACTS: Respondent Flores bought an Isuzu Carter Crew Cab
from petitioner. The RCBC financed the balance of the purchase
price. Its payment was secured by a chattel mortgage of the
same vehicle. However, defects of the car emerged when
respondent was using it. These defects persuaded respondent
Flores to rescind the contract with petitioner and stop the
payment of the balance for the aforesaid car.
As a result, RCBC bank opted to file a petition for Extrajudicial
Foreclosure of Chattel Mortgage. The car was then sold at a
public auction and RCBC acquired the same. It was later sold to
a third person. Petitioner contends that respondent has "no right
to rescind the contract of sale" because the motor vehicle in
question is already in the hands of a third party.
Hence, Article 1191 can no longer be availed of by the
respondent.
ISSUE: Whether or not Article 1191 can no longer be availed of
by respondent Flores.
RULING: Article 1191 is applicable. Rescission is proper if one of
the parties to a contract commits a substantial breach of its
provision. It creates an obligation to return the object of the
contract. It can be carried out only when the one who demands
rescission can return whatever he may obliged to restore.

POWER COMMERCIAL V. CA (June 20, 1997)


FACTS: Petitioner asbestos manufacturer Power Commercial
and industrial corporation bought the property of spouses
Reynaldo and Angelita Quiambao located in Makati City.

Rescission abrogates the contract from its inception and requires


a mutual restitution of the benefits received. Respondent is not
obliged to return the car; while petitioner is obliged to return what
has been paid.

Since there are lessees occupying the subject land, part of the
deed of sale is a warranty of respondents that will defend its title
COMPILED BY: WIGMORE #WIGMOREFOREVER

126

SALES Case Digest (Atty. Sarona)


Compiled by: Wigmore #wigmoreforever
PART IX: EXTINGUISHMENT OF SALE
A. Grounds
B. Conventional Redemption

KINGS PROPERTIES CORP., VS. CANUTO A. GALIDO


FACTS: This case involves an action for cancellation of
certificates of title, registration of deed of sale and issuance of
certificates of title filed by Canuto A. Galido before the RTC of
Antipolo City. On April 18, 1966, the heirs of Eniceo, namely
Rufina and Maria Eniceo, were awarded with Homestead Patent
consisting of four parcels of land located in San Isidro, Antipolo,
Rizal. The Antipolo property with a total area of 14.8882 hectares
was registered under OCT No. 535.
The issuance of the homestead patent was subject to the
following conditions:
To have and to hold the said tract of land, with the
appurtenances thereunto of right belonging unto the said Heirs of
Domingo Eniceo and to his heir or heirs and assigns forever,
subject to the provisions of sections 118, 121, 122 and 124 of
Commonwealth Act No. 141, as amended, which provide that
except in favor of the Government or any of its branches, units or
institutions, the land hereby acquired shall be inalienable and
shall not be subject to incumbrance for a period of five (5) years
next following the date of this patent, and shall not be liable for
the satisfaction of any debt contracted prior to the expiration of
that period; that it shall not be alienated, transferred or conveyed
after five (5) years and before twenty-five (25) years next
following the issuance of title, without the approval of the
Secretary of Agriculture and Natural Resources; that it shall not
be incumbered, alienated, or transferred to any person,
corporation, association, or partnership not qualified to acquire
public lands under the said Act and its amendments; x x x
On September 1973, a deed of sale covering the Antipolo
property was executed between Rufina Eniceo and Maria Eniceo
as vendors and respondent as vendee. The property was sold to
respondent for P250,000. A certain Carmen Aldana delivered the
owner's duplicate copy of OCT No. 535 to respondent.
On 1988, the Eniceo heirs registered with the Registry of Deeds
a Notice of Loss of the owner's copy of OCT No. 535.
RTC rendered a decision finding that the certified true copy of
OCT No. 535 contained no annotation in favor of any person,
corporation or entity. The RTC ordered the Registry of Deeds to
issue a second owner's copy of OCT No. 535 in favor of the
Eniceo heirs and declared the original owner's copy of OCT No.
535 canceled and considered no further value.
Petitioner states that as early as 1991, respondent knew of the
RTC decision because respondent filed a criminal case against
Rufina Eniceo and Leonila Bolinas for giving false testimony
upon a material fact during the trial. They alleged that sometime
in 1995, Bolinas came to the office of Alberto Tronio Jr.,
petitioner's general manager, and offered to sell the Antipolo
property. During an on-site inspection, Tronio saw a house and
ascertained that the occupants were Bolina's relatives. Tronio

also went to the Registry of Deeds to verify the records on file


and ascertained that OCT No. 535 was clean and had no lien
and encumbrances. After the necessary verification, petitioner
decided to buy the Antipolo property.
On March 20, 1995, the Eniceo heirs executed a deed of
absolute sale in favor of petitioner covering lots 3 and 4 of the
Antipolo property for P500,000.00.
On August 17, 1995, the Secretary of Department of
Environment and Natural Resources (DENR Secretary)
approved the deed of sale between the Eniceo heirs and
respondent. On January 1996, respondent filed a civil complaint
with the trial court against the Eniceo heirs and petitioner praying
for the cancellation of the certificates
of title issued in favor of petitioner, and the registration of the
deed of sale and issuance of a new transfer certificate of title in
favor of respondent.
The trial court rendered it's decision dismissing the case for lack
of legal and factual basis. However, the CA reversed the trial
court's decision.
ISSUE: Whether the deed of sale delivered to respondent should
be presumed an equitable mortgage pursuant to Article 1602(2)
and 1604 of the Civil Code.
HELD: Validity of the deed of sale to respondent
The contract between the Eniceo heirs and respondent executed
on 10 September 1973 was a perfected contract of sale. A
contract is perfected once there is consent of the contracting
parties on the object certain and on the cause of the obligation.
In the present case, the object of the sale is the Antipolo property
and the price certain is P250,000.
The contract of sale has also been consummated because the
vendors and vendee have performed their respective obligations
under the contract. In a contract of sale, the seller obligates
himself to transfer the ownership of the determinate thing sold,
and to deliver the same to the buyer, who obligates himself to
pay a price certain to the seller. The execution of the notarized
deed of sale and the delivery of the owners duplicate copy of
OCT No. 535 to respondent is tantamount to a constructive
delivery of the object of the sale.
Petitioner invokes the belated approval by the DENR Secretary,
made within 25 years from the issuance of the homestead, to
nullify the sale of the Antipolo property. The sale of the Antipolo
property cannot be annulled on the ground that the DENR
Secretary gave his approval after 21 years from the date the
deed of sale in favor of respondent was executed.
Equitable Mortgage
Petitioner contends that the deed of sale in favor of respondent is
an equitable mortgage because the Eniceo heirs remained in
possession of the Antipolo property despite the execution of the
deed of sale.
An equitable mortgage is one which although lacking in some
formality, or form or words, or other requisites demanded by a
statute, nevertheless reveals the intention of the parties to
charge real property as security for a debt, and contains nothing
impossible or contrary to law. The essential requisites of an
equitable mortgage are:
COMPILED BY: WIGMORE #WIGMOREFOREVER 127

SALES Case Digest (Atty. Sarona)


Compiled by: Wigmore #wigmoreforever
1.
2.

The parties entered into a contract denominated as a


contract of sale; and
Their intention was to secure existing debt by way of a
mortgage.

Petitioner claims that an equitable mortgage can be presumed


because the Eniceo heirs remained in possession of the Antipolo
property. Apart from the fact that the Eniceo heirs remained in
possession of the Antipolo property, petitioner has failed to
substantiate its claim that the contract of sale was intended
to secure an existing debt by way of mortgage. In fact, mere
tolerated possession is not enough to prove that the
transaction was an equitable mortgage.
Furthermore, petitioner has not shown any proof that the Eniceo
heirs were indebted to respondent. On the contrary, the deed of
sale executed in favor of respondent was drafted clearly to
convey that the Eniceo heirs sold and transferred the Antipolo
property to respondent. The deed of sale even inserted a
provision about defrayment of registration expenses to effect the
transfer of title to respondent.
The Court notes that the Eniceo heirs have not appealed the
CAs decision, hence, as to the Eniceo heirs, the CAs decision
that the contract was a sale and not an equitable mortgage is
now final. Since petitioner merely assumed the rights of the
Eniceo heirs, petitioner is now estopped from questioning the
deed of sale dated 10 September 1973.
HEIRS OF THE LATE SPOUSES BALITE VS. LIM
FACTS: Spouses Aurelio and Esperanza Balite were the owners
of a parcel of land. When Aurelio died intestate, his wife
Esperanza and their children inherited the subject property and
became co-owners thereof. Esperanza became ill and was in
dire need of money for her hospital expenses. She, through her
daughter, Criseta, offered to sell to Rodrigo Lim, her undivided
share for the price of P1 mil. Esperanza and Rodrigo agreed that
under the Deed of Absolute Sale, it will be made to appear that
the purchase price of the property was P150,000 although the
actual price agreed upon by them for the property was P1mil.
On April 16, 1996, Esperanza executed a Deed of Absolute Sale
in favor of Rodrigo. They also executed on the same day a Joint
Affidavit under which they declared that the real price of the
property was P1mil. payable to Esperanza by installments. Only
Esperanza and two of her children Antonio and Criseta knew
about the said transaction. When the rest of the children knew of
the sale, they wrote to the Register of Deeds saying that their
mother did not inform them of the sale of a portion of the said
property nor did they give consent thereto. Nonetheless, Rodrigo
made partial payments to Antonio who is authorized by his
mother through a Special Power of Attorney.
Esperanza signed a letter addressed to Rodrigo informing the
latter that her children did not agree to the sale of the property to
him and that she was withdrawing all her commitments until the
validity of the sale is finally resolved. Then Esperanza died
intestate and was survived by her children. Meanwhile, Rodrigo
caused to be published the Deed of Absolute Sale.
Petitioners filed a complaint against Rodrigo for the annulment of
sale, quieting of title, injunction and damages. Rodrigo secured a

loan from the Rizal commercial Banking Corporation in the


amount of P2mil and executed a Real Estate Mortgage over the
property as security thereof. On motion of the petitioners, they
were granted leave to file an amended complaint impleading the
bank as additional party defendant. The court issued an order
rejecting the amended complaint of the petitioners. Likewise, the
court dismissed the complaint and held that pursuant to Article
493 of the Civil Code, a co-owner is not invalidated by the
absence of the consent of the co-owners. Hence, the sale by
Esperanza of the property was valid; the excess from her
undivided share should be taken from the undivided shares of
Criseta and Antonio, who expressly agreed to and benefited from
the sale.
The CA likewise held that the sale was valid and binding insofar
as Esperanza's undivided share of the property was concerned.
It affirmed the RTC ruling that the lack of consent of the coowners did not nullify the sale.
ISSUE: Whether or not the Deed of Absolute Sale was valid. Deed of Absolute Sale was merely relatively simulated, it
remains valid and enforceable.
HELD:
Validity of the Sale
We have before us an example of a simulated contract. Article
1345 of the Civil Code provides that the simulation of a contract
may either be absolute or relative. In absolute simulation, there is
a colorable contract but without any substance, because the
parties have no intention to be bound by it. An absolutely
simulated contract is void, and the parties may recover from
each other what they may have given under the contract. On the
other hand, if the parties state a false cause in the contract to
conceal their real agreement, such a contract is relatively
simulated. Here, the parties real agreement binds them.
In the present case, the parties intended to be bound by the
Contract, even if it did not reflect the actual purchase price of the
property. That the parties intended the agreement to produce
legal effect is revealed by the letter of Esperanza Balite to
respondent dated October 23, 1996 and petitioners admission
that there was a partial payment of P320,000 made on the basis
of the Deed of Absolute Sale. There was an intention to transfer
the ownership of over 10,000 square meters of the property .
Clear from the letter is the fact that the objections of her children
prompted Esperanza to unilaterally withdraw from the
transaction.
Since the Deed of Absolute Sale was merely relatively simulated,
it remains valid and enforceable. All the essential requisites
prescribed by law for the validity and perfection of contracts are
present. However, the parties shall be bound by their real
agreement for a consideration of P1,000,000 as reflected in their
Joint Affidavit.
Deed of Sale not an Equitable Mortgage
For Articles 1602 and 1604 to apply, two requisites must concur:
one, the parties entered into a contract denominated as a
contract of sale; and, two, their intention was to secure an
existing debt by way of mortgage.
In the present case, however, the Contract does not merely
purport to be an absolute sale. The records and the documentary
COMPILED BY: WIGMORE #WIGMOREFOREVER 128

SALES Case Digest (Atty. Sarona)


Compiled by: Wigmore #wigmoreforever
evidence introduced by the parties indubitably show that the
Contract is, indeed, one of absolute sale. There is no clear and
convincing evidence that the parties agreed upon a
mortgage of the subject property.
Furthermore, the voluntary, written and unconditional acceptance
of contractual commitments negates the theory of equitable
mortgage. There is nothing doubtful about the terms of, or the
circumstances surrounding, the Deed of Sale that would call for
the application of Article 1602. The Joint Affidavit indisputably
confirmed that the transaction between the parties was a sale.
We find no basis to conclude that the purchase price of the
property was grossly inadequate. Petitioners did not present any
witness to testify as to the market values of real estate in the
subjects locale. They made their claim on the basis alone of the
P2,000,000 loan that respondent had been able to obtain from
the Rizal Commercial Banking Corporation. This move did not
sufficiently show the alleged inadequacy of the purchase price. A
mortgage is a mere security for a loan. There was no showing
that the property was the only security relied upon by the bank;
or that the borrowers had no credit worthiness, other than the
property offered as collateral.
Co-Ownership
The appellate court was correct in affirming the validity of the
sale of the property insofar as the pro indiviso share of
Esperanza Balite was concerned.
Article 493 of the Civil Code gives the owner of an undivided
interest in the property the right to freely sell and dispose of such
interest. The co-owner, however, has no right to sell or alienate a
specific or determinate part of the thing owned in common,
because such right over the thing is represented by an aliquot or
ideal portion without any physical division. Nonetheless, the
mere fact that the deed purports to transfer a concrete portion
does not per se render the sale void. The sale is valid, but only
with respect to the aliquot share of the selling co-owner.
Furthermore, the sale is subject to the results of the partition
upon the termination of the co-ownership.
Hence, the transaction between Esperanza Balite and
respondent could be legally recognized only in respect to
the formers pro indiviso share in the co-ownership. As a
matter of fact, the Deed of Absolute Sale executed between the
parties expressly referred to the 10,000-square-meter portion of
the land sold to respondent as the share of Esperanza in the
conjugal property. Her clear intention was to sell merely her ideal
or undivided share in it. No valid objection can be made against
that intent. Clearly then, the sale can be given effect to the extent
of 9,751 square meters, her ideal share in the property as found
by both the trial and the appellate courts.

On July 9, 1955, Leoncia and her three sons executed a deed


denominated Kasulatan ng Biling Mabibiling Muli, whereby they
sold the land and its existing improvements to the Spouses
Benedicto Francia and Monica Ajoco (Spouses Francia) for
P500.00, subject to the vendors right to repurchase for the same
amount sa oras na sila'y makinabang. Potencianas heirs did not
assent to that deed. Nonetheless, Teofilo and Jose, Jr. and their
respective families remained in possession of the property and
paid the realty taxes thereon.
Leoncia and her children did not repay the amount of P500.00.
The Spouses Francia both died intestate. Alejandro, the son of
Jose, Sr., first partially paid to the Spouses Francia the amount
of P265.00 for the obligation of Leoncia, his uncles and his
father. Alejandro later paid the balance of P235.00. Thus, on
August 11, 1970, the heirs of Spouses Francia executed a deed
entitled Pagsasa-ayos ng Pag-aari at Pagsasalin, whereby they
transferred and conveyed to Alejandro all their rights and
interests in the property for P500.00.
On August 21, 1970, Alejandro executed a Kasulatan ng
Pagmeme-ari, wherein he declared that he had acquired all the
rights and interests of the heirs of the Spouses Francia, including
the ownership of the property, after the vendors had failed to
repurchase within the given period. On the basis of the
Kasulatan ng Pagmeme-ari, Tax Declaration was issued to
Alejandro. From then on, he had paid the realty taxes for the
property.
Nevertheless, on October 17, 1970, Alejandro, his grandmother
(Leoncia), and his father (Jose, Sr.) executed a Magkakalakip na
Salaysay, by which Alejandro acknowledged the right of Leoncia,
Jose, Jr., and Jose, Sr. to repurchase the property at any time for
the same amount of P500.00.
On October 22, 1970, Leoncia died intestate. She was survived
by Jose, Sr., Teofilo, Jose, Jr. and the heirs of Potenciana. Even
after Leonicas death, Teofilo and Jose, Jr., with their respective
families, continued to reside in the property.
On September 2, 1993, Alejandro also died intestate. Surviving
him were his wife, Amanda, and their children. In 1994,
respondent Amanda Reyes asked the heirs of Teofilo and Jose,
Jr., to vacate the property because she and her children already
needed it. After the petitioners refused to comply, she filed a
complaint against the petitioners in the barangay, seeking their
eviction from the property. When no amicable settlement was
reached, the Barangay Lupon issued a certification to file action
to the respondents on September 26, 1994.
In the interim, petitioner Nenita R. de la Cruz and her brother
Romeo Reyes also constructed their respective houses on the
property.

HEIRS OF REYES VS. REYES


FACTS: Antonio Reyes and his wife, Leoncia Mag-isa Reyes
(Leoncia), were owners of a parcel of residential land located in
Pulilan, Bulacan. On that land they constructed their dwelling.
The couple had four children, namely: Jose, Sr., Teofilo, Jose, Jr.
and Potenciana. Antonio Reyes died intestate, and was survived
by Leoncia and their three sons. Potenciana also died intestate,
survived by her children.

ISSUE: Whether or not the Court of Appeals erred in finding that


respondents (were) already barred from claiming that the
transaction entered into by their predecessors-in-interest was an
equitable mortgage and not a pacto de retro sale.
RULING: The true agreement of the parties vis--vis the
Kasulatan ng Biling Mabibiling Muli was an equitable
mortgage, not a pacto de retro sale. There was no dispute that
the purported vendors had continued in the possession of the
COMPILED BY: WIGMORE #WIGMOREFOREVER 129

SALES Case Digest (Atty. Sarona)


Compiled by: Wigmore #wigmoreforever
property even after the execution of the agreement; and that the
property had remained declared for taxation purposes under
Leoncias name, with the realty taxes due being paid by Leoncia,
despite the execution of the agreement. Such established
circumstances are among the badges of an equitable mortgage
enumerated in Article 1602, paragraphs 2 and 5 of the Civil
Code, to wit:
Art. 1602. The contract shall be presumed to be an equitable
mortgage, in any of the following cases:
xxx
(2) When the vendor remains in possession as lessee or
otherwise;
xxx
(5) When the vendor binds himself to pay the taxes on the thing
sold;
xxx
The existence of any one of the conditions enumerated under
Article 1602 of the Civil Code, not a concurrence of all or of a
majority thereof, suffices to give rise to the presumption that the
contract is an equitable mortgage. Consequently, the contract
between the vendors and vendees (Spouses Francia) was an
equitable mortgage.
Are the petitioners now barred from claiming that the transaction
under the Kasulatan ng Biling Mabibiling Muli was an equitable
mortgage by their failure to redeem the property for a long period
of time?

conceal the true nature of a contract, that is, a loan secured by a


mortgage. It is a reality that grave financial distress renders
persons hard-pressed to meet even their basic needs or to
respond to an emergency, leaving no choice to them but to sign
deeds of absolute sale of property or deeds of sale with pacto de
retro if only to obtain the much-needed loan from unscrupulous
money lenders.[30] This reality precisely explains why the
pertinent provision of the Civil Code includes a peculiar rule
concerning the period of redemption, to wit:
Art. 1602. The contract shall be presumed to be an equitable
mortgage, in any of the following cases:
xxx
(3)When upon or after the expiration of the right to repurchase
another instrument extending the period of redemption or
granting a new period is executed;
xxx
Ostensibly, the law allows a new period of redemption to be
agreed upon or granted even after the expiration of the equitable
mortgagors right to repurchase, and treats such extension as
one of the indicators that the true agreement between the parties
is an equitable mortgage, not a sale with right to repurchase. It
was indubitable, therefore, that the Magkasanib na Salaysay
effectively afforded to Leoncia, Teofilo, Jose, Sr. and Jose, Jr. a
fresh period within which to pay to Alejandro the redemption
price of P500.00.

ALUDOS VS SUERTE
Considering that sa oras na silay makinabang, the period of
redemption stated in the Kasulatan ng Biling Mabibiling Muli,
signified that no definite period had been stated, the period to
redeem should be ten years from the execution of the contract,
pursuant to Articles 1142 and 1144 of the Civil Code. Thus, the
full redemption price should have been paid by July 9, 1955; and
upon the expiration of said 10-year period, mortgagees Spouses
Francia or their heirs should have foreclosed the mortgage, but
they did not do so. Instead, they accepted Alejandros payments,
until the debt was fully satisfied by August 11, 1970.
The acceptance of the payments even beyond the 10-year
period of redemption estopped the mortgagees heirs from
insisting that the period to redeem the property had already
expired. Their actions impliedly recognized the continued
existence of the equitable mortgage. The conduct of the original
parties as well as of their successors-in-interest manifested that
the parties to the Kasulatan ng Biling Mabibiling Muli really
intended their transaction to be an equitable mortgage, not a
pacto de retro sale.
Both the trial court and the CA declared that the Magkasanib na
Salaysay, which extended the redemption period of the
mortgaged property, was inefficacious, because the period to
redeem could no longer be extended after the original
redemption period had already expired.
The provisions of the Civil Code governing equitable mortgages
disguised as sale contracts, like the one herein, are primarily
designed to curtail the evils brought about by contracts of sale
with right to repurchase, particularly the circumvention of the
usury law and pactum commissorium.[29] Courts have taken
judicial notice of the well-known fact that contracts of sale with
right to repurchase have been frequently resorted to in order to

FACTS: Lomises acquired from Baguio City Government the


right to occupy two stalls in the Hangar Market in Baguio City.
Lomises entered into an agreement with respondent johnny
Suerte for the transfer of all improvements and rights over the
two market stalls. Suerte gave down payment and there is a
receipt. Suerte gave another payment but before the completion
of payment, Lomises backed out of the agreement and returned
what was paid and there was a receipt for this. Johnny did not
want the return of his money and wanted the continuation and
enforcement of his agreement. He filed for specific performance
with damages.
RTC: nullified the agreement between johnny and Lomises for
failure to secure the consent of the baguio city government and
held that Lomises are merely lessees and the government was
the lessor owner of stalls. Lomises appealed to CA: the real
agreement between the parties was merely one of loan and not
for sale. The load had been extinguished upon the return of
amount to Johnnys mother Domes.
CA: there are two agreements entered into between Johnny and
Lomises: one for assignment of leasehold rights and the other
was for the sale of the improvements on the market stalls. The
assignment of leasehold rights was void for lack of consent of the
lessor, Baguio government. The sale of improvements was valid
because there were Lomises private properties.
Petitioner: agreement was a loan.
ISSUE: W/N the CA was correct in characterizing the agreement
between Johnny and Lomises as a sale of improvements and
assignment of leasehold rights
COMPILED BY: WIGMORE #WIGMOREFOREVER

130

SALES Case Digest (Atty. Sarona)


Compiled by: Wigmore #wigmoreforever
HELD: What existed was an equitable mortgage, as
contemplated in Article 1602, in relation with Article 1604, of the
Civil Code. An equitable mortgage has been defined as one
which although lacking in some formality, or form or words, or
other requisites demanded by a statute, nevertheless reveals the
intention of the parties to charge real property as security
for a debt, there being no impossibility nor anything contrary to
law in this intent. Article 1602 of the Civil Code lists down the
circumstances that may indicate that a contract is an equitable
mortgage:
Art. 1602. The contract shall be presumed to be an
equitable mortgage, in any of the following cases:
When the price of a sale with right to repurchase is
unusually inadequate;
When the vendor remains in possession as lessee or
otherwise;
When upon or after the expiration of the right to repurchase
another instrument extending the period of redemption or
granting a new period is executed;
When the purchaser retains for himself a part of the
purchase price;
When the vendor binds himself to pay the taxes on the
thing sold;
In any other case where it may be fairly inferred that the
real intention of the parties is that the transaction shall
secure the payment of a debt or the performance of any
other obligation.
In any of the foregoing cases, any money, fruits, or other
benefit to be received by the vendee as rent or otherwise
shall be considered as interest.
Based on Lomises allegations in his pleadings, we consider
three circumstances to determine whether his claim is well
supported. First, Johnny was a mere college student dependent
on his parents for support when the agreement was executed,
and it was Johnnys mother, Domes, who was the party actually
interested in acquiring the market stalls. Second, Lomises
received only P48,000.00 of the P68,000.00 that Johnny claimed
he gave as down payment; Lomises said that the P20,000.00
represented interests on the loan. Third, Lomises retained
possession of the market stalls even after the execution of the
agreement. Whether separately or taken together, these
circumstances do not support a conclusion that the parties
only intended to enter into a contract of loan.
That Johnny was a mere student when the agreement was
executed does not indicate that he had no financial capacity to
pay the purchase price. At that time, Johnny was a 26-year old
third year engineering student who operated as a businessman
as a sideline activity and who helped his family sell goods in the
Hangar Market. During trial, Johnny was asked where he was to
get the funds to pay the P260,000.00 purchase price, and he
said he would get a loan from his grandfather. That he did not
have the full amount at the time the agreement was executed
does not necessarily negate his capacity to pay the purchase
price, since he had 16 months to complete the payment. Apart
from Lomises bare claim that it was Johnnys mother, Domes,
who was interested in acquiring his market stalls, we find no
other evidence supporting the claim that Johnny was merely
acting as a dummy for his mother.

Lomises contends that of the P68,000.00 given by Johnny, he


only received P48,000.00, with the remaining P20,000.00
retained by Johnny as interest on the loan. However, the
testimonies of the witnesses presented during trial, including
Lomises himself, negate this claim. Judge Rodolfo Rodrigo
(RTC of Baguio City, Branch VII) asked Lomises lawyer, Atty.
Lockey, if they deny receipt of the P68,000.00; Atty. Lockey said
that they were not denying receipt, and added that they had in
fact returned the same amount. Judge Rodrigo accurately
summarized their point by stating that there is no need to
dispute whether the P68,000.00 was given, because if [Lomises]
tried to return that x x x he had received that. Witness Atty. Albert
Umaming said he counted the money before he drafted the
October 9, 1985 receipt evidencing the return; he said that
Lomises returned P68,000.00 in total. Thus, if the transaction
was indeed a loan and the P20,000.00 interest was already
prepaid by Lomises, the return of the full amount of P68,000.00
by Lomises to Johnny (through his mother, Domes) would not
make sense.
That Lomises retained possession of the market stalls even after
the execution of his agreement with Johnny is also not an
indication that the true transaction between them was one of
loan. Johnny had yet to complete his payment and, until
Lomises decided to forego with their agreement, had four more
months to pay; until then, Lomises retained ownership and
possession of the market stalls.
Lomises cannot feign ignorance of the import of the terms of the
receipt of September 8, 1984 by claiming that he was an illiterate
old man. A witness (Ana Comnad) testified not only of the fact of
the sale, but also that Lomises daughter, Dolores, translated the
terms of the agreement from English to Ilocano for Lomises
benefit. Lomises himself admitted this fact. If Lomises believed
that the receipt of September 8, 1984 did not express the parties
true intent, he could have refused to sign it or subsequently
requested for a reformation of its terms. Lomises rejected the
agreement only after Johnny sought to enforce it.
Hence, the CA was correct in characterizing the agreement
between Johnny and Lomises as a sale of improvements and
assignment of leasehold rights.

C. Legal Redemption

PART XI: ASSIGNMENT

PART XII: LEASE

REGINA DIZON ET AL V. CA AND OVERLAND EXPRESS


LINES, INC.
G.R. No. 122544 January 28, 1999
FACTS: Overland Express Lines, Inc. entered into a Contract of
Lease with Option to Buy with petitioners involving a 1,755.80
square meter parcel of land situated at Diliman, Quezon City.
The term of the lease was for 1 year commencing from May
COMPILED BY: WIGMORE #WIGMOREFOREVER

131

SALES Case Digest (Atty. Sarona)


Compiled by: Wigmore #wigmoreforever
16,1974 up to May 15, 1975. During this period, Overland
Express Lines was granted an option to purchase for the amount
of P3,000.00 per square meter. Thereafter, the lease shall be on
a per month basis with a monthly rental of P3,000.00.
For failure of Overland Express Lines to pay the increased rental
of P8,000.00 per month effective June 1976, petitioners filed an
action for ejectment against it. Overland Express Lines were
ordered to vacate the leased premises and to pay the sum of
P624,000.00 representing rentals in arrears and/or as damages
in the form of reasonable compensation for the use and
occupation of the premises during the period of illegal detainer
from June 1976 to November 1982.
Overland Express Lines Inc. then endorsed P300,000.00 as
partial payment for the leased property and as an attempt to
resurrect the lapsed option of purchasing the property, which
petitioners accepted (through Alice A. Dizon,) with the issuance
of an official receipt.
ISSUES:
1. Whether Alice Dizon was authorized to receive the sum of
P300,000.00 on behalf of petitioners and validly consider it as
the partial payment of the property to be purchased by the
respondent(Overland Express Lines, Inc.).
2. Whether there was a perfected contract of sale between the
parties.
HELD:
1. Alice Dizon is not authorized to receive the sum of
P300,000.00 on behalf of petitioners, therefore will not validly
bind the petitioners with the private respondents to a contract of
sale.
Article 1874 of the Civil Code is explicit that: "When a sale of a
piece of land or any interest therein is through an agent, the
authority of the latter shall be in writing; otherwise, the sale shall
be void." There was no written proof and valid consent by the
petitioners (as co-owners of the leased premises) on the
supposed sale entered into by Alice A. Dizon, as petitioners
alleged agent, and Overland Express Lines.
The sum of P300,000.00 received by Alice Dizon cannot be
considered as partial payment for the purchase of the property
for the reason that the option given to the private respondent in
purchasing the property as an added condition in the contract of
lease already expired on May 1975.
As provided in Art 1670 of the Civil Code, the provision entitling
the lessee the option to purchase the leased premises is not
deemed incorporated in the impliedly renewed contract because
it is alien to the possession of the lessee. Private respondent's
right to exercise the option to purchase expired with the
termination of the original contract of lease for one year.
2. There was no perfected contract of sale in the first place
because Alice Dizon was not an authorized agent of petitioner,
therefore she cannot do any legal transactions with the
respondent.
As enshrined in Art 1868, By the contract of agency a person
binds himself to render some service or to do something in
representation or on behalf of another, with the consent or
authority of the latter.

VIEGELY SAMELO vs MANOTOK SERVICES, INC.


G.R. NO. 170509 June 27, 2012
FACTS: On January 31, 1997, the respondent entered into a
contract with the petitioner for the lease of a portion of the lot for
a period of 1 year. Upon the expiration of the lease contract on
December 31, 1997, the petitioner continued occupying the
subject premises without paying the rent. On August 5, 1998, the
respondent sent a letter to the petitioner demanding that she
vacate the subject premises and pay compensation for its use
and occupancy however, petitioner refused to heed these
demands.
On November 18, 1998, the respondent filed a complaint for
unlawful detainer against the petitioner befor the MeTC praying
that the petitioner be ordered to vacate the subject premises and
to pay compensation for its use and occupancy.
In her answer, the petitioner alleged that the respondent had no
right to collect rentals because the subject premises are located
inside the property of the Philippine national Railways (PNR).
She also added that the petitioner had no certificate of title over
the subject premises and further claimed that her signature in the
contract of lease was obtained through respondents
misrepresentation and likewise maintained that she is now the
owner of the subject premises as she had been in possession
since 1944.
The MeTC decided in favor of the respondent and held that the
only issue to be resolved in an unlawful detainer case is physical
possession or possession de facto, and that the respondent had
established its right of possession over the subject premises. It
added that the petitioners right under the lease contract already
ceased upon the expiration of the said contract. It further ruled
that the petitioner is already stopped from questioning the right of
the respondent over the subject premises when she entered into
a contract of lease with the respondent.
On appeal, the RTC set aside the decision of the MeTC and
dismissed the complaint for unlawful detainer. The RTC held that
the respondent had no right to collect rentals as it failed to show
that it had authority to administer subject premises and to enter
into a contract of lease with the petitioner.
Aggrieved by the reversal, the respondent filed a petition for
review with the CA which reversed and set aside the RTC
decision and reinstated the MeTC judgment. The CA held that
the petitioner is now estopped from questioning the right of the
respondent over the subject property. It explained that in an
action involving the possession of the subject premises, a tenant
cannot controvert the title of his landlord or assert any right
adverse to that title, without first delivering to the landlord the
premises acquired by virtue of the agreement between
themselves. It further held that the only issue in an ejectment suit
is physical or material possession. The issue of ownership is not
required to determine the issue of possession since the petitioner
tacitly admitted that she is a lessee of the subject premises.
Petitioner moved for reconsideration but the CA denied her in its
motion, hence this petition.
ISSUE: The ejectment suit is physical or material possession.
COMPILED BY: WIGMORE #WIGMOREFOREVER 132

SALES Case Digest (Atty. Sarona)


Compiled by: Wigmore #wigmoreforever
(The issue of ownership is not required to determine the issue of
possession since the petitioner tacitly admitted that she is a
lessee of the subject premises.)
RULING: The SC held that an action for unlawful detainer exists
when a person unlawfully withholds possession of any land or
building against or from a lessor, vendor, vendee or other
persons, after the expiration or termination of the right to hold
possession, by virtue of any contract, express or implied. The
only issue to be resolved in an unlawful detainer case is physical
or material possession of the property involved, independent of
any claim of ownership by any of the parties involved. Thus, any
attempt of the parties to inject the question of ownership into the
case is futile, except insofar as it might throw light on the right of
possession.
In the instant case, the lease contract was for the period of one
year with a monthly rental of P3,960 commencing on January 31,
1997 and expiring on December 31, 1997. It bears emphasis that
it was only on August 5, 1998 that a notice to vacate was sent
and the petitioner continued enjoying the subject premises for
more than 15 days, without objection from the respondent. By
the inaction of the respondent as lessor, there can be no
inference that it intended to discontinue the lease contract,
therefore, an implied new lease was therefore created pursuant
to Article 1670 of the Civil Code which provides:
Article 1670. If at the end of the contract of lease the lessee
should continue enjoying the thing leased for 15 days with the
acquiescence of the lessor, and unless a contrary by either party
has previously been given, it is understood that there is implied
new lease, not for the period of the original contract, but for the
time established in Articles 1682 and 1687. The other terms of
the original contract shall be revived.
An implied new lease or tacita reconduccion will set in when the
following requisites are found to exist:
(a) The term of the original contract of lease has expired;
(b) The lessor has not given the lessee a demand to vacate; and
(c) The lessee continued enjoying the thing leased for 15 days
with the acquiescence of the lessor.

act on the part of the lessor that it no longer consents to the


continued occupation by the lessee of its property. After such
notice, lessees right to continue in possession ceases and her
possession becomes one of detainer.
Wherefore, petition was denied and the decisions of CA were
affirmed with modification on the unpaid rentals due.

SPS. MAMARIL VS. BOY SCOUT OF THE PHILIPPINES


G.R. NO. 179382 | JANUARY 14, 2013
FACTS: PUJ operators Sps. Mamaril would park their 6
passenger jeepneys every night at BSPs compound in Malate,
Manila for a fee of P300.00 per month for each unit. One day,
one of the vehicles was missing and was never recovered.
According to the security guards Pea and Gaddi of AIB Security
Agency with whom BSP had contracted for its security and
protection, a male person who looked familiar to them took the
subject vehicle out of the compound. Sps. Mamaril prayed that
Pea and Gaddi, together with AIB and BSP, be held liable for:
(a) the value of the subject vehicle; (b) amount representing daily
loss of income/boundary reckoned from the day the vehicle was
lost; (c) exemplary damages; (d) moral damages; (e) attorney's
fees; and (f) cost of suit.
BSP denied any liability contending that not only did Sps.
Mamaril directly deal with AIB with respect to the manner by
which the parked vehicles would be handled, but the parking
ticket itself expressly stated that the "Management shall not be
responsible for loss of vehicle or any of its accessories or article
left therein." It also claimed that Sps. Mamaril erroneously relied
on the Guard Service Contract. Apart from not being parties
thereto, its provisions cover only the protection of BSP's
properties, its officers, and employees.
ISSUE: Whether or not BSP may be held liable for the loss of the
vehicle caused by the negligence of its security guards. NO
HELD: The proximate cause of the loss of Sps. Mamaril's vehicle
was the negligent act of security guards Pea and Gaddi in
allowing an unidentified person to drive out the subject vehicle.

Article 1687 of the CC on implied new lease provides:


Article 1687. If the period for the lease has not been fixed, it is
understood to be from year to year, if the rent is to be paid
annual; from month to month if it is monthly; from week to week,
if the rent is weekly; and from day to day, if the rent id to be paid
daily. However, even though a monthly rent is paid, and no
period for the lease has been set, the court may fix a longer term
for the lease after the lessee has occupied the premises for over
one year. If the rent id weekly, the court may likewise determine
a longer period after the lessee has been in possession for over
6 months. In case of daily rent, the courts may fix a longer period
after the lessee has stayed in the place for over one month.
Since the rent was paid on a monthly basis, the period of lease is
considered to be from month to month. A lease from month to
month is considered to be one with a definite period, which
expires at the end of each month upon a demand to vacate by
the lessor. When the respondent sent a letter to vacate to the
petitioner on August 5, 1998, the tacita reconduccion was
aborted, and the contract of lease is deemed to have expired at
the end of that month. A notice to vacate constitutes an express

The records are bereft of any finding of negligence on the part of


BSP. Neither will the vicarious liability of an employer under
Article 2180 of the Civil Code apply in this case. Pea and Gaddi
were assigned as security guards by AIB to BSP pursuant to the
Guard Service Contract. No employer-employee relationship
existed between BSP and the security guards assigned in its
premises. Sps. Mamaril are not parties to the Guard Service
Contract. Guard Service Contract between defendant-appellant
BSP and defendant AIB Security Agency is purely between the
parties therein.
Contracts take effect only between the parties, their assigns and
heirs, except in case where the rights and obligations arising
from the contract are not transmissible by their nature, or by
stipulation or by provision of law. The heir is not liable beyond the
value of the property he received from the decedent. If a contract
should contain some stipulation in favor of a third person, he may
demand its fulfillment provided he communicated his acceptance
to the obligor before its revocation. A mere incidental benefit or
interest of a person is not sufficient. The contracting parties must
have clearly and deliberately conferred a favor upon a third
COMPILED BY: WIGMORE #WIGMOREFOREVER 133

SALES Case Digest (Atty. Sarona)


Compiled by: Wigmore #wigmoreforever
person.
Thus, in order that a third person benefited by the second
paragraph of Article 1311, referred to as a stipulation pour autrui,
may demand its fulfillment, the following requisites must concur:
(1) There is a stipulation in favor of a third person; (2) The
stipulation is a part, not the whole, of the contract; (3) The
contracting parties clearly and deliberately conferred a favor to
the third person - the favor is not merely incidental; (4) The favor
is unconditional and uncompensated; (5) The third person
communicated his or her acceptance of the favor before its
revocation; and (6) The contracting parties do not represent, or
are not authorized, by the third party. However, none of the
foregoing elements obtains in this case. There is absolutely
nothing in the said contract that would indicate any obligation
and/or liability on the part of the parties therein in favor of third
persons such as herein plaintiffs-appellees.
Moreover, the Court concurs with the finding of the CA that
the contract between the parties herein was one of lease as
defined under Article 1643 of the Civil Code. It has been held
that the act of parking a vehicle in a garage, upon payment
of a fixed amount, is a lease.
A lessor-lessee relationship existed between Spouses
Mamaril and BSP. Article 1664 of the same Code states that
the lessor is not obliged to answer for a mere act of
trespass which a third person may cause on the use of the
thing leased; but the lessee shall have a direct action
against the intruder. Here, BSP was not remiss in its
obligation to provide Spouses Mamaril a suitable parking
space for their jeepneys as it even hired security guards to
secure the premises; hence, it should not be held liable for
the loss suffered by Spouses Mamaril.
The agreement with respect to the ingress and egress of
Sps. Mamaril's vehicles were coordinated only with AIB and
its security guards, without the knowledge and consent of
BSP. Accordingly, the mishandling of the parked vehicles
that resulted in herein complained loss should be recovered
only from the tort feasors (Pea and Gaddi) and their
employer, AIB; and not against the lessor, BSP.

END

COMPILED BY: WIGMORE #WIGMOREFOREVER

134

You might also like